Prévia do material em texto
u oee & % rm" TT:^ÍB^dreesç Dorin Andrica ^BBEDDDDDBBeSBBEBBi ^^BBBBr . WBBBBBBBB . ._..JDGIBBBfflr zll □□□□□□E3HBI ■i IjLjor ca □□□□□□□□□’ a:: :;..®asaBD □□^BBaapsi .•::£^e]hbbd ar . í jEmDúd£Lxr!^ íãHBBBIBI -•• _ Titu Andreescu Dorin Andrica Editora VestSeller Fortaleza - Ceará 1a edição - 2013 Números Complexos de A à Z I. Matemática II. Números Complexos CDD 510 Andreescu, Titu; Andrica, Dorin. Números Complexos de A a Z / Titu Andrescu, Dorin Andrica Fortaleza: Vestseller, 1a edição 2013, 341 p.; É proibida a reprodução parcial ou total por quaisquer meios sem autorização prévia do autor. Os transgressores serão punidos nos termos da lei. Denuncie o plágio, cópias ilegais, pirataria pela internet, sites para download pirata, comunidades piratas na internet anonimamente através do correio eletrônico da Editora: DireitoAutoral@VestSeller.com.br Traduzido da Edição Inglesa Complex Numbers from A to ...Z by Titu Andreescu and Dorin Andrica Copyright © 2005, Birkhãuser Boston Birkhãuser Boston is a part of Springer Science+Business Media All Rights Reserved Publicado no Brasil com exclusividade pela VestSeller www.vestSelier.com.br Editor responsável: Renato Brito Bastos Neto Editoração: Alex Sousa Tradução: Caio Richer Capa: Mary Burgess Dorin Andrica "Babes,-Bolyai" University Faculty of Mathematics 3400 Cluj-Napoca Romania Titu Andreescu University of Texas at Dallas School of Natural Sciences and Mathematics Richardson, TX 75083 U.S.A. Classificação dos assuntos de Matemática (2000): 00A05, 00A07, 30-99, 30A99, 97U40 Esta obra pode ser adquirida diretamente na LIVRARIA VESTSELLER através de sua página eletrônica www vestseller.com br mailto:DireitoAutoral@VestSeller.com.br http://www.vestSelier.com.br vestseller.com Jacques Hadamard O menor caminho entre duas verdades do domínio real passa através do domínio complexo. Sobre os Autores Dorin Andrica obteve seu PhD em 1992 na Universidade de Babes-Bolyai em Cluj- Napoca, Romênia, com a tese sobre pontos críticos e suas aplicações na geometria de subvariedades diferenciáveis. O professor Andrica tem sido o presidente do Departamento de Geometria em Babes-Bolyai desde 1995. Dorin tem escrito e contribuído com vários livros didáticos de matemática, livros de exercícios, artigos e artigos científicos em vários níveis. É um palestrante convidado para várias conferências universitárias ao redor do mundo - Áustria, Bulgaria, República Tcheca, Egito, França, Alemanha, Grécia, Holanda, Sérvia, Turquia e Estados Unidos. Ele é membro do Comitê Romeno de Olimpíadas de Matemática e membro do conselho editorial de várias revistas internacionais. Dorin tem sido membro da Canada-USA Mathcamps desde 2001. Titu Andreescu conseguiu seu bacharelado, mestrado e PhD na Universidade de Timisoara, Romênia. Sua tese de doutorado foi "Pesquisa sobre Análise Diofantina e suas Aplicações". Atualmente o professor ensina na Universidade do Texas, em Dallas. É o ex presidente da Olimpíada de Matemática dos Estados Unidos, atuando como diretor do MAA-AMC (1998-2003), treinador do time americano da Olimpíada Internacional de Matemática (IMO) por 10 anos (1993-2002), diretor do Programa de Verão de Olimpíadas de Matemática (1995-2002) e líder do time americano na IMO(1995-2002). Em 2002 Titu foi eleito membro do Conselho Consultivo da IMO, o corpo diretivo da mais prestigiosa competição de matemática do mundo. Recebeu da MAA em 1994 o prêmio Edyth May Sliffe pelo significante Ensino de Matemática em Universidades e um "Certificado de Apreciação” do presidente da MAA em 1995 por seus reconhecidos serviços prestados como treinador do Programa de Verão de Olimpíadas de Matemática na preparação do time americano para a excelente perfomance em Hong Kong, na IMO de 1994. As contribuições de Titu para vários livros didáticos e livros de exercícios são reconhecidas em todo o mundo. Sumário 1. 2. Prefácio ix 1.1.1. 1.1.2. 1.1.3. 1.1.4. 1.1.5. 1.1.6. 1.1.7. 1.1.8. 1.1.9. 2.1.1. 2.1.2. 2.1.3. 2.1.4. 2.1.5. 1.2.1. 1.2.2. 1.2.3. 1.2.4. 2.2.1. 2.2.2. 41 41 41 43 47 51 51 53 .53 .56 13 13 13 14 14 16 18 19 21 26 29 34 35 35 36 39 Notação xiii Números Complexos na Forma Algébrica....................................... 1.1. Representação Algébrica dos Números Complexos................. Definição dos números complexos.............................. Propriedades da adição................................................ Propriedades da multiplicação.................................... Números complexos na forma algébrica.................... Potências do número i................................................... Conjugado de um número complexo.......................... Módulo de um número complexo .............................. Resolvendo equações quadráticas ........................... Problemas ..................................................................... 1.2. Interpretação Geométrica das Operações Algébricas............. Interpretação geométrica de um número complexo. Interpretação geométrica do módulo......................... Interpretação geométrica das operações algébricas. Problemas .................................................................... Números Complexos na Forma Trigonométrica................................... 2.1. Representação Polar dos Números Complexos ............................... Coordenadas polar no plano............................................... Representação polar de um número complexo................ Operações com números complexos na forma polar...... Interpretação geométrica da multiplicação....................... Problemas ............................................................................ 2.2. As raizes enésimas da unidade......................................................... Definindo as raizes enésimas de um número complexo As raizes enésimas da unidade ........................................ 3.6.1. 3.6.2. 3.6.3. 2.2.3. 2.2.4. 3.1.1. 3.1.2. 3.1.3. 3.1.4. 3.1.5. 3.1.6. 3.5.1. 3.5.2. 3.5.3. 3.5.4. 3.5.5. 3.5.6. Equações binomiais Problemas ..... 102 102 108 112 115 119 124 124 125 126 128 66 66 66 66 70 71 73 74 78 81 84 90 90 91 93 64 64 4.6.1. 4.6.2. 4.6.3. 4.6.4. 4. Mais sobre Números Complexos e Geometria ............................... 4.1. Produto Real entre Dois Números Complexos............................ 4.2. Produto Complexo entre Dois Números Complexos................... 4.3. Área de um Polígono Convexo....................................................... 4.4. Cevianas Concorrentes e Pontos Importantes de um Triângulo. 4.5. Circunferência dos Nove Pontos de Euler.................................... 4.6. Algumas Distâncias Importantes em um Triângulo..................... Invariantes fundamentais de um triângulo.................... A distância OI................................................................... A distância ON.................................................................. A distância OH.................................................................. 3. Números Complexos e Geometria........................................................ 3.1. Simples Noções Geométricas e Propriedades............................... Distância entre dois pontos.............................................. Segmentos, raios e retas.................................................. Dividindo um segmento em uma razão dada................ Medida de um ângulo........................................................ Ângulo entre duas retas................................................... Rotação de um ponto........................................................ 3.2. Condições para Colinearidade. Ortogonalidade e Conciclicidade 3.3. Triângulos Semelhantes.................................................................... 3.4. Triângulos Equiláteros....................................................................... 3.5. Geometria Analítica no Plano Complexo........................................Equação da reta................................................................. Equação da reta determinada por dois pontos.............. Área de um triângulo......................................................... Equação da reta determinada por um ponto e uma direção.95 Pé da perpendicular de um ponto até uma reta......................97 Distância de ponto à reta............................................................ 97 3.6. A circunferência............................................................................................. 97 Equação da circunferência.........................................................97 Potência de ponto em relação a uma circunferência............ 99 Ângulo entre duas circunferências..........................................100 6. 4.13.1. 4.13.2. 4.13.3. 4.13.4. 4.13.5. 4.13.6. 4.13.7. 4.13.8. 6.1.1. 6.1.2. 6.1.3. 6.1.4. 6.1.5. 175 176 192 199 208 228 235 244 253 262 274 274 274 278 279 281 284 5.3. 5.4. 5.5. 5.6. 5.8. 5.9. Respostas, Dicas e Soluções dos Problemas Propostos.................. 6.1. Respostas, Dicas e Soluções dos Problemas.................................. Números complexos na representação algébrica........... Interpretação geométrica dos operações algébricas...... Representação polar dos números complexos............... As raizes enésimas da unidade........................................ Algumas transformações geométricas do plano complexo 4.9 1. 4.9.2. 4.10. Área do Triângulo Antipedal...................................................... 4.11. Teorema de Lagrange e suas Aplicações............................... 4.12. Centro de Euler de um Poligono Inscritivel............................. 4.13. Algumas Transformações Geométricas no Plano Complexo Translação.................................................................... Reflexão em relação ao eixo real............................. Reflexão em relação a um ponto.............................. Rotação........................................................................ Transformações isométricas no plano complexo ... Teorema de Morley..................................................... Homotetia..................................................................... Problemas.................................................................... 5. Problemas Importantes para Olimpíadas.............................. 5.1. Problemas envolvendo Módulos e Conjugados............... 5.2. Equações Algébricas e Polinõmios.................................... De Identidades Algébricas a Propriedades Geométricas. Resolvendo Problemas Geométricos................................. Resolvendo Problemas Trigonométricos.......................... Mais sobre Raizes Enésimas da Unidade........................ Problemas Envolvendo Polígonos..................................... Números Complexos e Combinatória................................ Problemas Diversos.............................................................. 4.7. Distância entre Dois Pontos no Plano de um Triângulo.........................128 4.7.1. Coordenadas baricêntricas.......................................................128 4.7.2. Distância entre dois pontos em coordenadas baricêntricas.... 130 4.8. Área de um Triângulo em Coordenadas Baricêntricas..........................133 4.9. Triângulos Ortopolares................................................................................139 Reta de Simson-Wallance e triângulo pedal...........................139 Condições necessárias e suficientes para ortopolaridade .. 146 150 154 162 169 165 166 166 167 167 169 173 175 Glossário............... Referências........... índice dos Autores, índice de Assuntos 6.2.1. 6.2.2. 6.2.3. 6.2.4. 6.2.5. 6.2.6. 6.2.7. 6.2.8. 6.2.9. 284 284 292 329 335 337 338 6.2. Soluções dos Problemas para Olimpíadas...................................... Problemas envolvendo Módulos e Conjugados (págs 175-176) Equações algébricas e polinômios (pág 181).................. De identidades algébricas a propriedades geométricas (pág 190)... 295 Resolvendo problemas geométricos (págs 211-213)...........297 Resolvendo problemas trigonométricos (pág 220)............... 309 Mais sobre Raizes Enésimas da unidade (págs. 228 - 229). 312 Problemas envolvendo polígonos (pág. 237)........................315 Números Complexos e Combinatória (pág. 245)................. 321 Problemas Diversos (pág. 252)............................................... 324 Prefácio Original (Edições em inglês) Resolver equações algébricas tem sido, historicamente, um dos tópicos favoritos dos matemáticos. Enquanto as equações do primeiro grau são sempre resolviveis nos números reais, nem todas as equações quadráticas apresentam a mesma propriedade. Por exemplo, a equação x2 + 1 = 0. Até o século 18, os matemáticos evitavam equações quadráticas que não fossem resolviveis nos K . Leonhard Euler acabou com a polêmica introduzindo o “número” no seu famoso livro Elements of Álgebra como "... nem nada, nem maior que nada, nem menor que nada..." e observou "... apesar disso, esses números nos levam á imaginação; eles existem na nossa mente e temos uma ideia suficiente sobre eles; ...nada nos impede de fazer uso desses números imaginários e empregá-los nos nossos cálculos". Euler denotou o número V-í de i e o chamou de unidade imaginária. Esse símbolo se tornou um dos mais úteis da matemática. Ele serve pra definir um número complexo como z = a + bi, onde a e b são números reais. O estudo dos números complexos continua e tem sido aprimorado nos dois últimos séculos e meio; na verdade, é impossível imaginar a matemática de hoje sem os números complexos. Toda a matemática faz uso deles de algum modo. Isso é verdade para outras disciplinas também, bem como, por exemplo, mecânica, física teórica, hidrodinâmica e quimica. Nosso principal objetivo é introduzir o leitor a esse assunto fascinante. O livro prossegue suavemente entre conceitos chaves e resultados elementares dos números complexos. O leitor tem a oportunidade de aprender como os números complexos podem ser empregados para resolver equações algébricas e entender a interpretação geométrica e operações deles. A parte teórica do livro é reforçada com vários exercícios dos mais diversos níveis de dificuldade. Nos Capítulos 3 e 4 abordamos importantes aplicações na geometria Euclideana. Muitos problemas de geometria podem ser resolvidos eficientemente e elegantemente usando números complexos. A riqueza de exemplos que fornecemos, a forma pessoal como apresentamos muitos tópicos, os inúmeros problemas originais e a atenção ao detalhar as soluções dos exercícios e problemas selecionados são somente algumas das principais características desse livro. Entre as técnicas apresentadas, por exemplo, estão o produto real e complexo de números complexos. Na linguagem dos números complexos, essas são as analogias aos produtos escalar e vetorial, respectivamente. Ao empregar esses dois produtos, se torna muito mais eficiente o modo como resolver vários problemas envolvendo números complexos. Após estudar essa parte, o leitor irá se familiarizar com o uso dessas técnicas. agradecer àqueles que nos forneceram sugestões pertinentes que contribuiram diretamente com a melhora do livro. Agradecimentos especiais para Daniel Vãcãretu, Nicolae Bijboacã, Gabriel Dospinescu e loan Serdean pela cuidadosa revisão da versão final do manuscrito. Também gostaríamos de Uma característica especial do livro é o Capítulo 5, uma seleção de importantes problemas de Olimpíadas e outras competições de matemática resolvidos utilizando os métodos presentes no livro. Esse trabalho não abrange todos os aspectos pertencentes aos números complexos. Não se trata de um livro de análise complexa, mas sim um degrau a mais no estudo desses, por isso que não usamos a notação padrão e" para z = cost + isent ou as outras séries expandidas. O livro reflete uma experiência única dos autores. Ele destila uma ampla literatura matemática,maioria da qual é desconhecida para o público ocidental, capturando a essência de uma rica cultura de resolução de problemas. Nosso trabalho é em parte baseado em uma versão Romena, Numere complexe de Ia Ala . Z, escrito por D. Andrica e N. Bijboacã e publicado pela Millennium em 2001 (veja nossa referência [10]). Estamos preservando o titulo da edição Romena e cerca de 35% do conteúdo do livro. Até mesmo esses 35% foram significantemente aperfeiçoados e aprimorados com material atualizado. O público alvo inclui estudantes de ensino médio e seus professores, graduandos, estudantes de competições matemáticas tais como estudantes de Olimpíadas ou da Putnam, seus treinadores e qualquer pessoa simplesmente interesseada em matemática. Esse livro pode servir como um curso de Números Complexos e Geometria Euclideana para futuros professores do ensino médio, dando a esses futuros educadores idéias sobre coisas que eles podem fazer com seus alunos mais brilhantes ou com um clube de matemática. Isso seria uma evolução bastante interessante e bem vinda. Titu Andreescu Dorin Andrica Outubro de 2004 Prefácio à 1â Edição Brasileira O livro "Números complexos de A a Z", de Titu Andreescu e Dorin Andrica, é um clássico, que explora magistralmente o caráter multidisciplinar da Matemática. O livro expõe com clareza a teoria e os aspectos históricos dos números complexos, e os utiliza como uma ferramenta extremamente eficiente para o estudo de resultados sutis e profundos de geometria, incluindo um grande número de problemas e idéias sofisticadas envolvendo e combinando números complexos, geometria e desigualdades. Os dois autores são romenos, e o livro se nutre da enorme tradição da Romênia e do leste europeu em Olimpíadas de Matemática. As primeiras Olimpíadas de Matemática registradas surgiram na Hungria, no final do século XIX, e a Romênia organizou a primeira Olimpíada Internacional de Matemática, a IMO. Titu Andreescu, um dos autores, liderou e treinou a famosa equipe norte-americana que, na IMO de 1994 conseguiu pela primeira vez a façanha de obter pontuação perfeita: todos os seus 6 integrantes obtiveram a pontuação máxima na competição - 42 pontos, feito que ainda não foi igualado em nenhuma outra IMO. O livro contém uma grande quantidade de problemas de olimpíadas para os quais as técnicas desenvolvidas fornecem soluções bonitas e elegantes. O livro preenche uma lacuna no Brasil; não são comuns os livros de matemática que, embora acessíveis a alunos e professores do ensino médio, contenham idéias sofisticadas e criativas, e discutam problemas desafiadores. Trata-se de uma obra indispensável para alunos, professores e amantes da matemática em geral, que encontrarão nele material instigante e profundo, que contribuirá para o desenvolvimento de seus talentos matemáticos. Em particular, trata-se de um material precioso para a preparação de alunos e professores que participam de Olimpíadas de Matemática, os quais se beneficiarão da experiência dos autores, da clareza das idéias e da variedade e profundidade dos problemas discutidos no livro. Carlos Gustavo T. de A. Moreira Pesquisador Titular do IMPA e Coordenador Geral da Olimpíada Brasileira de Matemática Notação z N o conjunto dos reais não nulos R2 o conjunto dos números complexos não nulos [a.b] o conjunto dos números reais x tais que a < x < b (a.b) o conjunto dos números reais x tais que a < x < b o conjugado do número complexo zz o módulo ou valor absoluto do número complexo z o vetor AB (AB) o segmento aberto determinado por A e B [AB] o segmento fechado determinado por A e B (AB o raio aberto de origem em A que contém B C(P; n) a circunferência centrada em P e de raio n o conjunto dos inteiros o conjunto dos inteiros positivos o conjunto dos números racionais o conjunto dos números reais o conjunto de pares de números reais o conjunto dos números complexos l£ AB Q R R- área[F] a área da figura F Un o conjunto das raizes enésimas da unidade 1 | Números Complexos na Forma Algébrica 13 1.1. REPRESENTAÇÃO ALGÉBRICA DE NÚMEROS COMPLEXOS para todo z, = (x,,y,) e R2 e z2 = (x2,y2) e R2 ■ R2 é Observações (1) Se z, = (x^O) e R2 e z2 = (x2,0) e R2, então z,.z2 = (x,x2, 0).. (2) Se z, = (O.y,) e R2 e z2 = (0,y2) e R2 , então z,.z2 = (-y1y2, 0).. Exemplos: (2) Sejam zi z, + z2 = 1z,z2 = 2' 4 1.1.1 Definição dos números complexos No que se segue, nós assumimos que a definição e propriedades do conjunto dos números reais R são conhecidas. Consideremos o conjunto R2=R xR = {(x.y) | x,y e R). Dois elementos (x,.y,) e (x2,y2) de R2 são iguais se e somente se x, = x2 e y, = y2. As operações de adição e multiplicação são definidas no conjunto R2 do seguinte modo: z, + z2 = (x„ y,) + (x2, y2) = (x, + x2, y, + y2) e R2 e z,.z2 = (x„ y1).(x2, y2) = (x,x2 - y,y2, x,y2 + x2y,) e R2, e 1 1 1 2 3' ^|-Números-Complexos-na-Forma-Algébrica O elemento z, + z2 e R2 é chamado de soma de z,,z2 e o elemento z,.z2 e chamado de produto de z,,z2 . (1) Sejam z, = (-5,6) e z2=(1,-2). Então z,+ z2 = (-5.6) + (1,-2) = (-4,4) e z,z2 = (-5, 6).(1, -2) = (-5 + 12. 10 + 6) = (7,16) = (4. l) e z2 = (-y. ^tão 4 = í-1. --3) (3 12 Números Complexos de A a Z14 A notação C * é usada para indicar o conjunto C \ {(0,0)}. z, +z2 = z2 + z, para todos z,,z2 e C . zi~^2 — (xi>yi) (x2.y2) -(xi~x2>yi”y?)ev z,.z2 = z2.z, para todos z,,z2eC Definição. O conjunto R2, junto com as operações de adição e multiplicação, é chamado de conjunto dos números complexos, denotado por C . Qualquer elemento z = (x.y) e C é chamado de número complexo. 1.1.2 Propriedades da adição A adição de números complexos satisfaz as seguintes propriedades: (a) Lei comutativa O número z,-z2 = z,+(-z2) é chamado diferença entre os números z, e z2. A operação que relaciona os números z, e z2 ao número z, - z2 é chamada subtração e é definida por 1.1.3 Propriedades da multiplicação A multiplicação de números complexos satisfaz as seguintes propriedades: (a) Lei comutativa (b) Lei associativa (z,+ z2) + z3 = z,+(z2+z3) para todos z,,z2,z3eC. A sentença é verdadeira devido à associatividade dos números reais. (a) Identidade aditiva Existe um único número complexo 0 = (0,0) tal quez + 0= 0 + z = z para todo z = (x,y)eC. (b) Inverso aditivo Para qualquer número complexo z = (x.y) existe um único -z = (—x,—y) e C tal que z + (—z) = (~z) + z = 0 • O leitor pode provar facilmente as sentenças (a), (c) e (d). De fato, se z, = (x,,y,) e C,z2 = (x2,y2) e C,z3 = (x3,y3) e C , então (z,+ z2) + z3 = [(x,,y,) + (x2,y2)] + (x3,y3) = (x, + x2,y, + y2) + (x3,y3) = = ((x, + xj + x3,(y, + y2) + y3) e z1+(z2+z3) = (x1,y,) + [(x2,y2) + (x3,y3)) = (xi.yi)+(x2 + x3.yz+ya) = (xi + (x2+xs).yi+(y? + ys)) 1 | Números Complexos na Forma Algébrica 15 x' = e y' = - z‘ e ~ •z = 1.Pela lei comutativa também temos z' = (x„ y,)z,.z- z 12 + Exemplos: 1) Se z = (1,2), então e 1z = (1. 0).(x, y) = (1x - Oy, 1y + Ox) = (x. y) (c) Identidade multiplicativa Existe um único número complexo 1 = (1,0) e C tal que z1 = 1z = z para todo z e C. Uma simples manipulação algébrica é tudo o que é necessário para verificar essas igualdades: z1 = (x. y).(1, 0) = (x1 - yO. xO + y1) = (x, y) = z = z (b) Lei associativa (z,.z2)z3 = z,(z2.z3) para todos z,,z2,z3 e 2 y x2 + y2 X x2 + y2' (d) Inverso multiplicativo Para qualquer número complexo z = (x,y)eC" existe um único número z“1 =(x’,y’)eC tal que z.z"1 = z"1.z = 1 Para achar z"’=(x',y'), observe que (x,y)#(0,0) implica x<0 ou y *0 e consequentemente x2 + y2 * 0 . A relação z.z-1 = 1 dá (x, y).(x’, y') = (1. 0), ou equivalentemente íxx’ - yy’ = 1 |yx’ + xy' = 0. Resolvendo esse sistema em relação a x' e y'. obtém-se x , y x2 + y2 6 y '777' portanto o inverso multiplicativo do número complexo z = (x,y) e C * é =1=Í_2S___ z ^x2 + y2’ x2+y2 1 -2 22' 12 + 22 I = í x,x + y,y -x,y + y,x ) t x2 + y2 x2 + y2 Dois números complexos z^íx^yJeC e z = (x,y)eC” determinam unicamenteum terceiro número chamado de quociente deles, representado por £1 e definido por z Números Complexos de A a Z16 para todo inteiro n < 0 . ,m-n2) 5) Quando z - 0 , nós definimos 0" = 0 para todo inteiro n > 0. é bijetiva e, além disso, (x,0) + (y,0) = (x + y,0) e (x,0) • (y,0) = (xy,0). O leitor não falhará ao notar que as operações algébricas em R x {0} são similares às operações em K, portanto podemos identificar o par (x,0) com o número x para todo xel? . Logo, podemos usar, pela bijeção de / , a notação (x,0) = x . 1.1.4 Números complexos na forma algébrica Para manipulação algébrica não é conveniente representar um número complexo como um par ordenado. Por essa razão, outra forma de representação é preferida. Definindo i = (0,1) obtemos z = (x, y) = (x,0) + (O.y) = (x,0) + (y,0).(0,1) = x + yi = (x,0) + (0,1).(y,0) = x + iy. (e) Lei distributiva z,(z2 + z3) = z,z2 + z,z3 para todos z1,z2,z3eC. As propriedades da multiplicação e adição acima mostram que o conjunto C de todos os números complexos, junto com essas operações, forma um campo. zn = z ■ z • • ■ z para todo inteiro n > 0 e zn = (z“1) n vezes Para introduzir essa nova representação algébrica, considere o conjunto Rx{0}, juntamente com as operações de adição e multiplicação definidas em R2. A função /: R-> R x {0} /(x) = (x,0) 11 2 25’ 25 _ 2(mn) . As seguintes propriedades valem para todos os complexos z,z,,z2 e C * e para todos os inteiros m,n: 1J Zm Zn ~ ^ = z- z" 3) (zm)n 4) (z^Zj)" = zÇ.zJ; f ílV = ÍL lzzj zj ’ 2) Se z, = (1.2) e z2 = (3,4), então Z, -4^1 = ( z2 (9 + 16 9 + 16j l Uma potência inteira de um número complexo z e C* é definida por z° = 1 ; z1 = z ; z2 = z ■ z ; 1 | Números Complexos na Forma Algébrica 17 Desse modo obtemos Em outras palavras, + lm(z)2.Re(z), z = x + yi, onde x,y são números reais. A relação I2 =-1 é válida. É fácil observar que a soma de dois números complexos é um número complexo cuja parte real (imaginária) é a soma das partes reais (imaginárias) dos números dados. Re(z, + z2) = Re(z), + Re(z)2 lm(z, +z2) = lm(z), +lm(z)2 Proposição. Qualquer número complexo z = (x,y) pode ser unicamente representado na forma Utilizando a representação algébrica, as operações usuais com números complexos podem ser feitas da seguinte maneira: 1. Adição z, + z2 = (x1 Re(z,z2) = Re(z)vRe(z)2 - lm(z),.lm(z)2 e lm(z,z2) = lm(z)vRe(z)2 y,i) + (x2 A fórmula i2 = -1 é obtida diretamente da multiplicação: i2 = i.i = (0, 1).(0, 1) = (-1, 0) = - 1. A expressão x + yi é chamada representação (forma) algébrica do número complexo z = (x.y), então podemos escrever C = |x + yi | x e R,y e R.i2 = -ij. De agora em diante representaremos o número complexo z = (x.y) por x + iy. O número real x = Re(z) é chamado parte real de número complexo z e, similarmente. y = lm(z) é chamado parte imaginária de z. Números complexos na forma iy.y e R - em outras palavras, números complexos cuja parte real é 0 - são chamados imaginários. Por outro lado, números complexos da forma iy.y e í* são chamados imaginários puros e o número complexo i è chamado de unidade imaginária. As seguintes relações são facilmente verificadas a) z, = z2 se e somente se Re(z), = Re(z)2 e lm(z), = lm(z)2 . b) z e R se e somente se lm(z) = 0 . c) zeC\R se e somente se lm(z)*0. y2i) = (x, + x2) + (y, + y2)i e 2. Multiplicação z,.z2 = (x, + y,i)(x2 + y2i) = (x,x2 - y1y2) - (x,y2 + x2y,)i e Números Complexos de A a Z18 1) ?.(Z1 + z2) = ^Z1 +^z2 • 2) ?.1(X.2z) - (aiX2)z ; Isto é, = (-i)’n. é o produto de um número real com um número complexo. As seguintes propriedades são óbvias: Para um número real X e um número complexo z = x + yi, Xz = X(x + yi) = Xx + Xyi e C . 3) (X, + X2)z = X,z + X2z para todos z,z„z2eC e X,X,X2eR. Na verdade, as relações 1) e 3) são casos especiais da lei distributiva e a relação 2) vem da lei associativa da multiplicação de números complexos. Re(z, - z2) = Re(z), - Re(z)2 lm(z, - z2) = lm(z).| - lm(z)2 3. Subtração z, - z2 =(x, + y,i) - (x2 + y2i) = (x, - x2) + (y1 - y2)i e C. 1.1.5 Potências do número i As fórmulas para potências de um número complexo com expoentes inteiros são preservadas para a forma algébrica z = x + iy . Tomando z = i, obtemos í° =1 ; i1=i ; i2=-1 ; i3=i2.i = -i; i4 = P.i = 1 ; i5 = i4.i = i ; i6 = i5.i = -1 ; i’ = i6.i = -i. Portanto in e {-1,1.—i,i J para todos os inteiros n > 0.Se n é um inteiro negativo, temos i" = (r’)-n = p) Pode-se provar por indução que, para qualquer inteiro n, j4n _ 1 j4n 1 j . j4n+2 _ j4n«-3 _ _j Exemplos. 1) Temos i105 + i23 + i20 - i34 = i4'28’1 + i4 5’3 + i4 5 - i4'8’2 = i - i + 1 + 1 =2. 2) Resolvamos a equação z3=18 + 26i, onde z = x + yi e x,y são inteiros. Podemos escrever (x + yi)3 = (x + yi)2(x + yi) = (x2 - y2 + 2xyi)(x + yi) 1 | Números Complexos na Forma Algébrica 19 = (x3 - 3xy2) + (3x2y - y3)i = 18 + 26i. 1.1.6 Conjugado de um número complexo z2 * 0 (o conjugado de um quocienteé o quociente dos conjugados).7) Para um número complexo z = x + yi o número z = x - yi è chamado de complexo conjugado ou conjugado complexo de z . Usando a definição de igualdade de números complexos, obtemos lx3 - 3xy2 = 18 [3x2y - y3 = 26. Tomando y = tx na igualdade 18(3x2y- y3) = 26(x3 -3xy2), observamos que xíO e y*0 implica 18(3t — t3) = 26(1 — 3t2). A última relação é equivalente a (3t - 1)(3t2 - 12t -13) = 0 . Prova. 1) Se z = x + yi, então a relação z = z é equivalente a x + yi = x-yi. Portanto 2yi = 0 , então y = 0 , logo z = x e R . 2) Temos z = x - yi e z = x - (-y)i = x + yi = z. 3) Observe que z■ z = (x + yi)(x - yi) = x2 + y2 > 0. 4) Note que A única solução racional desta equação é t = ) ; portanto. x = 3,y-1ez = 3ri. 8) As fórmulas Re(z) = - e lm(z) = Z^.Z são válidas para todo z e C. Proposição. 1) A relação z = z é válida se e somente se z e R. 2) Para qualquer número complexo z a relação z = z é válida. 3) Para qualquer número complexo z o número z • z e R é um real não-negativo. 4) z, + z2 = z, + z2 (o conjugado de uma soma é a soma dos conjugados). 5) z,.z2 = z,.z2 (o conjugado de um produto é o produto dos conjugados). 6) Para qualquer número complexo não-nulo z , a relação z = (z)"1 é válida. (l) = í ’ z, + z2 = (x, + x2) + (y, + y2)i = (x, + x2) - (y, + y2)i = (x, - y,i) + (x2 + y2i) = z, -r z2. Números Complexos de A a Z20 1, levando a 7) Observe que como desejado. □ 4') n k=1 5) Podemos escrever = (x,x2 - y,y2) + i(x,y2 + x2y,) = (x,x2 - y,y2) - i(x1y2 + x2y,) = (x, - iy,)(x - iy2) = z,.z2 8) Das relações z +z = (x + yi)+ (x-yi) = 2x ,z-z = (x + yi)-(x-yi) = 2yi segue que Z2 ( n 5') n zk = n Zr Para todo zk e C , k = 1, 2, n. \k=1 ' ' Como consequência de 5’) e 6) temos 5") (zn) = (z)n para qualquer inteiro n e para qualquer z e €. b) O complexo conjugado nos permite obter o quociente de dois números complexos como a seguir: z, ZpZ; = (x, + y^Xx; - y2i) x,x2 + y^; z2 z2.z2 x22 + y2 x2 + y| 1 , e consequentemente z (—) = As propriedades 4) e 5) são facilmente extendidas para ( n An Zzk = Zz,; \k-1 ) k=1 Comentários. a) Para obter a multiplicação inversa de um número complexo zeC* pode-se utilizar a seguinte fórmula: 1 z z z z 1 6) Como z —= 1, temos z (z-1) = (z)-1. -x,y2 + x2y1; xj + yl elm(z> = ¥Re(z) = ^ Z2 x - yi x2 + y2 __ í________ y__ i. x2 + y2 x2 + y2 1 | Números Complexos na Forma Algébrica 21 Exemplos. = 3 - iz = (2) Sejam z,.z2 e C . Prove que o número E = z,.z2 z,.z2 é um número real. 1.1.7 Módulo de um número complexo Proposição. As seguintes propriedades são satisfeitas: , z2 * 0 (o módulo de um quociente é o quociente dos módulos).(8) lzzl • Solução. Temos Ê = z,.z2 + z,.z2 = z,.z2 + z,.z2 = E . então Ee'í 80 - 60i 25 75 - 25I 25 -5 + 35i 25 w=n (9) N - lzzl 2 |z, - z2| < |z,| têm os módulos |Zi| = ^42+32 =5 , |z2| = 7o2 +(-3)2 =3 , |z3| = V? = 2. O número |z| = Jx2 + y2 é chamado de módulo ou de valor absoluto do número complexo z = x + yi. Por exemplo, os números complexosz, = 4 + 3i , z2 = -3i , z3 = 2 (1) Simplifique z = Solução. Podemos escrever (5 + 5i)(3 + 4i) + 20(4 - 3i) 9 - 16i2 16 - 9i2 5 + 5i 20 3 4i + 4 + 3i ' (1) —|z| < Re(z) < |z| e — |z| < lm(z) < |z|. (2) |z| > 0 para todo z e C . Além disso, temos |z| = 0 se e somente se z = 0 . (3) lzl = H = lzl- (4) z.z = |z|2. (5) |z,.z2| = |z,|.|z2| (o módulo de um produto é o produto dos módulos). (6) lzil - lzzl S |z, + z2| < |z,| + |z2|. (7) |z“1| = |z|"1. z*0. Números Complexos de A a Z22 e consequen- l2i z2| + |z2|,z2 portanto kll - |z2| S |z, + z2|. 1, ou (9) Podemos escrever (z-J = |z, - z2+z2| s|z,-z2| + |z2|, então |z,-z2|z|z,|-|z2| Observações consequentemente, |z,+z2| < |z,| + |z2|, como n £ Z |zk| para todo zk e C, k = 1,n . k=1 Por outro lado, |z, - z2| = |z, + (-z2)| < |z,| + |-z2| = |z,| + |z2|. o (1) A desigualdade |z, + z2| < |z,| + |z2| torna-se uma igualdade se e somente se Re(z,z2) = |z,||z2| ■ Isto é equivalente a z, = tz2, onde t é um número real não negativo. (2) As propriedades 5) e 6) são facilmente extendídas para (5’> |nzk|= nM; 161 N |-| = Á. Portanto |z’| = |z|-’. M kl kil kzl’ 1 11l(7) Note que a relação z ■ — = 1 implica |z| ■ — = (8) Temos z lèl= Hl= = 'z,''lz*1l= Prova. Pode-se facilmente checar que (1 )-(4) são válidas. (5) Temos |ziz2|2 = (ziz2)(ziz2) = (zizi)(z2z2) = lzi|2-|z2|2 temente ki z2| ~ lzil lz2|' Pois lzl-° para todo zeC. (6) Observe que z2| ~ (2i “ z2)(zi + z2) = (zi + z2)(zi + z2) = krl + zvz2 + ztz2 + lz2| Como z,.z2 = z,.z2 = z,.z2 segue que z,z2 + z,.z2 = 2Re(z,.z2) < 2k,.z2| = 2|zi| |z2| ■ portanto |z, +z2|2 s(|ziHz2|)2 ■ e desejado. Para obter a desigualdade do lado esquerdo note que kil = lzi + z2 + (~z2)| £ IZ1 + z2l + Fz2| = I21 1 | Números Complexos na Forma Algébrica 23 Como consequência de (5') e (7) temos (5") |zn| = |z|n para qualquer inteiro n e qualquer complexo z Problema 1. z2|2 + lzi - zz|2 = 2(|z,|2 + |z2|2) para todos números |z2|‘ Problema 2. Então A é um número real. Ache o máximo e minimo valor de |z| quando z e Ma . Prove a identidade complexos z,.z2. Solução. Utilizando novamente a propriedade 4 na proposição acima, temos zi Problema 3.___________________________________ Seja a um número real positivo e seja = {zeC*:|z + -| = a} 1 Z2 1 + -1.-L Z, Z2 Solução. Utilizando a propriedade 4 na proposição acima, obtemos |z, + z2|2 + |z, - z2|2 = (z, + z2)(z, + z2) + (z, - z2)(z, - z2) = lzlf + Z1Z2 + Z2Z1 + lz2|2 + lzl|2 - Z1Z2 - Z2Z1 ■ '■ '2 = 2(|z,|2 +|z2|2). z,.z, = |z,|2 = 1 e z,=A Z1 Analogamente, z2= —. Portanto representando por A o número no problema temos z2 Prove que se |z,| = |z2| = 1 e z,z2 * 1, então zi + z? é um número real. 1 + z,z2 à = Zl V2 = 1 + zrz2 ^^ = A. 1 + Z1Z2 Números Complexos oe A a Z24 , obtemos >2 |z|2 +a2 e , então , min|z| = Problema 4. Prove que para qualquer número complexo z, |z 2 2 ' _1_ 72 Tomando z = a + bi, com a,b e K chega-se a z2 = a2 - b2 + 2abi. Obtemos (1 + a2 - b2)2 + 4a2b2 < 1 e (1 + a)2 + b2 Prove que |z| = 1. max|z| = e |l + z21 < 1. Segue que |z| e ou |z2 + l| > 1. z2| < 3^-, para todo número complexo com Problema 5. Jf < |1 + z| + |1 - z + 1 kl2 e consequentemente (a2 + b2)2 + 2(a2 - b2) < 0 e 2(a2 + b2) + 4a + 1 < 0. Somando estas desigualdades implica (a2 + b2)2 + (2a + 1)2 <0, que é um absurdo. 1| * 4= ' 72 M2 a + ja2 + 4 2 Solução. Suponha por contradição que |1 + z| -a + Ja2 + 4 2 = |z + l| =(z + |z|4 + (Z + z)2 - 2|z|2 + 1 |z|2 Portanto |z|4 - |z|2.(a2 + 2) + 1 = -(z + z)2 < 0 e consequentemente a2 + 2 - x/a4 + 4a2 a2 + 2 + + 4a2 2'2' a + 7a2 + 4 . | . -a + 7a2 + 4 —2— ■min lzl »----- -2------- e os valores extremos são obtidos para os números complexos em M satisfazendo z = -z. Solução. Elevando ao quadrado os dois lados da equação a = |z + —| z2 + (z)2 |z|2 251 | Números Complexos na Forma Algébrica t2 = (1Temos z).(1 z) = 2 + 2Re(z), então podemoscomo na figura abaixo. -4 t0 Problema 6. Considere o conjunto H = {zeC:z = Em outras palavras, x é o ponto de mínimo da função R. /(y) = (y - 1)2 y2 = 2y2 - 2y + 1/:R Obtemos f observar Solução. Seja <o = y -1 + yi, com y e K . Basta provar que existe um único número x e x tal que (x - 1)2 + x2 < (y - 1)2 + y2 para todo y e R . 2 2 *2- Figura 1.1. x - 1 + xi, x e R}. Prove que existe um único número z e H tal que |z| < |w| para todo w e H . z + z2| = ^7 - 2t2| . Basta achar os valores extremos da função /: [0,2] —> lí ,/(t) = t + ^|7 - 2t2|. 1 1 portanto x = — e z = Solução. Seja t = |1 + z| e [0,2]. t2 - 2Re(2) = L_^. Então |1 - Números Complexos oe A a Z26 Problema 7. £ £ £ se torna |z| - |y| 2 t(|z| - |x|), e, consequentemente, |y| < (1 - t)|z| + t|x|. = 0.a Logo e então x, = Sejam x.y.z números complexos distintos tais que y = tx + (1 - t)z, t e (0,1). Prove que Esta é a desigualdade triangular para y = (1 - t)z + tx . A segunda desigualdade pode ser provada de maneira análoga, escrevendo a igualdade y = tx + (1 - t)z. Como y - x = (1 - t)(z - x). 1.1.8 Resolvendo equações quadráticas Agora somos capazes de resolver a equação quadrática com coeficientes reais ax2 + bx+c = 0,a*0 no caso que o seu discriminante A = b2 - 4ac é negativo. kl - M |z- x| -b + iV=à 2a kl - |y| k- y| |y| - kl |y - x| -b-i>/3à 2a Observe que as raizes são números complexos conjugados e a forma fatorada ax2 + bx + c = a(x - x,)(x - x2) é válida mesmo no caso A < 0 . Consideremos agora a equação quadrática geral com coeficientes complexos az2 +bz + c = 0,a*0. Completando o quadrado, facilmente nós obtemos a forma equivalente (x+áJ+è b '2 x + -— 2a, Solução. A relação y = tx + (1 - t)z é equivalente a z - y = t(z - x). A desigualdade kl - |y| kl - kl k - y| k - x| 1 | Números Complexos na Forma Algébrica 27 a Isto é equivalente a Essa equação tem as soluções y,.2 az2 + bz + c = a(z - z,)(z - z2) A' = (4 - 4i)2 - (63 -16i) = -63 -16i e onde A = b2-4ac também é chamado de discriminante da equação quadrática. Tomando y = 2az + b , a equação é reduzida a y2 = A = u + vi, onde u e v são números reais. também são preservadas quando os coeficientes da equação são elementos do campo de números complexos C . Solução. Temos A 4a2 Utilizando a mesma manipulação algébrica do caso com coeficientes reais, temos (-a’ + (sgnv)^Hij Problema 1.______________________________________ Resolva, em números complexos, a equação quadrática z2 - 8(1-i)z + 63-16Í = 0 . r = | A'| = >/632 +162 = 65 f-ac.onde A' = ^-|^ z,.2 = ^(-b + y,.2). A = °- 4a2 . .. . . -b c Observe que as relações entre raizes e coeficientes z, + z2 = — , z,z2 = — . assim como a forma fatorada a a ou (2az-b)2 = A , bf Z+2aJ onde r = |a| e sgn v é o sinal do número real v . As raizes da equação original são 28 Números Complexos de A a Z ±(1-8i).yi.2 = ± Problema 2. é um número real, como enunciado. Solução. Segue que z,2 = 4 — 4i ± (1 — 8i). Portanto z, = 5-12Í e z2 = 3 + 4i. Problema 3.___________________________________________________________ Sejam a, b, c números complexos não-nulos distintos com |a| = |b| = |c|. a) Prove que se uma raiz da equação az2 + bz + c = 0 tem módulo igual a 1, então b2 = ac. b) Se cada uma das equações '65-63 2 A equação y2 = —63 — 16t tem a solução . /65 + 63 ) V 2 J az2 + bz + c = 0 e bz2 + cz + a = 0 tem uma raiz de módulo 1, então |a — b| = |b — c| = |c — a|. 2 Re(x,x2) > -|x,x2| = -r2. então ^->0. Sejam p e q números complexos com q * 0 . Prove que se as raizes da equação quadrática x2 + px + q2 = 0 têm o mesmo valor absoluto, então E é um número real. d (1999 Olimpíada Romena de Matemática - Fase Final) 2 -TRe(x1x2) Portanto, — q Solução. Sejam x, e x2 raízes da equação e seja r = |xj = |x2|. Então p2 = (X1 + x2>2 = ÍL + Í2. + 2 = *1*2 + + 2 = 2 + q2 x,x2 x2 x, r2 r2 é um número real. Além disso, 1 | Números Complexos na Forma Algébrica 29 segue que = 1. Achamos que (z,+z2)2 =z1z2,i.e, 1.1.9 Problemas Questão 1. a) z, + z2+ z3; c) z,z2z3; d) z? + z| + z|; 0b) z,z2 + z2z3 + z3z,; A relação (1) é equivalente a (a - b)2 + (b - c)2 + (c - a)2 = 0, i.e., (a-b)2 +(b-c)2 +2(a-b)(b-c) + (c-a)2 =2(a-b)(b-c). Z2)(Z1 + Zj) ” V i-e> (Z1 + zz)í~ + b) Como já vimos, temos b2 = ac e c2 = ab . Multiplicando estas relações chegamos a b2c2 = a2bc , logo a2 = bc . Portanto a2 + b2 + c2 = ab + bc + ca (1) Considere os números complexos z, = (1,2), z2 = (-2,3) e z3 = (1, -1). Calcule os seguintes números complexos: a qual se reduz a b2 = ac, como desejado. e)i + i + í=; z2 z3 Z1 z? + A ZÍ + ZÍ Segue que (a - c)2 = (a - b)(b - c). Tomando valores absolutos, achamos P2 = '/a, onde a = |b - c|, p = |c - a|, y = |a - b|. De modo análogo, obtemos a2 = Py e y2 = aP . Adicionando estas relações chega-se a a2 + p2 + y2 = ap + Py + ya, i.e., (a-p)2 +(P~y)2 +(y-a)2 = 0 . Portanto a = p = y c 1 a) Sejam z,, z2 as raizes da equação com |z,| = 1. De z2 =------- II b zi |z2| = — i—| = 1. Como z, + z2 = — e |a| = |b|, temos |z, + z2|2 = 1. Isto é lal Izi| 3 equivalente a (Z1 = 1. Como z, + z, = - — a 30 Números Complexos de A a Z c) z (2,3) = (4,5); b) (2,3) + z = (-5,-1); Questão 3. b) z3+1 = 0. Questão 4. b) (1.1).z2 = (-1,7). Questão 6. z4.C Questão 7. Seja z0 = (a, b) e C . Ache z e C tal que z2 = z0. Questão 8. Seja z = (1, -1). Calcule zn , onde n é um inteiro positivo. Questão 9. a) (2 - i)(—3 + 2i)(5 - 4i); d) c) Resolva em C as equações: a) z2 +z + 1 = 0; Questão 10. Calcule: Questão 2. Resolva as equações: a) z + (-5,7) = (2,-1); Questão 5._________ Resolva as equações: a) z-(1,2) = (-1.3); ^4 Seja z = (a,b) e C . Calcule z2, z3 e -1 + ix/3 2 n k Seja z = (0,1) e C. Expresse £ z em termos do inteiro positivo n. k=0 Ache os números reais x e y em cada um dos seguintes casos: a) (1 - 2i)x + (1 + 2i)y = 1 + i; b)^4 + ^ = i; 3 + i 3-i c) (4 - 3i)x2 + (3 + 2i)xy = 4y2 - 42 + (3xy - 2y2)i. d,râ-<3'2>- H 5 - 8i 2 - 3i b) (2 - 4i)(5 + 2i) + (3 + 4i)(-6 - i); e) 3 * 7i 2 + 3i z. .x16 1 | Números Complexos na Forma Algébrica 31 :2000 +(-i)94. Questão 13. e R. Seja z e C tal que a) jZOOO + |1999 + j201 + |02 + j47 . b) En = 1 + i + i2 +i3 + ... + Í" para n21; i^. 2 Questão 12.________________________________ Resolva no conjunto dos complexos as equações: a) z2 = i; b) z2 = -i Questão 14.__________________ Prove que: a) E, = (2 + iV5)7 + (2 - iVã)7 Questão 15.________________________________________________________ Prove as seguintes identidades a) |z, + z2|2 + |z2 + z3|2 + |z3 + z,|2 = |z,|2 + |z2|2 + |z3|2 + |z, + z2 + z3|2; Questão 11. Calcule: Questão 18.______________________________________ Encontre todos os complexos z tais que 4z2 + 8|z|2 = 8 . Z1Z2|2 + ki " z2|2 Questão 17.___________________________________ Encontre todos os complexos z tais que |z| = 1 e |z2 + (z)2| = 1. |z + ip2.|z3 +-^-l < 2 . Prove que c) z2 = j b) 11 + e R ; b) E2 = Encontre todos os números complexos z 7 0 tais que z + — e R . z c) i1 - i2 - i3 •. - i2000 ; d) rs+(-i)-7 + (-i)13+i-100 20 + 5Í? 7 + 6i J = (1 + kil2)(1 + kzl2); c) |1 - z,z2|2 - |z, - z2|2 = (1 - |z,|2)(l - |z2|2); d) ki+z2+z3|2 + |-zi + z2+z3|2 +ki-z2+z3|2 +ki + z2-z3|2 = 4(lZr|2 +lZz|2 +IZ3|2)- Questão 16. Números Complexos de A a Z32 I) é um número Questão 27. n Encontre todos os inteiros positivos n tais que + c<o). Questão 26.___________________________________________________________ Sejam z,z2 e C números complexos tais que |z, + z2| = 5/3 e |z,| = |z2| = 1. Calcule IZ1 ~ Z2| • Questão 24.____________________________________________ Encontre todos os números complexos z tais que z = (z - 2)(z real. Questão 23.___________________________________________________________ Encontretodososreaismparaosquaisaequaçãoz3 + (3 + i)z2 - 3z - (m + I) = 0 possui pelo menos uma raiz real. -1 - i-13 2 -1 + ix/3 2 Questão 21.______________________________________________________ 1 J3Sejam a, b, c números reais e w = — + i——. Calcule (a + boi-rcw2)(a + bo>2 Questão 19. Encontre todos os complexos z tais que z3 = z. Questão 20._____________________________________ 11 11Suponha que z eC com Re(z) > 1. Prove que----- --- ,n = 2. 2 < 2 Questão 25. Encontre todos os complexos z tais que |z| = - . Questão 22. Resolva as equações: a) |z|-2z = 3-4i; d) iz2 + (1 + 2i)z + 1 = 0; b) |zj + z - 3 + 4I; e) z4 + 6(1 + i)z2 + 5 + 6i = 0 ; c) z3 = 2 + 11i, onde z = x + yi e x,y e Z ;f) (1 + i)z2 + 2 + 11i = 0. 1 | Números Complexos na Forma Algébrica 33 Questão 28. 2 um inteiro. Encontre o número de soluções da equação z' = iz.Seja n Questão 30. . Prove é real. c) i51 + 2i80 + 3i45 + 4i38 .5 + i 2-i' Questão 31.____________________________________________________________ Sejam z,, z2, z3 números complexos tais que z, +z2 +z3 =0 e Jz,) = |z2| = |z3| = 1. Prove que z2 + z| +z3 = 0 . v(u ~ z) ü.z - 1 Questão 35.______________________________________________________ Fatore (em polinômios de primeiro grau) os seguintes polinômios: a) x4+16; b)x3-27; c)x3+8; d)x4+x2+1. Questão 33.____________________________________________________________ Sejam z,, z2, z3 números complexos distintos tais que |z,| = |z2| = |z3| > 0 . Se z, +z2z3,z2 +z,z3 e z3 +z,z2 são números reais, prove que z,z2z3 =1. Questão 29._______________________________________________________ _ Sejam z,, z2, z3 números complexos com |z,| = |z21 = |z3| = R > 0 . Prove que IZ1 - z2|.|z2 - z3| + |z3 - Zl|.|z, - z2| + |z2 - z3|.|z3 - z,| < 9R2. Questão 34. _____________________________________________________ Sejam x, e \ as raizes da equação x2 - x + 1 =0. Calcule: a) x2000 + x2°°° ; b) xj999 + x2999 ; c) x" + x2 , para n e N . Questão 36.____________________________________________________________ Encontre todas as equações de segundo grau com coeficientes reais que possuem uma das seguintes raizes: , . a) (2 + i)(3-i); b)i±l; Questão 32.____________________________________________________________ Considere os números complexos z,, z2 zncom |z,| = |z2| = ... = |zn| = r > 0. Prove que o número E = (.^)fe+z3)-(Wz.)(Vz:) zlz2-zn Sejam u, v, w, z números complexos tais que |u| < 1, |v| = 1 e w = que |w| S1 se e somente se |z| < 1. Números Complexos de A a Z34 ocorre para quaisquer números complexos zv z2, z3. 1.2 INTERPRETAÇÃO GEOMÉTRICA DAS OPERAÇÕES ALGÉBRICAS M(x, y) y 0 X M(x, y) M"(-x, -y) Figura 1.2. M'(x* -y) M(x, y) ♦ 1.2.1 Interpretação Geométrica de um número complexo Temos definido um número complexo z = (x.y) = x + yi como sendo um par ordenado de números reais (x.y) e RxR , portanto é normal que um número complexo z = x + yi corresponda a um ponto M(x,y) no plano RxR. Por meio de uma introdução formal, vamos considerar P como sendo o conjunto dos pontos de um plano n dado que apresenta um sistema de coordenada xOy. Considere a função bijetora <p: C -> P, <p(z) = M(x,y). Definição. O ponto M(x,y) é chamado de lugar geométrico do número complexo z = x + yi. O número complexo z = X + yi é chamado de coordenada complexa do ponto M(x,y). Nós vamos usar a notação M(z) para indicar que as coordenadas complexas de M é o número complexo z. Questão 37._______________________________________________________ (Desigualdade de Hlawka) Prove que a seguinte desigualdade |z, + z2| + |z2 + z3| + |z3 + z,| < |z,| + |z2| + |z3| + |z, + z2 + z3| 1 | Números Complexos na Forma Algébrica 35 um vetor v = OM, onde M(x,y) é o lugar geométrico do número complexo z. M(x. y) T o XT Figura 1.3. onde i, j são os vetores do eixo-x e eixo-y, respectivamente. 1.2.2 Interpretação Geométrica do módulo - 7(xm-xo) +(yM-yo) ■ O lugar geométrico do conjugado complexo z de um número complexo z = x + yi é a reflexão M'(x,-y), ao longo do eixo-x, do ponto M(x,y)(veja na figura 1.2). O lugar geométrico do inverso -z de um número complexo z = x + yi é a reflexão M"(-x,-y),ao longo da origem, do ponto M(x,y)(veja na figura 1.2). A função bijetora ip localiza o conjunto R sobre o eixo-x, que é chamado de eixoreal. Além disso, os números complexos imaginários estão localizados sobre o eixo-y, que é chamado de eixo imaginário. O plano TI, cujos pontos são identificados como números complexos, é chamado de piano complexo Por outro lado, também podemos identificar um número complexo z = x + yi como Seja Vo o conjunto dos vetores cujos pontos iniciais são a origem O. Podemos, então, definir a função bijetora Vamos considerar um número complexo z = x + yi e seu lugar geométrico M(x,y) no plano complexo. A distância Euclidiana OM é dada pela fórmula <p': C —> Vo , <p'(z) = OM = v = x I + y j , portanto, OM = y/x2 + y2 = |z[ = I v|. Em outras palavras, o valor absoluto |z| de um número complexo z = x + yi é o comprimento do segmento OM ou a magnitude do vetor v = x i + y j . 36 Números Complexos de A a Z Observação 1.2.3 Interpretação geométrica das operações algébricas. a) Adição e subtração. Considere os números complexos z, = x,+y1i e Portanto, a soma z, +z2 corresponde á soma v,+ v2 . M(x, + x2, y, + y2) M2(x2,y2) V, M,(x„y,) 0 Figura 1.4. a) Para um dado número real positivo r, o conjunto dos números complexos de módulo r correspondem no plano complexo a C(O;r), nossa notação para o círculo C com centro em O e raio r. b) Os números complexos z com |z| < r correspondem aos pontos interiores ao circulo C; já os números complexos z com |z| > r correspondem aos pontos exteriores ao circulo C. z2 = x2 + y2i e os vetores correspondentes v, = x, I + y, j e v2 = x2 i + y2 j . Observe que a soma dos números complexos é z, +z2 = (x, + x2) + (y, + y2 )i, e a soma dos vetores é v, + v2 = (x, + x2) i + (y, + y2) j . 1 73Exemplo. Os números zk = ±—±-^-i, k = 1, 2, 3, 4 são representados no plano complexo por quatro pontos sobre um circulo unitário centrado na origem, já que l2t| = l2z| = lza| = Iza| = 1 • 1 | Números Complexos na Forma Algébrica 37 M(9, 6) M,(-2^5) 5-. M.(3, 5) M(4. 3) M;(6, 1)1 ■■ 4 6 3 6 9 M.(6. -2) Figura 1.5. Figura 1.6. e a diferença entre os vetores v, e v2 é Por isso, a diferença z, -z2 corresponde à diferença v,-v2 . M'(2. 4) M,(2. 3) M(5. 2) M,(-3. 1) M,(3, -2) M(-5, -2) M"(—2. -3) M(—2, -4) Figura 1.8.Figura 1.7. 3) Temos que (-3 + i)-(2 + 3i) = (-3 + i) + (-2-3i) =-5-2i; consequentemente o lugar geométrico da diferença desses dois números complexos é o ponto M(-5, -2) dado na figura 1.7. 6 5 Por outro lado, a diferença entre os números complexos z, e z2 é zi -z2 = (X1 ~ x2 ) + (v 1 — Yz)' ■ vi-v2=(x1-x2)i+(y1-y2) j . Exemplos. 1) Temos que (3 + 5i) + (6 + i) = 9 + 6i; o lugar geométrico da soma é dada na figura 1.5. 2) Observe que (6 - 2i)+ (-2 + 5i) = 4 + 3i. Portanto, o lugar geométrico da soma desses dois números complexos é o ponto M (4,3) (veja na figura 1.6). 38 Números Complexos oe A a Z Observação Adistância entre e M2(x2,y2) é igual ao módulo do número complexo apresentam a mesma orientação Logicamente, se X = 0 , então X v = 0 . y y M'(Xx, Xy) X>0 X< 0 M(x, y) M(x, y) 0 x x M'(Xx, Xy) Figura 1.9. 4) Perceba que (3-2i)-(-2-4i) = (3-2i) + (2 + 4i) = 5 +2i, assim obtêm-se o ponto M(-2,-4) como o lugar geométrico da diferença entre esses dois números complexos (veja na figura 1.8). z, -z2 ou a medida do vetor v,-v2 . Certamente, |MiM2| = |z, -z2| = |v,- v2| = 7(x2-xi)2+(yz-yi)2. Perceba que se X>0então os vetores Xv e e |x v| = X |v| . Quando X < 0, o vetor X v muda sua orientação para a oposta à ve|Xv| = - X|v|. a) Múltiplos reais de números complexos. y _( Considere o número complexo z = x + yi e o correspondente vetor v = x i + y j . Se X é um número real, então o múltiplo real Xz = Xx +iXy corresponde ao vetor X v = Xx i + Xy j 1 | Números Complexos na Forma Algébrica 39 M'(3, 6) M(—3, 2) 2 6 -3>(1.2)'2 1 3 x M'(6,-4) Figura 1.10. 0; y 6 a) (-5+ 4i)+ (2-3i) =-3+ i; b) (4-i) + (-6 + 4i) =-2 + 3i; c) (-3-2i)-(-5 + i) = 2-3i; a) |z-2| = 3; b) |z + i| < 1; c) |z-1 + 2i|>3; •> r<H) 1.2.4 Problemas Questão 1.____________________________________________________ Represente os lugares geométricos dos seguintes números complexos: z,=3 + i; z2=-4 + 2i; z3=-5-4í; z4 =5 —i; z5 = 1; z6 = -3i; z7 = 2i; z8 = -4. Exemplos. 1) Temos que 3(1 + 2i) = 3 + 6i; portanto M'(3,6) é o lugar geométrico do produto entre 3 e z = 1 + 2i. 2) Observe que -2(-3 + 2i) = 6 - 4i, e assim temos que o ponto M'(6,-4) é o lugar geométrico do produto entre -2 e z = -3 + 2i. z Questão 2,________________________________________________ Interprete geometricamente as seguintes equações: d) (8-i)-(5 + 3i) = 3-4i; e) 2(-4+2i) = -8 + 4il f) -3(-1 + 2i) = 3-6i. Questão 3.____________________________________________________ Encontre os lugares geométricos do número complexo z em cada caso: d) |z-2|-|z + 2| < 2; e) 0<Re(iz)<1; f) -1 < lm(z) < 1; 40 Números Complexos de A a Z Questão 5. Seja z, = 1 + i e z2 =-1-i. Encontre z3 e C tal que o triângulo z1,z2,z3éequilátero. Questão 4.______________________________________________ Encontre o conjunto de pontos P(x,y) no plano complexo tais que |^x2 + 4 + i^y - 4| = TÍÕ. Questão 6.____________________________________________________________ Encontre o lugar geométrico dos números complexos ztais que o triângulo de vértices em z,z2,z3 é retângulo. Questão 7,_____________________________________ Encontre o lugar geométrico dos complexos z tais que 2 | Números Complexos na Forma Trigonomêtrica 41 M(x, y) P(1,t‘) t* 0 Figura 2.1. Reciprocamente, vamos considerar um ponto M(x,y). O raio polar é r = Jx2 +y2 . Para determinar o seu argumento, vamos estudar os seguintes casos: 2.1.1 Coordenadas polar no plano Vamos considerar um plano de coordenadas e um ponto M(x,y) diferente da origem. O número real r = Jx2 + y2 é chamado de raiopolardo ponto M. O ângulo no sentido anti-horário f e [0.2rr) entre o vetor OM e o lado positivo do eixo-x é chamado de argumento polar do ponto M. O par (r,t*) é chamado de coordenada polar do ponto M. Vamos escrever M(r,t"). Perceba que a função h : Rx R \ {(0,0)} —> (0,oo)x[0,2n)t h((x,y)) = (r.t*) é bijetora. A origem O é o único ponto tal que r = 0; o argumento t* da origem não está definido. Para qualquer ponto M pertencente ao plano, existe um único ponto de interseção P entre o raio (OM e o círculo unitário centrado a origem. O ponto P tem o mesmo argumento t*. Usando as definições das funções seno e cosseno, encontramos que x = rcost' e y = rsent'. Portanto, é fácil encontrar as coordenadas cartesianas de um ponto a partir de suas coordenadas polares. ^Númerqs-Complexos-na-fqrma-trigonqmétrica 2.1 representação polar dos números complexos Números Complexos de A a Z42 X k = (-■t] portanto n/2, para y > 0 37t/2, para y < 0. ti = arctg(-1) + 2n = —^ + 2n = , portanto M, ^275,-^^ b) Se x = 0 e y * 0 , então Exemplos. 1. Vamos encontrar as coordenadas polares dos pontos M,(2,-2), M2 (-1,0), M3 (-275,-2), M4(73,1). M5(3,0), M6(-2,2), M7(0,1), Me(0,-4). Nesse caso, temos que r, = ^22 + (-2)2 = 275 ; Observe que r2 = 1, tj = arctgO + n = rt, portanto M2 (1,n). n _ 7rt = 6 + 7l~~6 , portanto M4 375 ——. portanto 2. Vamos encontrar as coordenadas cartesianas dos pontos M, | 2,— |, M21 3,— eM3(1,1). _ 2n Temos que x^Zcos— M,(-1.73). Perceba que x2 = 3cos-^ o 7n , y2 = 3sen — a) Se x í O, partindo de tgt' = — deduzimos que C = arctg+ kn, onde 0, para x > 0 e y > 0 1, para x < 0 e qualquer y 2, para x > 0 e y < 0. 73 n mTemos que r3 = 4, tj = arctg— + n = —+ rr = — , portanto M3 3^^_ 6 6 Perceba que r = 2, t4 = arctg— = —, portanto M4 2,— . 3 6 \ 6; Temos que r5 = 3, tj = arctgO + 0 = 0, portanto M5 (3,0). Temos que r6 = 275 , tj = arctg (-1) + n = _^ + n = ~ Perceba que r7 = 1, tj = , portanto M7 ■ Observe que r8 = 4, tj = ^ , portanto Ma í 1.-^-) ■ 7x 4 2n \ 3 = = “1. Yi = 2senTf = = . portanto 7n 375 4~ 2 ' 2 2 2 | Números Complexos na Forma Trigonométrica 43 Veja que x3 = cos1, y3 = sen1, portanto M3 (cos1,sen1). 2.1.2 Representação polar de um número complexo z = r[cos(t- 2kít) + isenft - 2krt)] = r(cost + isent), Portanto, dois números complexos z,,z2 -0 representados como z, = r, (cost, + isent,) são iguais se e somente se r, = r2 e t, -12 = 2kn, para algum inteiro k. e determine seus argumentos estendidos. Para um dado complexo z = x + yi podemos escrever a sua representação polar como 5rt T z = r(cost' +isent'), onde r e [0,a>) e f e [0.2n) são as coordenadas polar do lugar geométrico de z . O argumento polar t* do lugar geométrico de z é chamado de argumento de z, denotado por argz . O raio polar r do lugar geométrico de z é igual ao módulo de z. Para z * 0 , o módulo e o argumento de z são unicamente determinados. Considere z = rjcosf + isent‘ j e defina t = t‘+2kn para algum inteiro k. Então, e z2 = r2 (cos t2 + isent2) i.e. qualquer número complexo z pode ser representado como z = rfcost + isent), onde r > 0 e t e R . O conjunto Argz = {t: t" + 2kn,k eZj é chamado de argumento estendido do número complexo z . a) Como mostrado na figura abaixo, o lugar geométrico do ponto P, (-1.-1) situa-se no terceiro quadrante. Assim, r, = J(-1)2 +(-1)2 = 72 e t = arctg— + it = arctgl + it = — + jc = x 4 Exemplo 1. Escreva os seguintes números complexos na representação polar a) z, = -1 - i b) z2 = 2 + 2i c) z3 = -1 + i73 d) z4 = 1 - iv3 Números Complexos oe A a Z44 V2 •1 P,(-1.-D Figura 2.2. Assim e Argz, =z, e Argz2 = PX-1.V3) Vã t3* —2 Figura 2.3. rt 4 2rr 3 2kn | k e z|. c) O ponto P3(-1, Vã) situa-se no segundo quadrante, assim r3 = 2 e tj = arctg(-Vã) + 7t = -^ + rc = b) O ponto P2 (2,2) situa-se no primeiro quadrante, assim podemos escrever r2 = J(2)2 + (2)2 = 272 e tj = arctgl = . . 4 n 7t cos— + isen — 4 4 _ 2?t . 2n'lPortanto, z3 = 21 cos—+ isen—I = Vã^cosy + isen^ + 2kít | k e zl. Consequentemente, z2 = 2^2^ 2 | Números Complexos na Forma Trigonométrica 45 e Argz3 = L* 1 Vã Consequentemente, 2 2 e Argz = Figura 2.4. Exemplo 2. Encontre a representação polar dos seguintes números complexos a) z, = 2i, b) z2 = -1, c) z3 = 2 , d) z4 = -3i e determine seus argumentos estendidos. a) O ponto P-|(0,2) situa-se no lado positivo do eixo-y, assim r, = 2, t' = —, z. = 2| cos- + isen-| ’ ’ 2 ’ k 2 2) d) O ponto P4 (l.-Vã) situa-se no quarto quadrante (figura 2.4), assim r4 = 2 e t4 = arctg(-V3) + 2a = + 2a = . Argz„ / 5a 5a)2| cos — + isen—I. + 2ka | k e Z y + 2ka|kez|. b) O ponto P2 (-1,0) situa-se no lado negativo do eixo-x, assim r2 =1, t2 = n , z2 = cosa + isena e Argz2 = {a + 2ka | k e Z). e = + 2ka|k ez|. Números Complexos de A a Z46 Observação As fórmulas seguintes devem ser memorizadas: t* = arctg e neste caso sena 1 + cosa a 2' Problema 1.___________________________________________________________ Encontre a representação polar do número complexo z = 1 + cos a + isena, a e (0,2n). í71 1—,rr12 J t* = arctg(tgj) = arctg(tg|j = e o ponto P(1 + cosa,sena) situa-se no primeiro e o ponto P(1 + cosa,sena) situa-se no quarto Solução. O módulo é |z| = + cosa)2 + sen2a = ^2(1 + cosa) = ^4cos21- = 2|cos|-| _ a( a . a) z = 2cos- cos- + isen- . 2 2) e Argz„ d) O ponto P„ (0,-3) situa-se no lado negativo do eixo-y, assim r„ = 3, tj = z4 = 2Ícos^ + isen^ = + 2kn | k e Z j . 1 = cosO + isenO ; i = cos— + isen— I-1 = cosrt + isenn ; -i = cos —+ isen—■ 2 2 2 2 e _ af (a ) z = —2cos— cos - + rr21 12 ) + 2n = -- n+2n = — + n2 2 O argumento de z é determinado a seguir: a) Se a e (O.rt), então - e f 0,—| quadrante. Assim, 2 2' c) O ponto P3(2,0) situa- se no lado positivo do eixo-x, assim r3 = 2, tj = 0 , z3 = 2(cos0 + isenO) e Argz3 = {2kn | k e Z}. b) Se a e (n,2n), então — e quadrante. Assim, 2 a 2 3rr 2 2 | Números Complexos na Forma Trigonometrica 47 = ;t, então z = 0.c) Se Problema 2. 1. Solução. = |cos2x + isen2x + cos2x -isen2x| = 2|cos2x| assim cos2x = — ou cos2x = - 1.2.....8. z,z2 = r,r2 (cos(t, + t2) + isen(t, +12)). Prova. De fato, 2 2 ' Assim, existem oito soluções. zk = cosxk +isenxk , k Suponha que z, = r, (cost,+isent,) e z2 = r2(cost2 -s-isent2). Então (D 1 z,z2 = r,r2(cost, + isent,)(cost2 + isent2) = r,r2 ((cost, cos t2 - sent,sent2) + i(sent,cost2 + sent2cost,)) = r,r2 (cos(t, + t2) + isen(t, +12)). □ 5z 7^ ■X2=T.X3=T. 5n X3=T- 2.1.3 Operações com números complexos na forma polar 1. Multiplicação Proposição. Encontre todos os complexos z tal que |z| = 1 e = Seja z = cosx + isenx , x e [0,2n). Então 1 = |= + —| = 11n „ „ 1x, =----- . Se cos2x = — , então 6 2 Se cos 2x = —, então x, = —, 2 1 6 2a 4n -X'=T'X7=T’: |z|2 1 2 Números Complexos de A a Z48 Observação k - A prova por indução é imediata. Essa fórmula pode ser escrita como n (2) k=1 k=1 COS Prova. Aplicando a fórmula (2) para z = z1 = z2 =... = zn obtemos n-vezes n-vezes = rn(cos(nt) + isen(nt)). 2. A potência de um número complexo Proposição. (De Moivre1) Para z = r(cost + isent) e n e N, temos que z" = rn (cos(nt) + isen(nt)). (3) 'Abraham de Moivre (1667-1754), Matemático francês pioneiro na teoria das probabilidades e na trigonometria. z" = r ■ r ■ ■ ■ r cos t + 1 + ... + t + isen t + t+ t n-vezes n Zk = n rk í cos Etk+isen £tk k=1 k=1 Exemplo. Seja z, = 1-i e z2 = 73 +i. Então cos—+ isen—1 z2 = zícos^-r-isen-^ 4 4 ) e 7n 4 7n n') (— + — +isen • 4 6; n')'] « 23rr 23ttA + — = 2V2 cos----- + isen------ ■6 JJ l 12 12 ) a) Temos que |z,z2| = |z,||z2| ■ b) Temos que arg(z,z2) = argz,+argz2 - 2kn , onde 0, para argz, + argz2 < 2<t, 1, para argz,+argz2 a 2k. c) Também podemos escrever Arg(z,z2) = {argz, + argz2 + 2kn : k e Z}. d) A igualdade (1) pode ser estendida para n>2 números complexos. Se zk = rk(cosxk + isenxj, k = 1 n, então z,z2...zn =r,r2...r„(cos(t, +t2 +... + t„) + isen(t,+t2 +... + tn)). z, = 72 z,z2 = 272 23rr12 7Ü 6 n 6 2 | Números Complexos na Forma Trigonometrica 49 Observação Exemplo. 4 4 Problema. Proveque sen5t = 16sen5t- 20sen3t + 5sent; cos5t = 16cos5 t - 20cos3 t + 5cost.' Assim 3. Divisão z2 - r2(cost2 + isent2) * 0 . a) Temos que |zn | = |z|n b) Se r = 1, então (cost +isent)n = cos(nt) + isen(nt). c) Podemos escrever Argzn = {nargz + 2kn : k e Z}. Solução. Usando o teorema de De Moivre, para expandir (cost + isent)5, e o teorema binomial, temos que cos5t + isenõt = cos5 t + 5icos4 tsent +10i2 cos3 tsen2t 4-1 Oi3 cos2 tsen3t+ 5i4 costsen4t + i5sen5t. Proposição. Suponha que z, = r,(cost,+isent,), Então -sen2tj cos5t + isen5t = cos5 t- 10cos3 t(l -cos2 t) i^sent(l-sen2t) sent-10(l- Por meio de uma simples manipulação algébrica encontramos o resultado desejado. = 2soo (cos250rt + isen250n) = 2500 Vamos calcular (1 + i)1000 . A representação polar de 1 + i é 721 cos—+ isen^ |. Aplicando a fórmula de de Moivre, obtemos 4 / r~ \1000 (ti 7t = (72) I cost000—+ isen1000 — — = —fcosft, - t2) + isen(t, -12)] z2 r2 + 5cost(l-cos2t) sen2t)sen3t + sen5t j. (1 + i)1000 Números Complexos de A a Z50 Observação a) Temos que b) Podemos escrever Arg c) Para z, = 1 e z2 = z, Problema. Calcule z = = cos5rt + isen5rt = -1 Solução. Podemos escrever 4ít cos— 3 2io (M10(VM5 (-i-< l — = {argz-i -argz2 + 2kn : k e Z}; lZ2 ) 2^0 2io — = z 1 = -(cos(-t) + isen(-t)); 55n 55ncos + isen-—- ____ 3________ 3_ 40k 40ncos----- + isen------ 3 3 35n 35ncos----- + isen------2 _____ 2_ 40n **vhcos----- + isen------ 3 3 5n 5ncos— + isen — 6_____ 6 . 40?O 71 n ]cos — + isen — 6 6J .10 = —(cos(t1 -t2) + isen(t1 -t2)). r2 Prova. Temos que z1 rjcost,+isent1) r, (cost, +isent1)(cost2 -isent2) z2 r2 (cost2+isent2) r2 (cos212+sen2t2) = — ((cost.| cost2 + sent.|Sent2) + ^sent., cost2 — sent2 cos t-j)) r2 7n o (cos— + isen— -2 h4 4 J l 4ttisen— 3 d) A fórmula de De Moivre também é válida para expoentes inteiros negativos, i.e., temos que zn = rn(cos(nt) + isen(nt)). N r2 lZ2| 2 | Números Complexos na Forma Trigonometrica 51 Figura 2.5. C) P3(2,n); 2.1.5. Problemas Questão 1. Encontre as coordenadas polares dos seguintes pontos, dadas suas coordenadas cartesianas: a)M,(-3,3); b) M2(-4x/3,-4); c) M3(0,-5); d) M4(-2,-1); e) M5(4,-2). Questão 2, Encontre as coordenadas cartesianas dos seguintes pontos, dadas suas coordenadas polares: / 3\ b) P2I 4,2n-arcsen— I; d) P4(3,-n); 2.1.4 Interpretação geométrica da multiplicação Considere os números complexos z, = r, (cost; + isentj), z2 = r2 (cost2 +isent2) e seus lugares geométricos M,(r,,tj), M2(r2,t2). Sejam P,,P2 os pontos de interseção da circunferência C(O;1) com os raios (OM, e (OM2 . Construa o ponto P3eC(O;1) com argumento tj + t2 e escolha o ponto Me(OP3 tal que OMj = OM, OMj . Seja z3 o complexo de coordenadas M,. O ponto M3 (r,r2,tj -12 ) é o lugar geométrico do produto z,z2. Seja A o lugar geométrico do número complexo 1. Como OM3 _ OM2 , j e OMj _ OM2 OM, ~' OM2 ” OA e M2OM3 = AOM, , segue que os triângulos OAM, e OM2M3 são semelhantes. Para construir o lugar geométrico do quociente, perceba que a imagem de — é Mr Z2 0 Pe^.yj. Números Complexos de A a Z52 a) |z| = 2 ; b) |z + i|>2; e) z5 = 3 - 2i; f) z6 = -4i.b) z2 = - 4x/3i).(3 + 3i);c)a) d) 3(1 - i)(-5 5i).b) (1 + i)(-2-2i)-i; Verifique suas respostas usando a forma algébrica Questão 8. Encontre |z|, argz, Argz , argz , arg(-z) para a) z = (1-i)(6 + 6i); b) z = (7-7V3i)(-1-i). 2 Questão 3.__________________________________ Expresse arg(z) e arg(-z) em termos de arg(z). 1 c) z3 = - 2 d) Z4 = 9 -9i\/3 ; Questão 7.____________________________________________________________ Calcule os seguintes produtos usando a representação polar de um número complexo: Questão 6.___________________________________________________ _ Encontre as representações polares dos seguintes números complexos: a) z, = cosa-isena , a e [0,2jt); b) z2 = sena + i(1 + cosa), a e [0,27t); c) z3 = cosa+ sena+ i(sena-cosa), ae[0,2n); d) z4 = 1-cosa+ isena , a e [0,2tt) . i^; 2 Questão 4.___________________________________________________ Encontre os lugares geométricos dos complexos z em cada um dos seguintes casos: 3n e) argz> —; f) argz<|; g) arg(-z)6[|,^; h) |z + 1 + i| < 3 e 0 < argz < 6 i^pJ(-3 + 3i)(2V3 + 2i) c) |z-i|<3; 5nd) n < argz < —; Questão 5.____________________________________________ Encontre as representações polares dos seguintes números complexos: a) = 6 + 6iV3 ; 1 J3 . 4 +' 4 ’ 2 | Números Complexos na Forma Trigonometrica 53 Questão 9. i6 c) z = (l + i>/3)" (l - ix/3)".a) z = b) z = a) (1-cosa + isena)n para 2.2 AS RAÍZES ENÉSIMAS DA UNIDADE 2.2.1 Definindo as raizes enésimas de um número complexo d) Zk=tf| + isen k = 0, 1 n- 1. Considere um inteiro n > 2 e um número complexo z0 * 0. Assim como no campo dos números reais, a equação Zn-z0 =0 é usada para definir as raizes enésimas do complexo z0 . Portanto, nós chamamos de cada solução Z da equação (1) uma raiz enésima do número complexo z0 . Questão 10.____________________________________________________________ Prove que a fórmula de Moivre também é válida para expoentes inteiros negativos. 1 . (273 + 2i)4 Questão 11. Calcule a e [0,2n) e n e N ; b) z" + —, se z +1 = 73 . zn z Teorema. Seja z0 = r(cost"+isenf) um número complexo com r > 0 e C e [0,2n). O complexo z0 possui n raizes enésimas distintas, dadas pela fórmula t'+2kn . f+2kn'| cos-----------+ isen------------ , n n (-1 + ')“ + (73 - if Encontre |z| e argz para: (273 + 2i)“ (1 + j)' (1 - i)6 + (273 - 2if ' 54 Números Complexos de A a Z Prova. Vamos usar a representação polar do complexo Z com argumento estendido. Até agora as raizes da equação (1) são zk = x/r(cos<pk + isenq>k) para keZ. Zq^I.... Z„-i. <Pk arco MkMk_, é igual a argZk+1-argZk = Agora observe que 0 < ip0 < <p, < ... < ipn_, < 2n , portanto os números ipk, k e (0,1,...,n-1} são argumento reduzidos, i.e., <pk = ipk. Até agora tínhamos n raizes distintas de z0 Considere um inteiro k e seja r e {0,1 n — 1} o resto de k módulo n . Assim, k - nq + r para algum q = Z e É fácil ver que Zk = Zr. Assim (Zk :keZ} = {Zo ,Z, Zn_,‘. Em outras palavras, existem exatamente n raizes enésimas distintas de z0 , como foi afirmado. 11 n t*= — + (nq + n . 2n 0— = n Os lugares geométricos das raizes enésimas de um complexo z0 * 0 são os vértices de um n-ágono regular inscrito em uma circunferência com centro na origem e raio tf. Para provar isso, denote Mo, M] Mn_, como sendo os pontos de coordenadas complexas Z0,Z, Zn_r Como OMk = |Zk| = tf para k e {0,1 n — 1}, segue que os pontos Mk situam-se na circunferência c(O;tf). Por outro lado, a medida do f + 2(k + 1)n-(f + 2kn) 2n _ „ + r— + 2qrr = (pr + 2qrr. n Z = p(cosip + isen<p). Pela definição, temos que Z" = z0 ou equivalentemente pn (cos(n<p) + isen(nq>)) = r(cost" +isent'j. Obtemos que p" = r e n<p = t' + 2kn para todo k e Z ; consequentemente p = tf e p =l_ + k— para todo keZ. n n 2rr —, para todo n n k e (0,1 n - 2} e o arco remanescente arcMn_1M0 é — = 2rr - (n -1)—. 2 | Números Complexos na Forma Trigonometrica 55 Mn_iM0 são iguais , o poligono 4 4 I. k = 0.1. 2. ou, de forma explicita, " 12 O triângulo equilátero resultante M0M,M2 é mostrado na figura seguinte: M2 Figura 2.6. Exemplo. Vamos encontrar as raizes cúbicas do complexo z = 1 + i e representa- las no plano complexo. Zk = V2^cos( Usando coordenadas polares, os lugares geométricos dos complexos Z0,Z,,Z2 são A representação polar dez=1 + i é z = V2 cos-^- + isen-2- . As raizes cúbicas do complexo z ' ' Como todos os arcos M0Mi, M1M2, é regular. _ 3n 3nZ. = V2 cos— + isen —1 t 4 4 Zo = V2Ícos—+ isen—|, 0 V 12 12/ e _ 17n 17n1Z, = V2 cos----- + isen------ .2 l 12 12 J ( n . 2n + isen — + k — 112 3is ’ ‘t) 56 Números Complexos de A a Z 2.2.2 As raízes enésimas da unidade Explicitamente, nós temos + isen... en-i = cos 2kn 2kn Consequentemente eo =1, e. = cos— + isen— = — + i— = e e u i Q Q O O e-> = cos— + isen— = Z Q o 2 2 2(n - 1)n n Elas formam um triângulo equilátero inscrito na circunferência C(O;1) , como na figura a seguir. O conjunto |i,e,e2.... cn’’| é denotado por Un . Observe que o conjunto L)n é gerado pelo elemento c , i.e., os elementos de Un são as potências de e. Como dito antes, os lugares geométricos da raizes enésimas da unidade são os vértices de um polígono regular de n lados inscrito em uma circunferência unitária com um dos vértices em 1. As raizes da equação Zn -1 = 0 são chamadas raizes enésimas da unidade. Já que 1 = cosO + isenO , partindo da fórmula das raizes enésimas de um número complexo chegamos nas raizes enésimas da unidade, que são 2kn 2kit , ek = cos----- + isen----- , k e {0,1,2......n - 1}. _ . _ . 2rt . 2n 4n . 4n 2cn = cosO + isenO = 1; e, = cos— + isen— = e; e, = cos— + isen— = c ; n n n n 2(n - 1)ir n = e"-1 Vamos analisar alguns valores específicos de n. i) Para n = 2, a equação Z2 - 1 =0 tem como raizes -1 e 1, que são as raizes quadráticas da unidade. ii) Para n = 3 , as raizes cúbicas da unidades, i.e., as raizes da equação Z3 -1 = 0 2kn 2kn ,sao dadas por Ek = cos-y- + isen-y para k e {0,1,2}. 1 ;V3 2 2 ■ i^ = E2. 2 2rr 3 4ti 3 2ít 3 4n 3 2 | Números Complexos na Forma Trigonomêtrica 57 Figura 2.7. iii) Para n = 4 , as raizes quartas da unidade são para k = 0,1, 2, 3. Figura 2.8. 2kn 2krtEk = cos----- + isen----- * 4 4 De forma explicita, temos e0 = cosO + isen0 = 1; e, = cos^ + isen^ = i;e2 = cosrt+ isenrr = -1 3rt . 3rt e e, = cos— + isen— = -i. 3 2 2 Observe que U4 = {1, i, i2, P} = {1, i, -1, -i}. Os lugares geométricos das raizes quartas são vértices de um quadrado inscrito na circunferência C(O;1). Números Complexos oe A a Z58 Prova. a) Se q = pn, então Zq-1 = (Zn)P-1 = (Zn-l)(z(p'1)n +... + Z" +1) de onde segue a conclusão. e A raiz Ek eUn é chamada primitiva se para todo inteiro positivo m < n temos que sJP * 1. 2pn . 2prt ed = cos-1-— tisen-!— mm 2qn Proposição 1. a) Se n | q , então qualquer raiz de Zn -1 = 0 é raiz de Zq -1 = 0 . b) As raizes comuns de Zm -1 = 0 e Zn -1 = 0 são as raizes de Zd -1 = 0 , onde d = mdc(m,n),i.e.,Umr,Un=Ud. c) As raizes primitivas de Zm-1 = 0 são Ek = cos-—- + isen------, onde Osksm m m emdc(k.m) = 1. c) Primeiramente vamos encontrar o menor inteiro positivo p tal que ep = 1. Da relação eJ = 1 segue que = 2k n para algum inteiro positivo k . Isto é, k m — = k e Z . Seja d = mdcfk.m) e k = k'd , m = m'd , ondemdc(k',m') = 1. Nós obtemos = Já que k e m são primos entre si, obtemos m |p. m d m Portanto, o menor inteiro positivo p com sk = 1 é p = m’. Substituindo na relação m - m d , segue que p = , onde d = mdc (k,m). 2kn m 2kn m 2prt uma raiz de Zm -1 = 0 eb) Considere ----- - m Eq = cos——— + isen——— uma raiz de Zn-1 = 0. Já que |ep| = |Eq| = 1. temos ■ x ■ 2pn 2qn . que ep = Eq se e somente se argcp = argsq, i.e., — = —-—+ 2rn para algum inteiro r. A última relação é equivalente a —- — = r, isto é, pn-qm = rmn . Por m n outro lado nós temos que m = m d e n - nd , ondemdc(m',n’) = 1. Da relação pn - qm = rmn encontramos n’p - m'q = rm nd . Assim, m' | n p, portanto m | p. ... ■ ■ . . . . ... 2pn 2pmn 2p'n Isto e, p = pm para algum inteiro positivo p e arge = -i—= —------=------- m m d d Ed = 1. Reciprocamente, já que d | m e d | n (da propriedade a), qualquer raiz de Z°-1 = 0 é raiz de Zm-1 = 0 e Zn-1 = 0. 2 | Números Complexos na Forma Trigonomêtrica 59 Observação Prova. Prova. = 0. Da proposição 1.b) tem-se que as equações Zm-1 = 0 eZn-1 = 0 possuem como única raiz comum 1 se e somente semdc(m.n) = 1 . Se ek é uma raiz primitiva da unidade, então, da relação ek = 1. P = segue que p = m , i.e.,mdc(k.m) = 1. Se n | k , então k = qn para algum inteiro positivo q, e assim obtemos n-1 , n—1 n—1, .q n—1 Ztj =5>?n = S k =Z1 = n. j=0 j=0 j=0v ' j=0 m mdc(k,m) ’ 1-ck Seja r um inteiro positivo e considere h e {0,1 n -1}. Então (e'*h= (en p" = 1, i.e,, e'*" é uma raiz de Z" -1 = 0. „Er*n"1 são distintas. Suponha por *r + h2 e h, >h2, nós temos que er'>h’ = Er*h2. Então -1 j = 0. Porém, er*h2 * 0 implica sbl"h2 = 1. Levando em consideração Considere e =cos^ + isen-^. Então eeUn é uma raiz primitiva da unidade, consequentemente em = 1 se e somente se n | m . Suponha que n não divide k. Temos que Temos de provar somente que er,er+1 absurdo que para r + h, er>h2 íEhi-h2 que h, - h2 < n e que e é uma raiz primitiva de Z" -1 = 0, chegamos a um absurdo. Proposição 2. Se e e Un é uma raiz primitiva da unidade, então as raizes da equação zn-1 = 0 são Er,er+1 er*n“’, onde r é um inteiro positivo arbitrário. Proposição 3. Sejam e0,e, en_,as raizes enésimas da unidade. Para qualquer inteiro positivo k a seguinte relação ocorre: n, se n | k 0, caso contrárioj-o i=o (-0' ' J=o' ' 1 - E 60 Números Complexos oe A a Z Prova. Solução. = a - bi. (Competição Americana de Matemática 12A, 2002, Problema 24) Problema 2.___________________________________________________________ Dois polígonos regulares estão inscritos em uma mesma circunferência. O primeiro polígono possui 1982 lados e o segundo possui 2973 lados. Se os polígonos possuem vértices em comum, quantos são esses? Seja z = a + bi,z = a- que |z|2002 (a,b) = (0,0), ou |z| = 1. No caso em que |z| = 1, temos z' a z2003 = z • z = |z|2 = 1. Já que a equação z2003 existem no total 1 + 2003 = 2004 pares ordenados que satisfazem as condições requeridas. Proposição 4. Seja p um número primo e seja c = cos— + isen — . Se ág.a, ap.t são inteiros não nulos, a igualdade a0 + a,s +... + ap-je9’1 = 0 ocorre se e somente se a0 = a, =... = ap_,. z2002 = z. Perceba i) = 0 . Assim, |z| = 0 e = z, que é equivalente = 1 possui 2003 soluções distintas, Problema 1.___________________________________________________________ Encontre o número de pares ordenados (a,b) de números reais tais que , . ..2002(a + bi) Solução. O número de vértices comuns é dado pelo número de raizes comuns entre zl9B2-1 = 0 e z2973 -1 = 0. Aplicando a Proposição 1.b), o número desejado é d = mdc(1982,2973) = 991. Se a0 = a, = ... = ap.,, então a relação acima é claramente verdadeira. Reciprocamente, defina os polinômios f,geZ[X] por f = a0+a1X + ... + ap_1Xp“' e g = 1+X + ... +Xp~1. Se os polinômios f,g possuem raizes comuns, então mdc(f,g) divide g. Porém, é bem conhecido (por exemplo pelo critério de irredutibilidade de Eisenstein) que g é irredutível em Z . Consequentemente, mdc(f.g) = g, portanto g | f e assim obtemos que g = kf para algum inteiro k não nulo, i.e., a0 = a, = ... = ap_,. bi,e |z| = Va2 +b2 .Arelaçãodadasetoma = |z2002| = |z| = |z|, da qual segue que |z|(|z|2001 -1 .2002 ' 2 | Números Complexos na Forma Trigonométrica 61 Portanto z = 0. Problema 4. c) n . Tomando o módulo de cada lado, o b) Temos que 1 - ek = 1 - kn cos — n £ z(ek j + zck, + z£ek=O. k=0 Solução. Da condição dada, segue que (z - ek )(z - ek) < 1, o que nos dá |z|2 k = 0,1 n -1. Somando as relações ao se variar o k, obtemos n|z|2£z[5>k .k=0 n 2n-1 ’ 1 2^1-11 _ kitf kit . kit'! = 2sen— sen------- icos— , n 1 - - 1 assim |l-ek| = 2sen—, k = 1,2 n-1 e a identidade trigonométrica desejada segue de a) . Problema 3.____________________________________________________________ Seja e e Un uma raiz primitiva da unidade e seja z um complexo tal que |z-sk| s 1 para todo k = 0,1 n-1. Prove que z = 0. it 3- sen—sen-------sen 2n 2n Seja PqPi -Pn-1 um poligono regular inscrito em uma circunferência de raio 1. Prove que: , , a) Pqpi • pop2 ■ popn-i = ni c) sen^-sen|^----sen'-n 2n 2n 2n it 2n (n —1)it b) sen —sen------- sen-1------- — n n Solução. a) Sem perda de generalidade, vamos supor que os vértices do poligono são os lugares geométricos das raizes enésimas da unidade e Po =1. Considere o polinômio f = z" -1 = (z-1)(z-e)...(z-En-1j , onde e = cos —+ isen— . É fácil ver que n = f '(1) = (1 — e)(l — e2)...(l — e' resultado desejado segue. c) Considere um polígono regular Q0Q1...Q2n_1 inscrito na mesma circunferência cujos vértices são os lugares geométricos das raizes 2n-ésimas da unidade. De acordo com a), Q0Q, ■ Q0Q2 ■ ■ • Qo^zn-i - 2n ■ Agora, levando em conta que Q0Q2...Qn_2 também é um poligono regular. 2kit 2kir „ 2 kit „. kitcos---------isen------- = 2sen------- 2isen — n n n n kit .____kit 'j n n )' 62 Números Complexos de A a Z , k = 1,2..... n .Q0Q2k_, = 2sen O próximo problema estende a identidade trigonométrica do problema 4,b) a) b) para todo inteiro positivo impar n. As seguintes propriedades dos polinômios ciclotõmicos são bem conhecidas: 1) Se q > 1 é um inteiro impar, então (f2q (x) = <bq (-x). 2) Se n > 1, então <t>„ (1) = p, quando n é a potência de um primo p, 1, caso contrário. n 1Skán-1 mdc(k/i)=1 n ' 1<k<n-1 mdc(k.n)=1 deduzimos de a) que Q0Q2 ■ Q0Q4 ■ • Qo^zn-z = n ■ Combinando as duas últimas relações, obtemos Q0Qi Q0Q3 •QoQzn-i = 2. Um cálculo semelhante ao do item b leva à (2k-1)n 2n de onde segue o resultado desejado. Seja n um inteiro positivo e seja e„ =cos—+ isen—. O enésimo polinômio ciclotõmico é definido por On(x) = Çg = X6 + X3 +1 = X4 - X3 + X2 - X +1 <(>,, = X10 +X9 +X8 +... + X + 1 d>., = x4 -x2 +1 T 4 _ „1Z . V11 . „10 .<P«q = X + X + X + ... + X + I 1 _ v6 V5-l-V4 v3_uv2 vx1<p14 = X —X 4- X —X 4- X — X + 1 (|)15 = X8 - x7 + X5 - x4 + X3 - X 4-1 <|>16 = X 4-1 n n n (x - eí). Eksn-1 ' mdc(k.n) = 1 Claramente, o grau de <?n é <p(n) , onde ip éa função "totiente" de Euler. $n é um polinômio mônico com coeficientes inteiros e irredutível em Q . Os dezesseis primeiros polinômios ciclotõmicos são dados abaixo: <{>, = x —1 Ó2 — X +1 <j)3 = X2 + X +1 (f>4 = X2 +1 <i>5 = x4 +x3 + x2 +X + 1 <j>6 = X2 -X +1 <j>7 = X6 + X5 + X4 + X3 + X2 + X + 1 á8=x4+1 Problema 5.______________________________________________________ As seguintes identidades são válidas: sen , sempre que n não é potência de primo; M>(n) kn (-1) 2 COS----= ------ , n 2<p'n' 2 | Números Complexos na Forma Trigonomêtrica 63 kn kn kn kn krc n n n n Temos 2 kn kn kn1 = 4>n (1) = n n De onde segue a conclusão. 2kn 2kn kn kn n n 1 =*n (~1) = kn kn n n _ 2«?(n) 2’’',(n) 2<p(n) j*p(n) Solução. a) Como vimos no problema 4.b) . k o kn kn sen — n <p(n) H)~. n 1sksn-1 ^mdc(k.n)=1 n 1sksn-1 mdc(k,n)=1 n 1sksn-1 mdc(k,n)=1 n 1sksn-1 ^mdc(k.n)=1 n 1sksn-1 mdc(kji)=1 n kn b) Temos que . k . 2krt 2krt „ 2 krr kn1 + e„ = 1 + cos----- + isen----- = 2cos — + 2isen — n n n n — sen—| cos— + isen— i n kn sen — n = 2cos— cos— + isen— , k = 0,1 n-1. n 1 * - 1 kn cos — n ip(n) q>(n) cos n + isen n 2 2 2<p(n) <P(n) (-1) 2 n (1+eÜ) 1sksn-l ' ' mdc(k/i)=1 n (i-^) = (-i)' 1sksn-1 ' ' mdc(k,n)=1 kncos—. resultando na identidade desejada, n knf kn . kn 2cos—| cos— + isen— n n (i-cí)= 1sksn-1 ' ' mdc(k.n)=1 = (-1)’(n) =(-1)“ Como n é ímpar, da relação <t>2n (x) = én (-1) segue que <t>n (—1) = <t>2r> (1) = 1- Assim ip(n) <p (n) cos—--n +isen- n 2 2 1-e„ = 2sen— sen—-icos — n V n n n krr 2 kn f kn kn = —sen— cos— + isen— i n n 1ík<n-1 mdc(k,n)=1 onde usamos o fato que <p(n) é par e também a conhecida relação n k=4n<p(n). 1sksn-1 4 mdc(k,n)=1 kn kn -cos- n Números Complexos de A a Z64 2.2.3 Equações Binomiais Zk = 2 cos + isen para k e {0,1, 2} e 3 3 a) z = 1 + i; e) z = 7 - 24i. b) z = i; b) z = -27 ; Uma equação binomial é uma equação da forma Zn + a = 0 , onde a e C e n > 2 é um inteiro. Resolvê-la na variável Z significa encontrar as raízes enésimas do complexo -a . Isto é, de fato, uma simples equação polinomial de grau n com coeficientes complexos. Partindo do conhecido teorema fundamental da álgebra segue que essa equação tem exatamente n raízes complexas e, obviamente, as raizes são distintas. Exemplo. 1) Vamos encontrar as raízes de Z3 + 8 = 0 . Temos que -8 = 8 (cos n + isenn), portanto as raízes são 2) Vamos resolver a equação Z6 -Z3(1 + i) + i = 0 . Note que a equação é equivalente a (z3 - l)(z3 -i) = 0 . Resolvendo em Z as equações binomiais Z3 -1 = 0 e Z3 -i = 0 , obtemos as soluções rt + 2krt 3 rt + 2kn 3 J. k e {0,1. 2}. 2 Questão 2.__________________________________________________ Encontre as raizes cúbicas dos seguintes números complexos: a)z = -i; c)z = 2 + 2i; e)z = 18 + 26i. d) z = | 2.2.4 Problemas Questão 1. Encontre as raizes quadradas dos seguintes números complexos: c) z = -JL+-^=;72 72 d) z = -2(l + iV3); 2krt 2knek = cos----- + isen----- ,k 3 3 y + 2krt í + 2krc Zk =cos—— ------ + isen—— ------, para k e {0,1,2}. 3 | Números Complexos e Geometria 65 e) z = -7 + 24i. 6 d) z3-27i = 0.c) z3 + 64i = 0 ; c) z = i; d) z = -2i; Questão 4. Encontre as raizes quinta, sexta, sétima, oitava e a décima segunda dos números complexos dados acima. Questão 3._____________________________________________ Encontre as raízes quartas dos seguintes números complexos: a) z = 2-i7Í2; b) z = V3+i; Questão 7.___________ Resolva as equações: a) z7 - 2iz4 - iz3 - 2 = 0 ; b) z6+iz3+i-1 = 0; n-1). Questão 8.____________________________ Resolva a equação z4 = 5 (z -1) (z2 - z +1). Questão 5.___________________________ Seja Un = {e0,e,,e2.... En-1} ■ Prove Que: a) Ej sk e n, para todo j.k e {0,1,...,n -1}; b) e[1 e Un, para todo j e {0,1, c) (2-3i)z6+1 + 5i = 0; d) z1°+(-2 + i)z5-2i = 0. Questão 6. Resolva as equações: a) z3-125 = 0; b) z4+16 = 0; 66 Números Complexos de A a Z 3.1.1 Distância entre dois pontos [0,oo) é definida por d(z1,z2) = |z,-z2| e satisfaz b) (simetria): d(z,,z2) = d(z2,z,) para todo z,,z2eC. |a - z| + |z - b| = |a - b|. Usamos a notação A - M - B . Suponha que os complexos z, e z2 possuam lugares geométricos M, e M2. Assim, a distância entre os pontos M, e M2 é dada por M,M2 = |z, - z2|. partindo da propriedade de módulo. A igualdade ocorre se e somente se existe um número real positivo k tal que z3 -z, = k(z2 -z3) . 3.1.2 Segmentos, raios e retas Sejam A e B dois pontos distintos com coordenadas complexas a e b. Dizemos que o ponto M de coordenada complexa z está entre os pontos A e B se z * a.z r b e a seguinte igualdade ocorre: O conjunto (AB) = {M : A-M-B] é chamado de segmento aberto determinado pelos pontos A e B. O conjunto [AB] = (AB)u{A,B] representa o segmento fechado definido pelos pontos A e B. Para justificar c) observe que ki ~ z21 = |(z, - z3) + (z3 - z2 )| s |z, - z31 + |z3 - z2|. ^-Números-Complexos-e-Geometria---------------------------- 3.1 ALGUMAS NOÇÕES E PROPRIEDADES GEOMÉTRICAS SIMPLES A função distância d:CxC as seguintes propriedades: a) (positividade e não degeneração): d(z,,z2) > 0 para todo z,,z2 e C ; d(z,,z2) = 0 se e somente se z, = z2. c) (desigualdade triangular): d(z,,z2) < d(z1,z3) + d(z3,z2) para todo z,,z2,z3eC. 3 | Números Complexos e Geometria 67 k(b-z) Prova. Basta provar que 1) => 2) => 3) => 4) =■ 1). e B(b) são dois pontos distintos. As seguintes O primeiro caso está provado; para o segundo caso tome t = , consequentemente z = tb -(t -1)a = (1-t)a + tb , como afirmado. 1)=> 2). Já que M e (AB temos que A-M-B ou A-B-M. Existem os números t,l e (0,1) tais que z = (1- t)a + tb ou b = (1 -l)a + Iz. Teorema 2. Suponha que A(a) e B(b) são dois pontos distintos. As sentenças seguintes são equivalentes: 1) Me(AB); 2) existe um número real positivo t tal que z = (1 - t)a + tb , onde z é a coordenada complexa de M; 3) arg(z-a) = arg(b-a); 4) £2<=IT. b-a Teorema 1. Suponha que A(a) sentenças são equivalentes. 1) Me(AB); 2) existe um número real positivo k tal que z-a = k(b-z); 3) existe um número real t e (0,1) tal que z = (1 -t)a + tb , onde z é a coordenada complexa de M. Prova. Vamos primeiramente provar que 1) e 2) são equivalentes. Certamente, temos que Me(AB)se e somente se |a-z| + |z-b| = |a-b|. Isto é, d(a,z) + d(z,b) = d(a,b), ou equivalentemente, existe um real k > 0 tal que z - a = k (b - z). k tPara provar 2) <=> 3), defina t = -—- e (0,1) ou k = -—- > 0 . Assim, z - a = 1 kse e somente se z =------a +-------b. Isto é, z = (1-t)a + tb e assim está provado, k +1 k +1 ' ' O conjunto (AB = {M | A - M - B ou A - B - M) é chamado de raio aberto com ponto final em A contendo B. Números Complexos oe A a Z68 4) 0; 5) = 0. Prova. 2) => 3). Partindo de z = (1 - t)a + tb , t > 0 obtemos arg(z-a) = arg(b-a). Teorema 3. Suponha que A(a) e B(b) são dois pontos distintos. As seguintes sentenças são equivalentes: 1) M(z) situa-se na reta AB. z a Para obter as equivalências 1) <=> 2) <=> 3) observe que para um ponto C tal que C-A-B a reta AB é a união (AB u{A)<j(AC . Dai, então, aplique o Teorema 2. z - a = t(b - a), t > 0 .Assim, z 1 ã 1 b b 1 2) — eR. b-a 3) Existe um número real t tal que z = (1 - t)a + tb . |z - a z - ã| |b - a b - ã| " 3) => 4). Arelação arg^—- = arg(z-a)-arg(b-a) + 2kn paraalgum k e Z implica arg Z ~ a = 2kn,k e Z . Já que arg Z~ a e [0.2rt), segue que k = 0 e argí—- = 0 . b-a b-a b-a Portanto e R' • como desejado. Se t = 1, então z = b e M = B e (AB . Finalmente, se t > 1 então, definindo I = e (0.1), temos que b = Iz + (1 - l)a. Segue que A-B-M e M e (AB . A prova então está terminada. 4) => 1). Seja t = ——- e R". Assim z = a +t(b-a) = (1-t)a + tb, t > 0. Se te (0,1), b-a então M e (AB) (AB . 3 | Números Complexos e Geometria 69 Agora vamos provar as equivalências 2) <=> 4) o 5) . Certamente, temos que -------e R se e somente se -—2. z - a 2) é equivalente à 4). z z Além disso, temos que = 0 se e somente se = 0. = 0,A última relação é equivalente a assim obtemos que 4) é equivalente a 5), e assim está provado. Problema 1. (Olimpíada de Matemática da Romênia - Última fase, 1984) Solução. Figura 3.1. Seja z = az2+(1-a)z3,a<=R e considere os pontos A,,A2,A3,A de coordenadas complexas z1,z2,z3,z , respectivamente. Partindo da hipótese segue que o circuncentro do triângulo A,A2A3 é a origem do plano complexo. Perceba que o ponto A situa-se na reta A2A3 , portanto A,A = |z - z,| é maior que ou igual a altura A ,8 do triângulo A,A2A3 . a b z 1 ã 1 b 1 |z - a z - ã |b - a b - ã l^b-a J | = 0, assimobtemosque= 5----?.,ou,equivalentemente,b - a z - ã 0 ã 1 Sejam z1,z2,z3 números complexos tais que |z,| = |z2| = |z3| = R e z2 * z3 . Prove que min|az2 + (1 -a)z3 - z,| = —|z, - z2| • |z, - z3|. acK Zrx a a b-a b - ã 0 . . , z - aIstoé,--------- b - a z-a b-a z-a b-a z - ã b-a b - ã Números Complexos de A a Z70 Basta, então, provar que como afirmado. Assim, obtemos Exemplo. a-kb z =-------- . 1-k A1A2.A3A1 2R 3.1.3 Dividindo um segmento em uma razão dada. Considere dois pontos distintos A(a) e B(b). Um ponto M(z) sobre a reta AB divide o segmento AB na razão k e R \ {1} se a seguinte relação vetorial ocorre: Mà = k.MB Observe que para k < 0 o ponto M situa-se no segmento de reta que liga os pontos A e B. Se k e (0,1), então M e (AB \ [AB]. Finalmente, se k > 1, então M e (BA \ [AB]. Como consequência, perceba que para k = -1 obtemos que a coordenada do ponto médio do segmento [AB] é dada por zM = . A1B - 2rIz’ - zall Z1 ~ zs| _ A1A2.A,A3 Em termos de números complexos esta relação pode ser escrita como a-z = k(b-z) ou (1-k)z = a-kb. De fato, como R é o circunraio do triângulo A,A2A3 , temos que 2 A1A2.A2A3.A3A1 _______ 4R A2A3 AB = 2area[A1A2A3] A2A3 Sejam A(a), B(b), C(c) pontos não colineares no plano complexo. Então o r i a + b ponto médio M do segmento [AB] tem como coordenada complexa zM = ■. O baricentro G do triângulo ABC divide a mediana [CM] na razão 2:1 internamente, consequentemente sua coordenada complexa é dada por k = -2 , i.e., c + 2zm a + b + czG =------------51 =---------------- ■ G 1 + 2 3 3 | Números Complexos e Geometria 71 Proposição. A medida do ângulo diretamente orientado M,OM2 é igual a arg— . y M. O x Figura 3.2. Figura 3.3.como afirmado. Prova. Vamos considerar os dois seguintes casos. 3.1.4 Medida de um ângulo Lembre-se que um triângulo é orientado se um ordenamento de seus vértices é especifico. Ele é positivamente ou diretamente orientado se os vértices são orientados no sentido anti-horário. Caso contrário, dizemos que o triângulo é negativamente orientado. Considere dois pontos distintos M,(z,) e M2(z2), diferentes da origem do plano complexo. O ângulo M.,OM2 é orientado se os pontos M, e M2 são ordenados no sentido anti-horário (Figura 3.2 abaixo). Z1 a) Se o triângulo M,OM2 é negativamente orientado (Figura 3.2), então M,OM2 = xOM2 - xOM, = argz2 -argz, = arg b) Se o triângulo M,OM2 é positivamente orientado (Figura 3.3), então M/5m2 = 2n - lÇÕM, = 2n - arg^-. argíí- . zi Í2., Z1 ' já que o triângulo MjOM, é negativamente orientado. Portanto M,OM2 = 2n - arg — z2 = 2n - ^2rr - arg|^ = Números Complexos de A a Z72 Observação O resultado também ocorre se os pontos O,M,,M2 são colineares. portanto M,OM2 = argi = í == M2OM, = arg(-i) = M,(1 + i)M,(-1 + i) Figura 3.4. M,(i) M2(1) Figura 3.5. Exemplos. a) Suponha que z, = 1 + i e z2 = -1 + i. Então (veja na figura 3.4) z2 = ~1 + i = (-1 + i)(1-i) =. z, 1 + i 2 b) Suponha que z, = i e z2 = 1. Então £1 = 1 = _j, portanto (veja na figura 3.5) z, i M,OM2 = arg(-i) = e M2OM, = arg(i) = . 3 | Números Complexos e Geometria 73 Teorema. Considere três pontos distintos M,(z,), M2(z2)e M3(z3) .A medida do Prova. portanto Observação M2M,M3 +isen M2M,M3) 3.1.5 Ângulo entre duas retas 71 4 7n 4 ' A translação através do vetor -z, marca os pontos nos pontos O,M2,M3, de coordenadas complexas O,z2-z,,z3-z,. Além disso, temos que M2M,M3 = M2OM3 . Do resultado anterior, obtemos que M2OM3 = arg afirmado. Z3-Z1 z2-z, Usando a representação polar, partindo do resultado acima temos 1^3—?l|^cos^arg z3-z, z2-z. Z3 -z, z2-z, j + isen^arg M3M,M2 = arglj^ Exemplo. Suponha que z, = 4 + 3i, z2 = 4 + 7i, z3 = 8 + 7i. Então z2 -z, 4i i(1 -i) 1 + i z3 -z, “ 4 + 4i 2 T”' e ------ 2 M2M,M3 = arg—: = arg(1 — i) = z3 -z, —------ . como Z2-Z, Considere quatro pontos distintos M, (z,), i e {1,2,3.4}. A medida do ângulo Zq “ Z, Za ~ Zn determinado pelas retas M,M3 e M2M4 é igual a arg —----- - ou arg-------- A prova Z4 “ Z2 Z3 “ Z1 é obtida seguindo a mesma ideia do tópico passado. ----------- Zi — Z< ângulo orientado M2M,M3 é arg—----- 1 Z2 - Zj 74 Números Complexos de A a Z 3.1.6 Rotação de um ponto y M'(zc) M(z) a O x Figura 3.6. Agora temos todos os ingredientes para estabelecer o seguinte resultado: Proposição. Prova. □ Suponha que o ponto C é a rotação de B,em relação à A, de um ângulo a . Se a,b,c são as coordenadas dos pontos A,B,C , respectivamente, então c = a + (b-a)s, onde s = coso + isena A translação através do vetor -a marca os pontos A,B,C nos pontos O.B ,C , de coordenadas complexas O.b - a,c - a , respectivamente (veja na figura 3.7). O ponto C é a imagem de B por meio da rotação, em relação à origem, de um ângulo a , portanto c - a = (b -a)s , ou c = a + (b-a)c , como desejado. Considere um ângulo a e o número complexo dado por e = cosa + isena . Seja z = r(cost + isent) um número complexo e M é seu lugar geométrico. Partindo do produto zc = r(cos(t + «) + isen(t + a)) observamos que |ze| = re arg(ze) = argz + a . Segue que o lugar geométrico M de ze é a rotação de M, em relação à origem, de um ângulo a. 3 | Números Complexos e Geometria 75 V O Figura 3.7. Vamos chamar a fórmula da proposição acima de fórmula de rotação. e sua 2 portanto E situa-se na reta FB, como desejado. Solução. Considere o plano complexo com origem em F e eixos CK e FB, onde FB é o eixo imaginário. Problema 2.____________________________________________________________ Sobre os lados AB, BC, CD, DAdo quadrilátero ABCD, exteriormente ao quadrilátero, construimos quadrados com centros O1,O2,O3,O4 . respectivamente. Prove que O,O3 _L O2O4 e O,O3 = O2O4 . Problema 1.____________________________________________________________ Sejam ABCD e BNMK dois quadrados que não se sobrepõem e seja E o ponto médio de AN Se o ponto F é o pé da perpendicular partindo de B até a reta CK. prove que os pontos E, F, B são colineares. Sejam c,k,bi as coordenadas complexas dos pontos C, K, B com c.k.be x. A rotação com centro em B através de um ângulo 0 = j- marca o ponto C em A, portanto, A tem coordenada complexa a = b(1-i) + ci. Analogamente, o ponto N é obtido pela rotação do ponto K ao redor de B através do ângulo 0 = coordenada complexa é n = b(1 + i)-ki, ' O ponto médio E do segmento AN tem como coordenada complexa a+n . c-k. e =--------= b +------- 1, 2 2 76 Números Complexos de A a Z Segue que °i = o3 = e o4 = Entào = -i e ilK* consequentemente 0^3 = O2O4 , como desejado. 15B.30“ e miABR K F C B D A N Figura 3.9. E Figura 3.8. Problema 3.___________________________________________________________ No exterior do triângulo ABC construimos os triângulos ABR, BCP e CAQ tais que m(PBC) = m(CÃQ) = O3-O1 o4 -o2 a + m 2 ” 2 c + d + (c - d)i 2 b + c + (b - c)i 2 d + a + (d - a)i 2 c + d-a-b + i(c-d-a + b) a + d-b-c + i(d-a-b + c) a + (d-a)i. portanto O,O3 ± O2O4 . Além disso, hc^lLl_i| = 1; |o4-o2| n = c + (b-c)i, p = d + (c-d)i e q = e DAQQ os quadrados contruidos com centros 45", m(BCP) = m(QCA) = Solução. Sejam ABMM', BCNN , CDPP' O,,O2,O3,O4, respectivamente. Denote por letra minúscula a coordenada de cada ponto representado por letra maiscula, i.e., o, é a coordenada de O,, etc. O ponto M é obtido do ponto A por uma rotação com centro em B através de um ângulo 0 = —, assim m = b + (a - b)i. Do mesmo modo. a^b + (a ̂ = 3 | Números Complexos e Geometria 77 Figura 3.10. consequentemente P = Do mesmo modo, q = + Solução. Considere o plano complexo de origem em R e seja M o pé da perpendicular partindo de P até a reta BC. Por meio de uma simples manipulação algébrica, mostra-se que — = ieii", q consequentemente QR ± PR . Além disso, |p| = |iq| = |q|, portanto RP = RQ e assim está provado. b = a[ Prove que m^QRp) = 90’ e RQ = RP . O ponto B é obtido a partir de uma rotação do ponto A com centro em R através de um ângulo 8 = 150", assim c + 73a 1 + 73 E^ = ie£^ = iV3.b-m p-m Denote por letra minúscula a coordenada de um ponto denotado por letra maiúscula. Partindo de MP = MB e = 73 segue que c + 73b b - c . 1+73 + 1 + 73'' a - c . 1 + Tã1' 73 O 2 + 2 J Números Complexos de A a Z78 ORTOGONAL1DADE EPARA COLINEARIDADE, Prova. A colinearidade dos pontos M,,M2,M3 é equivalente a M2M,M3 e {0,n}. Segue que Proposição 2. Prova. Temos que M,M2XM3M4 se e somente se (M,M2,M3M4) e . Isto é Observação Suponha que M2 = M4 . Então M,M2 J. M3M2 se e somente se i, assim M,M2 1 M3M4 . ri 3tt 2'T 77 7t 2’T Nesta seção vamos considerar quatro pontos distintos M, (z,), i e {1,2,3,4}. Proposição 1. Os pontos M-pMj.Mj são colineares se e somente se Z2-Zi e{0,7t} ou, equivalentemente, Exemplos. 1) Considere os pontos M1(2-i),M2(-1 + 2i),M3(-2-i),M4(1 + 2i) . Com uma simples manipulação algébrica mostra-se que zi - Z2 .. Z3“Z4 Z3 -Z1 arg—---- 1 z2 - Z1 3.2 CONDIÇÕES CONCÍCLICIDADE 2) Considere os pontos M,(2 — i),M2 (-1 + 2i),M3 (1 + 2i),M4 (-2-i). Temos então z,-z, que —-----= -i, assim M,M2 1 M3M4 . z3 - Z4 —£16 R’ , como afirmado. z2 — Z1 . , , Z.-Z2equivalente a arg—---- — e z3"z4 As retas M,M2 e M3M4 são ortogonais se e somente se Zl~Z; e ®*. z3“z4 Zl Z; eIR'. z3“z2 . Obtemos que ——— e IR’. z3~z4 3 | Números Complexos e Geometria 79 Problema 1. Solução. = 0., i.e.. Observação e x. Proposição 3. Os pontos distintos M, (z^.Mj (z2),M3 (z3).M4 (z4) são conciclicos ou colineares se e somente se O número k é chamado de razão anarmónica dos quatro pontos M, (z, ),M2 (z2), M3(z3), M4(z4). _ í w, ) i.e., Re —— = V-W2 ) Re(w,.w2) = 0, como desejado. e {3tr,n} ,i.e., k < 0 . Sejam zvz2,z3 as coordenadas dos vértices A, B. C de um triângulo. Se w, = z, -z2 e w2 = z3 - z,, prove que  = 90 se e somente se Re(w,.w2) = 0. 1) Os pontos M,,M2,M3,M4 são colineares se e somente se £3~z2 eR- e z3~.z4 e !C z1~z2 z1-z4 2) Os pontos M,.M2,M3.M4 são conciclicos se e somente se :Í3jL£4.eR. mas ZiZÍleR e ! z1~z2 z1~z4 Z1 ~ z2 zr"z4 arg?3s£2_£3 z£4 Zt - z2 z, - z4 Para qualquer outro arranjo dos quatro pontos a prova é análoga. Perceba que k > 0 em três casos e k < 0 nos outros três. Prova. Suponha que os pontos são colineares. Podemos arranjar quatro pontos em uma circunferência de (4-1)1 = 31 = 6 diferentes modos. Considere o caso quando M,,M2,M3,M4 são dados nessa ordem. Então M1,M2,M3,M4 são conciclicos se e somente se M,M2M3 +M,M4M3 e {3n,n}. Isto é, arg——?2. + arg ——Eí. e |3n,rt}. Obtemos Z’ Zz Z’ Z4 Temos que à = 90 se e somente se ——— e ix . o que é equivalente á ■ W) e iR z3~z1 ~W2 0. A última igualdade é equivalente á Re —? I-Iw2| ) k = Í3^2 . £3^4. e R- Z1 - z2 Z1 - Z4 90 Números Complexos de A a Z 1-i Certamente, k = = 1 e R" e = 4eR". e R'.de um quadrilátero ciclico, então Segue que — eT, i.e., -1 0 , somente se t e a) Se te b) Se te , então 0 s 4t -2n < t < 2t < 3t < 2n ou 0 < argz4 < argz < argz2 < argz3 < 2n . z-í)(|z|’-n). Problema 2.___________________________________________________________ Encontre todos os complexos z tais que os pontos de coordenadas complexas z,z2,z3.z4 - nessa ordem - são os vértices de um quadrilátero ciclico. 1 + z + z2 z 2) Os pontos M, (2-i),M2 (3-2i),M3 (-1+2i),M„ (-2+ 3i) são colineares. —4 + 4i , 1-i , „ -4 + 4i -1 + i 4 - 4i Exemplos. 1) Os lugares geométricos dos números complexos 1,i,-1.-i são concíclicos. De fato, temos que a razão anarmõnica k=——-: -1 + ■ = -1 e R* e claramente -1-i -1 + i 1-i 1 + i — elíe77Fe5' ~ 1 . .z + =. Assim z Solução. Se os pontos de coordenadas complexas z, z2,z3,z4 - nessa ordem - são os vértices z3-z2 ,z3-z4 z - z2 z - z4 Se z e R, então os pontos de coordenadas complexas z,z2,z3,z4 são colineares, assim é desnecessário considerar o caso em que |z| = 1. Seja t = argz e [0,2n). Vamos provar que os pontos de coordenadas complexas z,z2,z3,z4 situam-se nessa ordem sobre a circunferência de raio unitário se e (°.t)-(y.24Defa,°. I 0,-| |, então 0 < t < 2t < 3t < 4t < 2n ou ' 0 < arg < argz2 < argz3 < argz4 < 2n . 1 Obtemos que z + —eR, i.e., z consequentememte z e R ou |z| = 1. c) Se te — ,n , então 0 s 3t -2n < t < 4t -2n < 2t < 2rt ou L ' 0 < argz3 < argz < argz4 < argz2 . 2n 3 3 | Números Complexos e Geometria 81 Da mesma maneira analisamos o caso em que t e [n,2n). 3.3. TRIÂNGULOS SEMELHANTES (1) . Obtemos queIsto é equivalente a □ Observação = 0 .1) A condição (1) e equivalente a 2 Considere seis pontos A,(a1),A2(a2),A3(a3),B,(b1),B2(b2).B3(b3) no plano complexo. Dizemos que os triângulos A,A2A3 e B,B2B3 são semelhantes se o ângulo em Ak é igual ao ângulo em Bk ,ke {1.2,3}. 1 a. |a2-a,| |b2-b1[ |a3-a,| |b3-b,| b? ~ b, b3-b, a2 ~ al a3 -a1 a2 ~ai a3 ~ ar b3 ~ b1 b3 -b, Para concluir, os números complexos que satisfazem a propriedade desejada sâo z = cos t + isent , com t e I 0, — I I —, n I. 1 1 a2 a3 b, b2 b3 bg-b, -i-0 b3 - b, “ -2 - 0 Proposição 1. Os triângulos A,A2A3 e B,B2B3são semelhantes, tendo a mesma orientação, se e somente se a2 ~ai a3 ~ai 1-0 _ 1 2i - 0 ~ 2i * Prova. Temos que AA,A2A3 - AB,B2B3 se e somente se a^ = e/r^ sbTbX • A,A3 B,B3 312 3,2 3n — 8, Ón — b« e arg —----- ^ = arg-^—-1 a3 ” 31 ‘D3-D1 2) Os triângulos A1 (0), A2 (1),A3 (2i) e B1 (0),B2 (—i),B3 (-2) são semelhantes, mas são orientados de modo oposto. Neste caso a condição (1) não é suficiente. Certamente, 82 Números Complexos de A a Z oposta, se e somente se Solução. tg^C Ab) = tg^ABcj = tg^BCAj. NA' BC Problema 1.___________________________________________________________ Sobre os lados AB, BC, CA de um triângulo ABC construimos os triângulos semelhantes ADB, BEC, CFA, tendo mesma orientação. Prove que os triângulos ABC e DEF possuem o mesmo baricentro. a2 ~ ai a3 - ai Solução. Denote por letra minúscula a coordenada de um ponto denotado por letra maiúscula. Os triângulos ADB.BEC,CFA são semelhantes com mesma orientação , assim d-ae-b f-c b - a PB'= segue que -------=--------= z c-b a-c e consequentemente d = a + (b-a)z , e = b + (c-b)z, f = c + (a-c)z . Então d+c+f a+b+c 3 3 ' portanto os triângulos ABC e DEF possuem o mesmo baricentro. Prova. A reflexão ao longo do eixo x dos pontos Bv B2, B3 marca os pontos M, (b,}.M2 (b2),M3 (b3). Os triângulos B,B2B3 e M,M2M3 são semelhantes e possuem orientação oposta, consequentemente, os triângulos A.AjAj e M.,tVI2M3 são semelhantes com mesma orientação. A conclusão segue da propriedade anterior. Perceba que da relação Proposição 2. Os triângulos A,A2A3 e B.,B2B3 são semelhantes, tendo orientação = b2 - b. b3 - b. Problema 2.______ ____ ________________________________________________ Sejam M, N, P os pontos médios dos lados AB, BC, CA do triângulo ABC. Sobre os bissetores perpendiculares aos segmentos [AB],[BC],[CA] os pontos C ,A ,B são escolhidos dentro do triângulo tais que PB prove que abc e AB BC CA A ,B ,C possuem o mesmo baricentro. 3 | Números Complexos e Geometria 83 Problema 3. Solução. ZF. zm = A A' O MZ B’ B -Re - zs e Seja ABO um triângulo equilátero com centro S e seja ABO um outro triângulo equilátero com mesma orientação e SíÁ.SíB’, Sejam MeN os pontos médios dos segmentos A B e AB . Prove que os triângulos SB M e SA N são semelhantes. (Shortlist - 30’ imo) Considere o plano complexo com origem em S tal que o ponto O situa-se no eixo real positivo. Então, as coordenadas dos pontos O,A,B são R.Re.Rc2 , respectivamente. Consequentemente, os triângulos AC B.BA C,CB A são semelhantes e podemos proceder como no problema anterior. N ' ~S zb*Za 2 Re2 + R - zc 2 Seja R o circunraio do triângulo ABO e seja e = cos-^ + isen^ . Figura 3.11. (R^) 2 2 zA-r.zB. Rs + R + z R(s + 1) + z -Rc2-z Zn “ ’ 2 2 ----------- 2 Seja R + z a coordenada do ponto B , assim R-zc é a coordenada do ponto A . Segue que os pontos médios M, N possuem coordenadas r(eM- 2 _R Z E R-zs 2 -2e Números Complexos de A a Z84 Agora temos que se e somente se 3.4 TRIÂNGULOS EQUILÁTEROSg) = o. 11 = o, 1 1 o = Z1 Z2 z2 Z3 Z1 Z2 z2 Z3 Z1 Z2 z2 Z3 z3 Z1 z3 Z1 z3 Z1 R + z -e(R + z) 2 R - ze R - ze -2ê A última relação é equivalente à e - e = 1, i.e., |e|2 = 1. Assim, os triângulos SB'M e SAN são semelhantes, com orientações opostas. Z2 ~ Z1 Z3~Z2 e) —— + z-z1 0 (zi 1 + tz2 1 2n 2ncos— + isen— • 3 3 Z1 + z2 + Z3 . 3 + ez3) = 0, onde s = Prova. O triângulo A,A2A3 é equilátero se e somente se A,A2A3 é semelhante, com mesma orientação, A2A3A,, ou 1 portanto a) <=■ g). Calculando o determinante, obtemos 1 = z,z2 + z2z3 + z3z, - (zz + zj; + zj) ZB‘ ~ ZS = ZA' ~ ZS ZM “ ZS ZN ~ ZS Proposição 1. Suponha que z,,z2,z3são as coordenadas dos vértices do triângulo A,A2A3 . As seguintes sentenças são equivalentes: a) A,A2A3 é um triângulo equilátero; b) |z,-z2| = |z2-z3| = |z3-z,|; c) z2 + z2 + z3 = z,z2 + z2z3 + z3z,; d) Z; ~Zl = z3 ~ z2 • z,-z2 ' 1 1--------+--------- = 0 , onde z = z - z2 z - z3 ,-rE2Z3)(z1-rE2Z2 1 3 | Números Complexos e Geometria 85 c) z2-z. 3 3 Figura 3.12. Por meio de uma simples manipulação algébrica mostra-se que d) o c). Já que a) o b) é óbvio, deixamos a cargo do leitor provar que a) <=> e). Os próximos resultados trazem um pouco de refinamento á questão. *)■= - (z, + ez2 + e2z3 ) (z, + e2z2 + ez. consequentemente g) o c) oi). .73'1 fl -i— Zi + — + i— z,.2 I 1 2 2 2 Prova. A1A2A3 é equilátero e orientado positivamente se e somente se A3 é obtido de A2 por uma rotação com centro em A1 através de um ângulo de — . Isto é, . . 3 z3 =z,+ cos—+ isen— (z2 -z,), assim a) <=> b). Para provar que b) <=> d), observe que b) é equivalente a b) z3 = Z1 + 2+l^“ l(z2 ~zi) = j 5nA rotação com centro em A, e ângulo de — marca A3 em A2. Analogamente mostra-se que a) <=> c). Proposição 2. Sejam z1pz2,z3 as coordenadas dos vértices A,,A2,A3de um triângulo orientado positivamente. As seguintes sentenças são equivalentes. a) A,A2A3 é um triângulo equilátero; b) z3 -z, = e(z2 -z,), onde e = cos^ + isen^; = c(z3 -z.) , onde e = cos —+ isen— ; k J 17 3 3 2 „ 2n 2nd) z, + ez2 + e z3 = 0 , onde e = cos—+ isen — . 2 1 2 1 2 86 Números Complexos de A a Z b) z3 -z, = e(z2 -z,), onde e = cos—+ isen—- ; c) z2-z, e ou, equivalentemente, lzi|-lz2| - |z2| |z3| - |Z3|-|Zl|- Prova. 2) => 1) ■ Tomando o módulo dos termos na relação dada obtemos lzi||z2| = |z2|-|z3| = |z3|-lzi|. Proposição 3. Sejam z,,z2,z3 as coordenadas dos vértices A,,A2,A3de um triângulo orientado negativamente. As seguintes sentenças são equivalentes: Prova. O triângulo equilátero A1A2A3 é negativamente orientado se e somente se A1A2A3 é um triângulo equilátero positivamente orientado. O restante segue da propriedade anterior. ° = zi + -õ + i^T Z2-Z1+ z2 =0' ou b) d). □ 2 2 Proposição 4. Sejam z,.z2,z3 as coordenadas dos vértices do triângulo equilátero A,A2A3 . Considere as sentenças: 1) A,A2A3 é um triângulo equilátero; 2) Z1-Z2 ~ z2-z3 = Z3-Z1’i 3) z? = z2.z3 e z2 = zvz3.. Então 2) => 1), 3) => 1) e 2) «=> 3). 1 2 73 2 1 2 1 2 73 2 1 2 73 2 i^ 2 Assim , 2 ( 1 • Tã'! ( 1 .Tin ( 1 .Tã'! z,+ez2^e Z3 =z,+ -- + i— z2+ ---i— z3 =z,+| -- + I—|z2 Yf 1 .Tã'! C1 .73'1 J[^2 2 J 1 ^2 2 J 2 i^ 2 a) A,A2A3 é um triângulo equilátero; , , 5n 5n ' - ■" 3 3 ’ = e(z3 - z,), onde e = cos—+ isen— ; , . 2n 2nd) z, + s z2 + ez3 = 0 , onde e = cos—+ isen— . 3 | Números Complexos e Geometria 87 Isto implica l21 — Z21 +|Z1+Z2| -2(lzi| +IZ2| )■ (D A soma das relações nos dá z2 + z| + zf = z,z2 + z2z3 + z3z,, portanto, o triângulo A^Aj é equilátero. Observe que também provamos que 2) => 3) e que as sentenças são reversíveis; consequentemente 2) <=■ 3). Como consequência, 3) => 1) e assim está provado. Problema 2.____________________________________________________________ Sejam z^Zj.Zj as coordenadas dos vértices do triângulo A,A2A3 . Se |z,| = |z2| = |z3| e z, + z2 + z3 = 0, prove que o triângulo A,A2 A3 é equilátero. Problema 1,___________________________________________________________ Sejam z,, z2, z3 coordenadas complexas não nulas dos vértices do triângulo A. Ay\. Se z2 = z2z3 e z2 = z,z3 , mostre que o triângulo A,A2A3 é equilátero. Solução. A seguinte identidade ocorre para quaisquer números complexos z, e z2 (veja o Problema 1 na Subseção 1.1.7): Solução. Multiplicando as relações z2 = z2z3 e z2=z,z3 obtemos z2z2=z,z2z3 e, consequentemente, z,z2 = z3 . Portanto z2 +z2 +z| = z,z2 + z2z3 +z3z,, assim o triângulo AqA2A3 é equilátero, de acordo com a Proposição 1 desta seção. De z3 + z2 + z3 = 0 segue que z, + z2 = -z; relações |z,| = |z2| = |z3| e(1) obtemos |z,-z2|2 =3|z,|2. Analogamente, encontramos as relações |z2-zs| =2lzi| e |z3 -z^2 = Slz^2. zi - z2z3, z2 - z3z}, z3 - zdz2. r = lzi| = lzzl = N e z, = r-, z2 = í-, z3 = Z1 z2 Z3 Retornando à relação dada temos ZL = £1 = Í2., ou : z2 z3 Z1 :3, portanto |z, +z2| = |z3|. Usando as 88 Números Complexos oe A a Z Solução. Figura 3.13. b respectivamente. Devemos verificar se c + a E + b e2 = 0 . De fato, Solução alternativa 2. Levando em consideração a hipótese |z,| = |z2| = |z3| segue que nós podemos considerar o plano complexo com origem no circuncentro do triângulo A1A2A3. Então, a coordenada do ortocentro H é zH = z, + z2 + z3 = 0 = zo. Assim, H = O e o triângulo A,A2A3 é equilátero. Combinando isso com a hipótese encontramos z? + z2 + z3 = z,z2 + z2z3+z3z, = 0, de onde a conclusão desejada segue pela Proposição 1. a-=l(a+b + c), b"=l(a + Problema 3.__________________________________ No exterior do triângulo ABC três triângulos equiláteros orientados positivamente AC'B, BA'C, CB'A são construídos. Prove que o baricentro desses triângulos são os vértices de um triângulo equilátero. (Teorema de Napoleão) 3(c" + a e + b s2) = (a + b + c') + (a ' + b + c)c + (a + b' + c)c2 = (b + a'e + ce2) + (c + b'e + ae2)e + (a + c’e + be2)e2 = 0 . Sejam a,b,c as coordenadas dos vértices A,B,C, respectivamente. Usando a propriedade 2. Temos a + c c + bs2 = 0 , b + a e + ce2 = 0 , c + b e + ac2 = 0 , onde a',b',c são as coordenadas dos pontos A’, B', C’. Os bancentros dos triângulos A BC.AB C,ABC têm coordenadas + c), c = j(a + b + c ), Solução alternativa 1. Se tomarmos o conjugado da relação, obtemos 1 1 1 — + — + — = 0. z, z2 z3 Portanto |z,-z2| = |z2-z3| = |z3-z,|, i.e., o triângulo A,A2A3 é equilátero. 3 | Números Complexos e Geometria 89 Solução. 2n/i] A ’2n/q CB 2n/q + (a “ bp)e ■ Portanto O triângulo A0B0C0 é equilátero se e somente se a* + b§ + Cq = aobo + boco + coao. A. Figura 3.14. Problema 4. Sobre os lados do triângulo ABC construimos três n-ãgonos regulares , externamente ao triângulo. Encontre todos os valores de n para os quais os centros dos n-ágonos são os vértices de um triângulo equilátero. (Olimpíada de Matemática da Balcãnia 1990 - Shortlist) Os ângulos AC0B,BA0C,AB0C possuem a mesma medida de — . Seja 2n 2rt n e = cos— + isen— n n e denote por a,b,c,a0,b0,c0 as coordenadas dos pontos A,B,C,A0,B0,C0 , respectivamente. Usando a fórmula de rotação, obtemos a = c0 +(b -c0)e ; b = a0 +(c - a0)c ; c = Sejam A0,B0,C0 os centros dos n-ágonos regulares construídos externamente sobre os lados BC.CA.AB , respectivamente. b — ce . c - aE „ a - be Números Complexos de A a Z90 3.5 GEOMETRIA ANALÍTICA NO PLANO COMPLEXO e y = ou, equivalentemente, z\ + C = 0. Seja a = a , m = A-Bi a =-------- e 2 g + g a - ã A + Bi 2 z + z 2 Proposição 2. Considere as retas dt e d2d2 de equações g,z + o.,z + 0, = 0 e ã2z + g2z + 02 = 0. 3.5.1 Equação da reta Proposição 1. A equação da reta no plano complexo éãz + az + p = 0, onde aeC, peR e z = x + iy e C. C‘ e p = C e R . Então az + az + p = 0, como afirmado. Se então B = 0 e temos uma reta vertical. Se a * g, então definimos o coeficiente angular da reta como A B Isso é equivalente a (l + e + e2 )[(a - b)2 + (b - c)2 + (c - a)2 ] = 0 . Segue que 1 + e + e2 = 0 , i.e., 35 = e obtemos n = 3. Portanto n = 3 é o único n 3 valorcom a propriedade desejada. Prova. A equação da reta no plano cartesiano é Ax + By + C = 0, onde A.B.Ce R e z + z z - z A2 + B2 * 0 . Se definirmos z = x+iy, então x = —-— e y = ———. Portanto, a^-3l3. - Bi —------ + C = 0, 2 2 g + g. g - g'' A-Bi + z---------- 2 respectivamente. As retas d, e d2 são: 1) paralelas se e somente se ; o, «2 Substituindo os resultados de a0,b0,c0 acima, obtemos (b-ce)2 + (c- as)2 + (a-bc)2 = (b-CE)(c- aE) + (c-as)(a -br) + (a -bs)(c - as). 3 | Números Complexos e Geometria 91 Prova. 0. plano cartesiano é = 0. Os resultados acerca do coeficiente angular correspondem às propriedades de inclinação. 3.5.2 Equação da reta determinada por dois pontos Proposição. z, z, 1 A equação da reta determinada pelos pontos P,(z,) e P2(z2) é z2 z2 1 = z z 1 A razão md = é chamada de coeficiente angular complexo da reta d de equação õ.z + az + p = 0. > + a2 j a2 - ã2 ' 2) perpendiculares se e somente se — «1 3) concorrentes se e somente se “1 + ^ = 0; “2 “2 , a« + ã-,. a2Portanto, —-----zH-i = — a, - a, < Prova. Aequação da reta determinada pelos pontos P^Xj.y,) e P2(x2,y2) no x, y, 1 x2 y2 1 x y 1 1) Temos que d, || d2 se e somente se m, = m2. , , ã, ã2 assim a,a< = a.a, e obtemos — = —. “1 “2 2) Temos que d, ± d2 se e somente se m,m2 = -1. Isto é, a2ã, + a2ã2 = 0, ou “1 + = o. a, a2 3) As retas d, e d2 são concorrentes se e somente se m, *m2 . Essa condição nos dá . a, a2 Números Complexos oe A a Z92 1 1 1 se e somente se = 0. = 0, como desejado. □ Observação = 0. coordenadas z, e z2 é m = m = zr z2 z3 1 1 1 1 1 1 4i Z1 + z, z2 + z2 z + z Z1 Z1 Isto é, z2 z2 z z zi - z. 2i z2 ~ z2 2Í Z ~ j 2 2) O coeficiente angular complexo de uma reta determinada pelos pontos de z2 ~ Z1 z2 - Z1' Z1 z2 z2 1 z3 Z3 1 De fato, a equação é 5 1 z2 1 = 0 « z,z2 + z2z + zz, - zz2 - z,z - z2z, = 0 z3 1 Usando números complexos temos zi + zi 2 z2 + z2 2 zi + z 2 1) Os pontos M1(z1),M2(z2),M3(z3) são colineares se e somente se Z1 1 Z1 - Z1 z2 - z2 z - z Z(Z2 ~ Zl) - Z(Z2 - Zl) + Z1Z2 - Z2Z1 = 0 Usando a definição de coeficiente angular complexo, obtemos Z2 ~ Z1 z2 - z,• 3 | Nümeros Complexos e Geometria 93 3.5.3 Área de um triângulo z. (1)4 Já que xk , k = 1,2 ,3 obtemos■ Yk z3 - z3 1 4 Z3Zl - Z2Z3 - Z<Z3 - Z2Z,)z2z3 Z2 Z3 x2 x3 1 1 1 Z, 1 Z2 1 Z3 1 A=1 2 z, 4 Prova. O determinante do teorema acima é z, z, 1 z2 z2 1 = (z2Z2 Z3 z3 1 Corolário. A área de um triângulo diretamente orientado A,A2A3 cujos vértices tém como coordenadas z1,z2,z3 é area[A,A2A3] = jlm(z,z2 + z2z3 + z3z,). = zk + zk 2 1 Zi + A = Zn 4" Zn8i 2 _2 Z3 + z3 como afirmado. □ Teorema. A área do triângulo A,A2A3 cujos vértices possuem coordenadas z1(z2,z3 é igual ao valor absoluto do número . z, 1 i z2 z2 1 . Z3 z3 1 É fácil ver que para um triângulo orientado positivamente A.,A2A3 , com vértices de coordenadas z1,z2,z3 ,a seguinte desigualdade ocorre: . z, z, 1 1 z2 z2 1 > 0. z3 z3 1 Prova. Usando coordenadas cartesianas, a área de um triângulo de vértices (xr y,), (x2, y2). (x3, y2) é igual ao valor absoluto do determinante Yi y2 y3 _ Zk zk 2i z, - 5, 1 z! z, 1 z2 - z2 1 = “ z2 z2 1 Z3 z3 1 1 4i 1 8i 94 Números Complexos de A a Z Problema 1. (3) m, . m3 0 - - X2)(1 - X3) Observação _ al ?-3a2 1 - x3 Da fórmula (3) chega-se ao conhecido teorema de Menelaus: Os pontos M,.M2,M3 são colineares se e somente se X,X2X3 = 1, i.e., M,A2 M2A3 M3A, Substituindo esse valor em (1), segue a fórmula desejada, o Veremos que a fórmula (2) pode ser estendida para um polígono convexo A,A2...An diretamente orientado (veja na Seção 4.3). = 1. M,A3 M2A^ M3A2 Considere o triângulo A,A2A3 e os pontos M,,M2,M3 situados sobre as retas AjAj.AiAj.A^j , respectivamente. Suponha que dividem os segmentos [A2A3].[A3Ai],[A,A2] nas razões , respectivamente. Então area [M,M2M3 ] 1 - X1X2X3 area[A1A2A3] " (1-X1)(1-X2)(1-X3) 1, = — Im 2 = —Im 2 Solução. As coordenadas dos pontos M1,M2,M3 são a2 ~ ^la3 m a3 ~ ^2a1 1-’ 2 1-X2 Aplicando a fórmula (2) encontramos que area[M1M2M3] = —Imfm/r^ + m2m3 + m-jmJ (ã2 - X1ã3)(a3 - A^aJ (ã3 - X2ã1)(ai - X3a2) (ã, - X3ã2)(a2 - X1a3) (1 - *1)(1 - M (1 - x2)(1 - *3) (1 - x3)(1 - X,) = (1 - MO-X2)(’ - x3)(ã,a2 + ã2a3 + ãsa,)] —---------- area[A,A2A3J. = [(z,z2 + z2z3 + z3z,) - (z,z2 + Z2Z3 + Z3Z,)] = 2ilm(z,z2 + z2z3 + Z3Z,) = -2ilm(z,z2 + z2z3 + z3z,) 3 | Números Complexos e Geometria 95 Problema 2. tal que a + b cos a + csena = 0 . Prove que 1 < area[ABC] csena |2 = (bcosa1 = |a|2 = | bcosa < 1 , de Solução. Observe que 72 72— sena + —cosa 2 2 < 1 + 72 2 2 (Olimpíada de Matemática da Romênia - Última fase, 2003) csena)(bcosa + |c|2sen2a + (bc + bcjsenacosa = 1 + Sejam a,b,c as coordenadas dos vértices A,B,C de um triângulo. É conhecido que |a| = |b| = |c| = 1 e que existe a e j t; J = |b|2cos2a + 3.5.4 Equação da reta determinada por um ponto e uma direção Proposição 1. Seja d:az + az + p = 0 uma reta e seja P0(z0) um ponto. A equação da reta paralela à d que passa por Po é z - zo = - —(z - z0). a)bc]| = ±| lm[(1 + sena + cosa)bcj| + sena + cosa)|lm(bc)| = ±(1 + sena + cosa)|lm(±icc)| + sena + cosa)|lm(±i)| = —(1 + sena + cosa) ]]=M1+^sen(a+3) + csena)j| sena - bcsena - becos a + bc)~] | = ±1 + 72 , . ... _ rt h 3i ,, 72 f ni Levando em consideração que — < a + — < — obtemos — < sen a + — . i- 4 4 4 2 l 4 Jonde segue a conclusão. n + v •*/ csena) b2 + c2 - ------- — cos a sena. bc Segue que b2 + c2 = 0 , assim b = ±ic. Aplicando a fórmula (2), obtemos area[ABC] = ±|lm(ãb + bc + ca)| = = ±|ImQ-bcosa - csena)b + bc - c(bcosa = ±| Imjj-cosa = ±|lm[bc - (sena + cos 96 Números Complexos oe A a Z Prova. y-y0 = Prova. Obtemos 2 z — z 2i + afz + z 2 Usando coordenadas cartesianas, a reta paralela à d que passa por Po(xo,yo) tem equação Isso é equivalente à (a - ã)(z -z0-z + z0) = (a + ã)(z + z - z0 - z0), OU a(z - z0) = -ã(z - ^). Zp + Zq 2 z0 - z0 ,a - afz + z 21 a + Usando coordenadas cartesianas, a reta que passa por Po e é perpendicular á d 1 a - ã , Atem equaçao y - y0 =-T--------(x - x0). i a + a' z - zo = f (z - Z0). □ Óbtemos z - z0 = -^-(z - z0). □ Isto é, (a - a)(z - z0 - z + z0) = -(a - a)(z - z0 + z - z0) ou (z - z0)(a + ã + a-ã) = (z- z0)(-a + ã + a +ã). .a + a, i i------ - x - x0 .a - a Usando números complexos, a equação toma a forma Z - z Zo - Zp a 2i 21 a - ã Obtemos a(z - z0) = ã(z - z0) e Proposição 2. Seja d:az + a z + P = 0 uma reta e seja P0(z0) um ponto. A reta que passa por Po e é perpendicular à d tem equação z - z0 = “(z - z0). 3 | Números Complexos e Geometria 97 3.5.5 Pé da perpendicular de um ponto até uma reta Prova. O ponto z é a solução do sistema A primeira equação nos dá z = Substituindo na segunda equação temos az - az0 = -az - 0 - a.Zg. Consequentemente z = como afirmado. □ 3.5.6 Distância de ponto à reta D = ■0 □0 3.6. A CIRCUNFERÊNCIA Prova. m, n, p e R, p < Prova. Usando o resultado anterior, podemos escrever 3.6.7 Equação de uma circunferência Proposição. A equação de uma circunferência no plano complexo é z.z + a.z + ã.z + 0 = 0, onde aeC e 0eR. a.z + a.z + 0 = 0 a(z - z0) = ã(z - Zo)’ + azo + p| ! 2H + « Zp + P| 2Va.ã A equação de uma circunferência no plano cartesiano é x2 + y2 + mx + ny + p = 0 , m2+n2 4 Proposição. Seja Po (z0) um ponto e seja d: az + a- z + 0 = O uma reta. O pé da az0 - azo - 0 2a + azo + p| 2s/ãá -az - 0 ã -az0 - azo - p 2a -zo| = | Proposição. A distância do ponto P0(z0) à reta d : ã.z + a.z + 0 = 0, a e C' é igual à _ I az0 - g zp - p ~ í 2a |az0 “0 perpendicular partindo de Po até d tem coordenadas z = gz0 - azo - 0 2a 98 Números Complexos de A a Z + p = 0. Problema. 2 ■ (1)z0 = Solução. z0(zi “ z2) (2) A equação da reta passando por P(z0) que é perpendicular à reta A,A2 pode ser escrita na forma Sejam z,,z2, z3 as coordenadas dos vértices do triângulo A,A2A3 .A coordenada z0 do circuncentro do triângulo A,A2A3 é 1 lz1 Eliminando z dessas duas equações, segue que z[(z2 - z3) + (z3 - z,)(z2 - z3)] = (z,- z3)(|z2 |2 - |z3|2) + (z2 - z3)(|z3 |2- |z,|2). Aplicando essa fórmula para os pontos médios dos lados [A2A3], [A,A3] e para as retas A2A3, A,A3 , encontramos as equações z(z2 - z3) + z(z2 - z3) = |z2|2 - |z3|2 z(z3 - z,) + z(z3 - z,) = |z3|2 - |z,|2. m + ni 2 m - ni 2 1 z3 |2 z(z, - z2) + z(z, - z2) = z0(z, - z2) + Tomando x = -Z + Z e Y = -—- obtemos 2 2i I |2 Z + Z Z-Z |z| + m—— + n—— + p = 0 ou z.z + z Tome a = m ni eCep = peRna equação acima e a proposição está provada. □ m2 n2 Note que o raio de uma circunferência é igual a r = .— +------p = Jnõ-p . V 4 4 Então a equação é equivalente a (z + a)(z + ã) = r2. Tomando y = -ã = —~-“i a equação da circunferência com centro em y e raio r se torna (z-y)(z-y) = r2. 1 z2 |z2|2 |z3í 1 1 1 Z, Z2 z3 Z, z2 z3 3 | Números Complexos e Geometria 99 assim z e a fórmula desejada segue. Observação (3)zo = 3.6.2 A potência de ponto em relação a uma circunferência Prova. + czzo + cxa - aã + 0 Seja O(-a) o centro da circunferência. A potência de Po em relação á circunferência de raio r é definida por p(z0) = OP02 -r2. Nesse caso obtemos Proposição. Considere um ponto P0(z0) e uma circunferência de equação zz + az + ãz + 0 = O,for tie £ e 0 e R. A potência de Po em relação à circunferência é p(z0) = zozo + <zz0 + õ.z0 + 0. 1 Z1 1 Z3 Z3 1 z3 |2 Podemos escrever essa fórmula na seguinte forma equivalente: z1zl(Z2 ~ Z3> + Z2Z2 (z3 ~ zi) + z3z3(z, - z2) 111 Z1 z2 z3 Z1 z2 Z3 1 1 Z1 z2 Z1 z3 p(zo) = Opo - r2 = |z0 + a|2 - r2 = zozo + ctz0 = Z0Z0 + °<Zo "T k Zq 1 Z2 lzi|2 N2 N' como proposto. Dadas duas circunferências de equações z.z + a,.z + ã,.z + 0, = 0 e z.z + a2 z + “2Z + 02 = 0. onde a,.a2 eC,0,,02 eR , o eixo radical delas é o lugar geométrico dos pontos que têm iguais potências de ponto em relação às circunferências. Se P(z) é um ponto deste lugar geométrico, então z.z + a,.z + ã,z + 0, = z.z + a2.z + ã2.z + 02, ou, equivalentemente, (a, - a2)z + (ã, - ã2)z + 0, - 02 = 0, queé a equação de uma reta. 100 Números Complexos de A a Z 3.6.3 Ângulo entre duas circunferências é válida.Proposição. A seguinte fórmula cose = Figura 3.15. Prova. como afirmado. Note que as circunferências são ortogonais se e somente se p, + p2 = a.yõ.2 + õ.^a2. Pi + P2 ~ (g1°2 + K1C2) 2rl r2 Seja T um ponto comum e sejam O, (-a,), O2(-a2) os centros das circunferências. O ângulo e é igual a O,TO2 ou a n-O,TO2 , logo O ângulo 0 entre duas circunfeências de equações z.z + ot,.z + a,.z + p, = 0 e z z + a2.z + õ2.z + p2 = 0, a,, a2 e C, p,, p2 e R , é o ângulo determinado pelas tangentes âs circunferências em um ponto comum. . __ . Ir,2 + r2 - O,O2| |aiai - Pi +a2a2 - p2 - |u, - cc2 |2 I cose = cos O, TO, = !-------- -----------íl = !--------------------------------------------------1 I I 2r1r2 2r1r2 10,0.1 ‘ <*20-2 ~ Pi ~ P2 ~ giai ~ Ct2g2 * gla2 + al^2 | _ | Pi + Pz ~ (a152 + ^1g2) | 2r,r2 2ryr2 4 | Mais sobre Números Complexos e Geometria 101 é a união de duas circunferências Solução. A relação |z2-a2| = |2az + b| é equivalente a |z2 -a2| = |2az + b|2, i.e., (z2 - a2)(z2 - ã2) = (2az + b)(2az + b). Logo |z|4-2a2|z|2 + a< - a2)2 = (a(z +z)+ b)2 . Problema 1.____________________________________________________________ Sejam a, b números reais tais que |b| < 2a2. Prove que o conjunto de pontos com coordenada z tal que |z2 -a2| = |2az + b| ortogonais. |z - a| = 72a2 +b ou |z + a| = 72a2-b. Portanto, os pontos com coordenadas z que satisfazem |z2-a2| = |2az + b| situam- se em duas circunferências de centros C, e C2, cujas coordenadas são a e -a . e raios R, = 72a2 + b e R2 = 72a2 - b . Além disso, C,C| = 4a2 = (72a2 +b)2 +(72a2-b)2 = R2 + R2 , logo as circunferências são ortogonais, como afirmado. = a2(z + z)2 + 2ab(z + z) + b2 , i.e., (|z|2 Segue que z ■ z - a2 = a(z + z) + b ou z z - a2 = -a(z + z) - b . Isso é equivalente a (z - a)(z - a) = 2a2 + b ou (z + a)(z + a) = 2a2 - b . Finalmente Podemos reescrever a última relação como |z|4 -a2(z2 + z2) + a4 = 4a2|z|2 + 2ab(z + z) + b2, i.e., |z|4 - a2[(z + z)2 - 2|z|2] + a4 = 4a2 |z|2 + 2ab(z + z) +b2 |z-a|2 = 2a2+b ou |z + a|2 = 2a2-b . (1) Como |b| < 2a2 , segue que 2a2+b>0 e 2a2 - b > 0. Logo as relações (1) são equivalentes a 102 Números Complexos de A a Z Sejam a e b dois números complexos. Observação De fato, seja M ------- O conceito de produto escalar de dois vetores é bem conhecido. Em seguida, introduziremos esse conceito para números complexos. Veremos que em várias situações o uso desse produto simplifica a solução do problema consideravelmente. Suponha que A e B sejam pontos com coordenadas a e b. Então o produto real ab é igual á potência de ponto da origem em relação à circunferência de diâmetro AB ^-IVIais-sobre-Números-Complexqs-e-Geqmetria- 4.1 O PRODUTO REAL ENTRE DOIS NÚMEROS COMPLEXOS Proposição 1. Para todos os números complexos a, b, c, z valem as seguintes relações: 1) a • a = |a|2. 2) a b = b ■ a , (o produto real é comutativo). 3) a (b + c) = a b + a c ; ( o produto real é distributivo em relação à adição). 4) («a) b = a(a b) = a (ab) para todo a e R . 5) a b = 0 se e somente se OA ± OB, onde A tem coordenada a e B tem coordenada b. 6) (az) (bz) = |z|2(a b). Dc foto, ç=;= f.<a + b l 2 circunferência, e seja r = — AB = —|a -b| o raio dessa circunferência. A potência de É fácil ver que a • b = ^(ab + ãb) = a ■ b ; logo a ■ b é um número real, o que justifica o nome desse produto. As seguintes propriedades são facilmente verificadas. Definição. Chamamos de produto real entre dois números complexos a e b o número dado por a b = ^-(ãb + ab). o ponto médio de [AB], e portanto o centro dessa 1. 2 2' ponto da origem em relação à circunferência é 4 | Mais sobre Números Complexos e Geometria 103 OM2-r2 = a b. como afirmado. A equivalência 2) <=> 3) segue imediatamente da definição de produto real, u Observação De fato, tomando A' diametralmente oposto a A no circuncirculo do triângulo ABC, o quadrilátero HBA'C é um paralelogramo. Se {M} = HA'mBC , então Prova. Tome os pontos M(b-a) e N(d-c) tais que OABM e OCDN sejam paralelogramos. Então temos AB 1 CD se e somente se OM J. ON. Isto é, m • n = (b - a) ■ (d - c) = 0 . usando a propriedade 5) do produto real. Proposição 3. O circuncentro do triângulo ABC está na origem do plano complexo. Se a, b, c são as coordenadas dos vértices A, B, C , então o ortocentro H tem como coordenada h = a + b + c. Prova. Usando o produto real de números complexos, as equações das alturas AA', BB'. CC do triângulo são (a + b)(ã + b) (a-b)(ã-b) ab-bã 4 4 2 AA':(z-a) (b-c) = 0, BB':(z-b)-(c-a) = 0, CC':(z-c) (a-b) = 0. Mostraremos que o ponto de coordenada h = a + b + c situa-se nas três alturas. De fato, temos (h - a) • (b - c) = 0 se e somente se (b + c) ■ (b - c) = 0 . A última relação é equivalente a bb-cc = 0 . ou |b|2 = |c|2 . Similarmente, Hê BB' e HeCC.e assim está provado. □ Se os números a, b, c, o, h são as coordenadas dos vértices do triângulo ABC, do circuncentro O e do ortocentro H do triângulo, então h = a+b + c-2o. Proposição 2. Suponha que A (a), B (b), C (c) e D (d) sejam quatro pontos distintos. As seguintes sentenças são equivalentes: 1) AB ± CD: 2) (b-a)-(c-d) = 0; 3) -—- eiR* (ou, equivalentemente, Re|-—- | - 0 ). d-c Vd~cJ b-a d-c 104 Números Complexos de A a Z , i.e., zH = a + b + c-2o . Problema 1. Solução. Solução. Usando as propriedades de produto real de números complexos, temos AB2+CD2 = BC2+DA2 se e somente se (b-a)-(b-a) +(d-c)(d-c) = (c-b) (c -b) +(a - d) (a - d). Isto é, a b + cd = bc + da e finalmente (c - a) ■ (d - b) = 0 , ou, equivalentemente, AC 1 BD. como desejado. f + a s = ------ , 2 Problema 2.__________________________________________________________ Sejam M, N, P, Q, R, S os pontos médios dos lados AB, BC, CD, DE,EF, FA de um hexágono. Prove que RN2 = MQ2 + PS2 se e somente se MQ ± PS . (Olimpíada Romena de Matemática - Fase Final, 1994) Figura 4.1. Usando as propriedades do produto real entre números complexos, temos RN2 =MQ2+PS2 Sejam a, b, c, d, e, f as coordenadas dos vértices do hexágono. Os pontos M, N, P, Q. R. S têm coordenadas a + b m =------- 2 respectivamente. Seja ABCD um quadrilátero convexo. Prove que AB2 + CD2 = AD2+BC2 se e somente se AC J. BD . b+c c+d d+e e+f n =--------- ■ p =---------- 1 Q =----------- i r = ,2 2 M 2 2 b + c zH + zA. zH + 2o - a zm- 2 - j g 4 | Mais sobre Números Complexos e Geometria 105 se e somente se Problema 3. Solução. Observação Problema 4. (Olimpíada Balcânica de Matemática. 1985) Solução. Considere o plano complexo com origem no ponto O e seja Rek as coordenadas dos vértices Ak, onde ek são as raízes n-ésimas da unidade, k = 1 n. Seja m a coordenada de M . Seja O a origem do plano complexo e sejam a, b, c. d, e as coordenadas dos pontos A, B, C, D, E, respectivamente. Então (e + f - b - c) (e + f - b - c) = (d + e- a-b)(d + e- a-b) + (f + a- c-d)(f + a- c-d). Isto é, (d + e - a - b) (f + a - c- d) = 0 ; Logo, MQ ± PS . como afirmado. Seja O o circuncentro do triângulo ABC, seja D o ponto médio do segmento AB e seja E o baricentro do triângulo ACD. Prove que as retas CD e OE são perpendiculares se e somente se AB = AC . n pois £ ek = 0. k=1 Seja A,A2...An um polígono regular inscrito em um circulo de centro O e raio R. Prove que, para todo ponto M no plano, a relação a seguir é válida: £MA2 = n(OM2+R2) k=1 Se M estiver no circuncirculo do polígono, então £ MA2 = 2nR2 . k-1 Usando as propriedades de produto real dos números complexos, temos Y MA2 = £ (m -Rrk) (m-REk) = £ (m m - 2Rsk m + R2sk r.k) k=1 k=1 k=1 -2RÍEskl \k=1 ) ■ m + R2 1 |ck|2 =n-OM2 +nR2 = n(OM2 +R2), k=1 = n|m|2 106 Números Complexos de A a Z (1)a b = ac As relações (1) e (2) mostram que CD J. OE se e somente se AB = AC . Solução. usando o produto real de números complexos, temos |b + c — a| = |a| Obtemos 2(R2 + b.c - a.c - a.b) = 0, i.e., a.a + b.c - a.c - a.b = 0 As retas CD e DE são perpendiculares se e somente se (d - c) e = 0 . Isto é, (a + b~2c)(3a + b + 2c) = 0. A última relação é equivalente a 3a.a + a.b + 2a.c + 3a.b + b.b + 2b.c - 6a.c - 2b.c - 4c.c = 0, isto é. Usando o produto real entre números complexos, se R é o circunraio do triângulo ABC . então aa = bb = cc = R2. Sejam A, B, C as imagens geométricas dos números complexos a, b, c, respectivamente. Escolha o circuncentro do triângulo ABC como sendo a origem do plano complexo e denote por R o circunraio do triângulo ABC . Então aã = bb = cc = R2 , 3a + b + 2c 6 a + c + d 3 Problema 5._________________________________________________________ Sejam a, b, c números complexos distintos tais que |a| = |b| = |c| e |b + c - a| = |a|. Prove que b + c = 0 . se e somente se |b + c - a|2 = |a|2 . . a +b d =------ 2 e e = Por outro lado. AB = AC é equivalente a |b — a|2 = |c - a|2 . Istoé, (b- a) (b-a) = (c — a) (c — a) ou b.b - 2a.b + a.a = c.c - 2a.c + a.a, logo ab = a-c (2) Isto é, (b + c-a)-(b + c-a) = |a|2, i.e., |a|2 +|b|2 +|c|2 +2b c-2a c-2a-b = |a|2 . 4 | Mais sobre Números Complexos e Geometria 107 Problema 6. Problema 7. MA2 +MB2 + MC2 + 9MG2 = 4(MA2 + MB2 +MC2) para todo ponto M no plano. A última relação mostra que (a - b) • (d - c) = 0 see somente se AB ± CD, como desejado. Segue que (a -b) (a -c) = 0 , logo AB ± AC, i.e., BAC = 90°. Portanto, [BC] é o diâmetro da circunferência circunscrita ao triângulo ABC , então b + c = 0 . Usando o produto real de números complexos, a relação BC2 + AD2 = 2(EG2 + FH2) se torna Seja G o baricentro do triângulo ABC e sejam A^B^C, os pontos médios dos lados BC, CA, AB , respectivamente. Prove que Solução. Denote por letra minúscula a coordenada do ponto denotado pela respectiva letra maiúscula. Então Sejam E, F, G, H os pontos médios dos lados AB. BC, CD, DA do quadrilátero convexo ABCD . Prove que as retas AB e CD sâo perpendiculares se e somente se BC2 + AD2 = 2(EG2 +FH2). c + d g =-------.2 Solução. Denote por letra minúscula a coordenada de um ponto denotado pela letra maiúscula correspondente. Então a +b e =------- , 2 2 , b + c t =-------- , 2 (c - b).(c - b) + (d - a).(d - a) 1 1 = — (c + d-a-b)(c + d-a-b) + — (a + d-b- c) (a + d-b-c). Isso é equivalente a c.c + b.b + d.d + a.a - 2b.c - 2a.d = a.a + b.b + c.c + d.d - 2a.c - 2b.d ou a.d + b.c = a.c + b.d . 108 Números Complexos oe A a Z Por outro lado, 4.2 O PRODUTO COMPLEXO ENTRE DOIS NÚMEROS COMPLEXOS Sejam a e b dois números complexos. O produto vetorial de dois vetores é um conceito central em álgebra vetorial, com várias aplicações em vários ramos da matemática e da ciência. A seguir, adaptamos esse produto para os números complexos. O leitor verá que essa nova interpretação tem várias vantagens nas resoluções de problemas envolvendo área ou colinearidade. portanto a prova está concluída. Observação -------------------------- para todo ponto M no plano. Uma boa generalização é dada no Teorema 5, Seção 4.11. b + c m-------- 2 b + cm-----— a +c m--------- 2 a + b m —— = 12|m|2 - 8(a + b + c) ■ m + = 12|m|2 -8(a + b + c)-m + b + c a>= — a + b ca usando o produto real de números complexos, temos M A2 + MB2 + MC2 + 9MG2 = (m - a) ■ (m - a) + (m - b) ■ (m - b) + (m - c) ■ (m - c) 2(|a|2 + |b|2 + |c|2) + 2a b + 2b c + 2c ■ a . c + a b’~' A seguinte generalização pode ser provada analogamente. Seja A1A2...An um poligono com baricentro G e seja Ai( o ponto médio do segmento [A,Aj),i < j,i, j e {1,2 n}. Então (n - 2) £ MA2 + n2MG2 = 4£MA? k=1 i<j a + b + c m-------- ----- 4(MA*+MB*+MC?) c + a^ ( 2(|a|2 + |b|2 + |c|2) + 2a b + 2b c + 2c■a , a + b + c9=-v- 4 | Mais sobre Números Complexos e Geometria 109 Observações- a x b = O Figura 4.2. Proposição 1. Suponha que a. b, c são números complexos. Então. 1) a x b = 0 see somente se a = 0 ou b = 0 ou a = Xb , onde X é um número real. 2) a x b = -b x a ; (o produto complexo é anticomutativo). 3) ax(b + c) = axb + axc (o produto complexo é distributivo em relação á adição). 4) a(a x b) = (aa) x b = a x (ab), para todo número real a. 5) Se A(a) e B(b) são pontos distintos entre si e distintos da origem, então a x b = 0 se e somente se O, A, B são colineares. a) Suponha que A(a) e B(b) sejam pontos distintos no plano complexo, diferentes da origem. O produto complexo entre os números a e b tem a seguinte interpretação geométrica útil: 2i areajAOB], se o triângulo OAB está positivamente orientado; -2iarea[AOB], se o triângulo OAB está negativamente orientado. Definição. O número complexo a x b = — (ab - ab) é chamado de produto complexo dos números a e b. ( b1 • sen arg— = i l a) ■l(ãb-ab) = a xb. De fato, se o triângulo OAB é positivamente (diretamente) orientado, então 2i ■ area[OAB] = i ■ OA ■ OB sen(ÁÕB) = i|a| • |b| • 1, 12fb b'!-Ia! tã-ãf — 1 - 1 -Note que a xb + a xb = — (ab-ab) + — (ab - ab) = 0 , então Re(axb) = 0, o que justifica a definição desse produto. As seguintes propriedades são facilmente verificáveis. 110 Números Complexos de A a Z area(ABC) = Além disso, uma manipulação algébrica simples mostra que como afirmado. A outra situação pode ser resolvida analogamente. Proposição 2. Suponha que A(a), B(b), e C(c) sejam pontos distintos. As seguintes propriedades são equivalentes: 1) Os pontos A, B, C são colineares. 2) (b-a)x(c-a) = 0. 3) axb + bxc + cxa = 0. Se o triângulo ABC é diretamente (positivamente) orientado. Para provar a fórmula acima, translade os pontos A, B. C com o vetor -c . As imagens de A, B.C são os pontos A',B’,O de coordenadas a-c,b-c,0, respectivamente. Os triângulos ABC e A’B'O são congruentes com a mesma orientação. Se ABC é positivamente orientado, então No outro caso, note que o triângulo OBA é positivamente orientado, logo 2i• areafOBA] = bxa = -a xb . b) Suponha que A(a),B(b),C(c)são três pontos no plano complexo. O produto complexo nos permite obter a seguinte fórmula para a área do triângulo ABC: ^(axb + bxc + cxa), se o triângulo ABC é positivamente orientado; 1 - —(axb + bxc + cxa), se o triângulo ABC é negativamente orientado. 1 area[ABC] = - lm(ab + bc + ca) 1 areafABC] = area[OA'BT = — ((a-c)x (b-c)) 1 1= — ((a - c) x b - (a - c) x c) = — (c x (a - c) - b x (a - c)) = —(cxa- cxc-bxa + bxc) = —(axb + bxc + cxa), 4 | Mais sobre Números Complexos e Geometria 111 Prova Prova. Problema 1. £'0’ A Figura 4.3. Proposição 3. Sejam A(a),B(b),C(c),D(d) quatro pontos, não colineares três a três. Então AB || CD se e somente se (b - a) * (d - c) = 0 . As retas AB e CD são paralelas se e somente se os pontos O, M. N são colineares. Usando a propriedade 5, isso é equivalente a 0 = m x n = (b - a) x (d - c) .o Escolha os pontos M(m) e N(n) tais que OABM e OCDN são paralelogramos; então m = b-a e n = d-c. Considere os pontos E’ e D' nos prolongamentos (BE e(CD tais que EE’ = 3BE e DD' = 3CD . Prove que: 1) os pontos D'. A,E' são colineares; 2) AD' AE'. Os pontos D e E estão sobre os lados AB e AC do triângulo ABC tais que AD AE 3 AB “ AC “ 4 Solução. Os pontos D. E. D',E' têm coordenadas: . a + 3b a + 3c d -------- , e ---------- , e' = 4e - 3b = a + 3c - 3b e d' = 4d - 3c = a - 3b - 3c . 4 4 respectivamente. Os pontos A, B. C são colineares se e somente se areajABC] = 0 , i.e.. a x b + b - c + cxa = 0.A última equação pode ser escrita na forma (b - a) x (c - a) = 0. u 112 Números Complexos oe A a Z Problema 2. Solução. Sejam a.b, c. d, e as coordenadas dos vértices A, B, C, D, E, respectivamente. C Figura 4.4. Os pontos M, N, P, Q, X, Y têm as seguintes coordenadas R, portanto Da Proposição 3 segue que XY || AB . 4.3 ÁREA DE UM POLÍGONO CONVEXO Seja ABCDE um pentágono convexo e sejam M, N, P, Q, X, Y os pontos médios dos segmentos BC, CD, DE, EA, MP, NO , respectivamente. Prove que XY || AB . Dizemos que o polígono convexo A,A2...An é diretamente (ou positivamente) orientado se, para qual quer ponto M situado no interior do polígono, os triângulos MAkAkíl, k = 1,2 n, são diretamente orientados, onde An<1 = A,. 1) Como (a-d')x(e’-d') = (3c-3b)x(6c-6b) = 18(c-b)x(c-b) = 0 .usando a Proposição 2 segue que os pontos D', A, E' são colineares. 2) Note que -^2- D'E' b + c c + d d + e e + a m =------ , n =--------. p =------- , q =--------2 2 2 H 2 -1, assim A é o ponto médio do segmento D'E'. 2 respectivamente Então —- = —4— = --l6 b-a b-a 4 (y-x)x(b-a) = -2(b-a)x(b-a) = 0. 4 b+c+d+e c+d+e+ax - ----------------- , y =----------------- 4 4 a -b 4 b-a 1134 | Mais sobre Números Complexos e Geometria + akak*1 + ak + 1 al)-a2a3 já que lm(aka1 + a,ak) = 0. já que para quaisquer números complexos z, w a relação lm(zw + zw) = 0 é válida. □ Observação An(an) são colineares se e = 4 £ lm(akak+1) + ±Zlm(za, 2 k=i 9 U-1 i Z(ãkak+1) \k=1 J Teorema. Considere um poligono convexo diretamente orientado com vértices de coordenadas a,, a*, .... an. Então area[A,A2...An] = -Im^aj + ã2a3 + ... + ã^a,, + ãna,). + ãk»lal + a1ak) 1, = — Im2 1 ~ 2lm(a’a2 + a2a3 +•■■ +ak-1ak + akal) 1 ._ “2 - + 2 lm(aKa1 + alak) = ^lrn(ala2 í akak.1 k=1 ) Da fórmula acima segue que os pontos A,(a,), A2(a2), ... somente se(aia2 + a2a3 + •■■an-ian + anai) = 0. Prova alternativa. Escolha um ponto M no interior do poligono. Aplicando a fórmula (2) da Subseção 3.5.3 temos n 1 n area[A1A2...An] = £area[MAkAk+1 J = -£Jm(zak + akak+1 + ã^z) = 4 £ lm(ãkak+1) + ^£lm(zak + ãkt1z) Z k=1 Z k=1 + —Imí z Z ak k=1 1 2 Prova. Usamos o principio da indução finita em n. O caso inicial n = 3 foi provado acima usando o produto complexo. Suponha que a afirmação é válida para n = k e note que area[AlA2...AxAK,1] = area[A,A2...AJ + areajA^.A,] 2lm(ãkak.i = ^lm(ãta2 + ã2a3 + ... + ãk_1ak + ãkak„, + ãkt1a,) . 1 2 Números Complexos de A a Z114 Problema 1. Solução. 2 ,k-2 lm(ne + n) =1) = n-1 Por outro lado, é claro que Obtemos (1) Observação Temos PkPk+i=|^1-Ek| = p(e-1)| = p||l-í| = n-1. Segue que são semelhantes e o resultado em n 8 sen2 — n area[P0P1...Pn_1] area[Q0Q,...Qn_,] area[Q0Ql...Qn_1] = ^Imí E akak+1 2 kk=O pkpk.1 QA+1 |1-e| = 2 sen—,k = 0,1, = 2 sen— ,k = 0,1,..., n-1. n Qk^k*1 - |ak-1 “ ak | - |ek 1| - 1 e n-1 . . n-i , já que Eê* = 0 e Et k-0 k=0 Isto é, os polígonos P0P,.e Q0Q1...Qn_1 (1) segue. :k+2 = 0. Im È(e-(Ê)k*1-E’ |_k=0 e"-1 ._ _ _ . r_, n 2n tt ti area[P0P1...Pn_1] = n arealPgOPJ = —sen— = n sen—cos- * 2|E-1|2 Considere ak = 1 + E + ... + Ek,k = 0,1 n-1, e observe que >1, f (ê)k+1 -1 Ekt2-1 kylm SMr-;---------- 7~ )2 ^k=0 E-1 S-1 1__ 2|c-f „ n a n , n2 sen—cos— = —cotg— n n 4 n Seja P0P,...Pn_, um polígono cujos vértices têm coordenadas 1, e.....En 1 e seja Q0Q,...Qn_, o polígono cujos vértices têm coordenadas 1,1 + e 1 + e + ... + e" , 2n . 2n . .. onde e = cos— + isen—. Ache a razao das areas desses polígonos, n n 1 2n--------- r-n sen— 2|e-1|--------- n n 7i n sen-cos - -------- - ------ — = 4sen2 —. n . 7t n 4COt9n 1154 | Majs sobre Números Complexos e Geometria Problema 2. [A,A2...An] n e ,k=1 sen AkOAk+2 > 0, k = 1,2 n, onde An+2 = A2. Proposição 1. Seja A1A2...An (n £ 5) um polígono convexo e seja Bk o ponto médio do segmento [AkAk+1], k = 1,2 n , onde Ant1 = A.!. Então a seguinte desigualdade é verdadeira: Considere os pontos A', ET.C' sobre os lados BC, CA, AB do triângulo ABC tais que AA', BB’, CC concorrem no ponto Q e sejam n m 1 f n + plm “ aKak 8 I k=1 = ^area 1 = —area 2 Nós usamos as relações Imí Z akakt1 I = Imí £ ak+1ak+2 I = 2 area[A1A2...An] kk=1 = ^lm £akak. 8 Vk’1 area[A1A2...An]. Solução. Sejam ak e bk as coordenadas dos pontos Ak e Bk,k = 1, 2 n. É claro que o polígono B,B2..Bn é convexo e se assumirmos que A,A2...An é positivamente orientado, então B1B2...Bn também tem essa propriedade. Escolha como a origem O do plano complexo um ponto situado no interior do polígono A,A2...An . Temos 1 n _ -XJm(akak+2) area[B,B2...Bn] = -l|m| 4.4. CEVIANAS TRIÂNGULO {A^-Ah] + ^lm £ akak+2 o l k=1 area[B,B2...Bn] > ^-area[A,A2...An]. CONCORRENTES E PONTOS IMPORTANTES DE UM m AC' p ' C'B BA' p CB' A'C-n’ B'A 1 8 1 8 bk=^(ak+akT,).k = 1.2.....n,e |( SbAu \k=1 1 ( n - + ãlm ak*lak+2 o \k=1 1 - 8 [A,A2...An] + - S OAk.OAk+2senAkOAk,2 > - o k=1 z 1 n - —= -|mZ(ak + aktl)(ak.i+akl2) O k=1 1. 8 1 = —area 2 116 Números Complexos de A a Z Prova. provar que AA',BB’, CC se encontram em Q. 0 Analogamente. Q situa-se nas retas BB' e CC’, assim a prova está completa. □ Alguns pontos importantes em um triângulo. Z| = 3) Se Q = H, o ortocentro do triângulo ABC , facilmente obtemos as relações zH ou (nb + pc-(n + p)a)x(nb + pc-(n + p)a) = 0, o que é óbvio, de acordo com a definição de produto complexo. Os pontos A, Q, A' são colineares se e somente se (q-a)x (a-a) - 0 . Isso é equivalente a (tg A)a + (tgB)b + (tg C)c tg A + tg B + tg C nb + pc n + p ma + nb m + n . Temos de aa + pb + yc a + P + y b' = ma + pc c’ = m + p ma + nb + pc m + n + p = ^-[aa + pb + yc], onde s = —(a + p + y). ( ma + nb + pc ( m+n+p Se a.b, c são as coordenadas dos pontos A, 8, C, respectivamente, então a coordenada do ponto Q é -ma + np + PC m + n + p 1) Se Q = G , o baricentro do triângulo ABC , temos m = n = p = 1. Então obtemos . , a+b+cnovamente que a coordenada de G e zG =------ ----- . 2) Suponha que os tamanhos dos lados do triângulo ABC são BC = a, CA = p, AB = y. Se Q = I, o incentro do triângulo ABC, então, usando o resultado conhecido a respeito do ângulo bissetor, segue que m = a, n = p, p = y. Portanto a coordenada de I é As coordenadas de A',B',C' são a' = nb bpc n + p respectivamente. Seja Q o ponto com coordenada q = BA’ tgC CB' tgA AC tgB A'C tgB ' B'A tg C ' C'B “ tg A ' Segue que m = tg A, n = tg B, p = tg C e a coordenada de H é dada por 1174 | Mais sobre Números Complexos e GeometriaObservação A fórmula acima também pode ser estendida para o caso limite quando o triângulo ±oo e zj = Segue que zK = BA' A'C CB’ ’ B'A CB' B'A ’Joseph-Diaz Gergonne(1771-1859), matemático francês, fundou o jornal Annales de Mathématiques Pures et Appliquées em 1810. ’Emile Michel Hyacinthe Lemoine (1840-1912), matemático francês, deu contribuições importantes para a geometria. —. Então A 2 -» 0 ■ Nesse caso zH = a , i.e., o ortocentro do ABC é um triângulo retângulo. De fato, assuma que A (tg B)b+ (tg C)c ;Q tg B + tg C tg A ' tg A triângulo ABC é o vértice A. BA' _ y2 A'C “ p2 1 AC' s-p ' CB 1 ' a2a + p2b + y2c a2 + P2 + y2 6) O ponto de NageP N é a interseção das cevianas AA', BB', CC , onde A', B'. C são os pontos de tangência das circunferências ex-inscritas com os lados g2 AC' _ p2 Y2 ’ C'B “ a2 ' 4) O ponto de Gergonne’ J é a interseção das cevianas AA',BB',CC', onde A', B', C são os pontos de tangência da circunferência inscrita com lados BC, CA, AB , respectivamente. Então 1 1 s-Y CB' S-g__________ 1 ' B’A 1 ' CB 1 ' s-p s-y s-a e a coordenada zd é obtida da mesma proposição, onde r„a + r^b + rTc r„ + rp + rY Aqui ra, rp, ry denotam os raios das três circunferências ex-inscritas ao triângulo. Não é difícil mostrar que as seguintes fórmulas são válidas: K K r K r«=-—ri,=T-r s-a s-p S-y 1 onde K = area[ABC] e s = —(a + p + y). 5) O ponto de Lemoine2 K é a interseção das simedianas do triângulo (a simediana é a reflexão da mediana pela bissetriz). Usando a notação da proposição obtemos 118 Números Complexos oe A a Z BC, CA, AB , respectivamente. Então, zN = Problema. a + P + y + (P-Y) + (y-a) 3Christian Heinrich von Nagel (1803-1882), matemático alemão. Suas contribuições para a geometria de triângulos foram incluídas no livro The Development of Modern Triangle Geometry [13], Solução. Considere o triângulo ABC, diretamente orientado no plano complexo centrado no ponto O. R2sen 2C " 2 R2sen 2A 2 R2sen 2B 2 x(a+b + c) Sejam a, p, y os comprimentos dos lados BC, CA, AB do triângulo ABC e suponha que a < p < y . Se os pontos O, I, H são o circuncentro, o incentro e o ortocentro do triângulo ABC , respectivamente, prove que areajOIH] = —(a - P)(P - y)(y - a), 8r onde r é o raio da circunferência inscrita ao triângulo ABC . = — [(s - a)a + (s - p)b + (s - y)c) s R2 1 = —[(a - P)sen 2C + (p - y)sen 2A + (y - a)sen 2B] = —(a - P)(P - y)(y - a), 4s 8r como desejado. = — [(a - P) • areajOAB] + (p - y) • areajOBC] + (y - a) ■ areajOCAj] 2s = -^-r[(a - P)a x b + (p - y)b x c + (y - a)c x a] 4si Usando o produto complexo e as coordenadas de le de H, temos rizA! n 1/1 l» ifaa + pb + yc areaflOH] = -(I x h) = - -------- ----- r— = ^(a-P) BA' s - y CB'_s-a AC s - p Ã7^” s-p ’ CÃ- s-y ’ CB ” s-a ’ e a proposição mencionada anteriormente nos dá a coordenada zN do ponto N de Nagel, (s - g)a (s - p)b + (s - y)c (s-a) + (s-p) + (s-y) = (1’t)a + (1’s)b + (1"s)C- 4 | Mais sobre Números Complexos e Geometria 119 4.5. CIRCUNFERÊNCIA DOS NOVE PONTOS DE EULER Figura 4.5. É óbvio que para os pontos A,. B,, C,. A",B", C" temos as seguintes coordenadas: Contudo, não é tão fácil encontrar as coordenadas de A', B', C. onde R é o raio da circunferência circunscrita ao triângulo ABC . Representemos por A,. B,. C, os pontos médios dos lados BC, CA, AB ,por A', B', C os pés das alturas e por A", B", C" os pontos médios dos segmentos AH, BH, CH, respectivamente. Dado um triângulo ABC , escolha seu circuncentro O para ser a origem do plano complexo e sejam a.b, c dos vértices A, B. C. Vimos na Seção 2.22, Proposição 3, que a coordenada do ortocentro H é zH = a + b + c. Proposição 1. Considere o ponto X(x) no plano do triângulo ABC. Seja P a projeção de X na reta BC . Então a coordenada de P é dada por 1 f bc > p = — x-----5-X + b + C2v R2 ) 1 1 1 ZA, = 2<b + c>' zB,=2<c + a>' zc1=2<a + b)' za- =a + ^(b + c), zB. =b + -l(c + a), zc. = c + ^(a + b). 1 2 120 Números Complexos de A a Z A coordenada p de P satisfaz ambas as equações; portanto temos (p - b) x (c - b) = 0 e (p - x) ■ (c - b) = 0. Segue que (x-b)b + x + bc /_ x -R2 k □ tf Prova. Essas equações são equivalentes a (p-b)(c-b)-(p-b)(c-b) = 0 e (p-x)(c-b) + (p-x)(c-b) = 0 . Somando uma relação com a outra, encontramos (2p-b-x)(c-b) + (b-x)(c-b) = 0. abc') R2 ; R2 b Prova. Usando o produto complexo e o produto real podemos escrever as equações das retas BC e XP como segue: BC:(z-b)x(c-b) = 0, XP:(z-x)(c-b) = 0. 1T.= — b + x - 2l Represente por O9 o ponto médio do segmento OH. De acordo com nossa hipótese inicial, segue que zOg = tf (a + b + c). Além disso, temos |a| = |b| = |c| = R , onde R é o circunraio do triângulo ABC . 1f . bca') 1f . cab zA.=-^a + b + c--^-J, zB. = —I a + b + c--^y- 1 r c - b z_ -j 1P = 4 b + x + ÊTb(x-b)]= 2 1( K , zr. = — a + b + c- c 2l c - b c r\"l 1 í bc _ , 1 b = — x------?x + b + c .'J R2 ) Da Proposição 1 segue que as coordenadas de A’,B',C' são Teorema 2. (A circunferência dos nove pontos.) Em qualquer triângulo ABC. os pontos A,, B,, C,, A', B', C, A",B", C" estão todos sobre a mesma circunferência, cujo centro está no ponto médio do segmento OH e o raio é metade do raio da circunferência circunscrita. 1 2 1 2 cab R2 1214 | Mais sobre Números Complexos e Geometria O9A'= |zA. - zOg de onde segue a propriedade desejada. □ 4 ZN ”3 c, logo podemos escrever zN = 3zG - 2z,. Aplicando o resultado mencionado acima e as propriedades do produto complexo obtemos (zG-z,)x(zN - Z|) = (zG-zJx[3(zG-z,)] = 0 ; logo os pontos I, H, N são colineares.0 4 Leonhard Euler (1707-1783), um dos matemáticos mais importantes, criou uma boa parte da análise e revisou quase todos os ramos da matemática pura que eram conhecidos na época, adicionando provas e arranjando tudo em uma forma consistente. Euler escreveu um número imenso de memórias sobre todos os tipos de assuntos matemáticos. Recomendamos o livro Euler. The Masterof Us All de William Dunham (The Mathematical Association of America, 1999) para mais detalhes a respeito das contribuições de Euler para a matemática. -R. 2 Ir.2 Prova. 1) Se o circuncentro O é a origem do plano complexo, temos zo = 0, 1 zG = — (a + b + c), zH = a + b + c. Logo esses pontos são colineares de acordo com a Proposição 2 na Seção 2.22. 2) Temos que z, = ^a +^b + ^c' ZG = -(a+b + c), e PVÍ1-1Q sj V sj Teorema 3._____ 1) (Reta de Euler4 de um triângulo.) Em qualquer triângulo ABC , os pontos O, G, H são colineares. 2) (Reta de Nagel de um triângulo.) Em qualquer triângulo ABC , os pontos I, G. N são colineares. | = ^lal = ■ e também OgB, = O9C, =^R . -z0J = i|a| = ^R , e também O0B" = O9C" = Observe que O9A, = |zA, - zOs Podemos escrever O9A" = |za- A distância O9A' também não é dificil de ser calculada: i 11 ( . bcã 1 1 . . . I| = |_^a + b + c__|__(a+b + c)| = —^-j-lbcãl = —l=-|ã| Ibllcl = -5-=- 2R2 1 2R2........... 2R2 1 Analogamente, temos OgB' = O9C = —R . Portanto 2 1 OgAi = OqB1 = OgCj = OgA ' = OgB ’ = OgC ' = Og A " = OgB" = OgC " = — R 1 2 a 122 Números Complexos de A a Z Observação Figura 4.6. é o ponto médio do segmento [IN], então sua coordenada éSe Problema 1. + —, c' =a' = Considere um ponto M sobre a circunferência circunscrita ao triângulo ABC . Prove que os centros dos nove pontos dos triângulos MBC, MCA, MAB são os vértices de um triângulo semelhante ao triângulo ABC . Solução. Sejam A',B',C os centros dos nove pontos dos triângulos MBC,MCD,MAB, respectivamente. Tome a origem do plano complexo como sendo o circuncentro do triângulo ABC . Represente por letra minúscula a coordenada do ponto representado pela letra maiúscula correspondente. Assim, Note que NG = Gl, logo os triângulos OGI e HGN sâo semelhantes. Conclui-se que as retas OI e NH são paralelas e assim temos a seguinte configuração básica do triângulo ABC (na Figura 4.6): m + b + c :2: m +a + b 2 (« + P)C1, \ (P + Y) (y + «)kzn =— (z, +z„ = ——— a +—---- -b +Gs 2 1 N 4s 4s 4s O ponto Gs é chamado de ponto de Spiecker do triângulo ABC e é fácil verificar que ele é o incentro do triângulo mediai A^C,. 4 | Mais sobre Números Complexos e Geometria 123 pois M situa-se no circuncirculo do triângulo ABC . Então portanto os triângulos A'B'C e ABC são semelhantes. que podem ser facilmente = 8R2, que é o mesmo que sen2 A + sen2 B + sen2 C = 2 . Problema 3. b'-a' c'-a' a-b a -c b-a c-a Seja ABCD um quadrilátero cíclico e sejam Ea,Eb,Ec,Ed os centros das circunferências dos nove pontos dos triângulos BCD, CDA, DAB, ABC , respectivamente. Prove que as retas AEa, BEb, CEc, DEd são concorrentes. Usando as propriedades do produto real, temos |a + b + c|2 = (a + b + c).(a + b + c) = a2 + b2 + c2 + 2(a.b + b.c + c.a) = 3R2 + 2(a.b + b.c + c.a) = 3R2 + (2R2 - a2 + 2R2 - p2 + 2R2 - /) = 9R2-(a2+p2+y2), Solução. Tome a origem do plano complexo no circuncentro O do triângulo ABC e represente por a, b, c as coordenadas dos vértices A, B, C respectivamente. Então o circuncirculo do triângulo ABC é tangente à circunferência dos nove pontos do triângulo ABC se e somente se OO9 = 5.. isso é equivalente a OO2 = , isto é, |a + b + c|2 = R2. Problema 2.____________________________________________________________ Mostre que o triângulo ABC é um triângulo retângulo se e somente se seu circuncirculo e sua circunferência dos nove pontos são tangentes. onde a, p, y são os comprimentos dos lados do triângulo ABC . Nós utilizamos as fórmulas ab = R2-—,bc = R2-—,ca = R2-—, 2 2 2 obtidas da definição de produto real entre números complexos (veja também o lema na Subseção 4.6.2). Portanto, a2+p2+y2 Podemos escrever a última relação como 1 - cos2A + 1 - cos 2B + 1 - cos2C = 4. Isso é equivalente a 2cos(A + B)cos(A-B) + 2cos2 C = 0, i.e., 4cosAcosBcosC = 0, de onde segue a conclusão desejada. 124 Números Complexos de A a Z 4.6 ALGUMAS DISTÂNCIAS IMPORTANTES EM UM TRIÂNGULO 4.6.1 Invariantes fundamentais de um triângulo Teorema 1. Os lados a, p, y são as raízes da equação cúbica t3 - 2st2 + (s2 + r2 + 4Rr)t - 4sRr = 0. Prova. Do teorema acima, usando as relações entre raízes e coeficientes, conclui-se que a + p + y = 2s, ap + py + ya = s2+r2+4Rr, aPy=4sRr. Solução. Tome a origem do plano complexo no circuncentro O do quadrilátero ABCD .Assim, as coordenadas dos centros das circunferências dos nove pontos são 3' , portanto está provado o que se pede. Isto é, a3 - 2sa2 + (s2 + r2 + 4Rr)a - 4sRr = 0 . Podemos mostrar analogamente que P e y são raizes da equação acima. □ Temos AEa : z = ka+ (1-k)ea,k e Tf , e as equações análogas para as retas BEb,CEc,DEd. Observe que o ponto de coordenada —(a + b + c + d) está nas quatro retas ^k = gr 4R(s-a)' Considere o triângulo ABC de lados a, p, y, o semiperimetro é s = ^(a + p + y), o inraio é r e o circunraio é R. Os números s, r, R são chamados de invariantes fundamentais do triângulo ABC. e sen2 — = 2 Provemos que a Da relação cos2 + sen2 y = 1, segue que — + 1111 ea=—(b + c + d), eb=—(c + d + a), ec=-(d + a + b), ed=—(a + b + c). -----= 1. 4R(s-g) A A satisfaz a equação. Temos a = 2Rsen A = 4Rsen—cos— e A 2 2 s — a = rcotg— = r-----2-, logo cos2 — = ——— 2 sen— 2 4Rr 2 4 | Majs sobre Números Complexos e Geometria 125 Corolário 2. Em qualquer triângulo ABC, as seguintes fórmulas são válidas: a2 + p2 + y2 = 2(s2 - r2 - 4Rr), a3 + p3 + y3 = 2s(s2 - 3r2 - 6Rr). Para provar a segunda identidade, podemos escrever 4.6.2. A distância OI Lema. Os produtos reais a ■ b, b • c, c • a são dados por de onde segue a primeira fórmula, o Assuma que o circuncentro O do triângulo ABC é a origem do plano complexo e sejam a, b, c as coordenadas dos vértices A, B, C, respectivamente. Prova. Como a coordenada do incentro é dada por Prova. Temos a2 +p2 +y2 = (a + p + y)2 - 2(aP + Py + ya) = 4s2 -2(s2 +r2 +4Rr) = 2s2 - 2r2 - 8Rr = 2(s2 -r2 - 4Rr). Teorema 4.________________________________ (Euler) A seguinte fórmula é verdadeira: OI2=R2-2Rr. Ab+ic 2s 2s az, =—a + 1 2s a.b = R2 - —, 2 c.a = R2 - £ 2 b.c = R2 - —, 2 Prova. Usando as propriedades do produto real, temos y2 = |a - b|2 = (a - b).(a - b) = a2 - 2a.b - b2 = 2R2 - 2a.b. a3 + P3 + y3 = (a + p + y)(a2 + p2 + y2 - ap - Py - ya) + 3apy = 2s(2s2 - 2r2 - 8Rr - s2 - r2 - 4Rr) +12sRr = 2s(s2 - 3r2 - 6Rr). o 126 Números Complexos de A a Z então podemos escrever 4K ' R > 2r A igualdade ocorre se e somente se o triângulo ABC é equilátero. Prova 4.6.3. A distância ON Teorema 6. Se N é o ponto de Nagel do triângulo ABC , então ON = R - 2r. Prova. |C. cyc A coordenada do ponto de Nagel é dada por zN Portanto Pelo Teorema 4. temos OI2 = R(R - 2r) > 0 , logo R ã 2r. A igualdade R - 2r = 0 é válida se e somente se OI2 = 0 , i.e., O = I. Portanto o triângulo ABC é equilátero. □ Corolário 5. (Desigualdade de Euler.) Em qualquer triângulo ABC a seguinte inequaçâo é válida: —a + 2s cyc ON2=|zn|2 = zn.zn = R2£(i-^J , _ i - r» “PTonde as conhecidas relações R = ——, r ABC. o 4K Ab+Xc 2s 2s OI2 = IzJ2 =f—a + — b +—cYf- 1 11 \2s 2s 2s J l = -L (a2 + P2 + y2 )R2 + 2 Y (ap)a ■ b 4s2 4s2 “ Usando o lema acima, encontramos que oi2 = -L (a2+p2+y2 )R2 + X “P í r2 - y-] 4s 4s l 2 J = -1. (a + P + Y)2R2 - £ “Py2 = R2 - -^2 aPY(a + P + Y) = R2-—aPY = R2-2-^I - = R2-2Rr. 2s ' 4K s —, são utilizadas. K é a área do triângulo s 1274 | Mais sobre Números Complexos e Geometria r2-e. N Figura 4.7. e GNO são semelhantes. Segue que as retas Gl GN Gq9 GO ' Aplicando a expressão do Corolário 2, concluímos que E = -2(s2 - r2 - 4Rr) + 2(s2 - 3r2 - 6Rr) + 8Rr = -4r2 + 4Rr . Logo ON2 = R2 - E = R2 - 4Rr + 4r2 = (R - 2r)2 e a fórmula desejada é provada pela desigualdade de Euler □ Teorema 7. (Feuerbach5 ) Em qualquer triângulo, o incírculo e a circunferência dos nove pontos de Euler são tangentes. Prova. = R2^3- a P + Y S Usando a configuração da Seção 4.5 observamos que — = R2-y]= R2Z 1-- cyck, S 5 Karl Wilhelm Feuerbach (1800-1834), geômetra alemão, publicou o resultado do Teorema 7 em 1822. = -£a2+-S«3 + 8Rr. cyc Scyc Portanto os triângulos GIO9 'l Y2 = z Y2 - - z (« + P)Y2 + 4 Z «Pr2 J cyc S cyc S cyc = ZY2-iz(2s-Y)Y2+^ = -Eg24E«3.8^4 cyc Scyc S cyc Scyc S |2+2zíi--Yi--]p I cyc\ SÁ R2-zÍi--Yi--'Iy2 = cycX. SA Sj Para calcular E , note que E = + 4 cycl S S2 1 2 128 Números Complexos oe A a Z 4.6.4., A distância OH Teorema 8. Se H é o ortocentro do triângulo ABC .então OH2 = 9R2 +2r2 +8Rr - 2s2. Prova. OH2 = |zH|2 = zH - zH =(a + b + c)-(a + b + c) = Z|a| +2£ab = 3R = 9R2 ~(a2+p2+y2) 4.7 DISTÂNCIA ENTRE DOIS PONTOS NO PLANO DE UM TRIÂNGULO 4.7.1 Coordenadas Barícêntrícas Assumindo que o circuncentro O é a origem do plano complexo, a coordenada de H é zH=a + b + c. Usando o produto real podemos escrever Considere um triângulo ABC e sejam a, p, y os tamanhos dos lados BC, CA, AB, respectivamente. Corolário 10. Em qualquer triângulo ABC a desigualdade a2+p2 + y2 <9R2 é verdade. A igualdade é válida se e somente se o triângulo é equilátero. O ponto de tangência desses dois círculos é representado por q> e é chamado de ponto de Feuerbach do triângulo. Za b. cyc' ’ cyc Aplicando as fórmulas do lema (p. 112)cycentão a primeira fórmula do Corolário 2, obtemos OH2=3R2 + 2zÍr2-— ] cycl 2 J = 9R2 - 2(s2 - r2 - 4Rr) = 9R2 + 2r2 + 8Rr - 2s2 . □ Corolário 9. As seguintes fórmulas são válidas: 1) OG2 = R2+—r2 + —Rr - — s2 ; 9 9 9 2) OOg = —R2+—r2 + 2Rrs2 . 9 4 2 2 1 IO e ON são paralelas e IO9=-ON. Aplicando o Teorema 6 obtemos 1 R 2 IO9 = — (R - 2r) = —-r = R9 - r, logo o incirculo é tangente à circunferência dos nove pontos. ° 1294 | Majs sobre Números Complexos e Geometria Pc = Figura 4.8. Procedemos de modo análogo no caso em que o ponto P está situado no exterior do triângulo ABC . Se o ponto P está situado na reta suporte de um lado do triângulo ABC (i.e., a reta determinada pelos dois vértices), então Os números reais pa, pb, pc são chamados de coordenadasbaricêntricas absolutas de P com respeito ao triângulo ABC . Os sinais dos números pa, pb, pc dependem das regiões do plano onde o ponto P está situado. O triângulo ABC determina sete tais regiões. ------ ------------c (1 + k,)(1 + k2) ' -------- ---------- b + (1 + k,)(1 + k2) 1 . k „ 1 V. k . . PBzD = ■----- b +------ c = O a +------ b +------ c, onde k = —. □ 1+k 1+k 1+k 1+k PC kik2 (1 + k,)(1 + k2) 1 + k, + k2+k,k2 „ (1 + k,)(1 + k2) Proposição 1. Sejam a, b, cas coordenadas dos vértices A, B. C e seja P um ponto no plano do triângulo. Se zp é a coordenada de P , então existem números reais únicos pa,pb,pc tais que zp = paa + pbb + pcc e pa+pb+pc=1. Prova. Assuma que P está no interior do triângulo ABC e considere o ponto A' tal que APnBC = {A'}. Sejam k, =k2 =e observe que zp = — . b + k2c PA' A'c 1 + k' Za' ~ 1 + k2 ' Portanto, nesse caso podemos escrever 1 zp =------- a + p 1 + k, Além disso, se considerarmos 1 k, Pa 1 + k,' Mb"(1 + k,)(1 + k2)' 1 k, k,k2 temos Pa + Pb + PC - — + (i + ki)(i + k2) + (1 + kl)(1 + k2) Números Complexos de A a Z130 Na tabela seguinte damos os sinais de na,pb,pc : V VI VIIII III IV Ma Mb Mc Prova. Escolha a origem do plano complexo no circuncentro O do triângulo ABC . Usando as propriedades de produto real, obtemos = Z(a2 -a,)2R2 + 2£(a2 -a,)(p2 -p,) R2 cyc cyc l J = R2(a2 + p2 + y2 - a, - p, - Y,)2 - Z(a2 - ccn)((J2 ~ Pi)Y2 cyc = -S(«2-“l)(p2-Pl)Y2 . cyc já que a, + p, + y, = a2 + p2 + y2 = 1 • ° Teorema 2. No plano do triângulo ABC considere dois pontos R, e P2 com coordenadas, zP1 e z^ , respectivamente. Se. z^ = aka + pkb + Ykc , onde czk, Pk, Yk são números reais tais que ak + pk + Yk = 1, k = 1, 2, então P1p22 = -Z(«2-«1)(P2-P1)y2 cyc 4.7.2 Distância entre dois pontos em coordenadas baricêntricas No que segue, para simplificar as fórmulas, usaremos o símbolo chamado de "somatório cíclico". Isto é, I f(x„ x2 xn), a soma dos temos considerados na ordem ciclica. O exemplo mais importante para nossos propósitos é X f(x„ x2, x3) — f(x-), x2, x3) + f(x2, x3, x,) + f(x3, Xp x2). cyc P]P2 -|2P2”ZP1| — |(a2 al)a + (P2 P,)b + (Y2 Yl)c| = Z(“2 - a,)2a a + 2£(a2 -a,)(p2 -P,)ab cyc cyc Y2 2 4 | Mais sobre Números Complexos e Geometria 131 P„. k = 1.2. P,P22 = Prova. As coordenadas dos pontos P, e P2 são = . k = 1, 2. k = 1, 2.ík = e a fórmula desejada segue. □ yk =1,k = 1.2. Segue que nesse caso, as coordenadas baricéntricas absolutas dos pontos P, e P2 são dadas por = ny ’ sk ■ onde mk,nk, pk são números reais não nulos, k = 1, 2, e Sk = mk + nk então Corolário 4. Para quaisquer números reais ak,pk,7k com ak+Pk a seguinte desigualdade é válida: E(“2-ai)(P2-Pi)Y2 20. cyc P.P2 =-z cyci Skmk + nk + Pk Substituindo na fórmula do Teorema 2, encontramos ( nz ni ¥ P2 Pi Y2 \S2 S, Jl.S2 S, J rr^a+nk^lPkE mk+nk +pk Pk mk + nk ” Pk — O2o2 Z(S/t2 — S2n1)(S1p2 - S2p1)az S1 S2 cyc = — E[S?n2p2 “ $1^2(11^2 +n2Pi)](X S-i S2 cyc -õM sis2 E <niP2 +Pin2)a2 "s? Sn2P2a2 -S2ZniPia2 S1 S2 L cyc cyc cyc j Z2Z2 S,S2 Z (0^2 + p1n2 )a2 - S2 Z n2p2a2 - S2 Z n^a2 o1o2L cyc cyc cyc J mk “k =------------ - --------- mk + nk + Pk Teorema 3. Os pontos A,. A2, B,. B2, C,.C2 estão situados sobre os lados BC. CA, AB do triângulo ABC tais que as retas AA^BB^CC, se encontram no ponto P, e as retas AA2,BB2, CC2 se encontram no ponto P2 . Se ^Ak Pk CBk . mk ACfc °k k = 1 2 AkC nk ' BkA pk ' CkB mk ' nk sk ■•k“ 132 Números Complexos de A a Z com a igualdade se e somente se a, = a2,p, = p2, y, = y2 ■ com a igualdade ocorrendo se e somente se = 2s3 + 2sr2 - 4sRr. equilátero. Z n,p,a2 = a2 + p2 + y2 = 2s2 - 2r2 - 8Rr. cyc Z(2a-p-y)(2p-a —y)y2 <0. cyc Aplicações. 1) Usemos a fórmula do Teorema 3 para calcular a distância Gl, onde G é o baricentro e I o incentro do triângulo. Temos m, = n, = p, = 1 e m2 = a, n2 = p, p2 = y ; portanto Por outro lado, Zn2p2o2 cyc Pi _ P2 mi _ m2 ni _ n2 n1 n2 Pi P2 ml m2 Corolário 5. Para quaisquer números reais não nulos mk, nk, pk, k = 1, 2 , com Sk = mk+nk + pk,k = 1,2, os comprimentos dos lados ot.p, y do triângulo ABC satisfazem a desigualdade Z (niPz + PA>)2 à f1 s n2p2cx2 + f2- Z n,P,a2 cyc o 2 cyc o1 cyc Então, Gl2 =^(s2+5r2-16Rr). 2) Provemos que em qualquer triângulo ABC de lados a,p,y, a seguinte desigualdade é valida: = a2Py + p2ya + y2aP = «Py(a + P + y) = 8s2Rr e Na desigualdade do Corolário 4, consideramos os pontos P, = G e P2 = I. Então „ 1 a ,, P y . , . .a, = p, = y, = — e a2 = —. P2 = 4-, y2 = — , e a desigualdade acima segue. 3 2s 2s 2s Temos a igualdade se e somente se P, = P2; isto é, G = I, então o triângulo é S, = Z m, = 3; S2 = Zm2 = ct + P + Y = 2s; cyc cyc z (n,p2 + n2p,)a2 = (p + y)a2 + (y + a)p2 + (a + p)y2 cyc = (a + P + y)(ap + py + ya) - 3aPy = 2s(s2 + r2 + 4rR) -12sRr 1334 | Mais sobre Números Complexos e Geometria 4.8 ÁREA DE UM TRIÂNGULO EM COORDENADAS BARICÉNTRICAS Prova. x a a xb Analogamente, achamos 2i area[P2OP3] = 2i area[P3OP,J = Considere o triângulo ABC com a,b,c as coordenadas de seus vértices, respectivamente. Sejam a. p, y os comprimentos dos lados BC. CA e AB . onde P,P2P3 têm a mesma orientação, então Suponha que os triângulos ABC e P,P2P3 são positivamente orientados. Se O denota a origem do plano complexo, então usando o produto complexo podemos escrever axb Yr Y2 “1 «2 cxa Pi P2 Yr Y2 «3 “1 Pr P2 Ps «1 “2 “3 2i areajABC] 1 1 2i area(ABC] 1 1 2i areajABC] 1 1 “2 «3 bxc bxc «i “2 area[P1P2P3] areajABC] Yi Yz Ys 2i area[P,OP2] = zPl x z, axb bxc Y2 Ys axb Y3 Y1 Teorema 1. Sejam PjfZp.), j = 1,2,3 . três pontos no plano do triângulo ABC com zPj =aja+ Pjb + YjC. u|,PrY| são as coordenadas baricêntricas de P. Se os triângulos ABC e ipj = (a,a + p,b + y,c) x (a2a + p2b + y2c) = (a,p2 - a2p,)a X b + (Pf/2 -p2'/,)bx c + (y,a2 - y2a,)c bxc Assumindo que a origem O estâ situada no interior do triângulo P,P2P3 , segue que area[P,P2P3] = area[P1OP2]-r-area[P2OP3] + area[P3OP,] = ^r(“i -“2 +a2 -a3 +ot3 -a,)axb-^(Yt -Yz + Y2 -Y3 +Y3 -Yr)bxc +(Yi«2 - Y2«1 + Y2«3 - Y3“2 + Y3“i - Yi«3)area[ ABC] Números Complexos oe A a Z134 «1 «3 Pa Va e a fórmula desejada é obtida. □ k3 (1 + k, + k1k2)(l + k2 + k2k3)(l + k3 + k3kj Figura 4.9. Prova. Portanto A coordenada de P3 é dada por Z?3 = C,A ' C,B " P3A fV- a + k3b + k3k,c 1 + k3 + k3k, Aplicando o teorema de Menelaus no triângulo AA,B achamos que C,A CB P3A, C,B ’ CA, ’ P3A “ ■ Se AA,r>BB, = {P,}. BB, oCC, = {P2} e CC, r> AA, = {P3}, então area[P,P2P3] = (1 - k,k2k3)2_ areajABC] C,A CB . . C,B CA, + a -r k3(l + k)zAi 1 + k3(l + k,) a + k3(l + k,)~L k^c 1 + k3 + k3k, = (Yi«2 - Y2ai + Y2a3 - Yaa2 + Yaai “ Yi«a)areaIABC1 Pi Yi1 Tl = area[ABC] 1 y2 «2 = areaIABC] a2 P2 Y2 1 Ya “a Corolário 2. Considere 0 triângulo ABC e os pontos A,,B,.C, situados nas retas BC. CA. AB, respectivamente, tais que ^ = k„ 5£ = k2 A,C 1 B,A 2 1354 | Mais sobre Números Complexos e Geometria 2p, u Observação j = 1,2.3. onde Sj = m, + nj + pj, j = 1.2,3 . Prova —(miazPj = ki 1 k3k, Quando k,=k2=k3=k, partindo do Corolário 2, obtemos o Problema 3 da 23a Olimpíada de Matemática de Putnam. ABC, P,P2P3 têm a mesma orientação, então area[P1P2P3] areajABC] 1 k2k3 k3 k1k2 k2 1 De modo análogo encontramos que k,k2a + b + k,c 1 + k, + k,k2 + njb + p|c), j = 1,2, 3. area[P,P2P3] - area[ABC] ________________1_______________ (1 + k1 + k1k2)(1 + k2 + k2k3)(1 + k3 -t- k3k3) k2a + k2k3b + c 1 + k2 + k2k3 ___________ ÇI-kMa)2____________ (1 + kl + k,k2)(1 + k2 + k2k3)(1 + k3 + k3kj 1 S,S2S3 Sejam A,, Bj, C1 os pontos nas retas BC, CA. AB , respectivamente, tais que BA) pj CBj m, AC, n, AjC Oj BjA Pj CjB rrij Em função das coordenadas do triângulo, as coordenadas dos pontos Pj são m^ + njb + PjC 1 mi+ni + Pi ~Si A fórmula acima segue diretamente do Teorema 1. □ Os triângulos ABC e P,P2P3 têm a mesma orientação; portanto, aplicando a fórmula do Teorema 1, achamos queCorolário 3. Se Pj é a interseção das retas AAj, BBj, CCj, j = 1, 2,3 , e os triângulos m, n, p, m2 n2 p2 m3 n3 P3 e Zp2 = Corolário 4. No triângulo ABC consideremos as cevianas AA',BB' e CC tais que A'B B'C CA ------ = m, -------= n, - = P- A'C B'A CB 136 Números Complexos de A a Z Prova. zR. =------c-r------ a. c2b 2 K,= BA, A,C area[P,P2P3] area(ABC) 1 A2C CB A3C " 2 ' BA 2 2 4 1 1 2 4 1 2 7 2 4 m 1 0 n 1 49 73 area[A’B'C] area[ABC] Então a seguinte fórmula é válida: area[A‘B’C] area[ABC] Aplicações.___________________________________________________________ 1) (Stemhaus6) Sejam A^Bj.C, pontos nas retas BC, CA, AB , respectivamente, j = 1,2,3 Assuma que 1 + mnp (1 + m)(1 + n)(1 + p) 1 + mnp (1 + m)(1 + n)(1 + p) 1 7-7-7 1 (1 + m)(1+n)(1 + p) 6Hugo Dyonizy Steinhaus (1887-1972), matemático polonês, deu contribuições em análise funcionais e outros ramos de matemática moderna. Observe que as coordenadas de A',B',C' são dadas por 1 . m 1 n z«. =-------b +------- c. z„. - c: 1 + m 1 + m 1 + n 1+n Aplicando a fórmula do Corolário 3 obtemos 0 1 0 P --------a J----------------area(ABC) 1 + I 1+P 1 + “ l pA “A y) 2) Se as cevianas AA',BB',CC são concorrentes no ponto P, denotemos por Kp a área do triângulo A'B'C . Podemos utilizar a fórmula do Corolário 4 para calcular as áreas de alguns triângulos determinados pelos pés das cevianas de alguns pontos notáveis em um triângulo. (i) Se I é o incentro do triângulo ABC temos Se Pj é a interseção das retas AA)r BBj, CCj, j = 1,2,3 , e os triângulos ABC, P,P2P3 têm a mesma orientação, então pelo Corolário 3 obtemos 1 2 4 1 p . zP. = ——a +—— b .c 1+p 1+p CB, 1 AC, 4 . BA2 4 CB2 2 AC2 1 . B^à “ 2 ’ C,B ~ 1 ' A2C ~ 1 ' B2A " 4 ’ C2B ~ 2 BA3 1 CB AC3 2 ------- 1 ' C3B " 4 ■ 1374 | Mais sobre Números Complexos e Geometria kh = area[ABC] 1 + area|ABC]Kn = area[ABC] Observação então para as cevianas isotõnicas correspondentes temos n Aplicando a fórmula do corolário 4, temos que A"B A"C 2 P s-p area[A'B'C] area[ABC] 1 + mnp (1 + m)(1 + n)(1 + p) 2a0ysr (a+ P)(P+ ■/)(* +ci) area[A"B"C"] area[ABC] 1+— mnp n n ) (iii) Para o ponto de Nagel do triângulo ABC podemos escrever ■i . s~Y s-g s-p s-p s-y s-g S-Y . S-P~j s-a ) 4area2[ABC] 2saPY 1 + 1 P Kj=^area[ABC] = ^ r at = — area[ABC] = — 2R 2R . Se procedermos da mesma maneira para o ponto de Gergonne J , encontramos a relação Duas cevianas AA' e AA" são isotõnicas se os pontos A’ e A" são simétricos em relação ao ponto médio do segmento BC . Assumindo que A'B B'C C'A------= m, -------= n, ——- = p, A'C---------B'A----------C'B 2(S-a)gp~YP)(S~Y)area[ABC] = _1_ ETC m' B"A -------------------------- area[ABC] = (a + p)(P + y)(y + «) 1 C"A ’ C"B (ii) Para o ortocentro H do triângulo ABC , obtemos 1 , tg C tg B tg A tg B tg A tg C ÍÇY1 + !9ÊY1+tgA tg 8 X tgAX tg C = (2 cos AcosBcosC)area[ABC] = (2cosAcosBcosC)sr. 138 Números Complexos de A a Z C , area[ABC] 1 areajABC] 1 = 2sR. c, zR =— a +—b +—c, zM' -J *3 o ' N Então areajIGN] = ■areajABC] = 0 , 4) (Reta de Nagel.) Usando a fórmula do Teorema 1, damos uma prova diferente para a reta de Nagel: os pontos I, G, N são colineares. Vimos que as coordenadas desses pontos são Logo area[A'B'C] = area[A"B"C"]. Um caso especial dessa relação é KN = Kj, já que os pontos N e J são isotônicos (i. e.. esses pontos são interseções de cevianas isotòmcas). 3) Considere os ex-centros l„,l(1,ly do triângulo ABC. Não é difícil ver que as coordenadas desses pontos são 1 1 1 1 JL 2s 2 3 area^lp!.,] = SuPy areajABC] 2s(s -cc)(S -P)(s - Y) saPy areajABC] 2 area2[ABC] 2supy 4 areajABC] Y 2(s-y) Y 2(s-y) P 2(s-p) P 2(s-p) P 2(S-P) Y 2(s-y) Y 2(s-y) Y 2(s-y) a 2s 2 3 1-- s a z, = —a ' 2s +Ab + X 2s 2s ______ aPY______ 8(s-a)(s-p)(s-y) _Y_ 2s 2 3 1-Ê 1-1 s s -P-b+ 2(s-P) -P-b + 2(s-p) a • p Z|r " 2(s-a)a + 2(s-p) Da fórmula do Teorema 1, segue que a ’2(s-a) a 2(s-a) a 2(s-a) C’ Z'0’2(s-a)a a z. =-----------a + 2(s - a) b-------—c. 2(s-y) 1 3 1 3 1 3 1394 | Mais sobre Números Complexos e Geometria logo os pontos I, G, N são colineares. 4.9 TRIÂNGULOS ORTOPOLARES 4.9.1 Reta de Simson-Wallance e triângulo pedal Teorema 1. Figura 4.10. (A reta8 de Simson7) Os pontos P, Q, R são colineares se e somente se M está no circuncirculo do triângulo ABC . Prova. Daremos um argumento geométrico padrão. Suponha que M está no circuncirculo do triângulo ABC. Sem perda de generalidade, podemos assumir que M está no arco BC. Para provar a colinearidade de R. P, Q . é suficiente mostrar que os ângulos BPR e CPQ são congruentes. Os quadriláteros PRBM e PCQM são inscritíveis (pois BRM = BPM e MPC - MQC = 180 ), logo Considere o triângulo ABC e seja M um ponto situado no plano do triângulo. Sejam P, Q, R as projeções de M nas retas BC, CA, AB , respectivamente. 7Robert Simson (1687-1768), matemático escocês. “Essa reta foi atribuída a Simson por Poncelet, mas agora é frequentemente conhecida como reta de Simson-Wallance já que na verdade ela não aparece em nenhum trabalho de Simson. William Wallance (1768-1843) também foi um matemático escocês, que provavelmente publicou o teorema acima a respeito da reta de Simson em 1799. 140 Números Complexos de A a Z Teorema 2. A área do triângulo pedal de X em relação ao triângulo ABC é dada por |xx-R2|area[PQR] = (1) Figura 4.11. onde R é o circunraio do triângulo ABC . Para provar a volta, notemos que se os pontos P, Q, R são colineares, então os ângulos BPR e CPQ são congruentes, pois ABM +ACM = 180', i.e., o quadrilátero ABMC é inscritivel. Portanto o ponto M está situado na circunferência circunscrita do triângulo ABC . □ Quando tvt está na circunferência circunscrita ao triângulo ABC , a reta do teorema acima é chamada de reta de Simson-Wallance de M em relação ao triângulo ABC . Continuamos com uma boa generalização da propriedade contida no Teorema 1. Para um ponto arbritário X no plano de um triângulo ABC , considere suas projeções P. Q e R nas retas BC, CA e AB, respectivamente. O triângulo PQR é chamado de triângulo pedal do ponto X em relação ao triângulo ABC Tomemos o circuncentro O do triângulo ABC como a origem do plano complexo. areajABC] 4R2 temos BPRsBMR e CPQsCMQ. Porém, BMR = 90° - ABM = 90°-MCQ, pois o quadrilátero ABMC também é inscritivel. Finalmente, obtemos BMR = 90°-MCQ = CMQ, portanto os ângulos BPR e CPQ são congruentes. ■ B PC 1414 | Majs sobre Números Complexos e Geometria x-----7x+c+a Segue que (2) Logo area[PQR] 16abc |xx-R2 |,area[PQR] Prova. Aplicando a fórmula da Proposição 1, Seção 4.5, obtemos as coordenadas p, q, r dos pontos P, Q, R , respectivamente: 2 2 Para as coordenadas p, q, r obtemos - 1 P = 2 area[PQR] = — q q 1 r r 1 i), r =-fx--^x+ã + bl ) 2^ R2 J i(a -b)(b -c)(a -c) 16abc i(a-b)(a-c) 16abc i(a-b)(a-c) R2 - cx 16abc area[ABC] 4R2 _i|n-p r-p| 4|r-p r-p| Levando em consideração a fórmula na Seção 2.5.3. temos . P P 1 c. — j. .j - 4 x-c R2-bx x-b x 1 R2 - bx x-b p4(x-Sx+b+4 1 í ca -, q = — x—-x + c + a2l R2 = i|q - p p - p| 4|r - q r - p|’ 1 í bc P = - x-----5-x + b + c2l R2 onde a, p, y são os comprimentos dos lados do triângulo ABC .□ 1-^- (x-c)R2 1--^- (x-b)R2 R2 i(a-b)(a-c) (b-c)x b-c R2-bx x-b ab _ . >x —=-x-rã + b .R2 J Procedendo para módulo achamos que _ |a-b||b-c||c-a| 2 apY 2 ’ 16|a||b||c| |XX’R |- l̂XX’R |- 1 / cx 1q-p = -(a-b)^1-^ < q-p = -Y~(a-b)(x-c)R2 e 2abc 1, /. bx'!e r-p = -(a-c)p-^ r-p = —!-(a-c)(x-b)R2. 2abc i(a-b)(b-c)(a-c) 2 16abc 1 2 1 2 ãb R2 cã R2 142 Números Complexos de A a Z Observações- IR?-R21-area[PQR] = Isso é equivalente a | x |2= R2 ± = R2 1± , uma das quais é degenerada em O Prova. (3) Teorema 3. Para qualquer ponto X no plano do triângulo ABC , podemos construir um triângulo com lados AX BC, BX ■ CA, CX ■ AB. Esse triângulo é semelhante ao triângulo pedal do ponto X em relação ao triânguloABC . Seja PQR o triângulo pedal de X em relação ao triângulo ABC . Pela fórmula (2) obtemos Sc !;í 1 -------- 4 r—4k— O e raio R, = R 11 +-------—— R2-cx R2(x-c)' 4k "l areafABC])' areafABC] 4R2 Segue que a área do triângulo PQR não depende do ponto X . A volta também é verdade. O lugar geométrico de todos os pontos X no plano do triângulo ABC tais que areajPQR] = k (constante) é definida por i - o2, 4R2k xx - R = —------— . areafABC] 4R2k areafABC] 1) A fórmula do Teorema 2 contém a propriedade da reta de Simson-Wallance. De fato, os pontos P,Q,R são colineares se e somente se area[PQR] = 0. Isto é, | xx - R2 |= 0, i.e., xx = R2. Segue que | x |= R , portanto X está no circuncirculo do triângulo ABC. 2) Se X está em um circulo de raio R, e centro O (o circuncentro do triângulo ABC ), então xx = R2 e, pelo Teorema 2, obtemos quando k =-^-areafABC]. q-p = l(a-b)(x-c) Se k > —areafABC], então o lugar geométrico é uma circunferência de centro 4 I 4k O e raio R, = R 11 +--------------- . y areafABC] Se k < ^-areafABC], então o lugar geométrico consiste em duas circunferências I 4k de centro O e raios R 11 ±-------7 area[ABC] 1434 | Mais sobre Números Complexos e Geometria Procedendo para modular em (3), segue que (4) logo por (4) obtemos a relação (5) Portanto (6) Corolário 4. area[A' B ’ C ’] = area[ABC] | xx - R21. (7) Prova. Corolário 5. (8) Corolário 6. R2-cx R2-cx x-c R2 No plano do triângulo ABC considere o ponto X e denote por A'B'C o triângulo com lados AX • BC, BX ■ CA, CX■AB. Então Por outro lado, 2 _1_ 2R X- — c Da expressão (6) segue que area[A'B'C] = 4R2area[PQR], onde PQR é o triângulo pedal de X em relação ao triângulo ABC . Utilizando esse resultado em (1), achamos a fórmula desejada. □ R2-cx x-c QR AXBC RP BXCA R2-cx x-c PQ CX AB e a conclusão segue, o (Teorema de Ptolomeu) O quadrilátero convexo ABC D é inscritivel se e somente se AC • BD = AB CD + BC ■ AD . (9) R2-cx R2-cx x-c x-c |q-p 1=-^-1 a-b || x-c |. Zr\ |q-p|=^21a-bIIx-c| (Desigualdade de Ptolomeu) Para qualquer quadrilátero ABCD a seguinte desigualdade é válida: AC BDS AB CD + BC AD. R2-cx R2(c - X) r2 x-c CX - R2 144 Nümeros Complexos de A a Z Prova. Corolário 7. Prova. No teorema 3 temos BC = CA = AB e a conclusão desejada segue. □ (10) Problema 1. AX QR BX RP 2>/3 3 9Dimitrie Pompeiu (1873-1954), matemático romeno, deu importantes contribuições nos campos de análise matemática, funções de variáveis complexas, e mecânica racional. Se a relação (9) é válida, então o triângulo A'B'C no corolário 4 é degenerado; i.e., area[A'B'C’] = 0 . Pela fórmula (7) segue que d d = R2. onde d é a coordenada de D e R é o circunraio do triângulo ABC . Logo, o ponto D está na circunferência circunscrita ao triângulo ABC . Solução. A primeira afirmação é apenas a propriedade contida no Corolário 7. Levando em consideração as relações (10), segue a área do triângulo de Pompeiu (Teorema de Pompeiu9) Para qualquer ponto X no plano do triângulo equilátero ABC , três segmentos com comprimentos XA, XB, XC podem ser tomados como lados de um triângulo. O triângulo do Corolário 7 é chamado de triângulo de Pompeiu de X em relação ao triângulo equilátero ABC . Esse triângulo é degenerado se e somente se X estiver na circunferência circunscrita ao triângulo ABC . Usando a segunda parte do Teorema 3, encontramos que o triângulo de Pompeiu do ponto X é semelhante ao triângulo pedal de X em relação ao triângulo ABC e CX PQ 2R a Sejam A, B e C pontos equidistantes na circunferência de raio unitário centrada em O e seja X qualquer ponto no interior da circunferência. Sejam dA,de,dc as distâncias de X para A, B, C, respectivamente. Mostre que há um triângulo com lados dA.dB,dc , e a distância de X para O . (Olimpíada de Matemática de Putnam 2003) Se o quadrilátero ABCD é inscritivel, então o triângulo pedal do ponto D em relação ao triângulo ABC é degenerado. De (6) obtemos a relação (9). □ 4 | Mais sobre Números Complexos e Geometria 145 area[A*B'C] (11) (12) Caso 1. de (12) encontramos que logo 1 3R2 Substituindo em (11) achamos |xO2-R2| = -^-7(u2+v2 Agora considere os dois seguintes casos: + w2)2 -2(u4 + v4 + w4). + w4).+ w2)2-2(u4+v4 + w2)2-2(u4+v4 + w“). |x-x-R2| = —|xO2-R2|. 2R2 = -i(u2 + v2 + w2) + J(u2 +v2 + w2)2 -2(u4 +v4 +w4), 3 V3 Se X estiver no interior do circuncirculo do triângulo ABC, então XO2 <R2. Usando a relação (também veja a fórmula (4) na Seção 4.11) XO2 =-(u2+v2+w2-3R2), a2 = —(u2 + v2 2 (Olimpíada de Matemática de GDR 1978) Solução. Os segmentos [XA], [XB], [XC] são os lados do triângulo de Pompeiu do ponto X em relação ao triângulo equilátero ABC. Denote esse triângulo por A'B'C’. Das relações (10) e do Teorema 2 segue que = area[PQR] = ^-areatABCjlx • x - R2| Por outro lado, usando a fórmula de Heron, obtemos depois de cálculos simples: area[A'B'C] = —7(u2+v2 Problema 2. Seja X um ponto no plano do triângulo equilátero ABC tal que X não está na circunferência circunscrita ao triângulo ABC e sejam XA = u, XB = v, XC = w . Expresse o comprimento do lado a do triângulo ABC em função dos números reais u, v, w . 2 do ponto X é yarea[PQR]. Pelo Teorema 2 temos que area[PQR] depende apenas da distância de P para O , como desejado. 146 Números Complexos de A a Z Caso 2. 4.9.2 Condições necessárias e suficientes para ortopolaridade |a| = |b| = |c| = |x| = |y| = |z| = R, onde R é o circunraio do triângulo ABC . Teorema 3. Os triângulos ABC e XYZ são triângulos ortopolares se e somente se abc = xyz . Prova. Dividimos em dois casos. As coordenadas de P, Q, R são denotadas por p, q, r, respectivamente. Usando a expressão da Proposição 1, Seção 4.5, temos Os pontos P, Q, R estão na mesma reta; isto é, a reta de Simson-Wallance do ponto X em relação ao triângulo ABC . Se X estiver no exterior da circunferência circunscrita ao triângulo ABC , então XO2 > R2 e, após alguns cálculos similares, achamos Sejam P. Q, R os pés das retas ortogonais do ponto X para as retas BC, CA, AB , respectivamente. Escolhamos o circuncentro O do triângulo ABC como a origem do plano complexo. Os pontos A. B, C, X, Y, Z têm coordenadas a, b, c, x,y, z com 2 2 + w2)2-2(u4 + v4 + w4).a2 = i(u2 + v2 10 Essa definição foi dada em 1915 pelo matemático romeno Traian Lalescu (1882- 1929). Ele é famoso por seu livro La géometne du tríangle publicado pela Librarie Vuibert, Paris, 1937. 1 ( bc_.'t 1 f ca_ > p = — x-----Tx+b+c , q = — x-—?x + c + a2< R2 ) 2< R2 ) Considere um triângulo ABC e os pontos X, Y, Z situados em sua circunferência circunscrita. Os triângulos ABC e XYZ sâo chamados triângulos ortopolares (ou S-triângulos)10 se a reta de Simson-Wallance do ponto X em relação ao triângulo ABC é perpendicular (ortogonal) a reta YZ . +w2)-^Tõ^7 3b _x----- -x + a + b . R2 ) 1 2 1 2 ca R2 1474 | Mais sobre Números Complexos e Geometria Caso 1. O ponto X não é um vértice do triângulo ABC . (y-z) = 0 logo Observação Problema 1. A última relação é equivalente a (abo-xyz)(a -b)(c-x)(y-z) = 0 e finalmente temos abc = xyz , como desejado. Devido à simetria da relação abc = xyz , observamos que a reta de Simson-Wallance de qualquer vértice do triângulo XYZ em relação a ABC é ortogonal ao lado oposto do triângulo XYZ . Além disso, a mesma propriedade é válida para os vértices do triângulo ABC . Logo, ABC e XYZ são triângulos ortopolares se e somente se XYZ e ABC são triângulos ortopolares. Portanto a relação de ortopolaridade é simétrica. Os triângulos medianos e órticos de um triângulo ABC são ortopolares na circunferência de nove-pontos. Então PQ é ortogonal a YZ se e somente se (p - q) ■ (y - z) = 0 . Isso é, [—>(’< ou (b -ã)(R2 - cx)(y-z) + (b -a)(R2 -cx)(y-z) = 0. Caso 2. O ponto X é um vértice do triângulo ABC . Sem perda de generalidade, assuma que X = B . Então a reta de Simson-Wallance do ponto X = B é a reta ortogonal de B para AC . Segue que BQ é ortogonala YZ se e somente se as retas AC e YZ são paralelas. Isso é equivalente a ac = yz . Como b = x , obtemos abc = xyz , como desejado. □ Obtemos I b a -4~(a - b)(c - x)(y - z) ——(a - b)(c - x)(y - z) = 0. abc xyz R2- —x c y z) í R2 (y-z) +(b-a) R2 -c— I x 148 Números Complexos de A a Z e a afirmação está provada. Solução. (b-c) = 0. bc - bc— + a a Problema 2.___________________________________________________________ As alturas do triângulo ABC encontram o circuncirculo nos pontos Ap Bv C,, respectivamente. Se ApBpC, são os pontos antipodais de A^B^C, sobre o circuncirculo de ABC, então ABC e A-^C, são triângulos ortopolares. 1f l. bc=— a + b + c------ 2 , 1 ( . ab• c = -la + b + c~ — (a -m)(b -<o)(c- co) = (m - co)(n-<o)(p - m) = —abc 8 Se m, n,p,a',b',c’ são coordenadas de M, N. P, A', B', C , então temos 1 1 1m = — (a+b), n = —(b + c), p = —(c + a) e a = — a + b + c------a2l F. ., 1f . cab =- a+b+c------ 2\ b Solução. Considere a origem do plano complexo no circuncentro do triângulo ABC . Sejam M, N. P os pontos médios de AB, BC, CA e sejam A', B', C os pés das alturas do triângulo ABC saindo de A, B, C, respectivamente. As coordenadas de ApB^C, são respectivamente. De fato, a abc equação da reta AH em termos de produto real é AH : (z-a)-(b-c) = 0 . Basta provar que o ponto de coordenada _ situa-se tanto em AH quanto no circuncirculo a do triângulo ABC. Primeiramente, perceba que I-—I = Msl = L_3. = r , assim este | a | |a| R ponto é situado sobre o circuncirculo do triângulo ABC. Agora vamos mostrar que o número complexo _ satisfaz a equação da reta AH. Isso é equivalente à a O centro dos nove pontos O9 é o ponto médio do segmento OH, onde H (a + b + c) é o ortocentro do triângulo ABC. A coordenada de O9 é o> = —(a + b + c). Agora observe que bc a ab 2 1494 | Majs sobre Números Complexos e Geometria 0 OU a Figura 4.12. bc ca ab ba obtemos que os triângulos ABC e A^C, são ortopolares. Problema 3. Figura 4.13. Finalmente, isso se torna (b-c) verdadeira. Usando a definição de produto real, se reduz à ( bc Sejam P e P’ dois pontos distintos sobre o circuncírculo do triângulo ABC tais que as retas AP e AP' são simétricas em relação à bissetriz do ângulo BAC . Prove que os triângulos ABC e APP' são ortopolares. ~ + a (b-c) + bc a ^ + aj(b_c) = f abc -Y. , í (,RF + aJ( ) + C V r2 r2) n Ab c J Segue que A^B^Ci possuem coordenadas respectivamente.Como abc = abc, c abc - R2 aR2 'l . , —— + a---------------- , uma igualdade claramente bc I 150 Números Complexos de A a Z (p - p')(b - c) - (p - p')(b - c) = 0. (P-P')[’ assim = 0 . 4.10 AREADO TRIÂNGULO ANTIPODAL Figura 4.14 1 PP' Solução. Vamos considerar como p e p' as coordenadas dos pontos P e P’, respectivamente. É fácil ver que as retas PP’ e BC são paralelas. Usando produto complexo, segue que (p - p')x (b -c) = 0 . Essa relação é equivalente à Portanto, bc = pp’, i.e., abc = app'. Partindo do Teorema 3 segue que ABC e APP' são triângulos ortopolares. b c Considere o triângulo ABC e o ponto M. As retas perpendiculares à MA,MB,MC partindo de A,B,C, respectivamente, determinam um triângulo; chamamos esse triângulo de triângulo antipedal de M em relação â ABC. Renomeie M’ como o ponto isogonal de M se os pares de retas AM, AM ;BM,BM ;CM,CM são isogonais, i.e., as seguintes relações ocorrem: MAC = MVXB.MBC = MTBÃ.MCà = tvFcB . Considerando a origem do plano complexo no circuncentro O do triângulo ABC, temos R2(P-P')(b-c)^ r2 R2k x n P P' P 1 1514 | Majs sobre Números Complexos e Geometria Prova (2) zo = zo, 2 4 ' m b c Teorema.______________________________________________________________ Considere M um ponto no plano do triângulo ABC, M' o ponto isogonal de M e A"B"C" o triângulo antipedal de M em relação à ABC. Então |r2-(om)2| 4R2 |P(M-)| 4R2 areafABC] area^A B C J onde p(M') é a potência de M' em relação ao circuncírculo do triângulo ABC. Areta bissetriz ao segmento [z,,z2],em a seguinte equação em termos de produto real: [z-^(z, + z2)j (z1 - z2) = 0 . Basta verificar sez0 satisfaz essa equação, com a implicação, por simetria, de que zo pertença âs bissetrizes perpendiculares aos segmentos [z2,z3] e [z^zj. m m 1 b b 1 c c 1 Considere o ponto O como a origem do plano complexo e sejam m,a,b,cas coordenadas de M,A,B,C. Assim R2 = aã = bb = cc e p(M) = R2 - mm. (1) Sejam O^Oj.CX, os circuncentros dos triângulos BMC,CMA,AMB .respectivamente. É fácil ver que 01,02,03são os pontos médios dos segmentos MA",MB",MC", respectivamente, e portanto area[O,O2O3] area[A"B"C"] p(M)(c - b) m 1 b 1 c 1 A coordenada do circuncentro do triângulo com vértices de coordenadas z,,z2,z3 é dada pela seguinte fórmula (veja a fórmula(1) na Subseção 3.6.1): zizi(zz ~ z3) + z2z2(z3 - z,) + z3z3(z, - z2) z, z, 1 z2 z2 1 z3 z3 1 A coordenada de O, é mm(b - c) + bb(c - m) + cc(m - b) (R2 mm)(c - b) m m 1‘ b b 1 c c 1 152 Números Complexos de A a Z Seja A = = mA. cyc Já que zOl . Zo2 = . zo3 = , temos [OíOgOs] _ >3 o. (3) As relações (2) e (3) implicam (4) _ P2(M) A3 ã 1 b 1 e considere c 1 •-L-A aPr 1 1 1 1 1 1 1 a = — A m c a m a b m ã b a b c ac ~b~ c - a a Z°1 4 4 -O V'1_ K . -.2 X- aD= mA-m-0+ £a| b—------ —a [ = mA + R — - cyc ~ cyc ab C area[ area[ABC] 1_ p2(M) 1 A A2 aPy = P2 (M) 1 A2 aPy b - c b - c S-ã p - b ã - b y = p2(M) 1 A3 ' < zo2 Zo, (a-b)p(M) y-A ■p4 |c-a c-ã aPy | a - b ã - b m 1 c 1 ã 1 areafABC] |A2aPy| area[A"B"C"] = 4p2(M) 1 eY = - e consequentemente aa + pb + yc = m , já que claramente A * 0 . Perceba que a.p.y são números reais e a + p + y = 1, portanto a,p,y são as coordenadas baricêntricas do ponto M. _(c-b)p(M) _(c-a)p(M) -------- ■ °2 ’ PA 2o, *o2 4'A m m 1 b b 1 c c 1 R2 c Com essa notação, obtemos (aa + pb + yc) A = m(ab - ac) -£ m(ab - ac) + X a(bc - bc) cyc cyc 4 - n v’ fu*2 c 1534 | Majs sobre Números Complexos e Geometria Como a,p,y são as coordenada baricêntricas de M, segue que . P' = (5) Por outro lado, temos ,2 A2 (6) p(M) = R2 - OM2 = ^apAB2 . cyc Por outro lado, se a,p,y são as coordenadas baricêntricas do ponto M, então seu ponto isogonal M' tem coordenadas baricêntricas dadas por Portanto p(M’) = ^a’p'AB2 = cyc ZM ~ aZA + PZB + • Usando o produto real encontramos que = (?■?) =|rarea OM2 = zMzM =(azA+pzB+yzc)(azA+pzB+yzc) (o.2+p2 + Y2)R2 + 2Xap[R2-^ cyc \ aPYAB2BC2CA2 p2(M) aPyAB2 BC2 CA2 (PyBC2 + ayCA2 +apAB2)2 AB2BC2CA2 R2 = (a2+p2+Y2)R2+2£apzA zB =( cyc cyc = (a + p + y)2 R2 - ^apAB2 = R2-£aPAB2 . cyc cyc Portanto, a potência de M' em relação ao circuncirculo do triângulo ABC pode ser expressa na forma [ABC]| = A conclusão desejada segue das relações (4), (5) e (6). □ Aplicações. 1) Se M é o ortocentro H, então M’éo circuncentro O e area[ABC] R2 1 area[A"B"C"| " 4R2 ” 4 ' a. - PyBC2 p, .. y«ca2 PyBC2 + ayCA2 + apAB2 ’ PyBC2 + gyCA2 + apAB2 «PAB2 PyBC2 + ayCA2 + apAB2 AB2 2 154 Números Complexos oe A a Z 4.11 TEOREMA DE LAGRANGE E SUAS APLICAÇÕES 'n No caso em que m, =... = mn = 1, o ponto G é o barícentro do conjunto {A! An). ,1. 4 area[ABC] area[A"B"C"] = Aplicando a desigualdade de Euler R > 2r (Corolário 5 na Subseção 4.6.2) segue que areafABC] area[A"B"C") cc2+P2+y2 36R2 area[ABC] |(2R + r)2-s2| area[A"B"C"] ~ 2R2 3) Se M é o ponto de Lemome K, então M'éo baricentro G e area[ABC] |R2-OG2| area[A"B''C"]= 4R2 ' Aplicando a fórmula do Corolário 9, Subseção 4.6.4 e a primeira fórmula do Corolário 2. Subseção 4.6.1, segue que area[ABC] 2(s2-r2-4Rr) area[A"B"C'] 36R2 onde a,p,y são os lados do triângulo ABC. Da desigualdade a2 + p2 + y2 < 9R2 (Corolário 10, Subseção 4.6.4) obtemos area[ABC] 1 area[A"B"C"] S 4 ’ 4) Se M é o incentro I do triângulo ABC. então M’ = I e, usando a fórmula de Euler, OI2 = R2 - 2Rr (veja o Teorema 4 na Subseção 4.6.2), encontramos que |r2-OI2| 2Rr r 4R2 4R2 4R 2) Se Méo circuncentro O, então M'éo ortocentro H e obtemos areafABC]_|R2-OH2| area[A"B"C"] “ 4R2 Usando a fórmula do Teorema 8, Subseção 4.6.4, segue que Considere os pontos distintos A1(z1),...,An(zn) no plano complexo. Sejam m, m, números reais não nulos tais que m, + ... + mn * 0 . Seja m = m, + ... + mn . O ponto G de coordenadas zG = -^(it^z, +... + mnzn) é chamado de baricentro do conjunto {A, An)em relação aos pesos m, mn. 1554 | Mais sobre Números Complexos e Geometria (1) Usando as propriedades do produto real obtemos para todo j = 1 n a igualdade Multiplicando por m( e fazendo o somatório para j = 1 n segue que Prova. Sem perda de generalidade, podemos supor que o baricentro G é a origem do plano complexo, isto é, zG = 0. Quando n = 3 e os pontos A,, A2. A3 não sao colineares. obtemos as coordenadas bancêntricas absolutas de G em relação ao triângulo A, A2A3(veja na Subseção 4.7.1); "Joseph Louis Lagrange (1736 - 1813), Matemático francês, um dos maiores matemáticos do século XVIII. Ele contribuiu de modo importante em todos os ramos da matemática e seus resultados influenciaram positivamente a ciência moderna. MAf =|zM-Zj|2 =(zM-z|)(zM-zi)=|zM|2 MA? =Izm|2-2zm-zí+|zí|2- m2 m = 21 m = m|zM|2-2zM- ’ f-‘z3 n I I2 ?,m.zi 1-1 Teorema 1. (Lagrange”) Considere os pontos A, An e os reais não nulos m, mn tais que m = m1+... + mr?O. Se G denota o baricentro do conjunto {A, An} em relação aos pesos m, mn, então para qualquer ponto M do plano a seguinte relação ocorrei n Êm,MA^ = mMG2 +Zm,GA2 i“1 j-1 Zm,MA2 = zm, í|zM|2 - 2zM ■ Zj + |Zj|2) (mzG) + Zmj|z||' j=1 = m|zM-zG|2 + Zmi|zj-zG|2 i=1 -2zm Zm/i + \í’1 ) i2 -2zMz|+|zi|2,i.e„ = m|zM|2 + Zmi|zJ|2 1=1 m1 = 7T ’ Rz2 m £ = mMG2 + SmjGA2 . □ Í=1 = m|zM|2 156 Números Complexos oe A a Z (2) Se m, = ... = mn = 1, do Teorema 1 obtem-se: (3) Observação (4) Corolário 2. Considere os pontos distintos A, An e os reais não nulos m,,...,mn tais que m, + ... + mn »0, Para qualquer ponto M do plano, a seguinte desigualdade ocorre: Aplicação. Vamos usar a fórmula (3) na determinação de algumas distâncias importantes em um triângulo. Vamos considerar o triângulo ABC e tomemos n = 3 na fórmula (3). Encontramos que para qualquer ponto M no plano do triângulo ABC a seguinte igualdade ocorre: MA2 + MB2 + MC2 = 3MG2 + GA2 + GB2 + GC2 ”Gottfried Wilhelm Leibniz (1646-1716) foi um filósofo,matemático e lógico Alemão que é, provavelmente, mais conhecido por ter inventado o cálculo diferencial e integral independentemente de Sir Isaac Newton. com a igualdade ocorrendo se e somente se M = G, o baricentro do conjunto (A, An} em relação aos pesos m, mn. Corolário 3. (Leibniz”) Considere os pontos distintos A, An e o baricentro G do conjunto {A,,. ,,An}. Para qualquer ponto M do plano, a seguinte igualdade ocorre : £MA2 = nMG2 +£GA2. j=1 j=1 fjrnjMA2 > ZmjGA? , Prova. A desigualdade (2) segue diretamente da relação de Lagrange (1). □ A igualdade (3) é equivalente â seguinte identidade: Para quaisquer complexos Z'Z’.... zn temos = n|z- Z’+ n' + 2-’| + z|z, . 1574 | Mais sobre Números Complexos e Geometria (6) 3) Considere em (4) M = I, incentro do triângulo ABC. Obtemos IA2 +IB2 +IC2 = 3IG2 +i(a2 + P2 +y2). Uma fórmula equivalente da distância OH foi obtida em termo das invanantes fundamentais do triângulo no Teorema 8, Subseção 4.6 4. IC = -±C' sen 2 2) Usando a colinearidade dos pontos O,G,H e a relação OH = 3OG (veja o Teorema 3.1 na Subseção 4.5) segue que OH2 = 9OG2 = 9R2 - (a2 + P2 + y2) onde G é o baricentro do triângulo ABC. Suponha que o circuncentro O do triângulo ABC é a origem do plano complexo. 1) Na relação (4) tomamos M = O e obtemos 3R2 = 3OG2 + GA2 + GB2 + GC2 . Aplicando a conhecida fórmula da mediana segue que Figura 4.15. Por outro lado, temos as seguintes relações: IA = ^Ã' ib = ^b' sen— sen — 2 2 GA2+GB2+GC2 =-^(rn2+m2+m2) cyc onde a,p,y são os lados do triângulo ABC. Encontramos OG2 =R2-l(a2+p2+y2) (5) Uma fórmula equivalente da distância OG é dada em termos das invariantes básicas do triângulo no Corolário 9, Subseção 4.6.4. Números Complexos de A a Z158 onde r é o inraio do triângulo ABC. Segue que 4(«2+P2+72) .r2 obtemos [s^|3y-3oPy] -16Rr), (7) Problema 1. |z3-Zt| = p.A coordenada de G é igual a Solução. Sejam A,B,C os lugares geométricos dos números complexos z,,z2,z3 e seja G o baricentro do triângulo ABC. z cyc =z cyc IG2 =^(s2+5r2-16Rr), consequentemente obtemos novamente a fórmula da Aplicação 1), Subseção 4.7.2. (s-P)(s-y) Py z1+z2+z3 3 ig2=- 3 ■ e |Zt-z2| = r. |z2-z3| = <x . 1 sen2^- = AXP7(s-“) = 4| K cyc K = -^-[s(s2 +r2 + 4Rr)-12sRrJ = -y(s2 +rz - onde usamos a fórmula da Subseção 4.6.1. Portanto IG2 =^s2+r2-8Rr-i(a2 + p2 + y2)] = —fs2+r2-8Rr(s2-r2-4Rr)"| = —(s2+5r2 3[ 3' 'J 9V onde a primeira fórmula do Corolário 2 foi usada. Isto é, ? A Levando em consideração a conhecida fórmula sen — = Pr(s-<x) (s-a)(s-p)(s-y) Sejam z1,z2,z3 números complexos distintos de módulo R. Prove que 9R2-|z1+z2+z3|2 73 |z,-z2||z2-z3| |z3-z,| R 1 1 , 1 2 A 2 B 2 senz — sem — senz —2 2 2) _y Py ^(s-P)(s-y) 1594 | Mais sobre Números Complexos e Geometria A desigualdade se torna (D R K = 7s(s-a)(s-p)(s-y) < (8) lzk|2) n N2) j-1 Aqui está a prova dessa famosa desigualdade, usando a fórmula de Herão e desigualdade das médias: Prova, Vamos supor que o baricentro G é a origem do plano complexo. Usando as propriedades do produto real, temos MAf =|zM-Zj|2 =(zM-zj)(zM-zi) = |zM|2 e AiAj = h - zk |2 = |zí |2 - 2z, ■ zk + |zk |2 , coordenada do ponto Ajt j = 1,2 n. 1si<ksn onde o número complexo zJ é a A relação (8) é equivalente à n2|zM|2 = nZ0zM|2 n . .2 nIstoé, nX|zj| =2n£zM Zj l-t j=i S (N2-2Zi-zk ick<n ' = S2 = (« + P + ■/)' =__________ 3v3 12n/3 ' 12^3 4>Í3 Agora vamos estender a relação de Leibniz do Corolário 3. Primeiramente, precisamos do seguinte resultado. Teorema 4. Seja n > 2 um inteiro positivo. Considere os pontos A, An distintos e seja G o baricentro do conjunto {A, A„]. Então, para qualquer ponto do plano, a seguinte fórmula é válida: n2MG2 = n£MA2 - £ A,Ak . j=1 1<i<k<n -2zMZj -2zm z,-|z,|2 / (s-a + s-P + s-y) I s3 Vs 27 V 27 3(a24.p2+y2) a2^ft2+72 9R2 -9OG2 73 aPy R Usando a fórmula OG2 = R2 -^(a2 +p2 + y2), (1) é equivalente à az+pZ+yz>2Pnã = ^S^3=4K73. 160 Números Complexos de A a Z = n(zM zc) = n(zM 0) = 0 . j-1 Observação Teorema 5. Para qualquer ponto M do plano, S MG2,k (9) A última relação pode ser obtida como mostrado a seguir: n k=1 Portanto, a relação (8) está provada. □ 1 í n W 0 = |zgI =ZqZG =~2 n l i=i n (n-k)^XMA2+n2(k 1)fnjMG2 = kn(n-1) 1 W 1SÍ,<. ,i^sn A fórmula (8) é equivalente à identidade seguinte: Para quaisquer complexos z,z„...,zn, temos ^Z|z-Zj|2-lz-Zl+"' + Zn| =1 £ |z|-zk|2. n j=i | n | n iíj<kán Levando em consideração a hipótese de que G é a origem do plano complexo, temos £zM Zj = zM- y.zl j=i Vi’1 Assim, a relação (8) è equivalente a E|zJ2 = + 2 Z z, zk 1ái<kán J -2 Z zrzk. 1si<ksn Se n=3, obtemos a fórmula (5). 2) Para qualquer ponto M do plano, a seguinte identidade ocorre: ZMAfU £ A, A2, j=1 riiskksn com a igualdade ocorrendo se e somente se M = G , o baricentro do conjunto {A,....An}. Seja n > 2 um inteiro positivo e seja k um inteiro tal que 2sk$n. Considere os pontos distintos A, An e seja G o baricentro do conjunto {A, A„}. Para os Índices i, <... <ik vamos denotar por G^ como sendo o baricentro do conjunto {A^ A,k | Temos o seguinte resultado: Aplicações. 1) Se A,,...,An são pontos sobre a circunferência de centro O e raio R, então, fazendo M = O em (8), segue que A, A2 = n2 (R2 - OG2) 1614 | Mais sobre Números Complexos e Geometria :1<i, (10) S GG2 (11) (12) (13)S E e (14) (15) (16) Das relações (16) e (13) obtemos (17) (18) ZG, S=1 1Sij <•••<!(( <n s=1 : E l£ij <—<*k E M. 1<i<ksn ÈGAi=k . j=1 lsi,<...<ihán < •■• < ik S n| é G. Aplicando ir ikAis kisp?qskAiPAi4 ' = MmG2+ ) cfcsn Considerando em (12) M = G e adicionando todasessas relações anteriores, segue que ■ E L 1sq<-<i|(Sns-1 1<i,<n1<p<q<n GG2 • +-'1-'k k 1SÍ1 <••• <i^ sn EMA2 =kMG2..k + f G,,.a EGA2 =k k2MG2,k £MA? = nMG2 + EGA? , A2A's S. MG2..ik -An] Prova. É fácil ver que o baricentro do conjunto |Gir a relação de Leibniz obtem-se Se rearranjarmos os temos da equação (17), obtemos (lkA?=k y GG. Jfí fn) 1 . ... . k í'n'| LJ l“1 1Si,<...«.sn M Aplicando a equação (8) no Teorema 5 para os conjuntos {A. Aik j , respectivamente, obtemos n2MG2=n£MA2~ £ A^J j=1 1<i<k<n = k£MA2- £ A, A2 s=1 1<p<q<k p M Tomando M = G, j em (15), segue que E SGA2=k S GG2 1ál-|< -Jfcánssl 1áij<- Números Complexos de A a Z162 Observação (n-k)l 2 Z - 2) Se k = 3 , de (9) obtemos que para qualquer ponto M no plano, a relação '■1'2'3 4.12 CENTRO DE EULER DE UM POLÍGONO INSCRITÍVEL ZE = n z é chamado de centro de Euler do polígono A,A2 An. Quando n = 3 é fácil ver que E = O9 , ou seja, o centro de Euler do circulo dos nove pontos. Considere um polígono A, An inscrito em uma circunferência centrada na origem de um plano complexo e sejam a1,a2 an as coordenadas dos seus vértices. Pela definição, o ponto E de coordenada Aplicações. 1) Quando k = 2, de (9) obtemos que para qualquer ponto M no plano, a seguinte relação ocorre: A relação (9) é equivalente á seguinte identidade: Para quaisquer complexos z,z,....,zn temos ' E mghZ-1 1,12 lsi,«2sn zi1+-+\ k ai +a2 + ... + an 2 n (n - 2)^MAj2 + n2MG2 =4 j-1 >2 Zi+...-FZn n Neste caso, é o ponto médio do segmento [a^A^J . (n - 3)(n - 2)£MA2 + 2n2 (n - 2)MG2 = 18 £ MG2 j=1 1Si-j<Í2<>3Sn ocorre. O ponto Gill2Í3 será o baricentro do triângulo A^A^A^ . Das relações (10).(11),(14) e (18) rapidamente encontramos a equação (9). □ = kn(n-1) £ {k)i?,lz-z'l2+n2(k-1)[k)lz- 1634 | Majs sobre Números Complexos e Geometria Observação (1) n2MG2 para M = E e M = O , obtemos n2 EG2 (2) (3) (4) = (n - 2)2 OE2 - 4OE2 +n2R2 = n(n-4) EO2 -n2R2 n-2 n b) Para n = 4 . o ponto E é também chamado de ponto de Mathot de um quadrilátero inscritivel A,A2A3A4. Proposição. Na notação acima, a seguinte relação ocorre: n Definindo s = ^a,, temos que i=1 £eA|2 = nR2 +(n-4)EO2. i=< Prova. Usando a igualdade (8) do Theorema 4, Seção 2.17, = nÊMA?- X A, A2 1=1 Isicjsn EG = |zE - zG| = |- nzG zH nOG OH E 2 2 2 2 Lembre que o ortocentro do polígono A, • A2 • • • An é o ponto H de coordenadas zH = a-i +a2 + -- + an. = n£EA?- £ A, A2, i=1 1<i<j<n e n2OG2=nR2- £ A,A2 lsi<jsn H-S Das relações (2), (3) e (4) encontramos que nfjEA2 = n2 -EG2 -n2 OG2 +n2R2 í=i a) Seja G(zg) e H(zh) o baricentro e o ortocentro de um polígono inscritivel A, A2 ■ ■■ A„ . Então ^OE. n 164 Números Complexos de A a Z ou, equivalentemente, (5) (6) (7) com a igualdade ocorrendo se e somente se o triângulo é equilátero. 2) Para n = 4 obtemos a interessante relação (8) 3) Para n > 4 , da relação (1) segue a desigualdade (9) ZROgA, (10) Usando a fórmula do Corolário 9.2, Subseção 4.6.4, podemos expressar o lado direito de (5) em termos das invariantes fundamentais do triângulo A,A2A3 : ^EA2 > nRz i=1 A igualdade é válida apenas no polígono A,A2...An com a propriedade E = O . Isso é equivalente a OgA^ + OgA2 + OgAg = 3R . 4) A desigualdade de Cauchy-Schwarz e a desigualdade (7) dão z 3 \2 3 ’ <(3R2)£O9A,2 < 9R2 i=1 OgA? +O9A|+O9A| =|R2-^r2-2Rr + |s2. Da expressão (5), segue que para qualquer triângulo A,A2A3 a seguinte desigualdade é válida: OgA? + O9A| + O9A2 < 3R2 , EEA2 = 4R2 . 1=1 O ponto E é o único ponto no plano do quadrilátero A,A2A3A4 satisfazendo a relação (8). Aplicações. 1) Para n 3 . da relação (1) obtemos O9A2 + O9A2 4- OgA2 = 3R2 - OO| EEA2 = nR2 + (n-4)EO2. como queríamos provar. □ 1654 | Mais sobre Números Complexos e Geometria (11) 6) Usando a relação 2EAj = 2|e - a,| 4.13 ALGUMAS TRANSFORMAÇÕES GEOMÉTRICAS NO PLANO COMPLEXO 4.13.1 Translação Seja z0 um número complexo fixado e seja tZQ o mapeamento definido por tZQ :£->£, t20(z) = z + z0. M'(t2()(z)) M(z) M.(z..) O Figura 4.16. ZEA, <4R. i=1 É óbvio que a composição de duas translações t2l e t22 satisfazem a relação IZ1 ° *z2 = 'z1+22 ■ = 2||-a.| = ls-ai|. as desigualdades (4), (5) tornam-se ^|-a, + a2 + a3| < 6R cyc e, respectivamente, |-a1 + a2 + a3 + a41 < 8R . cyc As desigualdades acima são válidas para todos os números complexos de mesmo módulo R. Levando em consideração a interpretação geométrica da adição de dois números complexos (veja Subseção 1.2.3). temos a Fig. 4.16. dando a representação geométrica de t^(z). Na Fig. 4.16 OM0M'M é um paralelogramo e OM1 é uma de suas diagonais. Portanto, o mapeamento t20 corresponde no plano complexo C á translação tOMo do vetor OM0 no caso de um plano Euclidiano. 4R2 ^EAi=16R4 H1 5) Usando a mesma desigualdade e a relação (8), obtemos f 4 \2 4 r£eaJ < \ i=t ) ou, equivalentemente, O mapeamento t2() é chamado de translação do plano complexo em relação ao número complexo z0 . Números Complexos de A a Z166 4.13.2 Reflexão em relação ao eixo real M(z) M'(s(z)) O mapeamento s0 é chamado de reflexão na origem. Se z0,z, sZQ(z) são as coordenadas dos pontos M0,M,M', então Mo é o ponto médio do segmento [MM'j, portanto M' é a reflexão de M por Mo (Fig. 4.19). Considere o mapeamento s0 : C —> C, s0(z) = -z . Como s0(z) + z = 0 , a origem O é o ponto médio do segmento [M(z)M'(z)j. Portanto, M' é a reflexão do ponto M pelo ponto O (Fig. 4.18). Também é óbvio que o conjunto T de todas as translações de um plano complexo é um grupo com respeito às composições de mapeamentos. O grupo (T,-) é Abeliano e sua unidade é to = 1c , a translação do número complexo 0. Figura 4.17. 4.13.3 Reflexão em relação a um ponto Considere um número complexo z0 fixado e o mapeamento s^iC—><C, szo(z) = 2zo-z- Considere o mapeamento s : C -> C, s(z) = z . Se M é o ponto com coordenada z, então o ponto M'(s(z)) é obtido refletindo M através do eixo real (veja Fig. 4.17). O mapeamento s é chamado de reflexão no eixo real. É claro que s » s = 1c. O mapeamento é chamado de reflexão em relação ao ponto M0(z0). É óbvio que a seguinte relação é válida: sZQ = sZ£| = 1C . 1674 | Mais sobre Números Complexos e Geometria O M‘(—z) Figura 4.18. M’(sr(z)) Mc(z..) M(z) O Figura 4.19. 4.13.4 Rotação ra (z) = az = p[cos(t -t-10) + i sen(t -s-10)]. 4.13.5 Transformações isométricas no plano complexo Seja a = cos t0 + i sen t0 um número complexo de módulo 1 e seja ra o mapeamento dado por ra :C -> C, ra(z) = az . Se z = pfcost +i sen t), então portanto M'(ra(z)) é obtido rotacionando o ponto M(z) . em torno da origem, de um ângulo t0 (Fig. 4.20). O mapeamento ra é chamado de rotação com centro O e ângulo t0 = arg a . Um mapeamento f: C —> C é chamado de isometha se preserva a distância, i.e.. para todo z„ z2 e C, |f(z,) - f(z2)| = |z, - z2|. Números Complexo de A a Z168 M’(Ya(z)) O Figura 4.20. Para a reflexão s pelo eixo real obtemos |s(z,)-s(z2)| = |z,-z2| = |z, Problema. Teorema 1. Translações, reflexões (pelo eixo real ou por um ponto) e rotações em torno do centro O são isometrias no plano complexo. Solução. Considere o plano complexo com origem no circuncentro e denote por letras minúsculas as coordenadas dos pontos denotados pelas respectivas letras maiusculas. Seja A,A2A3A4 um quadrilátero cíclico inscrito em uma circunferência de centro O e sejam H,, H2, H3, H4 os ortocentros dos triângulos A2A3A4. A,A3A4, A,A2A4, A,A2A3 , respectivamente. Prove que os quadriláteros A,A2A3A4 e H,H2H3H4 são congruentes. (Olimpíada Balcânica de Matemática, 1984) Podemos facilmente checar que a composição de duas isometrias também é uma isometria. O conjunto Iz0 (C) de todas as isometrias do plano complexo é um grupo em relação á composição dos mapeamentos e (T,<-) é um subgrupo dele. Prova Para a translação t2Q temos |tzo(Zl)-tzo(Z2)|=KZ1+2o)-(Z2+Zo)| = |Zl-Z2| -z2| = |z, -z2|, e o mesmo vale para a reflexão por umponto. Finalmente, se ra é uma rotação, então Ira(Z1) - ra(z2)| = Iazi -azz| = |a||zi - zz| = |Z1 - zz| ■ já que |a| = 1. 1694 | Majs sobre Números Complexos e Geometria 4.13 ALGUMAS TRANSFORMAÇÕES GEOMÉTRICAS NO PLANO COMPLEXO Teorema 2. Qualquer isometria é um mapeamento f :C->C com /(z) = az->-b ou /(z) = az + b, onde a, b e C e |a| = 1 Prova. Sejam b = /(0),c = /(1) e a = c-b. Então |a| = |c - b| = |/(1) -/(0)| = |1 - 0| = 1. 4.13.6 Teorema de Morley Em 1899, Frank Morley, então professor de matemática em Haverford College, chegou a um resultado tão surpreendente que entrou no folclore matemático com o nome “Milagre de Morley". O maravilhoso teorema de Morley enuncia que: Os três pontos de interseção das trissetrizes adjacentes dos ângulos de qualquer triângulo formam um triângulo equilátero. 0 teorema foi erroneamente atribuído a Napoleão Bonaparte, que fez algumas contribuição para a geometria. Existem várias provas desse bom resultado: as provas de J. Conway, D.J. Newman, L. Bankoff e a de N. Dergiades. Aqui apresentamos a nova prova publicada em 1998, por Alain Connes. Sua demonstração é derivada do seguinte resultado: Se s = a, + a2 + a3 + a4 , então h j = a2+a3+a4 ~ s — a^,h2 —s — a2, ^3 = ® , ^4 = ® a4 ■ Portanto o quadrilátero H,H2H3H4 é a reflexão do quadrilátero A,A2A3A4 pelo Considere a transformação g : C -> C, dada por g(z) = az + b. Não é difícil provar que g é uma isometria, com g(0) = b = /(O) e g(1) = a + b = c = /(1). Portanto, A = g“1»f è uma isometria, com 0 e 1 como pontos fixos. Por definição, segue que qualquer número real é um ponto fixo de A , assim h = 1c ou h = s, a reflexão no eixo real. Logo, g = f ou g = f-s e a prova está completa. O resultado acima mostra que qualquer isometria do plano complexo é a composição de uma rotação e de uma translação, ou a composição de uma rotação com a reflexão pela origem O e uma translação. s ponto de coordenada —. O resultado a seguir descreve todas as isometrias do plano complexo. 170 Números Complexo de A A Z Prova Note que (Zi°Z2)(z) = aia2z + aib2 + b1, a,a2 * 1, (Z2 « f3 )(z) = a2a3z + a2b3 + b2, a2a3*1, (Z3 3 /i)(z) = a3a,z + a3b, + b3, a3a,* 1. Fix(Z^Z2) Fíx(/2-/3) Fix(Z‘Z>) portanto (Z,3 - Z23 “ Z33)(z) = a?a2a3z + a?a^b3(a^ + a3 +1) + a?b2(a^ + a2 +1) + b,(af + a, +1). Teorema 3. (Alain Connes) Considere as transformações de um plano complexo f: C —> C, /[(z) = a^z + b|, i = 1.2,3 , onde todos os coeficientes a, são diferentes de zero. Assuma que as transformações f2,.f2^ f3. f3'-f\ e ,f^f2»f3 não são translações, i.e., equivalentemente, a,a2,a2a3,a3ava,a2a3 eC\{1). Então as seguintes sentenças são equivalentes: (1) (2) j3 = 1 e a + jp + j2y = 0 , onde j = a,a2a3 * 1 e a, p, y são os únicos pontos fixos das transformações /," f2, f2 ^f3, f3 f, respectivamente 4 Logo /,3 <• Z23 ' Z33 = ide se e somente se a3a2a3 = 1 e afa2b3(a3 + a3 +1) + a,b2(a2 + a2 +1) + b,(a2 + a, +1) = 0 . í 3^2 + b, 1 J > ~í 1-a,a2 J [ = í a2b3 + b2 ] í l 1-a2a3 J [ = [a3b,+b3l _ [ l 1-a3a, j l onde Fix(/) denota o conjunto dos pontos fixos da transformação / . Para os cubos de Zi.Z2.Z3 temos as fórmulas Z13(z) = a?z+b,(a?+a1+1). Z23(z) = a2z + b2(a2 +a2 +1), /^(z) = a3+b3(a| + a3 +1), aia3^2 +a3^- a3 “ í 3-|32^3 + ai^2 l al~j a2a3b1 + a2b3 a2 ” J 4 | Mais sobre Números Complexos e Geometria 171 1 a2-j a,a2b3j a,a2b2j a,a2b3 1 ■> 1 = (ai-j)(a2-j)(a -j) <aib2 j - b, - a, a3b2j - 3,33b, j - a,a2a2b, - b3j +a,a2b3j - b2j2 + a2a3b, + a,a3b2j2 + a3b, j2 + a2a^b,j + a2a3b3j - a2b3 j2 +a2bj2 + b2j2 + bj - a,a3b2j2 - a3b,j2 + a2a3b,j2 + a2a3bj2 +a, a2a2b, + a,a2a o, + a2a■:b, + a2a3b3 + a2a2a3b3 + a3a2a3b2 + a2b3) Va,-j)(a21-j)(a3-i)(-aib2j2 ~ -a2a3bi ~ a2a3b3 “ ala2^3J2 ” aia2^2^ ~ a2&3) +8-j 8283b-] + 3-]32a3bi + 8283b-] + ^2a3b3 a?a2a3b3 + 8^828302 + a2^3) = ’ ^-j)(a2-j)(a3-j)[a2a2b,(1 + a' + a?) + a’ a2a'b2(1 + a2 + a2) +a2b3(1 + a3+ a3 +a?a^)] +a, b2( 1 + a 2 + a2)i-b,(1 + a, + a2) ] Para provar a equivalência das sentenças (1) e (2) temos que mostrar que a + jp + j2y é diferente do termo livre de /,3 » /23 =>/33 por uma constante multiplicativa. De fato, usando a relação j3 = 1 e a implicação j2 + j +1 = 0 , obtemos com sucesso: a + jp + j27 = a + jP + (-1-j)y = a-y + j(p-y) _ aiasb2 a3b-j a2a3^i + ^2^3 j j| aia2bs al^2 a23jb-| + 82^3 a3-j a2-j \ a,-j a2-j J a,a2a3b2 + a2a3b, - a,a3b2j - a3b,j — a2a3b, - a2a3b3 + a2a3b,j + a2b3j (a2-j)(a3-j) ,:a,a2b3 + a,a2b2 - a,a2b3j - a,b2j - a,a2a3b, - a,a2b3 4- a2a3b,j + a2b3j (a,-j)(a2-j) ^2l~a2a3b,j2 - a,a3b2j -a3b,j-a2a3b, -a2a3b3 +a2b3j a3 _ i - a,b2j2 - b,j2 + a2a3b,j2 + a2b3j2 'l ai-j ) Números Complexo de A a Z172 C B’(P)C‘(Y} A(a) BA' Figura 4.21. Prova. (Alain Connes) Consideremos as rotações = rA 2x, f2 = rB 2y, f3 =rc.2z de Figura 4.23. Teorema 4. (Morley) Três pontos A'(a),B'(P).C'(y) das trissetrizes adjacentes dos ângulos de qualquer triângulo ABC formam um triângulo equilàtero. A" = G(A) Figura 4.22. 1 - 1 - 1 -centros A. B, C e de ângulos x = — A, y = — B, z =—C (Figura 4.21), Note que Fix(/, o/2) = {A'),Fix(/2 °/3) = {B'),Fix(/3 <•/,) = {C’J (veja Figura 4.22). 1734 | Mais sobre Números Complexos e Geometria 4.13.7 Homotetia O ponto M' é chamado de ponto homotético de M com centro O e magnitude k . M(z) M'(h.(z) k>0O Figura 4.24. Para provar que o triângulo A'B'C é equilátero é suficiente mostrar, pela Proposição 2 na Seção 2.4 e o Teorema 3 acima, que /,3 = /2 =• /33 = 1c. A composição sAC '>óab de reflexões jac e sAB pelas retas AC e AB é uma rotação pelo centro A de um ângulo 6x. Logo = sAC osAB e analogamente /23 = •'ba 0■sbc e fz = 'cs0 'ca ■ Segue que Dado um número real fixo não nulo k , a transformação hk :C —> C,hk(z) = kz , é chamada homotetia do plano complexo com centro O e magnitude k . Figuras 4.24 e 4.25 mostram a posição do ponto M'(hh(z)) nos casos k > 0 e k < 0. Os pontos M(z) e M'(hk(z)) são colineares com o centro O, que se encontra no segmento de reta MM' se e somente se k < 0. Além disso, a seguinte relação é válida: |OM'|=|k||OM|. " fz ' fz ~ sm: " s:-~ ' 5ba " sbc " scb " 5ca - ^c • 174 Números Complexo de A a Z O k < 0 M’(h.(z)) Figura 4.25. Se m, m,. m2, m3 são as coordenadas dos pontos M, M,. M2, M3 , temos Seja g a coordenada do baricentro do triângulo M3M2M3 . Então Problema. Seja M um ponto interior ao triângulo equilátero ABC e sejam M,. M2, M3 os pés das perpendiculares saindo de M para os lados BC, CA, AB, respectivamente. Ache o lugar geométrico do baricentro do triângulo M,M2M3 . Solução. Sejam 1,c,e2 as coordenadas dos pontos A,B,C, onde e = cos120°+i sen 120°. Lembre que e2 + e +1 = 0 É óbvio que a composição de duas homotetias Ak) e Ak também é homotética, isto é \oAk2 =h^2- e e3=1. e2)) = y. mi =-^(1 + e + m-em), m2 = —(e + e2 + m -m), m3 = (e2 +1 + m - e2m). 1 1 , g = — (m, +m2 + m3) = —(2(1 + c + e ) + 3m-m(1 + e + 3 6 O conjunto de todas as homotetias do plano complexo é um grupo Abeliano em relação á composição de transformações. A identidade do grupo (H,°) é A, = 1c, a homoteticidade de magnitude 1. 1755 | Problemas Importantes para Olimpíadas O lugar geométrico de G é o interior do triângulo obtido de ABC sob uma homotetia coordenadas 4.13.8 Problemas C, g(z) = -iz +1 + 2i é uma isometria. Analise 1 1 2'2 1. Prove que a composição de duas isometrias do plano complexo é uma isometria. 2. Uma isometria do plano complexo tem dois pontos fixados A e B . Prove que qualquer ponto M da reta AB é um ponto fixado da transformação. 3. Prove que qualquer isometria do plano complexo é uma composição de uma rotação com uma translação e possivelmente também com a reflexão no eixo real. 4. Prove que a transformação f:C->C,/(z) = iz + 4-i é uma isometria. Analise f como no problema 3. 5. Prove que a transformação g : C g como no problema 4. -s,ls2. 2 2 1 de centro O e magnitude — . Em outras palavras, os vértices desse triângulo têm 1 portanto OG = — OM . 176 NúmerosComplexo de A A Z 5.1 PROBLEMAS ENVOLVENDO MÓDULOS E CONJUGADOS Problema 1. Sejam z,. z2, z3 números complexos tais que |z,| = |z2| = |z3| = r > 0 e z, + z2 + z3 # 0. Prove que = r. Solução. Observe que z,.z, = z2.z2 = z3.z3 = r2 Então r2 r2 r2 r2 r2 r2 = r2 como queríamos provar. Problema 2. Sejam z,, z2 números complexos tais que |z,| = |z2| = r > 0 . Prove que z2 z3 ’ z2 z,z2 *t~ Z2z3 + z3z, z,z2 + z2z3 + z3z, z, + z2 + z3 z, + z2 + z3 z3 z, r^_ Z3 z,z2-r z2z3-r z3z1 z,+z2+z3 Zl-Z; ■ZiZ2 ^^Rroblemas-Impor-tantes-para-Olimpíadas------------------------- O uso de números complexos é útil para resolver problemas de Olimpíadas. Em várias situações, um problema bem complicado pode ser resolvido inesperadamente empregando números complexos. Apesar de que os métodos da geometria Euclidiana, geometria de coordenadas, álgebra vetorial e números complexos pareçam semelhantes, em muitas situações o uso do último tem múltiplas vantagens. Esse capitulo ilustra alguns tipos de problemas importantes para Olimpíadas, onde o método de números complexos funciona eficientemente. z,z2 + Z2Z3 + Z3Z, Z, Z2 Z, + Z2 + z3 ■ r^ z, Jz,z2 >2 , Zl+Z2 Z, r2+z,z2J v 2 ,2 1I Z2Z3 + Z3Z, I2 2, + Z2 + Z3 | 1775 | Problemas Importantes para OumpIadas r2 - z,z2 Definindo z, = r(cos2x+i sen 2x) e z2 =r(cos2y +i sen 2y) Similarmente, Logo como dito. Problema 3. Sejam z,, z2, z3 números complexos tais que |z,| = |z2| = |z31 = 1 Definindo z = z, + z2 + z3 , obtém-se z3 - 3z(z,z2 + z2z3 + z3z,) = -4z,z2z3 . Isso é equivalente a -4 I^-ZjZj , >cos2(x-y) 1 z3 1 Z2 r(z<+z2) r2+z,z2 r2(cos2x + isen2x + cos2y + isen2y) r2(l + cos(2x + 2y) + isen(2x + 2y)) cos(x - y) cos(x + y). obtemos r(z, + z2) r2 + z,z2 2 2*1. 2 z , rfo-Zg) R í 1z =z1z2z3 3zl— + r(zi-z2) sen(y-x) r2-z,z2 sen(y + x) -4z,z2z3 = z’ + z2 + z2 - 3z,z2z3 = (z, + z2 + z3)(zf + z2 + z2 - z,z2 - z2z3 - z3z,). Solução. A desigualdade desejada é equivalente a , x2 / \ I r(z3 -fz2) j ! í r(z,-z2) j lr2 + z,z2J (r2-z,z2J e_± + 3L + ^L + i = 0. Z2Z3 Z3Z3 z3z2 Prove que |z, + z2 + z3| e {1, 2}. Solução. A igualdade dada pode ser escrita como z’ +z2 +z3 'r7-yL2L'i 0 ou l cos2(x-y) sen2(x-y) J cos2(x + y) sen2(x+y) + sen2(x-y) =1 178 Nümeros Complexo de A A Z -z?z^ IZ1 + z2 + z3|2 = (z, + z2 + z3 )í -!- + -?- Por outro lado, da relação dada segue que z3 = -z,z2z3, resultando z3 = -z,z2 . Temos Suponha que z? + z2=0. Então z,+z2=0 ou z2-z,z2+z2=0 implicando z2 + z| = -2z,z2 ou z2 +z2 = z,z2. = z2z2z3 (z? + zl + z2 )(z? + zl + z^) - z2z2z3 = Z1Z2Z3“Z1Z2Z3 =°- (u + v)(v + w)(w + u) = (u + V + w)(uv + vw + wu) - uvw w = z3, segue que A última relação pode ser escrita como z’ = zizaz3[3z(z,+ z2+ z3)-4], i.e„ z3 = z,z2z3(3|z|2 - 4) para u = z?.v = 4 (z? + z32)(z32 + z3)(z3 4- z3) = (z? + z2 + z3)(z3z2 + z2z3 + z33z3) - z3z32z33 = Z?Z2Z3 (Z? + Z2 + z3 + p-+ p- Solução alternativa. Não é dificil ver que |z, + z| + z3| = 1. Usando a identidade algébrica Tomando os módulos dos dois lados obtém-se |z|3 = |3|z|2 -4|. Se |z| £ , então )z|3 - 3|z|2 +4 = 0, implicando |z| = 2. Se |z| < , então |z|3 + 3|z|2 -4 = 0, dando v3 |z| = 1, como queríamos provar. |Z, + Z2 + Z3 f = (z, + Z2 + Z3 )í -L + -1 + -L I k Z1 Z2 z3 ) = 3+ f— + —1 + f— + —- I + f — + Z3 1 lZ2 ZJ lZ3 Z2> lZ3 ZJ = 3 + Z1 +ZZ + Z3+Z1Z2 + Z3 + Z1Z2 = 3 + z? +z2 . Z,Z2 Z2Z3 Z3Z, Z,Z2 Isso resulta em |z,+z2+z3|2 =1 se z2+z2=-2z,z2 e |z,+z2+z3|2 = 4 se z2+z2 = z,z2 , de onde segue a conclusão. 1795 | Problemas Importantes para OumpIadas Problema 5. £ 2n. ,2n*l| .2k n z2n. .2n+1 ,2nr2 .2n,3| è 2n + 2 . :2n+1| Solução. Temos |z, + 1| + |z2+1| + |z,z2+1|>|z1+1| + |z,z2+1-(z2+1)| = |z1+1| + |z,z2-z2| sIzi+i| + Iz2|Iz1-i| = Izi+i| + |z<-i|s|z1 + i + z1-i| = 2|z,| = 2, como afirmado. n|l + z| + |l + z2| + |l + z3| Devemos provar que (n +1)|1 + z| + |l + z2| +... + |l + z l*H: Problema 4. Sejam z,,z2 números complexos com |z,| = |z21 = 1. Prove que |z,+1| + |z2+1| + |z,z2+1| > 2 Solução alternativa. Usamos indução em n. Para n = 1, devemos provar que |1 + z| + |l + z2| + |l + z3| a 2. De fato, 2 = |l + z + z3+1-z(l + z2)|s|l + z| + |z3 +l| + |z||l + z2| = |l + z| + |l + z2| + |l + z3|. Suponha que a desigualdade é válida para algum n, portanto |l+zta :”l) Seja n > 0 um inteiro e seja z um número complexo tal que |z| = 1 Prove que |l + z2n| + |l + z2n’1; k=T = z(|1-z: k=1vl como afirmado. z2k|z —z2k+1 + 1 + z2k| = 2n, n|l + z| + |l + z2| + |l + z3| Solução. Temos n|l + z|+|l + z2| + |l + z3| + ...+|' = s(|l + z| + |l + z2k+1| n ;2"Hn 'HH2kl l=W- IHI1-z2k '|+s|i + z: 1 k=r :2k| + |z + z2k Números Complexo oe A a Z180 ,2n+3 como pedido. (1) Por outro lado, s2 e considere a equação cúbica z3 - s,z2 + s2z - s3 = 0 1 2 9 S2 2/.2 3 Das relações (1) e (2) seguem que s2 = — s, = 2X e s3 = = = —— = X . 2 s, 2X A equação de raizes z,,z2,z3 se torna z3-2Xz2+ 2X2z-X3 = 0 . Que é equivalente a (z - X)(z2 - Xz + ,2n+3| com raizes z,,z2,z3. Como z2 + z2 + z3 = 0 , temos s2 = 2s2. Usando a hipótese indutiva, temos (n +1)|1 + z| + |l + z2| +... +|l + z: |l + z2n+3| =2n + |l (l + z2n+2) + 1 + z: S 2n + |l + z| + |l + z2n+2|+|" Z2n + |l + z-z X2) = 0. 1 73As raizes são X, Xs,-Xe2 , onde s = — + . Sem perda de generalidade, supomos que z, = X,z2 = Xe,z3 =-Xs2. Usando as relações s2 - s +1 = 0 e e3 = -1, segue que Problema 6. Sejam z,,z2,z3 números complexos tais que lzi| = |z2| = lz3| = 1; zi+z2+z3 *0; z?+z2+z3 = °- Prove que para todo inteiro n > 2, |z" +z2 +zj| e {0,1,2,3). Solução. Seja S1 = 21 +z2 +Z3‘ :2"+2| + |l + z2"*3| I + z| + |z||l + z2n+2| + |l + z" 2n + 2, = S3P- + + -J-] = s3(z, + z2 + z3) = s3.s,. (2) kZ1 Z2 Z3 ) As relações (1) e (2) implicam s2=2s3 s, e. consequentemente, |s,|2 = 2|s3 | • |s,| = 2|s,|. Como s, # 0, temos |s,| = 2, portanto s, = 2X , com |x| = 1. s2 “ Z1Z2 +Z2Z3 +Z3Z1' S3 “ Z1Z2Z3 1815 | Problemas Importantes para Olimpíadas (-1)ns2n|. resolvem o problema. Problema 7. z.Z + (z + z).|z-1| + |z-1|2. En = |z? + z$ +zj| = |xn + XV + (-1)" Xns2n | = |l + en + È fácil ver que Ek+6 = Ek para todos os inteiros k e que as igualdades Eo =3, E, = 2, E2 =0, E3 = 1, E„ = 0, Es = 2 , Encontre todos os complexo z tais que |z - |z +1|| = |z + |z -1||. Solução. Temos |z-|z +1|| = |z + |z -1|| se e somente se |z - |z +1||2 = |z + |z -1||2, i.e. (z-|z + 1|)-(7-|z + 1|) = (z + |z-1|)-(z+|z-1|). A última equação é equivalente a z-z-(z + z)|z + 1| + |z + i|2 = Que pode ser escrita como f -|z-f =(z + z) (|z + 1| + |z-1|), j.e., (z + 1)(z + l)-(z-1)(z-l) = (z + z)-(|z + l| + |z-l|). A última equação é equivalente a 2(z + z) = (z + z)(|z + 1| + |z-1|), i.e., z + z = 0, ou |z + 1| + |z-1| = 2. Solução alternativa. Obviamente, z2,z|,z2 são distintos. Caso contrário, se, por exemplo, z2 = z2 , então 1 = |z3| = l-(z2 + z2 )| = 2|zf| = 2 , um absurdo. De z2 + z2 + z3 = 0 segue que z2,z2,z2 são as coordenadas dos vértices de um triângulo equilátero. Assim, podemos supor que z2 = sz2 e z2 = e2z2, onde e2 + e +1 = 0 . Como z2 = c4z2 e z2 = s2z2, segue que z2 = ±e2z, e z3 = ±ez,. Então |z? +z£ +z3| = |(l + (±e)n +(±e2)njzí| = |l + (±e)n +(±e2f|e {0,1,2,3} pelo mesmo argumento usado no final da solução passada. 182 Números Complexo de A A Z {bi:beX}u{aeR:ae[-1,l]}. Problema 8. = 0 S A desigualdade triangular 2 = |(z +1) - (z -1)| < |z +1| + |z -1| mostra que as soluções da equação |z +1| + |z -1| = 2 satisfazem z +1 = t (1 - z), onde t é um número real e t a 0. A equação z + z = 0 possui soluções z = bi, b e R. Assim, as soluções da equação são Sejam z,. z2 zn números complexos tais que |z,| = |z21 = ... = |zn| > 0. Prove que n Zzk k=1 n se e somente se E zk = 0 k=1 (Olimpíada de Matemática da Romênia - Segunda Fase, 1987) Problema 9. Seja X um número real e seja n > 2 um inteiro. Resolva a equação X(z + zn) = i(z-zn). Solução. A equação é equivalente a zn(X + i) = z(-X + i). Tomando o módulo de ambosos lados da equação, obtemos |z|" = |z| = |z|, assim n Assim, S é um número real, portanto ReS = S = 0 se e somente se £ zk = 0. k=1 f( n \ / n 7, =I £ zk • £ - \k=1 ) l k=1r t -1Segue que z = ------ , portanto z é qualquer número real com -1S z < 1. t + 1 n n Zj ( n > ( n -1 \ Solução. Seja S = ES —■ Então S= £zk • E — j=1k=1 Zk \k=1 ) \k=1 Zk J e, já que zk ■ zk = r2 para todo k, temos 1835 | Problemas Importantes para Olimpíadas |z| = 1- Como Então zk = cos para k = são as outras soluções da equação (além de z = 0) Problema 10. Prove que Solução. Temos | 6z -i|2 <|2 + 3iz|2, i.e., (6z -i)(6z + i) < (2 + 3iz)(2 - 3iz). Encontramos 36z.z + 6iz - 6iz + 1 < 4 - 6iz + 6iz + 6iz + 9zz, e X. Prove que |z| = 1. Problema 11. Seja z um número complexo tal que z e C \ R e 1 + z + z2 1 - z + z2 |^Zg ' | £ 1 se e somente se |6z - i| < |2 + 3iz|. A última desigualdade é equivalente a |z| = 0 ou z"*’ Se |z| = 0 , então z = 0, que satisfaz a equação. Se |z| = 1, então z = - e a equaçao pode ser reescrita como z i.e., 27z.z < 3. Finalmente, zz < ou, equivalentemente. |z| < --, como desejado. I 6z-i I . . i i 1-------- < 1 se e somente se z < — . |2 + 3iz| 1 1 3 X + i 1, existe te [0,2^) tal que ?‘ + l = cost + isent. Z + i t + 2krt t + 2k" --------- + isen n+1-----------n+1 Números Complexo de A A Z184 e IR= 1 + 2 e S se e somente se Isto é, A última relação é equivalente a z + - = z + Problema 12. Sejam z,,z2 zn números complexos tais que |z,| = ... = |zn| = 1 = z,z Seja zk = cosak + isenak, onde ak são reais para todo k = 1,n. Então z = = n2 , Encontramos z = z ou |z| = 1. Como z não é um número real, segue que |z| = 1, como desejado. 1_ z 1 z 1-z + z2 Solução. Temos que 1 + z + z2 1-z + z2 1-z + z2 z z 1-z + z2 \k=1 J ^k=lZkJ Prove que z ê um número real e que 0 < z < n2. 2 >0. = n + 2 £ isicjsn como queríamos provar. n n V cos ak + i y senak k=1 k=1 i.e. (z - z)(l - |z|2) = 0. n n 22 cos ak - i 22 senak ,k-1 k=1 = £cosuk \,k=1 J Por outro lado, temos que n z = y (cos2 ak + sen2ak) + 2 y (cosc<icoso.j +senaisenaJJ k=1 1£i<j£n cos(ai-aj)<n + 2Pl = — 1 Solução. Perceba que zk =— para todo k Como zk = = flZk') \k=1 ) ^k=izkj Vk=lZk; \k=i ) segue que z é um número real. n Esenak k=1 n(n-1) n + 2-i------ 2 -1 + zeli, i.e., z + — e K . z ... 1 - z 1855 | Problemas Importantes para Oumpiadas Observação 2 n2, portanto 0 < z < n2. Problema 13. Sejam z,,z2,z3 números complexos tais que z, + z2 + z3 == 0 e |z,| = |z21 = |z3|. z2z2 = z3z, = r’ àM >0,•Re= Re como desejado. \k=1 Z3 1 Z2 1 Z3 Problema 14. Sejam x,y,z números complexos. a) Prove que |x| + |y| + |z| < |x +y-z|+ |x-y+ z|+ |-x +y+ z|. 1 z2 + z3 Z,+Z2+Z3 r2 Uma solução alternativa para demonstrar as desigualdades 0 s z < n2 éa que segue: z = [ízk ]Íz-'I = ÍZZk lÍÊíl lk=i Ak=1 ) '■k=1 Ak=r J Solução. Seja r = |z,| = ]z21 = |z31 > 0 . Então z,z, = -L- z, + z2 + z3J ' (Re(z, + z2+z3))2 r2|z,-z2+z3|2 Zi + z2 + z3 |z,+z2+z3|2 lReí----1--- J lz,+Z2+Z3 Por outro lado, temos — z. Prove que Rei — + lzi YêzJ=|zzJ2< Ak=1 ) |k=1 | Zj + Z2 4- Z3 |z,+z2+z3|2 z1+z2+z3 z1+z2 + z3 r2 r2 el + A z, z2 e, consequentemente, Rei — + — lz, z2 186 Números Complexo de A a Z Solução. Seja m = -x +y + z, p = x + y-z.n = x - y + z, y = z =------- . IxIs^(H+Ip|). Msj(lml+|p|)' lzl£j(lml+lnl) o que resulta |x| +1y| + |z| < |m| + |n| + |p|, como desejado. A desigualdade (1) pode ser escrita como A igualdade em (1) só ocorre se e somente se o triângulo ABC for equilátero, i.e , b) Se x.y.z são distintos e os complexos x + y-z, x - y+ z,-x+ y+ z possuem módulos iguais, prove que 2(|x||y|+ |z|) < |x + y-z|+ |x -y+ z|+ |-x+ y + z|. Sejam A, B, C pontos de coordenadas m,n,p . Observe que os números m,n,p são distintos e que |m| = |n| = |p| = R, com R sendo o circunraio do triângulo ABC. Defina como sendo a origem do plano complexo o circuncentro do triângulo ABC. O ortocentro H do triângulo ABC tem coordenada h = m + n + p.A igualdade desejada se torna |h - m| + |h - n| + |h - p| < |m| + |n| + |p| ou AH + BH + CH<3R. í C A - B ou o < 2sen-----cos-------- 12 2 a a 2y = —e, z = —e . 2 2 n + p m + p m + nTemos que x = - ——. y = —z= . Aplicando o módulo em ambos lados e fazendo a desigualde triangular, temos 2 . - . . 2ir . 2jim = a, n = as, p = ac , onde a e um parametro complexo e e = cos— + isen— ■ Neste caso, x = — |2 2A-B + sen-----.I 2 3 Que é equivalente a cosA +cosB + cosC < —. (1) _ A + B A — B _ 2 C 32cos--------cos-------- + 1-2sen— <- 2 2 2 2 1875 | Problemas Importantes para Olimpíadas 1) Encontre z0 tal que z0 + z, +... + zn = 2" . 2) Para o valor de z0 determinado acima, prove que Solução, a) Use indução para provar que zk ,2 ,2 2" ■ ,n n! como desejado. Problema 16. 2n n Sejam z,,z2,z3 números complexos tais que z, +z2 + z3 = z,z2+z2z3+z3z, = 0. Prove que |z,| =|z2| = |z3|. Assim z0 + z, + ... + zn = 2". se e somente se z0(1 + i)n = 2" , i e.. z0 =(1-i)°. = ik ^)z°' para todo k s '°'1.. n' ' Problema 15.______________________________________________________ Sejam z0,z,,z2.... zn números complexos tais que (k+1)zk.,-i(n-k)zk =0 para todo k e {0,1,2.... n-1). (3n + 1)n n! (3n + 1)n n! 2nf 2n + (2n-1) + ... + (n + 1) n |z0|2+|z,|2+... + |zn|2 b) Aplicando a desigualdade das médias, temos |z0|2+|Zl|2+... + |zn|2 = |z0|2^"j +("} + on = —2n(2n-1)-(n + 1) n! Solução. Substituindo z, + z2 = -z3 em z,z2 + z3 (z, + z3) = 0 obtemos z,z2 = z3 .portanto |z,| |z2| = |z3|2 . Analogamente, |z2||z3| = |z,|2 e |z3| |z,| = |z2|2 . Assim N2+Izz|2+Iz3|2 “NN + NN + NN’ Números Complexo oe A a Z188 i e- (N -N)2 + (N -lz31)2 + (lz31" |z>|)2 = 0 • obtendo |z,| = |z2| = |z3|. Solução. Definindo z = cost + isent, obtemos |1-z| = 7(1^ e [-1,1], Deixamos Z4-Z3 b) Prove que |z3 - z, | < |z4 - z2| 0 Solução alternativa. Usando as relações entre as raizes e os coeficientes, segue que z1,z2,z3 são as raizes do polinômio z3-p, onde p = z,z2z3. Assim, z3-p = z2-p = z3-p = 0, implicando z3 = z2 = z3, de onde segue a conclusão. t= sen— e 2 |l + z2| = + cos2t)2 + sen22t = 72 + 2cos2t = 2|cos Problema 17. Prove que para todo complexo z com |z| = 1 a seguinte desigualdade ocorre: 72 <|l-z| + |l + z2| < 4 . Problema 18. Sejam z,.z2.z3.z4 números complexos distintos tais que Re Z; ~ Zl = Re Z? ~ Za = 0 z„ -z, a) Encontre todos os reais x tais que Izr-zjl”+|zi-Z4|x s|z2-z4|x <|z2-z3|x + |z4-z3|x. cost)2 +sen2t = 72-2 cos t = 2|sen-^| t| = 2|l-2sen2||. V2Agora, basta provar que — < |a| + |l- 2a21 £ 2 , para a isso a encargo do leitor. 2 ' ' Solução. Considere os pontos A, B, C, D de coordenadas z1,z2,z3,z4 , respectivamente. As condições do enunciado Re^^l = Re^^- = Z4-Z1 Z4-Z3 1895 | Problemas Importantes para Olimpíadas Problema 19. Sejam x e y números complexos distintos tais que |x| = |y|. Prove que y|<M- Solução. Sejam x = a + ib e y = c + id,com a.b.c.deR o que é verdadeiro, já que x s y . Problema 20. Considere o conjunto A = {z e C: z = a + bi, a > 0. |z| < 1}. Solução alternativa. Considere os pontos X(x) e Y(y). No triângulo XOY temos que OX = OY . Assim OM < OX , onde M é o ponto médio do segmento [XY]. Sendo desigualdade é equivalente a (a + c)2 + (b + d2) < 4 (a2 + b2) ou (a-c)2 +(b-d)2 >0, 1-a-ib 1 + a + ib ' implicam BAD = BCD = 90° .Assim, |z, -z2| - AB e |z, - z4| = AD são as medidas dos lados do triângulo retângulo ABD, de hipotenusa BD = |z2 - z41. A desigualdade AB* + ADX < BDX ocorre para x > 2 . De modo análogo. |z2-z3| = BC e |z4-z3| = CD sâo os lados do triângulo retângulo BCD, portanto a desigualdade 8DX <BCX+CDX ocorre para x<2. Consequentemente, x = 2 . Finalmente, AC = |z3 - z,| < BD = |z4 - z2|, já que AC é uma corda da circunferência de diâmetro BD. e a2 + b2 = c2 + d2 . A a coordenada do ponto M, a desigualdade desejada segue. 1 — X Prove que para qualquer ze A existe um número x e A tal que z = - ----- 1 — X 1 — z Solução. Sejaze A. A equação z = —— tem raizes x =------ 1 + x 1 + z onde a>0 e a2 +bz < 1. 190 Números Complexo oe A a Z 0, já que |z| < 1. Problema 21. Considere o conjunto A = {z e C : |z| < 1}, e um número real a com |a| > 1 e a função f: A A. Problema 22. Problema 23. Considere a função f :R -» C, Problema 24. Sejam z,,z2 e c‘ tais que |z, + z2| = |z,| = |z2|. Calcule Problema 25. Prove que para quaisquer números complexos z,,z2 zn. a seguinte desigualdade ocorre f (z) = 1~az . Prove que f é bijetiva. z + a (1 + a)2. f(t) = —. 1 - ti Prove que f é injetiva e determine sua imagem. <1 se e somente se (1-a)2 (|z1| + |z2| + ... + |zn| + |zl+z2+... + z„|)2>2(|zn|2+... + |zn|2+|z1+z2+... + zn|2). Para provar que x e A . basta mostrar que |x| < 1 e que Re(x) > 0 . De fato, temos que Seja z um número complexo tal que |z| = 1 etanto Re(z) quanto lm(z) são números racionais. Prove que |z2n -l| é racional para todo inteiro nz1. 2 (1-a)2+b2 i.e., 0 < 4a , como queríamos mostrar. ... .. „ , . 1 -Izl2 Alem disso, Re(x) = —L-y Seguem mais alguns problemas envolvendo módulos e conjugados de números complexos. 1915 | Problemas Importantes para Olimpíadas Problema 26. Problema 28. Problema 29. Problema 30. z".Re(z) = (z)n .Im(z). onde n é um inteiro. 1 x + y Sejam z,,z2 z„ números complexos distintos de mesmo módulo tais que Z3Z4 "zn-1zn + Z1Z4 ''’ zn-1Zn + -- + Z,Z2 •■Zn_2 =0 . Prove que z,z2 + z2z3 +... + zn_,zn = 0 . Problema 27._______________________________________________________ Encontre todos os reais positivos x e y que satisfazem o sistema de equações Problema 31.___________________________________________ Encontre o lugar geométrico dos números complexos z tais que |l-2|a|| 72 ■ —'1 = 472. X-ryJ (Olimpíada de Matemática do Vietnã. 1996) Sejam a e z números complexos tais que |z + a| = 1. Prove que |z2 + a2| > Sejam z,.z2 z2n números complexos tais que |z,| = |z2| = ... = |z2n| e argz, < argz2 < ... < argz2n < rr Prove que |Z1 + z2n | S |z2 + z2n-l| S • S |z„ + Z„J . Sejam z^Zj.Zj números complexos. Prove que z,+z2+z3=0 se e somente se lzl| = lz2 + z3| • Iz2| = |z3+zl| e IZ3| = IZ1+Z2|- 192 Números Complexo oe A a Z Problema 32. Problema 33. Prove que 5.2 EQUAÇÕES ALGÉBRICAS E POLINÓMIOS assim, |z,| = |z2| onde a.b.c e C’ e sejam z,.z2 suas raizes. Prove que se — é um número real, c Sejam k,n inteiros positivos e sejam z,,z2,...,zn números complexos não nulos de mesmo módulo tais que +z2 + ... + z„ = 0 . Problema 1. Considere a equação quadrática a2z2 + abz + c2 = 0 lzi + z2| 2 Se |t| > 2 , as raizes da equação são z12 = e. obviamente, — é um número real. Sejam a.b números reais com a + b = 1 e sejam zvz2 números complexos com |z,| = |z2| = 1. Prove que |az, +bz2|> então |z,| = |z2| ou ^•eR. z2 Solução. Seja t - - e R . Então b = tc e A = (ab)2 - 4a2 • c2 = a2c2 (t2 - 4 j. -tac±acjt<^ = _c_ (_t ± . . 4 J . 111 n à à 4 Se |t| < 2 . as raizes da equação são z12 =y-(-t±ix/4-t2 j, Icl = j—, . como afirmado.Ial 1935 | Problemas Importantes para Olimpíadas |z,|-|z2| + ^,1^2! -1 < 0 se e somente se (1 -í-|z2|)(|z1| -1) < 0 Consequentemente, |z,| < 1 e |z2| < 1, como desejado. Solução. Como z,+z2=-p e z,z2 = q , a desigualdade |p| + |q|<1 implica |z, + z2| + |z,z2| < 1. Porém. ||z,| -|z2|| < |z, +z2|, assim Solução. Sejam x, e x2 as raizes complexas do polinômio f = x2 + ax - b e sejam y, e y2 as raizes complexas do polinômio g = x2 + |a| x + |bj. e |z2|-|z,| -e|z2Hz^l — 1 < 0 se e somente se (1 + |z,|)(|z2| -1) < 0 . Problema 2. Sejam a.b.c.z números complexos tais que |a| = |b| = |c| > 0 e az2 + bz + c = 0 . Prove que Problema 3. Sejam p,q números complexo tais que |p| + |q| < 1. Prove que os módulos das raizes da equação z2 + pz + q = 0 são menores que 1. assim r|z2| < r|z| Por outro lado, |c| = |-az2 Problema 4, Seja f = x2 + ax + b uma equação quadrática polinomial de coeficiente complexos com ambas raizes de módulo 1. Prove que f = x2 + |a|x + |b| apresenta a mesma propriedade. Solução. Seja r = |a| = |b| = |c| > 0 . Temos |az21 = |-bz -c| < |b| |z| <- |c|, . Segue que |z|2 — |z| — 1 < 0 , portanto |z| < 1 . bz| <|a||z|2+b|z|, tal que |z|2+|z|-1 a 0 . Portanto. |z| z 1 e assim está provado. ^£|z|s^2. 2 11 2 Números Complexo de A a Z194 É fácil ver que |y,| = |y2| = 1, como queríamos provar. lado, z,z2z3 = Solução. Sejam x,,x2,x3,x4 as raizes reais do polinômio f. Então Temos de provar que se |x,| = |x2| = 1 , então |y,| = |y2| = 1. Como x, x2 = b e x, + x2 = -a , então |b| = |x,||x21 = 1 e |a| < |x,| +|x2| = 2 . O polinômio quadrático g = x2 + |a|x +1 tem discriminante a = |a|2 - 4 s 0 , assim Problema 5, Sejam a.b números complexos não nulos. Prove que a equação az3 + bz2 + bz + ã = 0 tem pelo menos uma raiz com módulo igual a 1. Problema 6. Seja f = x4 +ax3 +bx2 + cx-d um polinômio de coeficientes reais e raizes reais. Prove que se |f (i)| = 1, então a = b= c = d = O. 1 z’=r 1 1 Solução. Observe que se z é uma raiz da equação, então — é também uma raiz da z 1 2 —Se zk = — para algum k = 1,2,3 , então | zk | = zkzk = 1 e assim está provado. Zk Se z, * — para todo k = 1,2,3 , devemos considerar, sem perda de generalidade, zk 1 que z, = —, Zj g As duas primeiras igualdades dão z,z2z3 = —,, assim |z,||z2|2 |z3| = 1. Por outro , portanto |z3||z2||z3| = 1. Segue que |z2| = 1, como afirmado. 111 equação Consequentemente, se z,,z2,z3 são raizes da equação, então —, —, — são as mesmas raizes, só que não necessariamente nessa ordem. zi z? z3 -|a|+ 1^4-|a|2 X1.2 - 2 1955 | Problemas Importantes para Olimpíadas Problema 7. Assim r"cosna + racosct+ 1 = 0 e rnsenna + rasena = 0 . Solução. Seja z = r(cosa + isenct) uma raiz não real da equação, onde e a# i. Substituindo essa raiz na equação encontramos r" cos ncx + ra cos a +1 + i (r"sen na + rasena Problema 8. Seja n > 3 um inteiro e seja a um número real diferente de 0. Mostre que qualquer raiz não nula z da equação xn +ax +1 = 0 satisfaz a desigualdade ^112 + 220b +102 (a2 + b2) Jl12(a2 +b2) + 220b + 102 Prove que se 11z'° +10iz9 +10ÍZ-11 = 0 , então |z[ = 1. (Competição de Matemática Putnam, 1989) Como |f (i)| = 1, deduzimos que x, = x2 = x3 = x4 = 0 e. consequentemente, a = b = c = d = 0, como queriamos provar. _ |l1-10iz| ~|l1z + 10i| ) = °- a e (0.2—) kl* (Olimpíada de Matemática da Romênia - Última Fase. 1995) Sejam f(a.b) e g(a,b) o numerador e o denominador do lado direito da equação, respectivamente. Se |z| > 1, então a2 + b2 > 1, portanto g(a,b) > f (a.b). implicando |z9| < 1, um absurdo. Se |zj < 1 , então a2+b2<1, portanto g(a.b) < f (a.b), implicando |z9| > 1, de novo um absurdo. Logo, |z| = 1. w9 f = (x-x,)(x-x2)(x-x3)(x-x4) e Solução. A equação pode ser reescrita como z9 = ———— .Se z = a + bi, então 11z-í-10i Números Complexo oe A a Z196 assim k.i1 + xk P(-1) 1~x„ 1 + xk P(1) Isso prova que é um número real, como queríamos provar. p(1) P(-1)l kJlll + Xk para algum X e C', e =pj í-L-íú _ r_i)2n p = ZÍ2L ) kJlt 1 + Xk J { 1 k'.l1 + Xk P(-1) |senci| < rn (n - 1)(sena). Como sena fl-xh) 1-x„ U + XkJ 1+xk Problema 9.__________________________________________________________ _ Suponha que P seja um polinõmio com coeficientes complexos e grau par. Se todas as raizes de P são números complexos não reais de módulo 1, prove que P(1) e R se e somente se P(—1) e R * 0, segue que rn >------,i.e., Izl > n-1 1 Multiplicando a primeira igualdade por sena e a segunda por cosa e então subtraindo-as, encontramos que rnsen(n-1)a = sena . Segue que r"|sen(n-1)a| = |sena| . A desigualdade |senka| S k|sena| é válida para qualquer inteiro positivo k. A prova é baseada apenas em um simples argumento indutivo sobre k. Aplicando essa desigualdade, de rn|sen(n-1)a| = |sena| obtemos que 1-A Xk. xk- 1 1 + ± xk + 1 Xk P(1) Solução. Basta provar que x ' e R . Sejam x„x2 x2n as raizes de P. Assim P(x) = X(x - x,)(x -x2)- -(x -x2n) p(1) _ ^•(1-x1)(1-x2) "(1-x2n) _ 2n 1-xkP(-1) X(-1-x,)(-1-x2) (-1-x2n) k-i1 + xk’ Da hipótese, temos que |xk | = 1 para todo k = 1,2,...,2n. Então 1-x> 1 + xk P(1) P(-1) 5 | Problemas Importantes para Olimpíadas 197 Problema 10. (18a IMO - Lista de problemas) Solução. Escreva x = z + z‘ , onde z é um número complexo não nulo. Assim . Por meio de um simples argumento indutivo, .2" -z = z -1 Segue que z' -1 = 0 e z‘ -1 =0 temos -1=0 diferentes de 1, temos z + z'-1 = w + w' se -1 = 0 e z' Claramente, o grau do polinômio Pn é 2". Como vimos antes, todas as raizes s = 1 2n. Considere a sequência de polinômios definida por P,(x) = x2 -2 e P, (x) = Pi(P,_, (x)) para j = 2,3 Mostre que para qualquer inteiro positivo n, as raizes da equação P„(x) = x são todas reais e distintas. Pi(x) = xz-2 = (Z -r Z* (z: = z. Também observe que para quaisquer duas raizes z e w de raizes é 1 + ^2n -2) + ^2" = 2" e todas elas são distintas. i2”*’ -1 = 0 é z = 1 (veja a Proposição 1 na Seção 2.2). . i.e., zfz2"'1 )2 - 2 = z2 + z' — 1 j = 0. Como mdc(2n -1, 2" +1) = 1, a única raiz comum entre as equações z2""1 z2”’1.iP-i-l z2"*’ k = 1 2n-2 e z = cos—+ isen ^Sn , 2n +1 2n +1 Além disso, para qualquer raiz da equação |z| = 1, i.e., í z2n-i :2"-1-1 z2n+1z2n-1 = z-2" + Z-^0 -z-2" + z-2n .2" se e somente mostra-se que para todo inteiro positivo n temos Pn (x) = z‘ Aequação Pn (x) = x éequivalentea z2” +Z-2" = z + z“1.Obtemos z‘ Levando em consideração a simetria da expressão z + z , o número total dessas (z-w)(l-(zw)~1j = 0 . Isso é equivalente a zw =1, i.e., w = z-1 = z , uma contradição com o fato de que a única raiz comum entre z2""1 -1 = 0 e z2"*1 -1 = 0 é 1. - , , 2kn 2krtde Pn (x) = x são dadas por x = z + z-', onde z = 1, z = cos ----- + isen ——- , Números Complexo de A a Z198 Seguem alguns outros problemas envolvendo equações algébricas e polinõmios. Problema 11. Prove que f[(x2+l) = (2a-c)2. Problema 14. real x, a relação f (x)f (2x2) = f (2x3 + x) . (21* IMO - Lista de Problemas) Sejam a,b,c números complexos com a * 0 . Prove que se as raizes da equação az2 + bz + c = 0 têm mesmo módulo, então ab |c| = |a|bc. Problema 15.__________________________________________________________ Encontre todos os polinõmios f de coeficientes reais satisfazendo, para qualquer Problema 13,_______________________________________ Considere a equação de coficientes reais x6 + ax5 + bx4 + cx3 + bx2 + ax +1 = 0 , e denote por x,,x2 x6 as raizes dessa equação. Sejam a e b números complexos e seja P(z) = az2 + bz + i. Prove que existe um z0 e C com |z0| = 1 tal que |p(z0)| z 1 + |a|. Problema 12.______________________________________________________ ___ Sejam z„z2 as raizes da equação z2+z + 1 = 0 esejam z3,z4 as raizes da equação z2 - z +1 = 0. Encontre todos os inteiros n tais que z" + z2 = z3 + z4 . 1995 | Problemas Importantes para OumpIadas 5.3 DE IDENTIDADES ALGÉBRICAS A PROPRIEDADES GEOMÉTRICAS = 0.(1) Observações- Subtraindo a relação (3) da relação (2), encontramos (a'-a)(b-c) + (b'-b)(c-a) + (c'-c)(a -b) = 0 . (4) Tomando o módulo em ambos lados, segue que |a’— a||b — c| < |b'-b||c-a| + |c'-c||a-b|. (5) Por outro lado, claramente, a seguinte relação ocorre: a(b -c) + b(c -a) + c(a -b) = 0 . (3) Levando em consideração que |b -c| = |c-a| = |a — b|, obtemos AA' < BB’+CC. De modo análogo, provamos as desigualdades BB' < CC+ AA'eCC < AA'+BB', de onde segue a conclusão desejada. 1) Se ABC eA’B'C são dois triângulos semelhantes situados no mesmo plano e de mesma orientação, então da relação (5) segue a desigualdade AA'BC < BB'-CA + CC-AB . (6) Esse é a desigualdade de Ptolomeu generalizada. A desigualdade de Ptolomeu é obtida quando o triângulo A'B'C degenera em um ponto. 2) Levando em consideração a desigualdade (6), também temos que BB'CA SCC-AB +AA'-BC e CC'-AB S AA' BC + BB' CA . Segue que para quaisquer dois triângulos semelhantes ABC e A'B'C de mesma orientação e Problema 1,____________________________________________________________ Considere os triângulos equiláteros ABC e A'B'C, ambos situados no mesmo plano e com mesma orientação. Mostre que os segmentos [AA1], [BB1], [CC] podem ser os lados de um triângulo. Solução. Sejam a,b,c as coordenadas dos vértices A, B, C e sejam a', b', c' as coordenadas dos vértices A', B', C. Como os triângulos ABC e A'8'C são semelhantes, temos a relação (veja a Observação 1 na Seção 3.3): 1 1 1 abc a' b' c' Isto é, a'(b-c) + b'(c -a) + c'(a-b) = 0 . (2) 200 Números Complexo de A A Z >1.(2) ai, Observações- = 2Rr, assim R > 2r. PC PA ya 2) Se P é o baricentro G do triângulo ABC nós obtemos a seguinte desigualdade envolvendo as medianas ma,mn,mY: 3) No caso quando o triângulo A'B'C degenera em um ponto M, da propriedade do nosso problema segue que os segmentos MA. MB, MC são os lados de um triângulo, i.e., teorema de Ptolomeu (veja também a Subseção 4.9.1). 1) Se P é o circuncentro O do triângulo ABC, podemos obter a desigualdade de Euler R > 2r. Certamente, neste caso a desigualdade é equivalente a R2 (a + p + y) > aPy . Portanto PB-PC Py i.e., a desigualdade desejada. situados no mesmo plano, podemos construir um triângulo com lados medindo AA' BC, BB' CA, CCAB . «Py 2s Problema 2,__________________________________________ Seja P um ponto arbitrário no plano do triângulo ABC. Então a ■ PB PC + P ■ PC • PA + y ■ PA ■ PB > apy , onde a.p.y são os lados de ABC. área [ABC] s PA PB aP ^.£PI = 2r. 2s 4R Levando em consideração que |a| = PA, |b| = PB, |c| = PC e que |b — c| = <z, |c — a| = p, |a — b| = y , a desigualdade (2) se torna R2>^P^ a + p + y Solução. Considere a origem do plano complexo em P e sejam a, b, c as coordenadas dos vértices do triângulo ABC. Da identidade algébrica bc ca ab . . --------------------------+---------------------------4-----------------------------= 1 ( 1 ) (a-b)(a-c) (b-c)(b-a) (c-a)(c-b) Tomando o módulo, segue que |b||c| . |c||a| , |a||b| [a —b||a —c| [b —c||b —a| |c —a||c —b| 2015 | Probleíaas Importantes para Olimpíadas Seja zi = z2 = Z3 = Z,+Z2+Z3 =1, .2 Z2 segue que Z1Z2 Z3 Z2Z3 Z1 Solução. Seja P a origem do plano complexo e sejam a, b, c as coordenadas de A, B. C, respectivamente. A igualdade do problema é equivalente a |ab(a -b)| + |bc(b-c)| + |ca(c -a)| = |(a -b)(b - c)(c - a)|. se e somente se P é o ortocentro do triângulo ABC. (Olimpíada de Matemática da China, 1998) com a segunda equação decorrente da identidade (1) do problema anterior. Iremos provar que P é ortocentro do triângulo ABC se e somente se z^Zj.z-j são números reais positivos. De fato, se P é ortocentro, então, já que o triângulo ABC ê acutângulo, segue que P está no interior de ABC. Assim existem reais positivos r,.r2,r3 tais que implicando z, = r,r2 > 0, z2 = r2r3 > 0, z3 = r3q > 0 e assim está provado. Reciprocamente, suponha que z1,z2,z3 são todos reais positivos. Como ,------- são imaginários puros, portanto APJ.BC e r: b BP 1 CA , o que mostra que P é ortocentro do triângulo ABC. z3zi _[ a b-c. Problema 3.__________________________________________________________ Seja ABC um triângulo acutângulo e seja P um ponto do seu interior. Prove que a ■ PB ■ PC + p ■ PC • PA + Y • PA ■ PB = a0y , ab (a-c)(b-c)' bc (b-a)(c-a)’ ca (c-b)(a-b)' a b c b-c'c-a'a-b a . b . c k 7 —7 7"r21’ 7 k "-r3' ’b-c c-a a-b mam3 m^m, mYma > 9 cxp Py y“ " 4 com a igualdade ocorrendo se e somente se o triângulo ABC é equilátero. Uma boa aplicação para o caso dos triângulo acutângulos é dada no próximo problema. Segue que |z,|-t-|z2|-1-|z3| =1 e 202 Números Complexo de A a Z Problema 4. onde R é o circunraio do triângulo ABC. a GB GC + p GC GA + y-GA GB £aPy . (1) i.e., R, + R2 + R3 s 3R . Problema 5. Solução. Perceba que k, k2 k3 = Porém, área [ABC] = . Duas relações semelhantes Seja ABC um triângulo e seja P um ponto do seu interior. Sejam R,,R2,R3 os raios dos circuncirculos dos triângulos PBC, PCA, PAB, respectivamente. As retas PA, PB, PCintersectam os lados BC, CA, AB em ApB^C,, respectivamente. Sejam também Solução. No Problema 2, considere P como sendo o baricentro do triângulo ABC. Então área [PBC] área [ABC]’ PBj. BB,' Seja G o baricentro do triângulo ABC e sejam R1,R2,R3 os circunraios dos triângulos GBC, GCA, GAB, respectivamente. Então R, +R2 + R3 è* 3R , área [PAB] área[ABC] área[PCA] área[ABC]' ct-PB-PC 4R, e área [PBC] = . PAr v k’=ÃÃ? k2 = onde a,p,y são as medidas dos lados do triângulo ABC. Porém a GB ■ GC = 4R, • área [GBC] = 4R, • iárea [ABC]. k3 CC, ’ Prove que k,R, + k2R2 + k3R3 á R , onde R é o circunraio do triângulo ABC. (Seletiva Romena para a IMO - 2004) 1 1 Analogamente, p-GC GA = 4R2 ■—área[ABC], y GA GB = 4R3 •—área[ABC]. Assim, a desigualdade (1) é equivalente a i(R, + R2 + R3) • área [ABC] > 4R ■ área [ ABC], «Pr 4R ocorrem para área[PCA] e área[PAB], 2035 | Problemas Importantes para Olimpíadas A desigualdade desejada é equivalente a + R >R onde G é o baricentro do triângulo ABC. Problema 7. área[ABCD] > Solução. A identidade x3(y-z) + y3(z-x) + z3(x-y) = (x-y)(y-z)(z-x)(x + y + z) (1) ocorre para quaisquer complexos x, y, z. Tomando o módulo, obtemos a desigualdade |x3 (y - z)| + |y3 (z - x)| + |z3 (x - y)| à |x -y||y - z||; Sejam a, b, c, m as coordenadas dos pontos A, B, C, M, respectivamente. Em (2), considerando x = m-a, y = m-b, z = m- c, obtemos AM3 -a + BM3 ■ p + CM3 -y > 3aPyMG. (3) Seja ABCD um quadrilátero cíclico inscrito em uma circunferência C(O;R) com lados medindo a,p,y,S e diagonais medindo d, e d2. Então Solução. Tome o centro O como sendo a origem do plano complexo e defina a, b, c, d como sendo as coordenadas dos vértices A, B, C, D, respectivamente. Da famosa identidade de Euler |z-x||x + y + z|.(2) PPCPA apy aPy que se reduz à desigualdade do Problema 2. No caso quando o triângulo ABC é acutângulo, do Problema 3 segue que a igualdade ocorre se e somente se P é ortocentro de ABC. Problema 6.___________________________________________________________ Para qualquer ponto M do plano do triângulo ABC a seguinte desigualdade ocorre: AM3senA + BM3senB + CM3senC â 6 MG ■ área [ABC], y-PAPB a?y aPySd^j 8R4 Usando a fórmula área [ABC] = e aplicando lei dos senos, a desigualdade desejada segue. 4R R--PB'-PC+R a3 ui(a-b)(a-c)(a-d) 1 204 Números Complexo de A A Z 2 1 . (2) e as três outras Prove que a, b, c são as coordenadas dos vértices de um triângulo equilátero. Seja s = a + p = -1 e p = ab.Arelação (1) se torna (a6+P6)-p(a4+p4) + p2(a2+p2)-p3 =1.(2) tomando o módulo, segue que s cyc A desigualdade (2) é equivalente a X cyc Problema 8._____________________________________ Sejam a, b, c números complexos distintos tais que (a-b)7 + (b-c)7 + (c-a)7 = 0. -aps+p6=1 (1) la|3 |a-b||a-c||a-d| R3 AB AC AD x7 + y7 + z7 = 0 . Já que z * 0, podemos definir a = — e P = — . Assim a + p = -1 z z e a7 +p7 = -1. Então a6-a5p + a4p2-a3p3 *1 (3) + a2p4 Solução. Definindo x = a-b, y = b-c, z = c-a obtemos x + y + z = 0 e Como a2 + p2 = s2 - 2p = 1 - 2p, a4 +P4 =(a2 +p2)2-2a2p2 =(1-2p)2-2p2 = 1-4p + 2p2, a6 + p6 = (a2 + P2 ) ((a4 + p4) - a2p2) = (1 - 2p)(l - 4p + p2), a igualdade (2) é equivalente a ou ^R3 BD CD BC > aPy8d1d2 ■ (4) cyc Contudo, conhecemos a relação BD-CD BC = 4R área[BCD] semelhantes. A desigualdade (4) pode ser escrita da forma 4R4 (área [ ABC]+ área[BCD] + área[CDA] + áreafDAB]) 2 aPy6d,d2 ou, equivalentemente, 8R4área[ABCD] > aPySd,d2 . 2055 | Problemas Importantes para OumpIadas i.e..I sto xy + yz + tx > yz Solução. Considere o plano complexo tal que 1, i, -1, -i são as coordenadas dos vértices A, B, C, D do quadrado. Se z é a coordenada do ponto M, então temos a identidade Solução alternativa. Seja x = a-b, y = b-c, z = c-a . Como x + y + z = 0 e x7 + y7 + z7 = 0 , encontramos que (x + y)7 - x7 - y7 = 0. Isso é equivalente a 7xy(x + y)(x2 + xy + y2)2 =0. Porém, xyz * 0, portanto x2 + xy + y2 = 0, i.e., x3 = y3. Da simetria do problema. x3 = y3 = z3 , assim, |x| = |y| = |z|. Isolando o primeiro termo da soma obtemos i(z + 1)(z + i)-1(z + i)(z-i)-i(z-1)(z-i) = -1(z-i)(z + 1). Aplicando a desigualdade triangular encontramos |z-i||z + i| + |z + l||z + i| + |z + i||z-l|>|z-l||z-i| ou yz + zt + tx a xy . Da mesma forma, provamos que xy + zt + tx > yz, e xy + yz + zt > tx , como queríamos provar. Problema 9,___________________________________________________________ Seja M um ponto do plano do quadrado ABCD e seja MA = x, MB = y, MC = z, MD = t. Prove que os números xy, yz, zt, tx são os lados de um quadrilátero. (1 - 2p)(l - 4p + p2) - p(l - 4p + 2p2) + p2 (1 - 2p) - p3 = 1. é , 1 - 4p + p2 - 2p + 8p2 - 2p3 - p + 4p2 - 2p3 + p2 - 2p3 - p3 = 1 , -7p3 +14p2 - 7p + 1 = 1. Obtemos -7p(p-1)2 = 0 , assim p = 0 ou p = 1. Se p = 0, então a = 0 ou 0 = 0 e, consequentemente, x = 0 ou y = 0 . Segue que a = b ou b = c,o que é falso; logo p = 1. De a0 = 1 e a + 0 = -1 deduzimos que a e 0 são as raizes da equação quadrática x2 + x +1 = 0 . Portanto a3 = 03 = 1 e |a| = |0| = 1. Logo, |x| = |y| = |z[ ou |a — b| = |b — c| = |c — a|, como afirmado. 1(z-i)(z+ 1) + i(z+ 1)(z+ i)-1(z + i)(z-1)-i(z-1)(z-i) = 0 . (1) Números Complexo oe A à Z206 Problema 10. Sejam z,,z2,z3 números complexos distintos tais que |z,| = |z21 = |z3| = R. Prove que Solução. A seguinte identidade é facilmente verificável 1 Solução alternativa. Sejam Do Problema 29 na Seção 1.1 temos ap + Py + ya < 9R2 . 29 Observação : „ n . «Pr 4K l.e., a+p + y = — 1 ya 1 + — > ya 1 1 segue que ap + py 4sr ' ' r'“ R Obtemos R s 2r, i.e., a desigualdade de Euler para um triângulo. 9 aP + Py + ya a = |z2-z3|, p = |z3-z1|. Y = 1^1 -Z2| • lzr|2 |Z1-Z2|IZ1-Z3|cyc Considere o triângulo de vértices em z1,z2,z3 cujo circuncentro é a origem do plano complexo Assim, o circunraio R é igual a |z,| = |z2| = |z3| e seus lados são a = |z2-z3|, P = |z,-z3|, y = |z1-z2| 1 + 1 + 1 1 |z, - Z2 ||z, - z31 + |z2 - z,| |z2 - z31 + |z3 - z, | |z3 - z21 R2 ’ Usando a desigualdade (ap + py + ya)í —+ — l ap Py >J_- p2 , como queríamos provar. ____É___ +____ á____+____ _____ = 1. (z1-z2)(z,-z3) (z2-z,)(z2-z3) (z3-z,)(z3-z2) A desigualdade acima é equivalente a — + — ap py ya Rz = R2Si----- ií----- !•^|z,-z3||Zl-z3| Tomando o módulo, encontramos que = y z?___ ^(z1-z2)(z1"z3) i.e., a desigualdade desejada. 5 | Problemas Importantes para Olimpíadas 207 2) Prove que R2 (R2 -4r2) è 4r2^8R2 -(ct2 + P2 +y2)j. Ix!3 |y " zl + M3 lz - xl + lz|3 |x - y| 2: |x - y||y - z||: Observação -1 + cos2 C > 0 . 2) Se P é o circuncentro O do triângulo ABC, após algumas transformações elementares, a desigualdade anterior se torna R2>6r OG. Elevando ao quadrado ambos os lados obtemos R4 à36r2OG2. Usando a famosa relação ocorre para quaisquer complexos x, y, z. Tomando o módulo em ambos os lados obtemos Sejam a,b,c,zp as coordenadas de A, B, C, P, respectivamente. Em (2), tome x = zp-a, y = zp-b, z = zp - c e assim obtemos a desigualdade desejada. Solução. 1) A identidade x3 (y-z) +y3(z-x) + z3(x-y) = (x-y)(y-z)(z-x)(x + y+ z) (1) |z-x||x + y + z|. 1 + cos2A 1 + cos2B ----------------------1--------------------- 2 2 o que se reduz a cos(A+ B)cos(A-B) + cos2 C > 0 , que é equivalente a cosC[cos(A -B)-cos(A + B)] > 0 , i.e., cos A cos B cos C < 0 . Seguem alguns outros problemas envolvendo esse tópico. Problema 11.________________________________________________ Seja ABC um triângulo e seja P um ponto no seu plano. Prove que a PA3 + p • PB3 + y • PC3 2 3aPy ■ PG , onde G é o baricentro de ABC. A desigualdade (2) aperfeiçoa a desigualdade de Euler para a classe dos triângulos obtusos. Isso é equivalente a provar que a2 + p2 + y2 < 8R2 em qualquer triângulo desses. A última relação pode ser escrita como sen2A + sen2B + sen2C < 2 ou cos2 A + cos2 B - sen2C > 0 . Isto é, OG2=R2—(a2+p2+y2) encontramos R4 2 36R2r2 -4r2(a2 + p2 + y2). de onde segue a conclusão. Números Complexo oe A a Z20B Problema12. Problema 13. Os números complexos z,, i = 1,2,3,4,5 têm o mesmo módulo não nulo e MAB = NAC e Prove que (39a IMO - Lista de Problemas) 5.4 RESOLVENDO PROBLEMAS GEOMÉTRICOS BM-BN BA BC AM -AN AB-AC Sejam a, b, c, d números complexos distintos com |a| = |b| = |c| = |d| e a + b + c + d = 0. Assim, os lugares geométricos de a, b, c, d são os vértices de um retângulo. b) Sejam A^Bj.C, pontos sobre os lados BC, AC e AB, respectivamente, tais que os ângulos do triângulo A,B,C, sâo nessa ordem a,p,y . Prove que Problema 1.___________________________________________________________ Sobre cada lado de um paralelogramo, um quadrado é desenhado externamente à figura. Prove que os centros do quadrado sâo vértices de outro quadrado. Problema 15._____________________________________ Sejam M e N pontos interiores ao triângulo ABC tais que MBà = NBC. Problema 14,_____________________________________ Seja ABC um triângulo. a) Prove que se M é um ponto qualquer do plano, então AMsenA < BMsenB + CMsenC . fzi = fz| =0. 1=1 1=1 Prove que zvz2,...,z5 são as coordenadas dos vértices de um pentágono regular. (Olimpíada de Matemática da Romênia - Última fase, 2003) ^AA-jSena £ BC sen a. cyc cyc (Olimpíada de Matemática da Romênia - Segunda fase, 2003) CM ■ CN CA-CB- ' 2095 | Problemas Importantes para Olimpíadas OU (Teorema de Pappus) ãa = 0 zz Que é equivalente a z(a - b) - z(a - b) = ab - ab, i.e., z + abz = a + b. Solução. Considere o plano complexo com origem no ponto de interseção das diagonais e sejam a,b,-a,-b as coordenadas dos vértices A, B, C, D, respectivamente. Problema 2.___________________________________________________________ Dado um ponto sobre o circuncirculo de um quadrilátero ciclico, prove que os produtos das distâncias do ponto para qualquer par de lados opostos ou para as diagonais são iguais. m - abm + a 2 Usando as fórmulas de rotação, obtemos b = zo, + (a-zo,)(-i) ed(M, AB) = |m = argi = ^. (m - a)(m - b) I 2m = argi = í, Seja M, o pé da perpendicular do ponto M, sobre o circuncirculo, à reta AB. Se m é a coordenada de M, então (veja a Proposição 1 na Seção 4.5) m - abm + a + b zm, - 2 1 b b 1 1 Solução. Sejam a, b, c, d as coordenadas dos vértices A, B, C, D do quadrilátero e considere o plano complexo de origem no circuncentro de ABCD. Sem perda de generalidade, suponha que o circunraio seja igual a 1. A equação da reta AB é - m, | = | m - já que mm = 1. Analogamente, = -—— °2 1 + i ’ Segue que C4C\O2 = arg Z°2 ~ Z°1 = z°4 “ zo, portanto O,O2 = O,O4 e Ó^O3Õ4 = -b-ai zo3----\: >3 1 + i a-bi-b-ai = arg----- ——------ -a + bi-b-ai zo4-zo, -a + bi + b + ai = arg—2------L = arg-------— -----r zo2-zo3 a-bi + b + ai portanto O3O4 = O3O2 . Logo, 0,020304 é um quadrado. b + ai 2°’ - ITT -a + bi z°< 210 Números Complexo de A à Z d(M,CD) = d(M,DA) = , d(M,AC) = Problema 3. (“O problema das cinco moedas”, Tzitzeica’) zb - ze +(zc ~ze)' e ZE 'Gheorghe Tzitzeica (1873 - 1939), Matemático romeno, teve grandes contribuições na geometria. Solução. A rotação em torno de E através de um ângulo de 90° leva o ponto C até B, assim Analogamente, BC = O,O2 e AC = O2O3 , logo os triângulos ABC e O,O2O3 são congruentes. Consequentemente, os seus circunraios são iguais. Como OO, = OO2 = OO3 = r, os circunraios dos triângulos O,O2O3 e ABC são iguais ar, como queríamos provar. Analogamente, d(M,BC) = : ZB ~ zçí 1-i Solução. Considere o plano complexo com origem em O e sejam z,,z2,z3 as coordenadas dos centros O,,O2.O3, respectivamente. Segue que os pontos A, B, C têm coordenadas z, + z2, z2 + z3, z3 + z,, assim AB = |(z1 + z2) —(z2 + z3)| = |z3 - z3| = 0,03 . (m-b)(m-c) 2m (m-d)(m-a) 2m (m-c)(m-d) 2m (m-a)(m-c} 2m Problema 4.___________________________________________________________ Sobre os lados AB e BC do triângulo ABC são construídos quadrados com centros em D e em E tais que os pontos C e D situam-se sobre 0 mesmo lado da reta AB e os pontos A e E situam-se em lados opostos à reta BC. Prove que 0 ângulo entre as retas AC e DE é igual a 450. e d(M,BD) = (m-b)(m-d)| I 2m Portanto. d(M.AB) d(M.CD) = d(M,BC) d(M,DA) = d(M,AC) d(M,BD) . como afirmado. Três circunferências iguais CjO^r), C2(O2;r) e C3(O3;r) têm um ponto em comum O. As circunferências C, e C2 , C2 e C3 , C3 e C, se encontram novamente nos pontos A, B, C, respectivamente. Prove que o circunraio do triângulo ABC é igual a r. 5 | ProblEíxas Importantes para Olimpíadas 211 Analogamente, zD Figura 5.1. Observação Se os quadrados são substituídos nas mesmas condições por retângulos com centros D e E, então o ângulo entre as retas AC e DE é igual a 90°-BAD . m . za -Za' 1-i Problema 5.____________________________________________________________ Sobre os lados AB e BC do triângulo ABC são construídos externamente os triângulos equiláteros ABN e ACM. Se P, Q, R são os pontos médios dos segmentos BC, AM. AN. respectivamente, prove que o triângulo PQR é equilátero. Solução. Considere o plano complexo deorigememAe denote por letra minúscula a coordenada de um ponto denotada pela respectiva letra maiúscula. A rotação em relação ao centro A através de um ângulo de 60° leva os pontos N e C até B e M, respectivamente. Definindo e = cos60°+isen60°. temos b = n-r. e O ângulo entre as retas AC e DE é igual a arnzc-zA arn (zc-Za)0-í) d i y--------------- — •---------- ZE -ZO como queríamos provar. „ . b + c m c • s nb br.5 br.2 m = c ■ e . Loqo, p -------. q = — = -—■. r = — = — =------= ——■. 2 2 2 2 2e 2 2 Para provar que o triângulo PQR é equilátero, usando a Proposição 1 na Seção = arg -v -V—. = arg^ = arg(1 + i) = £, zB - zci - zB + zAi -i 4 212 Números Complexo de A a Z 3.4, basta observar que p2 + q2 + r2 = pq + qr + rp . Problema 6. Figura 5.2. c' = c-e. É fácil ver que m2 + n2 + p2 = mn + np + pm; portanto MNP é um triângulo equilátero (veja a Proposição 1 na Seção 3.4). iR' Solução. Considere o plano complexo com origem em A e sejam b, c, g, e, m as coordenadas dos pontos B, C, G, E, M. Se M. N. P são os pontos médios dos lados AA', BB‘, CC, respectivamente, prove que o triângulo MNP é equilátero. Seja AA’BB'CC um hexágono inscrito na circunferência C(O;R) tal que AA' = BB' = CC' = R. be + cn =------- 2 Problema 7.___________________________________________________________ Sobre os lados AB e AC do triângulo ABC, os quadrados ABDE e ACFG são construídos externamente. Se M é o ponto médio do lado BC, prove que AM ± EG e EG = 2AM . ce + a P = — Os pontos M. N, P têm coordenadas m = ac^b Solução. Considere o plano complexo de origem no circuncentro O e sejam a, b, c, a’, b', c' as coordenadas dos vértices A, B, C, A, B', C', respectivamente. Se e = cos60D+isen60°, então a’ = a e, b' = b-e, . .. b + c m-a -(b + c) iObserve que g = ci, e = —bi, m = , assim --------= ------ = — e 2135 | Problemas Importantes para Olimpíadas Portanto, AM ± EG e 2AM = EG . Figura 5.3. C Figura 5.4. 2c + a 3 0 ponto D é obtido de uma rotação do ponto M em torno do centro N e através de um ângulo de 60°. Assim c + 2a S“ 3 onde e = cos60°+isen60°. Analogamente c + 2a + (c -a)e 3 a + 2b n =-------- , 3 2a+ b m =-------- , 3 2b + c b + 2c P =---------- ■ q =-----------3 3 e f = s + (r-s)e = Solução. Denote por letras minúsculas as coordenadas de um ponto denotado por letras maiúsculas. Assim, Problema 8.___________________________________________________________ Os lados AB, BC e CA do triângulo ABC sâo divididos em três partes iguais pelos pontos M, N: P, Q e R, S, respectivamente. Os triângulos equiláteros MND, PQE, RSF são construídos externamente ao triângulo ABC. Prove que o triângulo DEF é equilátero. e |m-a| = l|e-g|. , , b + 2c + (b-c)c e = q + (p - q)c =---------- ----------- 214 Números Complexo de A a Z = E . temos FDE = 60° e FD = FE , portanto o triângulo DEF é equilátero. Solução. Considere o plano complexo tais que os pontos A, B, C, D têm coordenadas Figura 5.5. +2cos(xUma vez que f-d e-d c(b + c-2a + (c Problema 9.___________________________________________________________ Seja ABCD um quadrado com lado medindo a e considere um ponto P sobre o incirculo do quadrado. Encontre o valor de PA2 + PB2 + PC2 + PD2. a2 aV2 af„ „ ( 4— + 2---------- 2cosx + 2cos x+— + " 2) a72 4a=—. + a-2b)(-E2)) Zc- 2 . z a^i ZB=—I. Seja zp = —(cosx+ isenx) a coordenada do ponto P. 7 ^i Então PA2 + PB2 + PC2 + PD2 = |zA - zp |2 + |zB - zp|2 + |zc - zP|2 + |zD - zP |2 c+a-2b + (b + c- 2a)c 2c-a-b + (2b-a-c)s 2c-a-b + (2b-a-c)e e(b + c -2a) + (c + a — 2b)(s — 1) 2c - a - b + (2b - a - c) e = X(2A-zP)(zA-zP) = 4a :2a^ cyc 7 2 2 n) + 2cos^x + -^-^ + 4-^- = 2a2 + 0 + a2 = 3a2 a 2 2155 | Problemas Importantes para Olimpíadas Solução. Sejam a, b, d as coordenadas dos pontos A, B. D, respectivamente. Figura 5.6. A fórmula de rotação dá e zo Portanto, QM d. OM e OM = QM , como queríamos provar. a + d + (d-a)i 2 Problema 10._____________ _____________________________________________ Sobre os lados AB e AD do triângulo ABD são construídos externamente os quadrados ABEF e ADGH de centros O e Q. respectivamente. Se M é o ponto médio do lado BD, prove que OMQ é um triângulo isósceles com ângulo reto em M. a-z0 b-z0 Problema 11.___________________________________________________________ Sobre os lados de um quadrilátero convexo ABCD são construídos externamente os triângulos equiláteros ABM e CDP e internamente os triângulos equiláteros BCN e ADQ. Descreva a forma do quadrilátero MNPQ. (23a IMO - Lista de Problemas) A coordenada do ponto médio M do segmento [BD] é zM assim z0-zM = a-d + (a-b)i = . Zq-zm a-b + (d-a)i d-zo . b + a + (a - b)i -------— = i, assim zo = a-zQ 2 Números Complexo de A a Z216 Figura 5.7. p =c + (d-c)e, onde portanto ou MNPQ é um paralelogramo ou os pontos M, N, P, Q são colineares. q = a + (d-a)s , Problema 12.________________________________________________________ Sobre os lados do triângulo ABC são construídos externamente os quadrados ABMM’, ACNN' e BCPP'. Sejam A’, B’, C os pontos médios dos segmentos M'N', P’M, PN, respectivamente. Prove que os triângulos ABC e A'B‘C têm o mesmo baricentro. e = cos 60° +isen60°. É fácil verque m + p = a+ c + (b + d- a-c)e = n + q, Solução. Denote por letra minúscula a coordenada de um ponto denotado pela respectiva letra maiúscula. Usando a fórmula de rotação, obtemos m = a + (b-a)c, n = c + (b-c)e, 2175 | Problemas Importantes para Olimpíadas N' A' M' D* M B P' P Figura 5.8. m' = a + (b-a)(-i). assim, a' = Analogamente, b' = c' =e Uma vez que a'+b'+c' = = a + b + c, segue que eles, de fato, possuem o mesmo baricentro. Problema 13.___________________________________________________________ Seja ABC um triângulo acutãngulo. Sobre o mesmo lado da reta AC. são construídos os triângulos isósceles DAB, BCE, AFC com ângulos retos em A. C, F, respectivamente. m’+n’ 2 2a + (c-b)i 2 . . ., 2b + (a-c)i , 2c + (b-a)i ’ 2 6 C 2 ' Os triângulos A'B'C e ABC possuem o mesmo baricentro se e somente se a'+b'+c' a + b + c 3 ~ 3 2a + 2b + 2c + (c -b + a-c + b-a)i 2 Usando a fórmula de rotação, obtemos n' = a + (c-a)i e Solução. Denote por letra minúscula a coordenada de um ponto denotado pela respectiva letra maiúscula. 218 Números Complexo de A a Z Prove que os pontos D, E, F são colineares. Figura 5.9. Solução. Denote por letras minúsculas as coordenadas dos pontos denotados pelas respectivas letras maiúsculas. Como ABCD é um paralelogramo, temos a + c = b + d. Problema 14.__________________________________________________________ Sobre os lados AB e CD do paralelogramo ABCD são construídos externamente os triângulos equiláteros ABE e CDF. Sobre os lados AD e BC são construídos externamente os quadrados de centros G e H. Prove que EHFG é um paralelogramo. Solução. Denote por letras minúsculas as coordenadas dos pontos denotados pelas respectivas letras maiúsculas. A fórmula de rotação nos dá d = a + (b-a)(-i), e = c + (b-c)i, a = f + (c-f)i.e = c + (b - c)i, Então f = = a+.c-+(.a_-£)i=^£ 1-i 2 2 logo, os pontos F, D. E são colineares. 5 | Problemas Importantes para Olimpíadas 219 As rotações de 90° em torno dos centros G e H levam os pontos A e C até os pontos Observe que g + h = a + c e e + f = Problema 15. C '/i Figura 5.10. OU Então m = e Seja ABC um triângulo retângulo com C = 90° e seja D o pé da altura que parte do vértice C. SeMeN são os pontos médios dos segmentos [DC] e [BD], prove que as retas AM e CN são perpendiculares. Solução. Considere o plano complexo de origem no ponto C e sejam a, b, d. m. n as coordenadas dos pontos A, B, D, M, N, respectivamente. b + d n =-------= 2 d ab 2 “ 2(a + b) gj D e B, respectivamente. Então, d-g = (a- g)i e b-h = (c-h)i, assim g = ——- a + c-(b + d)s 1-e 1—s logo, EHFG é um paralelogramo. Os triângulos ABC e CDB são semelhantes de mesma orientação, assim a-dO-d . ab= ou d =------- . d-0 d-b a+b 2ab + b2 2(a + b)’ e = cos 60° +isen60° . Assim e =-?—— e f = -——. 1-E 1-E d + b-(a+c)i (a + c)-(a+c)i í-1 T^i a + c-(ac)E b-ci e h =------- . 1-i As rotações de 60° em torno dos centros E e F levam os pontos B e D aos pontos A e C, respectivamente. Então, a-e = (b-e)s e c-f = (d-f)r., onde 220 Números Complexo oe A A Z logo AM 1 CN . (m -a) (n -c) = -a A conclusão segue da Proposição 2 na Seção 4.1. como queríamos provar. Problema 17. onde e = cos 60° +isen60° . O ponto B situa-se sobre o segmento [AC]. Os triângulos equiláteros ABE e BCF são construídos sobre o mesmo lado da reta AC. Se M e N são os pontos médios dos segmentos AF e CE, prove que o triângulo BMN é equilátero. Problema 16.__________________________________________________________ Seja ABC um triângulo equilátero com circunraio igual a 1. Prove que para qualquer ponto P sobre o circuncirculo temos PA2 tP32 +PC2 = 6 . 2a+ b 2(a + b) Solução alternativa. Usando as propriedades de produto real da Proposição 1, Seção 4.1, e levando em consideração que CA X CB , temos ab 2(a + b) ab 2abTb2 2(a + b) Solução. Denote por letras minúsculas as coordenadas dos pontos denotados pelas respectivas letras maiúsculas. O ponto E é obtido do ponto B por meio de uma rotação de 60° em torno do centro A, assim e = a + (b-a)c, Analogamente, f = b + (c - b)e . 2ab + b2 ] 2(a+b)J 2 (ab) = 0- = (z - l)(z - 1) + (z - e)(z - e) + (z - e)(z - e 2) = 3|z|2 - (1 + e + e2)z - (1 + ê + ê2)z + 1 + | e|2 + | e212 = 3 - O.z - O.z + 1 + 1 s =argHHPortanto, arg——~ = arg Solução. Considere o plano complexo tal que as coordenadas dos pontos A, B,C são as raizes cúbicas da unidade 1,e,e2 , respectivamente, e seja z a coordenada do ponto P. Então, |z| = 1 e assim temos PA2 +PB2 +PC2 =|z-1|2 +|z-e|2 +|z-e2| 2a+ b a —------ r 2(a + b) L 2a+ b ■ b----------------l 2(a + b) 2215 | Problemas Importantes para Olimpíadas As coordenadas dos pontos M e N são n =m = e (c + a -2b)s + (b -a)s2 . Isto é, a-b = (a-b)e +(b-a)(e-1). como queríamos provar. Figura 5.11. 2 2 Logo, AM ± MN e AM = NM, como queríamos provar. Solução. Considere o plano complexo com centro em O tal que 1, i, -1, -i sejam as coordenadas dos pontos A, B, C, D, respectivamente. a -m n - m 1--2 se e somente se a-b + (c-b)s = Problema 18,__________________________________________________________ Seja ABCD um quadrado de centro O e sejam M, N os pontos médios dos segmentos BO, CD, respectivamente. Prove que o triângulo AMN é isósceles e retângulo. 2-i -1 - 2i c + a + (b-a)e 2 a + b + (c-b)r. 2 n = ——-, assim 2 Basta provar que ——- = e. Certamente, temos m-b = (n-b)s n - b Os pontos M e N têm coordenadas m = e 222 Números Complexo de A A Z Problema 19. Solução. Os triângulos A1A2B3,A2A3B,,A3A1B2 são semelhantes de mesma ou Suponha que a primeira condição é válida. 2 q +e2a2), i.e., (23a IMO) No plano do triângulo não-equilátero A,A2A3 considere os pontos B,,B2,B3 tais que os triângulos A,A2B3, A2A3B.| e A3A,B2 são semelhantesde mesma orientação. Prove que o triângulo B,B2B3 é equilátero se e somente se os triângulos A,A2B3, A2A3B,, A3A,B2 são isósceles em relação âs bases A,A2 , A2A3, A3A, e o ângulo da base mede 30°. O triângulo B,B2B3 é equilátero se e somente se b, + sb2 + s2b3 = 0 ou b, + eb3+e2b2 = 0 . :2z(a, -a2) = 0, i.e., ca, +E2a, -s2a2 j = 0 . Problema 20.__________________________________________________________ As diagonais AC e CE de um hexágono regular ABCDEF são divididas pelos pontos interiores MeN, respectivamente, de tal forma que AM CN -----=------= r. AC CE Determine r sabendo que os pontos B, M, N são colineares. Então, temos b, + sb2 + E2b3 = 0 se e somente se a3 + z(a2 -a3) + Ea1 +sz(a3 -a,) + e2a2 ■ a 3 + sa^ + E2a2 + z(a2 ~ a3 + sa3 — orientação, assim —f—— = ——— = ——— = z . Então, al -a2 a2 ~a3 a3 " a1 b3 =a2+z(a,-a2), b, = a3+z(a2-a3), b2 = a, +z(a3-a,). — , ------- - . _ _loz = +------- ------------------------ (1-e)(a3 +ea, +e2a2) o que mostra que os triângulos A,A2B3, A2A3B, e A3A,B2 são isósceles com ângulos da base medindo 30°. Perceba que a3 + Ea, + e2a2 * 0 , uma vez que o triângulo A^jAj não é equilátero. A última relação é equivalente a <=> z[a2 (1 -e)(1 + e)-a1e(1-E)-a3 (1 -e)] = -(a3 -s-ea, + a 3+ ea, + e a2------_ _1_ _ JL.(COS30°-i-isen30°), 2235 | Problemas Importantes para Olimpíadas D c N M E BO 1-r r -1 + is/3(2r-1) “ -(1 + 3r) + i7ã(1-r) 1 + 1-73 2 se e somente se 73(1- r)-(1 + 3r) - 73 (2r -1) = 0 . e n = s2r + e (1 — r), n-1 = e3r+ s(1-r)-1 = -r + e U’ F A Figura 5.12. m-1 assim ’ 1 as coordenadas dos pontos M e N são m = sr + e5 (1 - r) respectivamente. Solução. Considere o plano complexo de origem no centro do hexágono regular de tal forma que 1,e,e2,e3,e4,e5 são as coordenadas dos vértices B, C, D. E. F, A, onde , , 1 1 Isso é equivalente a 1 — r = 6r — r — 1, i.e., r = —. Segue que r = —=■ . 3 v3 *-í^(1-r). 2 2 r MC NEComo-----= — MA NC m — 1 • Os pontos B, M, N são colineares se e somente se -------e R . Temos n-1 cr- e2 (1 — r) — 1 (l-r) = -l + ^(2r-1) e c = cos — + isen— = 3 3 m -1 = cr + e5 (1 — r) — 1 = 1 + ix/3 -1 + i73 2 r 2 l±^(1-r)-1 = _! 2 V 1 2 224 Números Complexo de A a Z d- = 0. provar. |4a — s| = |4b — s|. ¥ s-a 3 ' Solução. Considere a, b, c, d, g as coordenadas dos pontos A, B, C, D, G, respectivamente. Usando as propriedades de produto escalar de números complexos, temos GA ± GD se e somente se (a -g) (d -g) = 0 , i.e., Problema 22.__________________________________________________________ Considere um quadrilátero convexo ABCD com os lados opostos AD e BC não paralelos. Sejam G,,G2,G3,G4 os baricentros dos triângulos BCD, ACD, ABD, ABC, respectivamente. Prove que se AG, = BG2 e CG3 = DG4 , então ABCD é um trapézio isósceles. Problema 21._________________________________________________________ Seja G o baricentro do quadrilátero ABCD. Prove que se as retas GA e GD são perpendiculares, então AD é congruente ao segmento de reta ligando os pontos médios dos lados AD e BC. a + b + c + d 4 0 e, assim, obtemos s-b 92 = — s-c 93=— s-d 9í~ Solução. Denote por letras minúsculas as coordenadas dos pontos denotados pelas respectivas letras maiúsculas. Definindo s = a+ b + c + d, obtemos A relação AG, = BG2 pode ser escrita como |a-g,| = |b-g2|, isto é, ! a + b + c + d'l a-------------------l 4 ) Isto é. (3a - b - c - d) (3d - a - b - c) [a-b-c + d + 2(a-d)]-[a-b-c + d-2(a-d)] = 0. A última relação é equivalente a e N são --- d e - ^C , assim a relação (1) mostra que MN = AD , como queríamos b + c + d 9’-------~ (a + d-b-c) (a+ d-b-c) = 4(a-d) (a-d), i.e., [a*d -■-*C| =|a-d|2.(1) Sejam M e N os pontos médios dos lados AD e BC. As coordenadas dos pontos M 2255 | Problemas Importantes para OumpIadas Isto é, 2 Obtemos aa - ad - ad + dd = bb - bc - bc + cc, i.e., |a — d|2 = |b - c|2. Assim AD = BC. (3) Somando as relações (1) e (2), obtemos |c|2 j = (a-b-d + c)-(a -rb + c + d). (2a ~ 2c)(2b ~ zo) = -(2a ~ ZB) (2o ~ 2c) - (2a ~ 2o) (2c ~ 2b ) Usando o produto escalar de números complexos , a última relação é equivalente (4a-s)(4a -s) = (4b -s) (4b -s), i.e., 16|a|2 -8a • s = 16|b|2 -8b s . |d|2) = (a-b-c + d)-(a+b + c + d). d|2 -|b + c|2 , i.e., 2(aã - bb - cc + dd) = ac + ad + ãd + dd - bb - bc - bc - cc. 2(|a|2-|b|2-|c|2 (|a|2-|b|2-|c|2+|d|2) = |a+ 2(|a|2-|b|2-|d|2 Encontramos 2^|a|2-|b|2) = (a-b) s. (1) Analogamente, temos Problema 23.___________________________________________________________ Prove que em qualquer quadrilátero ABCD, AC2 BD2 = AB2 CD2 +AD2 BC2-2AB BC CD• DA■ cos(A + C). (Relação de Bretschneider ou primeira generalização do Teorema de Ptolomeu) CG3 = DG4 se e somente se 2 Qc|2 — |d|2) = (c — d) • s. (2) Subtraindo as relações (1) e (2), encontramos Solução. Sejam zA,zB,zc,zD as coordenadas dos pontos A, B, C, D no plano complexo com origem em A e com o ponto B sobre o eixo real positivo (veja a Figura 5.13). Usando a identidade e, de modo semelhante, encontramos AC = BD. (4) Das relações (3) e (4) deduzimos que AB || CD e, consequentemente, ABCD é um trapézio isósceles. Números Complexo de A a Z226 Figura 5.13. Basta provar que z + z = -2AB BC • CD • DA • cos(A + C). e assim está provado. Observação Já que cos(A +C) > -1, essa relação dá a desigualdade de Ptolomeu, ACBDs AB DC + AD BC, com a igualdade ocorrendo apenas para quadriláteros inscritíveis. Temos zA -zB = AB(cosrr + isenrc), zD -zc = DC[cos(2n-B -C) + isen(2rr-B -C)], zA -zD = DA[cos(n - A) +lsen(7t-A)] e zc -ZD = BC[cos(n + B) + isen(n + B)] . Então z + z = 2Rez = 2AB BC CD DAcos(5n- A - C) = -2AB • BC ■ CD ■ DA • COS (A + C) desenvolvendo o produto, obtemos AC2 ■ BD2 = AB2 • DC2 + AD■BC2 + z + z, onde z = (zA -zB)(z0-zc)(zA-zD)(zc - zB). e (zA "Zc)(zB "Zo) = "(za -zB)(zD -zc)-(zA - zD)(zc - zB), 2275 | Problemas Importantes para Oumpíadas por isso 2 Re (zA - zB) (zA - zD) (zc - zB )(zc - zD) = 2abcdcos(A + D + D + C) = 2abcdcos(2rr-B + D) = 2abcdcos(B-D) Problema 24. Seja ABCD um quadrilátero com AB = a. BC = b, CD = c, DA = d, AC = d, e BC = d2. Prove que d|[^a2d2 + b2c2 -2abcdcos(B-D)J = d2|^a2b2 +c2d2 -2abcdcos(A -C)j . (Segunda generalização do teorema de Ptolomeu) Solução. Sejam zA,zB,zc,zD , respectivamente, as coordenadas dos pontos A. B, C, D no plano complexo de origem em D e o ponto C no eixo real positivo (veja a figura no problema anterior, porém com notação diferente). Multiplicando as identidades (zB -zD)[(zA -zB)(zA -zB)-(zc -zD)(zc -zD)] = (zc “za)’[(zb -za)(zb -zc)-(zd “za)(zd -zc)] e (ZB - ZD )[(ZA - ZB )(ZA - ZO ) - (ZC - ZB )(ZC " ZO )] = (ZC - ZA ) }(ZB - ZA )(ZB - ZC ) - (ZD - ZA )(ZD " ZC )] além disso df [a2 d2 + b2 ■ c2 - (zA - zB) (zA - zD )(zc - zB )(zc - zD) -(zc-ZbXzc-ZoÍÍZa-ZbXza-Zo) d? [a2 • b2 + c2 ■ d2 -(zB -zA)(zB -zc)(zD -zA )(zD -zc)] —(zo -za)(Zo -ZcXzb -za)(zb -Zc)} Basta, então, provar que 2Re(zA -zB)(zA -zD)(zc -zB)(zc -Zo) = 2abcdcos(B-D) e 2Re(zB - zA )(zB - zc )(zB - zA )(zD - zc) = 2abcdcos( A - C). Temos zB - zA = a[cos(n + A + D) + isen(rr + A + D)], zB -zc = b[cos(n-C) + isen(n-C)], zD-zA = d[cos(n-D) + isen(rr-D)J, zD-zc =c[cosn + isenrr], zA-zB = a[cos(A + D) +isen(A + D)]. zA -zD = d[cosD +isenD], zc -zB = b[cosB +isenB], zc - z0 = c [cos 0 + isenO]; 228 Números Complexo de A A Z (1) e, consequentemente A relação se torna d2|jad + bc)2 -2abcd(1- Problema 25.__________________________________________________________ Sejam três triângulos equiláteros OAB, OCD e OEF em um plano. Prove que os pontos médios dos segmentos BC, DE e FA são os vértices de um triângulo equilátero. Se ABCD é um quadrilátero inscritlvel, então B + D = A + C = rr. Segue que cos(B-A) = cos(2B - n) = -cos 2B e cos(A-C) = cos(2A -n) = -cos2A. cos2B)j = d2 j\ab + cd)2 - 2abcd(1-cos2A)J. d = cs, d + e CE + e f+a n =-------—--------- , p =-------2 2 K 2 Que é equivalente a d2 (ad + bc)2 - 4abcdd2sen2B = d2 (ab + cd)2 - 2abcdd2sen2A. A lei dos senos aplicada nos triângulosABC e ABD de circunraios iguais a R nos dá d, = 2RsenB e d2 = 2RsenA , portanto, d,senA = d2senB . A relação (1), então, é equivalente a e 2 Re(zB - zA) (zB - zc) (zD - zA )(zD - zc) = 2abcdcos(n + A + D + 7t-C + n-D + n) = 2abcdcos(4n +A - C) = 2abcdcos(A - C), como queríamos provar. Observação ---------------------------------------------------------------------- d2(ad + bc)2 = df (ab + cd)2 d2 ab + cd d, ad + bc A relação (2) é conhecida como o segundo teorema de Ptolomeu. Solução: Considere o plano complexo com origem em O e assuma que os triângulos OAB, OCD, OEF são orientados de modo positivo. Indicando por letra minúscula a coordenada de um ponto indicado pela respectiva letra maiúscula. Seja e = cos 60° +isen60°. Então: b = ac, d = ce, f = eE, b + c ac + c d + e CE + e f+a es + a e m =-------=--------- , n =------- =--------- . p =-------=--------- . 2 2 2 2 2 22 2295 | Problemas Importantes para OumpIadas Assim, m + s2n + e4p = m + e2n - ep e está provado o que se pede. Pedimos ao leitor que resolva os problemas seguintes usando números complexos. Problema 26. Sejam os pontos A”, B", C" sobre os segmentos B'C',C'A',A'B' tais que: Prove que os triângulos ABC e A"B"C"são semelhantes. Problema 28. Problema 29. A"C A"B' C"B C"A' B'C B’A O triângulo MNP é equilátero se e somente se m + ian + <o2p = 0, onde <o = cos120°+isen120°= c2 Seja ABC um triângulo tal que AC2 + AB2 = 5BC2. Prove que as medianas que partem dos vértices B e C são perpendiculares. A'B A'C Seja ABCD um quadrilátero inscrito em um circuloC(O;R). Prove que AB2 + BC2 + CD2 + DA2 = 8R2 se e somente se AC ± BD ou uma das diagonais é diâmetro de C. Problema 27.__________________________________________________________ Sobre os lados BC, CA, AB do triângulo ABC são escolhidos os pontos A, B', C tais que: = —(as + c - c + es2 - es2 - ea) = 0, CA , ------= k C'B B"A' , -------= k B"C R m„ Prove que em qualquer triângulo a desigualdade seguinte é válida — à —— , com a igualdade ocorrendo somente para triângulos equiláteros. 230 Números Complexo de A a Z Problema 31. Problema 33. Problema 34. Prove que os pontos médios das diagonais dos quadriláteros ABCD e MNPQ são os vértices de um paralelogramo. AD + BC cos (A + B) = ABcos A + CDcosD BCsen(A + B) = ABsenA-CDsenD Problema 30._________________________________________________________ Sobre os lados do quadrilátero convexo ABCD são desenhados os triângulos equiláteros ABM, BCN, CDP e DAQ externamente à figura. Prove que os quadriláteros ABCD e MNPQ tem o mesmo baricentro. Seja ABCD um quadrilátero e considere as rotações R,,R2,R3,R4 com centros A, B, C, D através do mesmo ângulo a e de mesma orientação. Os pontos M, N, P, Q são as imagens dos pontos A, B, C, D sobre as rotações R2.R3.R4.Rv respectivamente. se e somente se  = —. 3 Sejam Og,l,G 0 centro do círculo dos nove pontos, o incentro e o baricentro, respectivamente, do triângulo ABC. Prove que as retas O9G eAI são perpendiculares Sejam dois círculos <n, e co2em um plano, com centros O, e O2, respectivamente. M, e M2 são dois pontos de m, e <o2 .respectivamente, tais que as retas O-iM, e O2M2 se intersectam. M, e M2 são pontos de o>, e co2, respectivamente, tais que os ângulos M^M, e M2O2M2 .medidos no sentido horário, são iguais. Determine o lugar geométrico do ponto médio de [M,M2] Seja P o ponto de interseção das retas O^, eO2M2. O circuncirculo do triângulo M,PM2intersecta o circuncirculo do triângulo O,PO2 em P e em Q. Prove que Q é fixo, i.e., independente da posição de M,eM2. (Olimpíada Vietnamita de Matemática de 2000) Problema 32,_________________________________________________________ Prove que em qualquer quadrilátero ABCD inscritivel ocorrem as seguintes gualdades: 2315 | Problemas Importantes para Olimpíadas Problema 35. Problema 36. (27a IMO) (21a IMO) (19a IMO) Problema 39. Os triângulos isósceles A3A,O2 e A,A2O3 são construídos externamente ao longo dos lados do triângulo A,A2A3 com O2A3 = O2A, eO3A, = O3A2 . Seja O, um Sejam um triângulo A,A2A3 e um ponto Po em um plano. Definimos As = As_3 para todos>4. Construimos uma sequência de pontos P0,P1,P2,... tais que Pk+1 é a imagem de Pk por meio de uma rotação com centro em Ak_, (k = 0,1, 2,...) através de um ângulo de 120° no sentido horário. Prove que se P1986 = Po então o triângulo A,A2A3 é equilátero. Seja ABC um triângulo equilátero e M um ponto interior ao ângulo BAC . Os pontos D e E sâo as imagens dos pontos B e C por meio de uma rotação de 120° no sentido anti-horário e horário, respectivamente, com centro em M. Prove que o 4° vértice do paralelogramo de lados MD e ME é o simétrico do ponto A em relação á M. Problema 40.___________________________________________________________ Prove que para qualquer ponto M inserido no paralelogramo ABCD é válida a Problema 37.__________________________________________________________ Duas circunferências se intersectam em um plano. Seja A um dos pontos de interseção. Duas partículas partem simultaneamente de A com velocidades constantes, cada partícula percorrendo sua própria circunferência na mesma direção. Depois de uma volta, as duas partículas retornam simultaneamente ao ponto A. Prove que existe um ponto P, fixo no plano, tal que em qualquer hora as distâncias de P às posições de cada partícula são iguais. Problema 38.__________________________________________________________ Dentro do quadrado ABCD, estão inseridos os triângulos equiláteros ABK, BCL, CDM, DAN. Prove que os pontos médios dos segmentos KL, LM, MN, NK e os pontos médios dos segmentos AK, BK, BL, CL, CM, DM, DN, AN são os vértices de um dodecágono regular. --------- 1-------- ponto no lado oposto da retaA2A3, partindo de A,, com e O,A3A2.= — A,O3A2 e T é o pé da perpendicular partindo de O, até A2A3 . Prove que A,O, _L O2O3 e que AP, _2 O,T O2O3 A2A3 (Olimpíada Iraniana de Matemática de 2000) 232 Números Complexo de A à Z seguinte desigualdade: MA ■ MC + MB- MD a AB■BC. (39a IMO - Lista de Problemas) Problema 42. 5.5 RESOLVENDO PROBLEMAS TRIGONOMÉTRICOS Problema 1. z + z3 + z5 + z7 + z9 = p 2 8 • Seja ABC um triângulo tal que ACB = 2ABC. D é um ponto do lado BC tal que CD = 2BD . O segmento AD é prolongado até E de tal modo que AD = DE. Prove que: ÉCB + 180°= 2ÉBC . (39a IMO - Lista de Problemas) 1_ 2 Problema 41._________________________________________________________ Seja ABC um triângulo, H é o seu ortocentro, O é o seu circuncentro e R é o seu circunraio. O ponto D é o simétrico de A em relação à BC, E é o simétrico de B em relação â CAe F o simétrico de C em relação à AB. Prove que D, E, F são colineares se e somente se OH = 2R . Problema 2.____________________________________ Calcule o valor do produto P = cos20°-cos40°-cos80° Solução 1.Definindo z = cos20°+isen20° temosque z9 = -1, z = cos20° - isen20° _ n 3ít 5rt 7n 9aProve que cos— 4-cos— + cos— + cos— + cos—11 11 11 11 11 Z9 4-1 Z4 4-1 Z9 4-1e, cos 20° = ----- cos 40° = ±-—-, cos 80° = .Então,2z 2z2 2z“ (z2 4- l)(z4 4-l)(z® 4-l) (z2 -l)(z2 +1)(z4 +1)(z8 +1) 8z7 8z7(z2-l) z16-1 _ —z7—1 8(z9-z7) 8(-1-z7) Solução: Definindo z = cos^-r-isen^ temos que: z11-z -1-z 1 z2 -1 z2 -1 1-z Tomando a parte real de ambos os lados da igualdade encontramos a resposta. 5 | Problemas Importantes para Olimpíadas 233 Problema 3. cos x + cos y + cos z = 0 •e cos2x + cos2y + cos2z = 0 .e — =-2z,z2z3(z, + z2 + z3) Problema 4. ,2 .10 .14 A identidade é equivalente a 2z Sejam x.y.z números reais tais que senx + seny + senz = 0 Solução 2. Trata-se de um problema clássico com solução também clássica. Seja S = cos 20 cos 40 cos 80 , então: 3 2 - - 2z3z2z3(z1 + z2 + z3) — 0 . Portanto, (cos2x+ cos2y+ cos2z) + i(sen2x + sen2y + sen2z) = 0 e a conclusão se torna óbvia. Prove que sen 2x + sen 2y + sen 2z = 0 Q Prove que cos210° + cos2 50°+cos2 70°= — . 2 Solução. Definindo z = cos 10°+isen10° , temos que z9 =ie :24 + 2z14 + z4 + z: S sen20 = sen20cos20cos40cos80= ^sen40cos40cos80 1 11= —sen80cos80 = —cos160 = —sen20. 4 8 8 z10+1 2z5 ' ' z10 +1? 2z5 J .28 Z” +1 2z7 ' x2 z" +r 2z7 cos 10°=-------- , cos 50° = 2z LogoS = —. 8 Perceba que essa solução clássica é imaginária, sem qualquer motivação. A solução por meio de números complexos, no entanto, é uma conta simples e direta. Isto é, z16 + 2z14+ z12+ 2z14 + 1 = 6z14,i.e. Solução. Definindo z, = cosx + isenx, z2 = cosy-s-iseny, z3 = cosz + isenz , nos temos z, + z2 +z3 = 0 e |z-|| = |z2| =|z3| = 1 Assim, z? + z| + z| = (z, + z2 + z3)2 -2(z3z2 + z2z3 + z3z,) = - 2z,z2z3 ÍA + A + = _ 2z,z2z3 (z, + z2 + z3) \Z1 Z2 Z3 J .14 COS70°= 234 Nümeros Complexo de A à Z Isto é, Problema 5. Resolva a equação cos x + cos 2x - cos 3x = 1 Solução: Definindo z = cos x+ isenx temos Que é equivalente a (z6 -z5-z4 -z3) + (z3 -z2 -z + l) = 0 | k e Z k Portanto, x e {kn | k e Z) |keZ Problema 6. e Sabendo quez ou n T = S q ■senkx. k=1 .18 equivalentemente (z4 +l)(z n + 2kn 3 Assim, z = 1ou z = -1 ou z3 =-1 e, consequentemente, x e {2kn | k e 7/,} ou cosx = - 2z A equação pode ser reescrita como ou (z3 +l)(z3 -z2 - z + l) = 0. Finalmente (z3 +l)(z-1)2(z + 1) = 0 cos2x =-----z-, 2z2 n Calcule as somas S = £ q* coskx k=l cos3x = Z t1 . 2z3 Z2 +1 z4 +1 z6 +1 . . 4 2 5 6 4 n 3 n ------- +----- 5----------5- = 1, l.e., z4 + z2 + z5 + z - z -1 - 2z = 0 . 2z 2z2 2z3 Z2B + Z16+ z24 ■^ = 0,(z4+l)(z,e o que é óbvio. = -1, obtemos z16 +z12 -z10 -z6 +z4 +1 = 0 :12-z6+l) = 0. :12+z4+1 = 0. x e {n + 2kn|k eZ} ou x e {• '2k + 1 ---------- 71 2 5 | Problemas Importantes para OumpIadas 235 ,n-r1 Observação e sen (2n + 1)x 2 sen(nx)-sen(n + 1)x + senx 2(1-cosx) n De fato, temos E coskx = k=1 n e senkx k=1 n Xcoskx = k=1 X COS------ COS 2 X sen — 2 nx sen — cos 2 nx sen—sen 2 x -sen— 2 nx sen—cos 2 Solução. Temos que 1 + S + iT = E çf (cos kx + isenkx) = E qk (cos x + isenx)k k=0 k=0 n ysenkx = k-1 -2sen2 — 2 [cos(n + 1)x + isen(n + 1)x] 1-qcosx-iqsenx cos (n +1) x - iqn*1sen(n +1) x] [1 - q cos x + iqsen x] q2 -2qcosx +1 x sen- 2 (2n + 1)x 2 (n + 1)x 2 (2n + 1)x 2 4sen2 — 2 _ [i-q' nx sen — sen 2 o x 2sen —sen 2 1-qn’1 1 - qn+1 (cos x + isenx)0*1 1-qcosx-iqsenx 2sen- 2 „ X X „ X2sen—cos — 2sen—cos 2 2 2 4sen2 — 2 (n + 1)x 2 (2n + 1)x 2 2sen —2 q"*2; e T = —— (n + 1)x 2 x sen —2 Se q = 1, então encontramos os conhecidos somatórios: (n+1)x 2 x sen— 2 cos nx - cos(n +1) x - (1 - cos x) 2(1-cosx) , „ q 2cosnx-q 1cos(n+ 1)x-qcosx+1 por isso 1 + S =------------------- - ------- -i------ ------------------- q -2qcosx + 1 !sen(nx) - qn*’sen(n + 1)x + qsenx q2 -2qcosx + 1 Números Complexo de A a Z236 Problema 7. Essa expressão se reduz à z6 - z4 + z2 -1 = iz3 . Já que z5 = i, Problema 8. (5a IMO) z * 1 e produto acima é zero. A condição é equivalente a z(z2 - z + l) = —ly. A soma dada é 1-z 3a Prova. 1 1-z Os pontos A,,A2 A10 estão igualmente distribuídos em uma circunferência de raio R (nessa ordem). Prove que/\^t - A,A2 = R . 2 2 2 2 1 _______ 1_______ 1-(cost + isent) (l-cost)-isent Portanto, temos de provar que Rei —1 = 11-z J z6 -1 z2 — 1 , a relação anterior é equivalente a z8 - z6 + z4 - z2 +1 = 0 . Contudo, isso é verdade 7t 2n _ / 3 o \ cos—cos— + cos— = Re z -z +z7 7 7' > Solução. Seja z = cosy^ + isen^ • Sem Perda de generalidade, tomamos R = 1. . Essa relação segue do lema abaixo: Lema. Se z = cost + isent ez # 1, então Re—— 1-z ,, , ir 2n 3nMostre que cos y-cos—+ cos— pois (z8 - z8 + z4 - z2 + l)(z2 +1) = z' Precisamos, então, mostrar que 2sen^--2sen^ = 1, z2 — 1 Definindo z = cosa+ isena , entãosena =--------. Teremos de provar que 2iz :’°+1 = 0e z2+1*0. Solução. Seja z = cos-y + isen-y . Entãoz7+1 = 0. Já que z7 +1 = (z +1)(z6 - z5 + z4 -z3 +z2 -z + l) = 0 segue que o segundo fator do 5 | Problemas Importantes para OumpIadas 237 1 Problema 9. ,n+1 nós obtemos as fórmulas: (1) ncos (2) Usando a relação (2) obtemos: 2sen 2°+4 sen 4°+... +178 sen 178°= 2(sen2°+2sen2 2°+... + 89sen89-2°) = 2 = 90 cot 1o . Problema 10. Seja n um inteiro positivo. Ache os números reais a0 e aw,k,l = 1,n, k > I. tais que: n X kcoskt = k=1 n y ksenkt = k=1 1-cos(n + 1)t 4sen2 — 2 sen(n + 1)t 4sen2 — 2 90cos179° sen1° (n + 1)sen t cos- 2 2sen— 2 X, ak, cos2(k-l)x 1sl<ksn sen90-2° 4sen21° -z")] = t . t sen—+ icos—2 ? „ t2sen—2 (2n + 1)t 2 2sen-2 2=l + i 2 2sen2 --2isen-cos—2 2 2 :nt1-z2) + ... + (z' (2n + 1)t 2 2sen- 2 ^n+l 1 Finalmente, —(2sen2°+4sen4°+... + 178sen178°+180sen180°) = cot1° 1_________ sen--icos—| 2 2J -z) + (z' -z(z-<7■íHí2"' Prove que a média aritmética dos números ksenk°(k = 2,4,6,...,180) é igual a cot1° Solução. Defina z = cos t + isent. Da identidade z + 2z2 + ... + nzn =(z + ... + zn) + (z2 +... + zn) + ... nzn*1 z-1 2sen-| 2 2' sen2nx sen2x 0 90cos179° 2sen1° J 238 Números Complexo de A â Z 2 nós obtemos S2 + S2 = por isso , e Problema 12. cos 2pa = a0 + a, sen2i para todo real x * ma, m e Z. (Competição Regional de Matemática da Romênia "Grigore Moisil", 1995). = n + 2 £ (cos2kxcos2lx + sen2kxsen2lx) =n + 2 £ cos2(k-l)x, 1 sl<kín 1 ^l<k^n sen(nx) senx sen(nx) senx fc)+cos: para qualquer inteiro n á 2 . (n~1)n n Problema 13.__________________________________________________________ Para qualquer inteiro p > 0 existem números reais a0,a, ap com ap * 0 tais que ... + ap -(sen2a)P, para todo a e R . 2 = n + 2 £ cos2(k-l)x Definindo ap = n e au = 2. 1 < I < k ê n, o problema está solucionado. Seguem mais alguns problemas. Por outro lado, S2 +S| = (cos2x + cos4x + ... + cos2nx)2 +(sen2x + sen4x +... + sen2nx)2 ... + cos2nProve que 1 + cos2n = n4'n(2 + (n)) Problema 11. Calcule a soma das duas séries n-ésimas a seguir para 0 = 30° : i) cosG cos (20) cos (30) cos((n-1)0) cos0 cos2 0 cos3 0 cosn-10 ii) cos0cos0 + cos2 0cos(20) + cos3 0cos(30) + ... + cosn 0cos(n0) (Olimpíada Crux de Matemática, 2003) „ n sen(nx)cos(n+ 1)x Solução. Usando as identidades S, = £cos2jx =-------—------ ------- — j=i senx „ n sen(nx)sen(n + 1)x e S2 = £ sen 2jx =------------------------ — j=i senx 2395 | Problemas Importantes para Olimpíadas 5.6 MAIS SOBRE RAÍZES ENÉSIMAS DA UNIDADE Problema 1. Seja n > 3 e k > 2 inteiros positivos e considere o número complexo e 0 = 1-Z + Z2-Z3+... + (-1)‘ a) Se k è par, então + isen ou e 27. -1= 0. i.e.,A última relação é equivalente a cos ou Isto é, sen2 — = cos2 — n n aklcos2(k-l)x 1 S I < k < n 2(k + 1) n kn sen — ____n_ kn cos — n kn sen — n 7t cos — n n — cos Temos |0| = 1 se e somente se |sen—| = |cos—| (k-1)n n a) Se k é par, prove que 0n = 1 se e somente se n for par e dividir k - 1 ou k+1. b) Se k é impar, prove que 6n =1 se e somente se n dividir k - 1 ou k + 1. e |e| = 2k 2hz = cos— + isen — n n 2kn 2n ncos------ 1- cos— = 0 . n n (Li^cos^ n n —— e 2Z + 1. Isto é equivalente à afirmação de n ser par e — dividir k - 1 ou n 2 k + 1.Portanto, é suficiente provar que 9" = 1 é equivalente à |0| = 1. Aimplicação direta é óbvia. Reciprocamente, se|fi| = 1. então n = 2t,t e Z., com t dividindo k - 1 ou k + 1. Já que k é par, os números k - 1 e k + 1 são impares, por isso t = 2I +1 e n = 4I + 2,l e Z. )k-1zk-' sen2nx sen2x 0 1 + (-1)k-1zk Solução. Já que z * -1, temos 0 = —-—-------- (k-1)n cos-5------ — n Números Complexo de A a Z240 e 0n = -cos(k — 1)ti = 1,Então, 0 = ±i cos 1 + zk0 = logo = 0 , i.e., n divide k-1 ou k+1.Dai. sen k = nt ± 1 e 0 = (-1)1 + isencos Dai, 0n = (-1)k±1 (cos(k -1) = 1, como desejado. Problema 2. Solução. Perceba que s3 = 1, E2 + E +1 = 0 e 1987 = 662 ■ 3 +1. Então como desejado. Se k é impar, então Basta provar que 0" =1 é equivalente a |9| = 1. Já que a implicação no sentido direto e óbvia, vamos provar a recíproca. Se |@| = 1, então k ± 1 = nt, t e 7.. Então (k + 1) n (k-1) n kn cos— n 71 cos —n :3k+2)(l + B3k*3)](l + E1987) (k-1)* n (k-1)* n —+ isen (k-1)* n (1+E)(1+E*)...(1+E’^)=n[(i+E3k-)(i+E: (k-1)* n Isto é, cos2 —= cos2 — n n ã —sen Calcule (1 + e)[i + e2)--(i + e1987) . k-1 k-1cos----- 7r + isen----- n n n . 2kn 2kn1 + cos---- + isen----- _______n_______ n__ . 2ti 2ti1 + cos— + isen— n n k?i ( k7i k7i cos— cos — + isen — n 1 71 1 H ílcos— cos —+ isen— n Considere a raiz cúbica da unidade e = cos—+ isen — . 3 3 isen(k — 1)tt) = (-1)k±1 (-1)k~1 Temos |0| = 1 se e somente se |cos~| = |cos~| • 2k7T 271cos---- = cos----- n n kTi n n J k7t n 7T n 2415 | Problemas Importantes para Olimpíadas (1 + e) = 2* Então, 1 — e" + e2n = E2)(1_£2+EÍ)...(1_E"Portanto, (l- se n = 0(mod3), se n s 1(mod3),e. se n s 2(mod3) Problema 4. 1, -2e, -2e2, :2") = 266'!662 (-e2) = 2' se n s 0(mod3), se n s 1(mod3), se n s 2(mod3), o produto de quaisquer três fatores consecutivos do produto dado é igual a 1-(-2e)(-2e2) = 22. = 2662 Solução. Perceba que 1 + e + e2=0 1 + e-e2 = -2e2 . Problema 3, Seja e * 1 uma raiz cúbica da unidade. Calcule (1-c + e2)(i-e2+c4) ••■(l-En+e2n). Prove que o número complexo z = ^4 tem módulo igual a 1, porém, z não é uma raiz enésima da unidade, para qualquer inteiro n . = (1 + e)[2(0 + 1)]662 2n 2\ _2^] = nj(1 + c)(l + e2 )(1 +1)](1 + e) = (1 + e)[2(l + e + e2 + e3 )]6“ ,66? 1 + 'JS 2 e e3 = 1 . Portanto, 1-c + c2=-2se 242 Números Complexo de A ã 2 + (2i)".= (2-i)n |(2i-1)n'22i + ...+Que é equivalente a (2i)n = (-2 + i) = (-2 + i)(a + bi), com a,b e Z. Problema 5. Solução. Suponha que exista (1-P) = = 1, portanto 1~PeUn, como desejado. Reciprocamente, se (1 + o.) P,1-P e Un, defina 1, temos Solução. Obviamente, |z| = 1. Suponha por absurdo que existe um inteiro n>1 tal que zn = 1. Então (2 + i)n = (2 - i)n , e fazendo 2 + i = (2 -1) + 2i, temos (2 + i)n = (2 - i)n n n-1 1 (1 + a)n (1-P)n pn Tomando o módulo de ambos membros da igualdade obtemos 2" = 5(a2 +b2), um absurdo. Seja Un o conjuntos de todas as raizes enésimas da unidade. Prove que as afirmações seguintes são equivalentes. a) Existe a e Un tal que 1 + a e Un ; b) Existe |5 e Un tal que 1 - [5 e Un . (Olimpíada de Matemática da Romênia - Segunda fase,1990) pn=fA)n =\1 + aJ )n'12i+... = 1, portanto P e Un . Temos também que 1 - p = —— e a + 1 1 a e U tal que 1 + a e U Definindo p =------ , temos 1 + a 1-Pa = —jj2- . Ja que a = que a e Un e 1 + a e U„, como desejado. = 1 e (1 + a)n = —= 2435 | Problemas Importantes para Olimpíadas Observação Mostre que |1 - e| à (e1 + (n-1)) Portanto, n < |1 -e|(1 + 2 + ... +(n -1)) = |1 - e| Problema 6. Seja ní3 um inteiro positivo e seja e * 1 uma raiz enésima da unidade. n = |e-l||en' , 1 . . 2n . 2n1 + a = — ± i-— = cos— ± isen— 2 2 6 6 Já que (1 + a)n = 1, segue que 6 divide n. 2 n-1 Se k é um inteiro positivo tal que n não divide k , então I 1 sen— >----- .| n | n-1 (Olimpíada de Matemática da Romênia - Última Fase, 1988) + 2cn~3 +...+ (n-1)|<|e-l|[|cn-2| + 2|cn-3| + ... n(n-1) 2 ’ 2 i.e., nos encontramos a desigualdade |1 - e >------ . Além disso, a igualdade não é n-1 1 ■'ã Reciprocamente, se n é um múltiplo de 6, então tanto a = -^ + '-^- Quant0 As afirmações a) e b) são equivalentes com 6 | n. De fato, se a,1 + a e U„, então |a| = |l + a| = 1. Dai segue que 1 = |1 + o.|2 = (1 + a)(1 + S) = 1 + a + ã + | a |2 = 1 Vã 1 + a = 2+'~2~ Pertencem â un ■ 1 1 \/3 1+a+ã+1=2+a+—, i.e., a = — ± i —. portanto a 2 2 Solução. 1) Temos que En -1 = (e -1)(en 1 +... + e +1) .portanto, levando em conta o fato deque e « 1, encontramos en_1 +... + e +1 = 0 . A última igualdade é equivalente :n“1 -1) +... + (e -1) = -n , i.e., (e - 1)[e"'2 + 2e"’3 +... + (n-2)e + (n -1)] = -n . Tomando o módulo em ambos os lados. 244 Números Complexo de A a Z no plano complexo não são 1-cos---- Problema 7. pn = (-jn - l)(in -1) = ~(i2" _ 1) = -((-1)" _ 1) = -(-1 _ 1) = 2 Pn = possível já que os lugares geométricos de 1,s e' colineares. n (i+e) n H + e) n (M(e-i) FI |e + -| = ceUn Portanto, |1-e| Seja Un o conjunto das raizes enésimas da unidade. Prove que: 0, sen = 0(mod4) 2, sen$1(mod4) -4, se n = 2(mod4) 2, sens3(mod4) Solução. Considere o polinõmio f(x) = Xn -1 = H (X-s) seUn n * tsUn Denotando por P„o produto do problema, temos: pn= n fc+-]= n í—]= teUn \ £ / £6Un \ e y Se n s 2(mod4). então (-1)n 1 2kn 2kn . X 2kn . 2kn Considere e = cos-----+ isen------ , temos que 1 - e = 1 -cos---------isen------ . n n n n 2kn 'j2 2 2kn „ „ 2kn . 2 kn - + sen----- = 2-2cos------ = 4sen —. n ) n n n Aplicando a desigualdade em 1), encontra-se a desigualdade desejada. = -1, (- i)n = in = i2 = - 1, in = -1, portanto (-1)"f(0) (-1)"-1 (-1)n-1 Se n s 0(mod4), então i" = 1 e Pn = 0 . Se n s 1(mod2), então (-1)n~1 = 1 e 2kn n 2kn n 1 8 5 | Problemas Importantes para Olimpíadas 245 Problema 8. ) = 0 . para todo k natural. b) (2z + 1)2n'1 + (2z - 1)2n'1 =0. . A conclusão segue e assim está Problema 9. Seja n um inteiro positivo impar e e01e, en_, as raizes enésimas da unidade. Prove que para quaisquer números complexos a e b (Olimpíada de Matemática da Romênia - Segunda fase, 2000) Solução. Se ab = 0, a afirmação é óbvia, então considere o caso quando a = 0 e bsO. ,w) + 2 = o n (a + bs^) = an +bn k=0 ' Solução. Temos que m2n 1 = e 1 + ra + m2 + ... + cn2" = 0 Então —+ w + w2 +... + <i)n + <»n(co + <o2 obtemos 2z + 1 =-ran(2z-1). Levando em = 1, obtemos (2z + 1)2n+1 =-(2z - 1)2nt1 o . 2n . 2nSeja co = cos--------+ isen---------, 2n +1 2n +1 n > 0 , e seja z = — + o> + <o2 +... + oi" . 1 „ „ . 1e"-1— = 0 , portanto z =----------- 2 2c,>" +1 e z2k"1 1-^-1 Temos z = —-------= -z. Portanto z2k 24 + 1 Prove que: a) lm(z2k) = Re(z2k+1 = z2k = -z2k‘1 Se n s 3(mod4), então (-1)n-1 = 1 e Pn = (-in - 1)(i" -1) = (i3 - 1)(-i3 -1) = -(i6 -1) = -((-1)3 -1) = 2 , e assim está provado. Da relação z + ®n(z~2^ + consideração que oi2"’1 provado. 1 o>" ' dessas duas igualdades. Números Complexo de A a Z246 Problema 10. (a2 +to)-(a2 +eti). para quaisquer números complexos a e b. (Olimpíada de Matemática da Romênia - Segunda fase, 2000) Começaremos com um lema útil. Lema. Se b0,são as raízes enésimas da unidade, com n um inteiro positivo impar, então lí’(A + Bs2) = A"+Bn, para quaisquer números complexos A e B. f A" „] %1fA > -—+1 = -n—+Ek .Ib" J kJolB Temos (a-iSo)- (“-iEn-i) e portanto = Solução. Se b = 0, a afirmação é óbvia. Se não for, seja n = 2(2s +1). Seja um número complexo a tal que a2 = r e 0 polinômio b f = Xn-1 = (X-s0)(X-e1)...(X-En.l). (“ + ÍEo)-(“+icn-l) n-1 ( - ]”[ (a + bE2) = a2 +b2 n-1 £ Prova. Usando a identidade xn -1 = ]"[ (x“ek) Para x = — temos k=o B de onde segue a conclusão. Como n é inteiro, a função f :Un ->Uné bijetiva. Para provar isso, é suficiente mostrar que ela é injetiva. Certamente, assumindo que f(x) = f(y), segue que (x-y)(x + y) = 0. Se x + y = 0, então xn = (—y)n ,i .e., 1 = -1,um absurdo. Logo, x = y . n—1, — . n—1. . Do lema, temos que ]”[ (a ** bek) = fl (a *->ej) = 3n +bn . Seja n um inteiro positivo par tal que — é impar e sejam e0,e, as raizes enésimas da unidade Prove que A" Bn 5 | Problemas Importantes para Olimpíadas 247 Assim, sk2) ,2 a' Os problemas seguintes também envolvem raizes enésimas da unidade. Problema 11. Para todo inteiro positivo k definimos Uk = |z e C | zk = ij Problema 12. são reais. Problema 13. (Crux Mathematicorum, 2003) Problema 14. 1, n > 1. Prove que |z-1|2. = b2(2s*”^ Sejam a,b,c,d,a números complexos tais que ]a| = |b| * 0 e |c| = |d| * 0 . Prove que todas as raizes da equação c(bx + aa)n - d(ax + ba)n = 0 , n > 1, = b"f(^f(-^ = b"[(a2)2-+l]2=b" Suponha que z* 1é um número complexo tal que z |nz_(n + 2)|áM^ O n(a+br^b"rf^^H"np- k=0v ' k=0\U ) k-0' Prove que para quaisquer inteiros m e n com 0 < m < n temos U, oU2 u...uUm a Un_mil uUn_m_2 u...uUn. (Competição Regional de Matemática da Romênia -“Grigore Moisil”, 1997) i2-1+b: b2s*’ Seja M um conjunto de números complexos tais que se x, y e M, então — e M. Proveque y se o conjunto M tem n elementos, então M é o conjunto as raizes enésimas da unidade. Números Complexo de A A Z248 5.7 PROBLEMAS ENVOLVENDO POLÍGONOS Problema 1. Além disso, 4OA2 = 4R2 - A,Aj2 . 2 zi+2i é igual a Prove que £ z é um inteiro ZeA Prove que para qualquer inteiro k, pode-se escolher um conjunto A que satisfaz a Problema 15.___________________________________________________ _____ O conjunto finito A de números complexos tem a propriedade: z e A implica z" e A, para qualquer inteiro positivo n. (n-1)(n-2) 2 = 4R2 2 1 ■ 1si<jsn AjAj' condição acima e £z = k. zeA (Olimpíada de Matemática da Romênia - Última fase, 2003) X _!__H A A .2 \2J Sejam z1(z2 zn números complexos distintos tais que |zj = |z2| =... = |zn|. Prove 2 > Zj + z coordenada do ponto médio Ad do segmento [AA] é igual a ' 1, para 1 < i < j < n. Assim, |zj +zj2 = 4OA2 e |zj -zj2 = A,A,2 que £ ------ *■ 1íi<jín Z( — Zj A soma S 1si<j<n Zj — Zj 4OA2 1si<jsn AjAj 4R2-A,AJ2 1si<jsn AjAj2 Solução. Considere os pontos A1,A2,...,An de coordenadas z,,z2 zn. O polígono A,A2...An está inscrito na circunferência de centro na origem e raio R = |z,|. A 5 | Problemas Importantes para Olimpíadas 249 A desigualdade AM-MH nos dá az.+zi 2 £ Problema 2. a^i 4-a^i) = 2R2 = n(n-1)R2 Problema 3. , para todo p = 1,n. Temos Sejam z1,z2,...,zn as coordenadas dos vértices de um poligono regular com o circuncentro na origem do plano complexo. Prove que existem i, j.k e [1,2 nj tais que Z| + Z) = zk se e somente se 6 divide n. Seja A1A2...An um poligono e sejam a1,a2 a„ as coordenadas dos seus vértices A,,A2 An . Se [aj = |a2| = ... = |an| = R , prove que Z 1<i<j<n Z, — Zj 12 n2 como queríamos provar. (n-1)(n-2). 2 Já que X A,A2 < n2R2 , segue que M Z A,A,2 2 > 4R2 Solução. Seja e = cos—+ isen—. Então, z = z,e' n n Z laj-t-aJ2 zn(n-2)R2 iskjsn1 1 Solução. Temos £ |ai+aj|2 = £ (a, + aj)(ã[ + ãj) 1ái<jsn lsi<jsn = E (|aj2 +|a,|2 1ái<ján (pA n n n — + EaiSi = n(n - 1)R2 + X £ a^ -Xa.a, ) i*j i=1j=1 i=1 + [.ZaiJzai^-nR2 =n(n-2)R2+|xa,| an(n-2)R2, como queríamos provar. Z a,A,2 1si<jsn z — 1si<jsn AjAj 250 Números Complexo de A a Z que Zj + Zj = zk se e somente se 1 + c1'1 = ek ' . I. e., + isen , i.e., n = 6(k -i) = 3(j - i), portanto 6 divide n. Zj + Zj = zk, como desejado. Problema 4. = 0. ,2k-2 = 0 e assim está provado. Problema 5. Solução. Suponha que o centro do polígono é a origem do plano complexo, assim Solução. Sem perda de generalidade, vamos assumir que o centro do polígono é a origem do plano complexo. (j-i)n n Seja n > 4 um inteiro e sejam a.j,a2 an as coordenadas dos vértices de um polígono regular. Prove que a^2 + a2^3 +... + ana-, = a^a^ + a284 +... + ana2 ■ n O lado direito da igualdade é igual a z,z2 + z2z3 + ...+ znz, = X zizk+i k=1 Seja zk = z,ek-1, onde c = cos— + isen—, k = 1 n. n n n 3 Sejam z,,z2,...,zn as coordenadas dos vértices de um polígono regular. Prove que zf + zf +... + z2 = z,z2 + z2z3 +... + znz,. O outro lado é igual a zf +zf + ... + Z2 = £z;2 = £ zfe' k=l k=1 = Z?E 2(k -i)n n 2(k - i)n n 2(k — i)n n 1-e2n 1-s2 „ (j-i)nf (j-i)n . (j-i)nl 2 cos-—í- cos-—— + isen-—— | = cos n A última igualdade é equivalente a -z2^l■21 1-s2 Reciprocamente, se 6 divide n, tome i = 1, j = — + 1 , k = — + 1 e temos que 3 6 4^’ k=1 (j-i)n n (j-i)n n 2515 | Problemas Importantes para Olimpíadas aia2 + a2a3 Problema 6. Sejam z,,z2 znnúmeros complexos distintos tais que |z,| = |z2| = ... = |zn| - 1. Solução. Vamos analisar, primeiramente, a implicação a) => b). zk = z,e 2—...*(n—1)A relação b) se torna z" (l + sn ,2n Que é equivalente a n = n(-1)' 1 = (-1)' , o que é = n, Considere as afirmações: a) z,,z2 zn são as coordenadas dos vértices de um poligono regular. b) z?+zS+... + zS=n(-ir1z,z2...zn. Verifique, com prova, se as implicações a) => b) e b) => a) são verdades. ;n<n-1>) = n(-1)n-1z^1* ak = a,Ek’1, k = 1 n , com e = cos— + isen — . n n O lado esquerdo da igualdade é igual a ... + ana, =a?£e: n(n-1) r1E 2 Seja s = cos—+ isen—. Já que z,,z2 zn são as coordenadas dos vértices de n n um poligono regular, sem perda de generalidade vamos supor que :k~1 , para k = In . :2k-’=a2E^4- = 0. 1 - e n _ g2n O lado direito da igualdade é igual a a2 £ E2k = a2s2------ — = 0 , e assim está provado. k’1 1-E r’Zlz2.. + Zn| = n. Nós obtemos 1 = (-1)"*1 (cos(n-1)rt+ isen(n - 1)n), i.e.. 1 = (—1)n-1(—1)n-1 verdade. Portanto, a implicação a) => b)é verdade. Vamos provar agora que a implicação b) => a) também é válida. Observe que |n ■(-1)nt1z1z2...zn| = n|z,||z2|...|zn| Assim |z" +zn2 + ... n(n-1)2n n(n-1)2n cos—------ -— + isen—-------— 2 n 2 n 252 Números Complexo de A A Z Solução. Considere o plano complexo de origem no centro do polígono e seja 1 a coordenada de A,. I rr cos—, 1 n o que é verdade, usando a transformação soma-produto no lado esquerdo da equação.. Problema 7.__________________________________________________________ Sejam A, B, C três vértices consecutivos de um poligono regular de n vértices e considere o ponto M sobre o circuncirculo tal que os pontos B e M estejam em lados opostos da reta AC. MB = |z_ -e| = 2sen| —- —| ' m 1 1,2 nj 4senf— - — 112 nj e MC = |zm-s2| = Então MA = |zm -1| = ^(cost-1)2 A igualdade MA + MC = 2MC cos— é equivalente a n „ * o rt 2^2sen—+ 2sen--------- =2 1,2 nj estão separados pela reta AC, segue que — < t. n Pela desigualdade triangular, temos n = |z" +z2 +--|-z"|^|z';|+|z;|+-+|z;|=i+n-...+i = n. n-vezes Portanto, os números complexos z",z^ zn possuem o mesmo argumento. Já que |z"| = |z21 = ... = |z"| = 1, dai segue que z" = zn2 =... = z" = a , onde a é um número complexo com |a| = 1. Os números complexos z^z2...z„ são distintos, portanto, são as raizes enésimas de a, e consequentemente as coordenadas dos vértices de um poligono regular. Prove que MA + MC = 2MBcos—. n (Generalização do teorema de Van Schouten; veja a primeira observação abaixo) sen2t = V2-2cost = 2sen—; 2 „ (‘ 2n'l2sen---------12 n J Se s = cos— + isen— , então ek 1 é a coordenada de Ak, k = 1,n . n n Sem perda de generalidade,suponha que A = A,, B = A2 e C = A3 . Seja zm = cos t + isent, t e [0,2n) a coordenada do ponto M. Já que os pontos B e M 2535 | Problemas Importantes para OumpIadas Observação , nós obtemos 1) Se n = 3 então nós obtemos o teorema de van Schooten: Para qualquer ponto M sobre o circuncirculo de um triângulo equilátero ABC tal que M pertença ao arco AC , a relação seguinte ocorre: MA + MC = MB. Perceba que este resultado também segue do Teorema de Ptolomeu. 2) Se n = 4 , então para qualquer ponto M sobre o circuncirculo do quadrado ABCD tal que B e M estejam em lados opostos da reta AC, temos a relação MA + MC = 72MB. Solução. Considere o plano complexo de origem no centro do quadrado tal que os pontos A, B, C, D têm coordenadas 1,i,-1,-i, respectivamente. Seja z = a + bi a coordenada do ponto P, onde a,b e R e a2 + b2 = 1. A soma S„(P) é igual a Sn(P) = [(a -1)2 + b2 ]2 + [a2 + (b -1)2 ]2 + [(a +1)2 + b2 ]2 + [a2 + (b +1)2 ]2 = 22 jjl + a)2 +(1-a)2 + (1 + b)z + (1-b)2 j. Problema 8.___________________________________________________________ Seja P um ponto sobre o circuncirculo do quadrado ABCD. Encontre todos os inteiros n > 0 tais que a soma Sn(P) = PAn + PBn + PCn + PDn independa do ponto P. 2 + 72 4 n para n > 9. 72 72' 2 ' 2 n nx Sn(E) = 2(2-72)2 +2(2 + 72)5. Já que Sn(P) independe de P, segue que Sn(A) = Sn(E) ou n+2 n n 2 2 +2" =2(2-72)2 +2(2 + 72)2 . n >2(2-72)2 para todo n>1. Nós também temos que n+2 É óbvio que 2 2 n 2n > 2(2 + 72)2 para todo n 2 9. A última desigualdade é equivalente a _1_ 4 n-2 Defina P = A(1,0). Então Sn(A) = 2 2 + 2" . Para P = e| 254 Números Complexo de A à Z Portanto a igualdade S„(A) = Sn(E) ocorre somente para n < 8. Agora é fácil verificar que Sn(P) é constante somente para n e {2,4,6}. O termo do lado esquerdo da desigualdade decresceem relação à n, assim percebe- 2^72? 4 J Solução. Primeiramente suponha que todas as raízes de P são iguais e escreva P(x) = a(z -z0)n , para algum a,z0 eC e n e N . Se A,,A2 An são pontos distintos deR2 tais que f(A,) = f(A2) = ... = f(An), então A, An estão situados em um circulo de centro (Re(z0),lm(z0)) e raio l^f(A,/a)| , implicando que os pontos são os vértices de um polígono convexo. 1 se que — > Problema 9. A função f: R2 -> R é chamada Olímpica se ela obedece a seguinte propriedade: dados n>3 pontos distintos A1,A!,...,AneR2 se f(A,) = f(A2) = ... = f(An) então os pontos A,.A2 An são os vértices de um polígono convexo. Seja P e C [X] um polinômio não constante. Prove que a função f: R -> R, definida por f(x,y) = ]P(x + iy)|, é Olimpica se e somente se todas as raizes de P são iguais. (Olimpíada de Matemática da Romênia - Última fase, 2000) Reciprocamente, suponha que nem todas as raizes de P são iguais e escreva P(x) = (z-z1)(z-z2)Q(z) onde z, e z2 são raízes distintas de P(x) tais que |z1-z2| é mínimo. Seja / a reta contendo Z, = (Re(z1),lm(z1)) e Z2 =(Re(z2),lm(z2)), e seja z3=^(z1 + z2) tal que Z3 = (Re(z3),lm(z3)) éo ponto médio do segmento [Z,Z2 ]. Denote também svs2 como sendo os raios Z3Z, e Z3Z2 e seja d = f (Z3) > 0. Devemos ter r > 0 , senão z3 seria uma raiz de P tal que |z, — z3| < |z, — z21, o que é impossível. Como f (Z3) = 0, lim f(Z) = +» z3->» Zesj 2555 | Problemas Importantes para Olimpíadas Problema 10. Problema 11. 2 um inteiro e I: R2 Identificaremos R2 como sendo o plano complexo e sejaSolução. 0 . Solução. Posicionando o hexágono no plano complexo e definindo a = B-A,b = C-B f = A - F. A identidade do produto implica |ace| = |bdf|, e igualdade Prove que f é identicamente nula. (Olimpíada de Matemática da Romênia - Última fase,1996) Seja n > 2 um inteiro e f: R2 ->R uma função tal que para qualquer n-ágono regular A,A2...An, f (A,) + f (A2) +... + f (An) = 0 . e f é contínua, existe um ponto Z4 e s,, no lado de Z, oposto a Z3 tal que f (Z4) = r. De modo análogo, existe Z5 e s2 no lado de Z2 oposto a Z3 tal que f (Z5) = r. Portanto, f(Z3) = f(Z4) = f(Z5) e Z3,Z4,Z5 não são vértices de um polígono convexo. Logo, f não é Olímpica. Em um hexágono convexo ABCDEF ,  + C + Ê = 360° e AB CD • EF = BC ■ DE FA . Prove que AB • FC EC = BF • DE ■ CA . (Olimpíada de Matemática da Polônia,1999) -b _q _f relacionada aos ângulos implica que---------- — é real e positivo. Assim, ace = -bdf a c e . Além disso, a+b+c+d+e+f=0. Multiplicando essa igualdade por ad e adicionando ace + bdf = 0 obtemos a2d + abd + acd + ad2 + ade + adf + ace + bdf = 0 , que na forma fatorada fica a(d + e)(c + d) +d(a + b)(f+ a) = 0 . Portanto |a(d + e)(c + d)| = |d(a + b)(f+a)|, que é o que queríamos provar. Ç = cos — + isen—. Então, a condição se torna, para qualquer zeC e qualquer n n n real positivo t, £f (z + tÇ') = 0 ■ Em particular, para cada k = 1 n, temos que £ f (z - ) = 0 . Fazendo o somatório com k variando, temos £ X f (z - (1 - ) = m=1k=1 ' ' ' ' Números Complexo de A a Z256 Seguem alguns problemas propostos. (31a IMO) Problema 13. = 0 e a2n = (21a IMO) Problema 14. Problema 15. Seja A,A2...An um polígono regular de circunraio igual a 1. Encontre o máximo Problema 16. Seja A,A2...A2n um polígono regular de circunraio igual a 1 e considere um ponto = 2n. Sejam A,B,C três vértices consecutivos de um polígono regular e consideremos M como sendo um ponto sobre o maior arco AC do circuncirculo. Prove que MA • MC = MB2 - AB2. Sejam Ae E vértices opostos de um octógono regular. Seja an o número de caminhos de comprimento n na forma (P0,P,,...,Pn) onde P| são os vértices de octógono e os caminhos são construídos usando a regra: Po = A , Pn = E , P; e Pj+1 são vértices adjacentes para i = 0.... n-1 e P, * E para todo i = 0.....n-1. Problema 12._________________________________________________________ Prove que existe um 1990-ágono convexo com as duas seguintes propriedades: todos os ângulos são iguais; a medida dos lados são os números 12,22.32....19892,19902 em alguma ordem. Prove que a2n_, e y = 2 - v2 . Para m = n , a soma interna é nf(z); para qualquer outro m, a soma interna percorre o polígono regular, assim é 0. Portanto, f (z) = 0 para todo zeC. P sobre o circuncirculo. Prove que £ PA2+, ■ PA2ik+1 k=0 n valor de maxfjPAj quando P percorre o circuncirculo. (Competição Regional de Matemática da Romênia “Grigore Moisil”, 1992) •72' -yn'1), para todo n = 1,2,3......onde x = 2 + -J2 2575 | Problemas Importantes para Olimpíadas 5.8 NÚMEROS COMPLEXOS E COMBINATÓRIA Problema 1. 2k+1 i6n 0- Problema 2. Calcule a soma s„ Solução. S„+ÍTn já que Considere o número complexo z = cosa + isena e a soma Tn que 1 ” i>/3 na sen—. 2 3n-1, z k=0 ' = ;-y=lm^26n (cos2nrt + isen2nn)J = n ínA = £ penka .Temos k-o^kj (cos a + isena)k = E ín Icoska , onde k-oLU 1 / r~\6n—— lm(l + ix/3j = a e [O.n]. Da relação (1) segue que a e [0,n]. )kf V = l2k + 1J = SÍ 6n Wk =k=ov2k + lj' I Solução. Temos que 3n-1 . S (-1)' k=0 Sn+iT„=^2cos|J^ a acos— + isen— 2 2 \n í íl . 71cos—+ isen— 3 3. „ • o 2a o- a a n cx i1 + cosa + isena = 2cos —+ 2isen—cos— = 2cos — 2 2 2 2' na . na | . cos— + isen— , i.e.,2 2 J S„ =^2cos^ f na T acos—e Tn = 2cos— / 2 n V 2 Í 6,1 l(-3)k 4 zí 6n Wy iV3 k.ol2k + l/ > í k-(2k + lJ3n-h xkCalcule a soma £ (-1) k=0 = E | |(coska + isenka)= e| k=o<kJ k=o' ■âC?-'1*1»'™ A representação polar do número complexo 1 z é 1 1 ij3 258 Números Complexo de A à Z Problema 3. 2 e e observe que (1 + i)n = xn+iyn. (1) Tomando o módulo em cada lado da igualdade segue que O que é equivalente à x2 + y2 = 2". Observação Podemos escreveras formas explicitas para xn e yn como segue. Observe que ,n n Problema 4. fm'| í'm'1 fm'| fm'| l 0 H p J + l<2pJ + |<3pJ + Solução. Começamos com a seguinte observação simples porém útil: Se f e R[X] ■ (1) 22cos— 4 n 1 n 3 n 5 mkn cos------ PP 2 = 2". Solução. Denote por xn =[q]"[2] + [4) Prove a identidade |í"l-í” | + | | lloj l2j (4J Se m e p são inteiros positivos e m > p , então í u V"1+ Z f COS——i k-1 < P ) \An _ / cos—+ isen—I =22|i 4 4JJ t - DTt Da relação (1) obtemos xn = 22 . 2n . 2n , . . . , e um polmomio, f = a0 + a,X +... + amXm e e = cos— + lsen“ e a P-esima raiz da unidade, então para todos os reais n a seguinte relação ocorre: a0+apxP+a2px2p+... = -i(f(x) + f(Ex) + ... + f(Ep‘,x)j nn . nn | cos— + isen— .4 4 ) o? nne y„ = 22sen—. (1 + i)n=[V2[( |Xn+Íyn| = |(1+i)n|=|l+í|n=22. 5 | Problemas Importantes para Olimpíadas 259 .(p-w _ obtemos (1 (2) .m no lado direito da equação. Considere o caso quando p é primo. Usando a relação (1) para o polinõmio 2 P p, se p | k, 0, casocontrário, p Considere agora o caso quando p é um inteiro positivo par. Uma vez que e2 = -1, temos que Para provar (1) usamos a relação 1 + sk +e2k +... mkrt cos------ P = —í 2m + (1 + e)m Pk p-1 p-1 1 P-11 i 12/ b \m 2 / „t,\mSp=-z(l + s ) = - S 1 + ek +E 1 + sp-k p Pk=ov ' P k-0k ' k»1k ' Usando a relação (l + ep-k)m=(l + ?)m=(l + ek)m Substituindo em (3) obtemos 2m+E(l + ekf k-1v ' P-1 +2mf I k=1! 2m i 2m y*, I k.V rCOS----- < l PJ l f krtY" cos-- l p J > ep-k = ? ■ —,m encontramos „mf knYY rnkrr mkrt'! - 2 cos— cos----------isen------- . I P } l P P J ' eü 2m 2 | = — 1 + 2Z k=1 mkrt mkncos---------isen------- P P . f = (1 + X)m P~1 , k Z 1 + ek k-|-i mkn mkncos------ + isen------- P P . mkrt mkn 1cos------ - + isen------ . P P ) . a- Substituindo x = 1 na relação (2) encontramos HoHprM „ k 2kn 2krt L ,Como c = cos + isen , segue que para todo k = 0,1 p-1 P P mkrt P P Números Complexo de A a Z260 Solução. Sejam Eo.E, Ep_, as p-ésimas raizes da unidade. Então + <-?)■(!) (2) ^O^kf = + isen Observação A seguinte relação trigonométrica ocorre: = 0 - (3) n m 2 P mkncos------ P „n( knYf = 2 cos— cos l P J l e usando as relações (1) e (2) a igualdade desejada aparece._k-m E0 (n-2m)kn P (n-2m)k7t P (n-2m)krr P (n-2m)kn P n COS' .n sen , _k-m ... + £P-1 2’’ + Z2m k=1 ( k^TíCOS----- I l p) l PçY kn^l 2 cos— k=0\ P J Levando em consideração que 2mkrt 2mkrtcos---------- isen--------- P P . 2n p-V knY — £ cos— p k=oL P J 2m 2"'l - — 1 + 2S ” k=r 7 i í kn l cos— l P ) mkn . mkn cos-------- isen------- P P . nkn nkn cos----- + isen------ P P . e’Ek "* (1 + ek)" k=0 Usando o resultado da Proposição 3, Subseção 2.2.2, segue que = ÍP, sep|(k-m) (O, casocontrário „ kíIT2cos— P ) 2‘1 2m i 2m k.1 mkrt mkncos------- usen------ P P . Problema 5.____________________ A seguinte identidade ocorre: n > ( n 'j m + pj ^m + 2pj P 2615 | Problemas Importantes para Olimpíadas Problema 6. Considere os inteiros a„,bn,cn , onde 2) Usando a identidade ,2 e a relação acima segue que a2 + b2 + c2 - anbn - b„cn - c„an = 1. Observação Do Problema 5 segue que 2n + cos 2" + 2cos2" +cos 2 3 2nn cos----- 3 (n-4)n 3 ’ + 3) Multiplicando a relação acima por 2 encontramos (an - t>n )2 + (&„ - Cn )2 + (c„ - a„ )2 = 2. (1) Da relação (1) segue que dois dos números an,bn,cn são iguais e o terceiro difere em um. Portanto, an + bn + Cn - 3a„bncn = (an + bn + cn )(an + bns + cns2)(an + b„s2 + cnc) = 2n (1 + s)n (1 + s2 )n = 2n (-s2)" (-s)n = 2n. uma raiz cúbica da unidade diferente de 1. Temos que (1 + s)n = an+bns + cne2 (1 + £2) = an + bne2 + cne W. Mostre que: a3+b3+c3-3anbncn = 2n. an +b2 + c2 -anbn-bncn -cnan =1. Dois dos inteiros an,bn,cn são iguais e o terceiro difere em um. (n~2M 3 ) (n-4)*y = —í 2n +2cos 3Í X3X- XX3 1 C" = 3 Solução. 1) Seja e (l4-1)n=an+bn+c„. J = ^2n+2cos-^'|. (2n-4)n 3 (2n-8)rt‘ 3 (-1)" cos a-4 b-4 + (-1)n cos 2n+Cosy + (-1)n (n~2> 3 x3 +y3 +z3 -3xyz = (x + y + z)(x2 +y2 +z2 -xy-yz-zx) 3 262 Nümeros Complexo de A á Z Problema 7. Problema 8. (Competição Juvenil,1999) -m = £ ec’- UCjSaj Quantos inteiros positivos de n dígitos escolhidos do conjunto {2,3,7,9) são divisíveis por 3 ? (Competição Regional de Matemática da Romênia "Traian Lalescu", 2003) Solução. Sejam xn,yn,zn o número de todos os inteiros positivos de n dígitos 2,3,7 ou 9 que são congruentes a 0,1 e 2 módulo 3. Temos que encontrar xn. k = ^(4n -1). Finalmente, x„=k+1 = l(4"+2). Seja n um número primo e sejam a,,a2 am inteiros positivos. Considere f (k) o e f(0) + f(l)c + ... + f(n-1)c' :m = fl(e + s2 +... + eaQ + Xa‘) = S XC1" ' isqsaj xn + syn + C2Z„ = E ^11^2*7)3^4 = / 2 + E3 + e7 + f9y = r il*Í2*Í3*Í4=" Segue que xn - 1 + eyn + c2zn = 0. Aplicando a Proposição 4 da Subseção 2.2.2 obtemos xn -1 = yn = zn = k . Então 3k = xn + yn + z„ -1 = 4n -1 e encontramos Considere c = cos + isen . É fácil ver que xn + yn + zn = 4n e Solução. Seja e = cos —+ isen —. Perceba que a seguinte relação ocorre: n n n(x+x2 i=1 v m número de m-uplas (c, cm) satisfazendo l^c, ^a, e £Cj =k(modn). Mostre i=1 que f (0) = f (1) = ... = f (n-1) se e somente se n | af para algum j e {1 m). É fácil ver que an = bnse e somente se ns 1(mod3), a„ = cnse e somente se n a 2(mod3), bn = cn se e somente se n a 0(mod3). 2635 | Problemas Importantes para Olimpíadas f(O) = f(1) = ... = f(n-1) Problema 9. (36a IMO) . Levando que pk = x0+... + xp_,-2 Para um conjunto finito A de números reais, denote por | A | o número cardinal de Ae por m(A)a soma dos elementos de A. Seja p um primo e A = {1,2 2p). Encontre o número de todos os subconjuntos B c A tais que | B |= p e pjm(B). temos que = 0 . Isso é Solução. O caso p = 2 é trivial. Considere p 2 3 e s = cos —+ isen —. Denote por x, o número de subconjuntos B c Portando, o número desejado é x0=2 + k = 2 +—Í|P|-2| PklPj J equivalente a n(e + s2 + ... + ca') = 0, i.e., e + e2 + ... + sa' = 0 para algum j <= {1,...,mj. Segue que e3' -1 = 0, i.e., n | a,. Aplicando o resultado da Proposição 4, Subseção 2.2.2, se e somente se f(0) + f(1)e + ... + f(n-1)En"1 Z x.e' = 2 ,i.e., x0 - 2 + x1e + ... + x.1ep 1 = 0 . i=o Da Proposição 4, Subseção 2.2.2, segue que x0 - 2 = x, = ... = Xp., = k . Encontramos Então Exjsl = y Em,B* = y ec’’' Cp j-0 BcA,|B| = p lSC!S...£CpS2p A última soma é o coeficiente de Xp em (X + e)(x + e2)...(x + e2p) em consideração a relação Xp-1 = (X-1)(X-e)...{X-ep“1) obtemos que (X + e)(x + e2)...(x + c2p) = (xp +1)2 , assim o coeficiente de Xp é 2. Portanto, 2rt . 2n P P A com a propriedade | B |= p e m(B) a j(mod p). 264 Números Complexo de A a Z Problema 10. S„ = Rn +iV2Sn . (1) l(-2)k ■ Problema 11. 1) 2n + Supondo por absurdo que Sn a0(mod 5) para algum inteiro positivo n. Então da relação (3) obtemos que R2 = +3(mod 5), um absurdo já que qualquer quadrado perfeito é congruente a 0.1 ou 4 módulo 5. Seguem alguns problemas envolvendo números complexos e combinatória. 2 5 não é divisível por 5 para qualquer inteiro n > 0. (16a IMO) Solução. Uma vez que 23 = -2(mod 5), um problema equivalente é provar que onde Rn = £ k=0 2n + 1 2k nn cos— + 5 n I Prove que o número y k-01 nn cos— 4 2nncos---- 5 l'2n + 1'i 3k <2k + lJ 2 coskt, onde t e [O.n]. = + Rão é divisivel por 5. Expandindo (l + i\/2) separando as partes reais e as imaginárias, obtemos n (nY Calcule a soma sn = £ k.oçk ) Problema 12._______________ Prove as seguintes identidades: (Olimpíada de Matemática da Romênia - Segunda fase, 1981) 2n-1 2 +22 2n+1 e então (>/5+l)n 2^1—1 , \2n+1(1 + í>/2) Tomando o módulo em ambos os lados de (1) segue que 32n*1 = R2 + 2S2 (2) Já que 32 = -1(mod 5), a relação (2) se torna R2 + 2S2 = ±3(mod 5). (3) 2655 | Problemas Importantes para OumpIadas Problema 13. Considere os inteiros An.B„,Cn definidos por Problema 14. divisível por número inteiro sn 5.9 PROBLEMAS DIVERSOS Problema 1. Dois quadrados unitários K,, K2 de centros M,N estão situados no plano de tal forma que MN = 4 .Dois lados de K, são paralelos à reta MN, e uma das diagonais de K2 situa-se sobre MN. Encontre o lugar geomérico do ponto médio de XY quando X.Y variam no interior de K,.K2 , respectivamente. (Olimpiada de Matemática da Bulgária, 1997) Seja p > 3 um primo e sejam m,n inteiros positivos divisíveis por p tais que n é m impar. Para cada m-upla (c, cm), c, e {1.2 n}, com a propriedade que p | Tc, M consideremos o produto c, ••■cm. Prove que a soma de todos esses produtos é 0 I Pj As seguintes identidades ocorrem: 1) A*+B*+C*-AnBn-BnCn-CnAn=3"; 2) An+AnBn+B„ =3"-1. Problema 15. Seja k um inteiro positivo e a = 4k -1. Prove que para qualquer inteiro positivo n. o -(MM?).. = ^l-í^a + (^a2 -^^a3 +— e divisível por 2n 1. (Olimpiada de Matemática da Romênia - Segunda fase,1984) 266 Números Complexo de A a Z Solução. Definindo os números complexos como M = -2 , N = 2 . Então, o lugar -y3 =o}. Prove que A m B = CnC . (Competição de Matemática Putnam, 1987) + 3i. Isso é equivalente à (17a IMO) são números Solução. Existem infinitos pontos com coordenadas racionais sobre o círculo unitário. Isso é um conhecido resultado decorrente dos triângulos Pitagóricos e da correspondente equação: 2 i 1 2’2 Problema 3. ________________________________________________________ Determine, com prova, se é ou não possível ter 1975 pontos sobre um circulo unitário tais que as distâncias entre dois pontos quaisquer são números racionais (as distâncias são tomadas ao longo da corda). C = |(x,y): x3 -3xy2 +3y = l| . D = |(x,y): 3x2y-3x [ B = |(x,y):2xy n2 = p2. Qualquer ponto A(xA,yA) pode ser representado por um número complexo zA = xA +iyA = cosaA +isenaA onde aA é o argumento do número complexo zA e cosaA,senaA racionais. •J2|x + y| ,|x - y| < . O resultado é um octógono de vértices , e assim por diante. Solução. Seja z = x + yi. As equações que definem A e B são as partes real e imaginária da equação z2 = z"1 + 3i e , de modo análogo, as equações que definem C e D são as partes real e imaginária de z3 -3iz = 1. Assim, para qualquer real x e y, temos que (x,y)e AnB se e somente se z2 = z' z3-3iz = 1, i.e., (x,y) e CnD. Logo, A oB = CnD . geométrico é o conjunto de pontos da forma -(w + xi) + (y + zi), onde |w| ,|x| < - e (1 + T2)i + 2 Problema 2._______________________________________ As curvas A,B.C e D estão definidas no plano como segue: A = {(x.y):x2-y2=-yA_ m2 + y =3. x2 + y2 J 2675 | Problemas Importantes para Olimpíadas Solução. Em um estágio, o turista atravessa o número complexo Problema 5. (27a IMO) Sejam A, B, C pontos fixos no plano. Um homem parte de um certo ponto Po e caminha em direção à A. Em A, ele desvia 60‘ para a esquerda e caminha para P, tal que PoA = AP,. Depois de executar a mesma ação 1986 vezes sucessivamente ao redor dos pontos A,B,C,A,B,C ele retorna ao ponto de inicio. Prove que ABC é um triângulo equilátero e que os vértices A. B, C estão arranjados no sentido antihorário. Tomando sobre o circulo unitário os números complexos da forma zA = cos2ctA + isen2aA Solução. Por conveniência, sejam A,,A2,A3,A4,A5,... os pontos A.B.C.A.B..... respectivamente, e seja Po a origem. Após do k-ésimo passo, a posição Pk será Problema 4.___________________________________________________________ Um turista viaja ao longo de uma cidade em estágios. Cada estágio consiste de três segmentos de medida de 100 metros separados por um desvio horário de 60 . Entre o último segmento de um estágio e o primeiro segmento do próximo estágio, o turista faz um desvio antihorário de 60' . Qual a distância que o turista irá ficar da sua posição inicial após 1997 estágios? (Olimpíada de Matemática Rioplatense. 1997) temos para dois pontos desses: |zA - zbI ~ \/(cos 2aA - cos 2aB )Z + (sen2aA - sen2aB )2 = ^2[l-cos2(a8 -aA)] = ^2-2sen2(aB -aA) = 2|sen(aB - = 2|senaB cos aA -senaA cos aB| e Q Resposta: Sim, é possível. “a)| x = 100 + 100c+ 100(c)2 = 100-100x/3i, onde e = cos—+ isen— . 3 3 Portanto, em 1997 estágios, o turista atravessará o número complexo Z = X + XE + XE2 +... + XE1"6 = xl-í------ = XE2 . 1 - E Consequentemente, o turista situa-se a |z| = |xe2| = |x| = 200 metros de sua posição inicial. :B cosaA - senaA 268 Números Complexo de A a Z Problema 6. uma contradição. Problema 7. então a,b,c sao números reais. (Teste de Seleção do Time Romeno,IMO 2001) Sejam a,n inteiros e seja p um primo tal que p>|a| + 1. Prove que o polinômio f(x) = xn+ax+p não pode ser representado como o produto de dois outros polinômios não constantes de coeficientes inteiros. (Olimpíada de Matemática da Romênia, 1999) + a|s|zn-1| + lal£l + lal' + E2Ak_2 +... + C& Prove que se a,b,c são números complexos tais que (a + b)(a + c) = b • (b + c)(b + a) = c (c + a)(c + b) = a |zn-’ Solução. Seja z uma raiz complexa do polinômio. Devemos provar que |z| > 1. Suponha que |z| < 1. Então, de zn + az = -p deduzimos que p = |zn + az| = |z||: o que contradiz a hipótese. Agora, suponha que f = gh é uma decomposição de f em polinômios não constantes de coeficientes inteiros. Então, p = f (0) = g(O)h(O), e também |g(0)| = 1 ou |h(0)| = 1. Suponha sem perda de generalidade que |g(0)| = 1. Se z,,z2 zk são as raizes de g , então elas também são raizes de f. Portanto 1 = |9(0)| = |z1z2---zk| = |z1||z2|--|zk|>1, Pk - \ +(Pk-i "Ak)t , para k = 1,2 onde e = cos-^ + isen^ . Facilmente obtemos Pk = (1 -s)(Ak +cAk_, + e2Ak_2 + ... + ek"1A1). A condição P = PI9M é equivalente a A1986 + eA1985 + ... + e1984A2 +e1985A, =0, a qual, tendo em mente que A, = A4 = A7 = .... A2 = A5 = A8 =.... A3 = A8 = Ag =.... se reduz à 662(A3 + eA2 + e2A1) = (1 + e3 +... + e1983 )(A3 + eA2 + e2A1) = 0 , e a afirmação segue da Proposição 2 da Seção 3.4. 2695 | Problemas Importantes para Olimpíadas Solução alternativa. Subtraia a segunda equação da primeira. Obtemos (a + b)(a-b) = b-c. Analogamente, (b + c)(b-c) = c- a e(c->-a)(c-a) = a- b. Podemos ver que se dois dos números são iguais, então todos os três são iguais e a conclusão é óbvia. Suponha que os números são distintos. Então, depois de multiplicar as igualdades acima, obtemos (a + b)(b + c)(c+ a) = 1. e depois: b(b + c) = c(c + a) = a(a + b) = 1. Agora, se um dos números é real, segue imediatamente que todos os três são reais. Suponha que todos os números não são reais. Então, arga.argb.argc e (0,2rt). Dois dos números arga.argb.argc estão contidos ou em (0, rt) ou em [n,2n). Suponha que esses dois são arga.argb e que arga < argb . Então arga < arg(a + b) < argb e arga < arga(a + b) < arg(a^b) < argb. Isso é uma contradição, já que a(a + b) = 1. Solução. Seja P(x) = x3 -sx2 +qx-p um polinômio de raizes a.b.c.Temos que s = a + b + c,q = ab + bc ca.p = abc . As igualdades dadas são equivalentes a sa + bc = b sb + ca = c (1) sc + ab = a Somando as igualdades, obtemos q = s-s2. Multiplicando as igualdades em (1) por a,b,c, respectivamente, e somando-as obtemos s(a2 +b2 + c2) + 3p = q ou, depois de uma pequena manipulação algébrica, 3p = -3s3 + s2 + s. (2) Se escrevermos as equações dadas na forma (s-c)(s-b) = b , (s-a)(s-c) = c, (s-b)(s-a) = a, Obtemos ((s-a)(s -b)(s-c))2 = abc e, por meio de algumas contas e usando (2), obtemos finalmente s(4s-3)(s + 1)2 = 0 . Se s = 0, então P(x) = x3, portanto a = b = c = 0. Se s =-1 . então P(x) = x3 + x2-2x-1, que tem como raizes 2cos^-,2cos^,2cos^- (isso não ê óbvio, mas podemos ver que P muda de sinal nos intervalos (-2,-1),(-1,0),(1,2) 3 da reta real, consequentemente, suas raizes são reais). Finalmente, se s = - , então 3 3 1 1P(x) = x3 —x2 + — x------ , que tem como raizes a = b = c = — .V ’ 4 16 64 4 270 Números Complexo de A A Z (Olimpíada de Matemática da Rússia, 2000) Problema 9. O par (z,,z2) de números complexos não nulos tem a seguinte propriedade: existe um número real a e [—2,2] tal que z2 ~az,z2 + z2 = 0 . Prove que todos os pares (z",zj), n = 2,3,..., têm a mesma propriedade. (Olimpíada de Matemática da Romênia - Segunda fase,2001) z ç’4k=çV 1 k.y lí-1. x+m-1 y-rm-1 z „ Problema 8. _______________________________________________________ Encontre o menor inteiro n tal que um quadrado nxn pode ser particionado em quadrados da forma 40 x 40 e 49 x 49 , com ambos os tipos de quadrados presentes na partição. Solução. Podemos particionar um quadrado 2000x2000 em quadrados 40x40 e 49x49 : particione um canto 1960x1960 do quadrado em quadrados 49x49 e então particione a porção restante em quadrados 40x40. Agora, vamos mostrar que n deve ser no minimo 2000. Suponha que um quadrado nxn foi particionado em quadrados 40x40 e 49x49 , usando pelo menos um de . „ 2n . 2" 2n 2a . ,cada tipo. Seja Ç = cos— + isen— e Ç = cos—+ isen—. Oriente o quadrado nxn de tal modo que dois lados são horizontais, e o numere as linhas e colunas dos quadrados unitários partindo do topo esquerdo: 0, 1,2 n - 1. Para 0 í j, k < n-1 escreva Ç'çk em um quadrado (j,k). Se um quadrado mxm tem seu canto esquerdo superior em (x.y), então a soma dos números escritos nele é Çm -l¥-1] Ç-1 A s-1 J' A primeira fração em parênteses é 0 se m = 40 e a segunda fração é 0 se m = 49. Portanto, a soma dos números escritos dentro de cada quadrado na partição é 0, assim a soma de todos os números deve ser 0. Contudo, aplicando a fórmula acima com (m,x,y) = (n,0,0), encontramos que a soma de todos os números é igual a 0 somente se Çn -1 ou Ç" - 1 for igual a 0. Portanto, n deve ser ou um múltiplo de 40 ou um múltiplo de 49. Sejam a e b o número de quadrados 40x40 e 49x49, respectivamente. A área do quadrado é igual a 402 a + 492 b = n2. Se 40|n, então 402|b e consequentemente b > 402. Portanto, n2 > 492 -402 = 19602 ; como n é um múltiplo de 40, n > 50 ■ 40 = 2000 . Se, ao contrário, 49 |n , então 492 |a , a > 492, e de novo n2 > 19602 . Como n é um múltiplo de 49, n S 41 • 49 = 2009 > 2000 . Em ambos os casos, n > 2000 e portanto 2000 é o menor valor possível de n. 2715 | Problemas Importantes para Olimpíadas t = ea 2cosa (1) 2cosna, n = 1,2,... 1. O valor mínimo é -4 e é obtido para x = 1 ,i.e., para z = a(1±i), a#0. Problema11. 0 . Então. - \2Imz5 lm5 z Problema 10. Encontre Sejam z,,z2,z3 números complexos, nem todos reais, tais que |z,| = |z2| = |z3| = 1 e 2(z, +z2 +z3)-3z,z2z3 e IR . Im(z)~ lm5 z a ±i\/4 -a2 2 Solução alternativa. Como ae[-2,2], podemos escrever a = 2cosa . A relação z2 - az,z2 + z2 = 0 é equivalente a z2 z, e, por meio de um simples argumento de indução, de (1) segue que z5 z? Definindo x = . Imz5 min —=— zsC\2 lm5 z e todos os valores de z para os quais o mínimo ocorre. obtemos — = 5x2-10x+ 1 = 5(x-1)2 -4. Solução. Sejam a.b números reais tais que z = a + bi. b lm(z)5 =5a4b-10a2b3+b5 e = -1of-'l + ' ' I rv*5 ) <b J Prove que max(argz1,argz2,argz3) > — . 6 t2-at + 1 = 0. Temos que Solução. Denote t = -^L, t e C . A relação z?-az,z2+z2 = 0 é equivalente z2 A = a2 - 4 < 0 , assim /g2 x _ _2 Zn |t| = ,— +---------= 1. Se t = cosa + isena então —L = tn = cosna + isenna e V 4 4 z2 podemos escrever z2n - anz2zj + z2n = 0 , onde an = 2cosna e [-2,2]. 272 Números Complexo de A a Z Suponha por absurdo que max(t,,t2,t3) A função seno é côncava em • (2) Das relações (1) e (2) obtemos (Olimpíada de Matemática da Romênia - Segunda Fase, 2002) Então, sen3t < 2sent. Segue que 4sen3t - sent > 0. 71 6 ' Problema 13.__________________________________ Resolva nos números complexos o seguinte sistema: x(x - y)(x -z) = 3 y(y-x)(y-z) = 3. z(z-x)(z-y) = 3 t, +12 +13 3 tr +12 +13 3 r_ x)0,— , assim L 6J Solução. Seja zk = costk +isentk,k e {1,2,3}. A condição 2(z,+z2+z3)-3z,z2z3 e R implica 2(sent, +sent2 +sent3) = 3sen(t, + t2 +t3). (1) Portanto, max(t,,t2,t3) > , como desejado. Seguem mais alguns problemas. t3) < sen Problema 12._____________________________________ Resolva nos números complexos o sistema de equações x|y| + y|x| = 2z2 •y|z| + z|y| = 2x2 z|x| + x|z| = 2y2 —(sent, +sent2 +sen assim t,,t2,t3 < — .Seja 6 t _ Oi +1?+ *3) e 3 i e., sen2t > . Assim, sent ~ e então tà|, o que contradiz com t e sen(t, +t, +t3) ------——-—— < sen 2 6 | Respostas, Dicas e Soluções dos Problemas Propostos 273 Problema 14. Problema 15. Seja ABCDE um pentágono cíclico, inscrito em uma circunferência de centro O. com ângulos B = 120‘, C = 120 , D = 130 , Ê = 100 . Mostre que as diagonais BD e CE se encontram em um ponto pertencente ao diâmetro AO. (Teste de Seleção do Time Romeno,IMO 2002) Sejam X,Y,Z,T quatro pontos no plano. Os segmentos [XY] e [ZT] estão conectados se existe um ponto O no plano tal que os triângulos OXY e OZT são isósceles em O. Seja ABCDEF um hexágono convexo tal que os pares de segmentos [AB],[CE] e [BD],[EF] estão conectados. Mostre que os pontos A. C. D e F são os vértices de um paralelogramo e que os segmentos [BC] e [AE] estão conectados. (Olimpíada de Matemática da Romênia - Última fase, 2002) 274 Números Complexo de A a Z f) 3. a) z1 = • Z2 = b2 ,sgn b. -,-sgn b.z2 = 1. a) z, +z2 +z3 =(0,4); c) z,z2z3 =(-9,7); 1 73 2’ 2 b) z,z2 + z2z3 + z3z, = (-4,5); d) z? +z|+z| =(-8,-10); Respostas,-Dicas-e-Sol-uções-dos-Rroblemas-Rropostos- Aqui seguem as respostas e soluções dos problemas apresentados nos capítulos anteriores Nós mantivemos o título da subseção que contém o problema e o número do problema proposto. 1 73) 2' 2 r 7a2 -i-b2 —2 (1,0), para n = 4k; (1,1), para n = 4k + 1; (0,1), para n = 4k + 2; (0,0), para n = 4k + 3. 5. a) z = (1,1); b) z, = (2,1), z2 = (-2,-1). 6. z2 = (a2 -b2,2ab); z3 = (a3 - 3ab2,3a2b -b3); z4 = (a4 - 6a2b2 + b4,4a3b-4ab3). ;b) z, = (-1.0), z2 = ■ z3 = Z?+ZÍ ... ZÍ+ZÍ -a + 7a2 +b2 2 a + 7a2 -rb2 2 6.1 RESPOSTAS, DICAS E SOLUÇÕES DOS PROBLEMAS 6.1.1 Números Complexos na forma algébrica (pág. 18-21) 1 73) ~2 ~ ' §,_A];d)z = (_9,7). 4. Z zk = k=0 311 65Y 130’837’ a e)ÍL+^ + £3=f Z2 Z3 z, (. -a + Tã2^ 2. a) z = (7,-8); b) z = (-7,-4); c) z = 152 72 ) 221' 221J 2756 | Respostas. Dicas e Soluções dos Problemas Propostos 8. Para todo inteiro nâo negativo k temos ,k,k zz 9. a) x = - c) x = 0. y = 0 .b)x = —2, y = 8; 10. a) 8 + 51i; b) 4 - 43I; c) 2; 11- a) -i; = 1 + i. E4k*2 = '. E4k-3 = 0' c) 1; d) -3i. c) z,2 = ± 13. z e R ou z = x + iy, com x2 +y2 = 1. 14. a) E, = E,; b) E2 =E2. 15. Substituímos a fórmula da definição de módulo. 16. Da identidade 4 13 2 2 72. 2 ' 1' z + — z 72 2 h + 73 773-1.) 2 2 T b) E4k - 1 ■ E4k*1 fs2+3H.3 = Z3 :4k-2 = (0.-2(-4)k); I 1I'obtemos z + - .72 ., 72 .72 -i— ; b) z. =-------- 1—2 1 2 2 ;4k=(H)k.o); = ^—2 (—4)k.—2(—4)k); para k >0. 1 3 —. y = —;4 4 .. 11 577. . 61 d)----------------- 1; e) —■ 4 2 13 z4k»1 a (z4+l)2=z4 2 1 Vã ou (z4 -z2 +l)(z4 + z2 +l) = 0 . As soluções são ±—i±— e z4k*3 2)(z’ + z2) = 1.17.Aequação | z2 - z 2| = léequivalenteá | z2 - z 2| = 1. Istoé, (z2 + z - z ( 1 'l2 Encontramos que (z2+z 2 j =1ouíz2+-yl = 1 .Aúltima equação é equivalente T'Z2 = +4“+3Íz+“ 11 z3 l z; = |z + — |, a > 0. Uma vez que a3 -3a - 2 = (a -2)(a2 + 2a +1) = (a - 2)(a +1)2 . temos que a < 2, como desejado. „ . 72 . 72 72 12- a) Z1 =^ + ,~Y' Z2 =^~ ou a3-3a-2 < 0 , onde a 276 Números Complexo oe A a Z 18. ZG 19. ze(0,1,-1,i,-i} . 21. a2 + b2 + c2 - ab - bc - ca. d) z12 = 23. m e {1,5}. 24. z = -2y + 2 + iy, y e R . 25. z = x + iy, com x2 + y2 = 1. 29. Seja a = |z2-z3|, p = |z3-z1|, y = |z1-z2|. Já que a seguinte desigualdade, ap + py + ya < a2 + p2 + y2 ocorre e a2 + P2 + y2 = 3 (|z,|2 + |z212 + |z312 - |z, + z2 + z312 ) 26. De |z, + z2| = 73 segue que |z, +z2|2 = 3 , i.e., (z, + z2)(z, +z2) = 3 . Obtemos que |z,|2 + (z,z2 +z,z2) + |z2|2 = 3 . Isto é, zlz2+z,z2=1. Por outro lado, temos kl - z2r = |z,|2 -(z,z2 + Z?z2) + |z212 = 2 -1 = 1, assim, |z, - z2| = 1. l1 1l 20. Observe que —- (2 - z)(2 - z) < z.z. Segue que 4 < 2(z + 1 -J327. Definindo e = - — + i^- e percebendo que e3 = 1, obtemos que n = 3k, k e Z. 28. Perceba que z = 0 é uma solução. Para z * 0 tomando o módulo em ambos os lados, obtemos |z|n-1 = |z|, i.e., ]z| = 1. A equação é equivalente á zn = iz ■ z , que se reduz a z" = i. Logo, o número total de soluções é n + 1. 1 — é equivalente a |2 - z| < |z|, e consequentemente z) = 4Re(z), como pedido. c) z = 2 + i;22.a) z12 = ^y^ + 2i; -2 ±73.2 2 " 2' b) z = -- + 4i; 6 +—i; e) z2 =-1, z2 =-5-6i; f)z2=-— i 2 2 2 2776 | Respostas, Dicas e Soluções dos Problemas Propostos segue que ap + Py + ya < 9R2 . Obtemos (u- z)(u - z) < (uz - 1)(uz -1) j.e„ |u|2 + |z|2 - |u|2 |z|2 -1 < 0 . Finalmente, Já que |u| < 1, segue que |w| < 1 se e somente se |z| < 1, como desejado. 31. z2 + z| + z3 = (z, + z2 + z3)2 -2(z,z2 + z2z3 + z3z,) -2z,z2z3 (z, + z2 + z3) = 0. E consequentemente EeR. z,z2 +z3 e X se e somente se á3(|z,|2+|z2|2+|z3|2) = 9R2, (|u|2-lj(|z|2-l) > 0 . _ +z2 Az2 +z3 Z, Z2 33. Perceba que z,.z, = z2.z2 = z3.z3 = r2 e z,z2 + z3 = z,.z2 + z3. 30. Observe que |w| = |v| < 1 se e somente se |u - z| < |uz - 1|. Isso é equivalente a |u — z|2 < |üz-1|2. z, + z2 z2 - z3 Z1Z2 z2z3 r2"----------------- Z1Z2 ' Zn r2 32. A relação |zk| =r implica zk = — para k e {1,2 n}. Então MH] - Z„ _ Z2Z3 -r Z, z2z3-r2z, Z,-1 z2-1 z3-1 z,-r2 z,-r2 z3-r2 Zn^Z, Zr,z—= E. Entã0_±_ = ^^3_ = ^3±^ ziZ2z3 z,z2 + rz3 z,z3 + rz2 (Zl-1)(z2-Z3) = Z, -1 = z2-1 = z3-1 z,-z2 = 1 (z2-z3)(z,-r2) z,-r2 z2-r2 z3-r2 z, - z2 Assim, z,z2z3 = r2 e consequentemente r3 = r2. Portanto, r = 1 e z,z2z3 = 1. como desejado. 278 Números Complexo de A â Z 6.1.2 Interpretação geométrica das operações algébricas (pág. 27) 3. a) Circunferência de centro em (2,0) e raio 3. b) Circulo de centro em (0,-1) e raio 1. e, analogamente, 2|z2 + z3| • |z3 + z,| < 2|z31 ■ |z, + z2 + z3| + 2|z2||z,| , 2|z3 +Zi|-|zi +z2| <2^1-1^ +z2 +z3| + 2|z2||z3| . 37. Temos 2|z, + z21 ■ |z2 + z31 = 2 |z2 (z, + z2 + z3) + z,z31 < 2 |z2| • |z, + z2 + z31 + 2|z, ] ]z3|, b) X3 -27 = x3 - 33 = (x-3)(x-3e)(x-3e2], onde e = - c) x3 rB = x3 +23 =(x <-2)(x + 1 + ix/3)(x + 1-i73). d) x4 + x2 +1 = (x2 - e)(x2 -e2) = (x2 -e'2)(x2 - e2) 34. Veja que x3 = x2 = -1. a)-1; b) 1; c) Considere n e {6k,6k ±1,6k ±2,6k± 3}. = (x-e)(x + e)(x-e)(x + e) = e = -^ + í̂ . Somando todas essas desigualdades obtemos |z, +z2|2 +|z2 +z3|2 +|z3 +z,|2 =|Z1|2 +|z2|2 + |z3|2 +|z, +z2 + z3|2 o que nos dá (|z, + z212 + |z2 + z312 + |z3 + z, |2 ) < (|z, | + |z2| + |z31 + |z, + z2 + z312). De onde segue a conclusão. 35. a) x4 +16 = x4 +24 = (x2 +4i)(x2 -4i) = [x2+(V2(1 + i))2][x2-(V2(1 + i))2] = (x + ^(-1 + i))(x + V2(1-i))(x-V2(1 + i))(x + 72(1 + i)). 1 73. — +—I. 2 2 36. a) x2-14X + 50 = 0; b)x2-—x + —= 0; c)x2+4x + 8 = 0. 5 5 2796 | Respostas. Dicas e Soluções dos Problemas Propostos c) Exterior do circulo de centro em (1,-2) e raio 3. 6.1.3 Representação polar dos números complexos (págs 39-41) arg(-z)=3. 12. ' 5 '2. a) x = 1, y = n/3 ; d) x - -3, y = 0 ; c) x = -2, y = 0: f) x = 0, y = -4. 16x = —, y = — 5 R2|x<- 4. a) Circunferência de raio 2 centrada na origem. b) Circunferência de centro em (0.-1) e raio 2 e seu exterior. c) Circulo de centro (0,1) e raio 3. d) Interior do ângulo determinado pelas retas y = 0. x < 0 e y = x. x < 0 . e) Quarto quadrante e o raio (OY‘. c) r = 5, t = >t; se argz * 0 se argz = 0 ’ b) e) x = 0,y = 1; b) r = 8, t’ = ; O 1. a) r = 3xÍ2 == t = —; 4 d) r = x/5. t" = arctg-1 + n / -j \e) r = 2^5. t' = arctgl -- 1 + 2x. rt + argz, se argz e [0.x) -x + argz. se argz e [x,2x) i,3x2 -y2 -3 <oj.d) M = |(x,y)eR2 |xa-j|u|(x,y)e e) M = {(x,y) e K2 | -1 < y < o|. f) M = |(x,y) e R2 | -1 < y < 1}. g) M = [(x.y) e iR2 | x2 + y2 -3x + 2 = 0|. h) A união das retas de equações x = ~ j e y = 0. 4. M = {(x,y) e R2 | y = 10- x2, y > 4}. 5. z3 =>/3(1-i) e zj =>/3(1 + i). 6. M = {(x, y) e R2 | x2 + y2 + x = 0, x * 0, x * -1} u{(0,y) eR2 | y j Clu{(-1,y) e R2 | y * o|. 7. A união das circunferências de equações x2+y2-2y-1 = 0 e x2 + y2+2y-1 = 0 . 280 Números Complexo de A A Z f) Primeiro quadrante e o raio (OX . [0,2n); b) z2 7. 5n T a 2 7n a + — 4 . c) z3 = T^cos^a + se a e [0.ti); se a e [0,n); 6. a) z, = cos(2n -a) + isen(2n -a), a e se a e [n,2n); se a e [n,2n); g) Interior do ângulo determinado pelas retas y = _^"x' x < 0 e y = V3x, x < 0 . h) A interseção do circulo de centro (-1,-1) e raio 3 com o interior do ângulo 12l cos— + isen— I; 3 3j 2) í 2)12n - arctg - I + isenl 2n - arctg— I ; determinado pelas retas y = 0, xiO e y = — x,x > 0. 1f — < 21 1( 2n . 2nlb) z, = — cos-— + isen— ;2 2< 3 3 ) d) z, = 18| cos—+ isen—|; (3 3 J =2H[cos(i4)+isen(i4)] „i ai r fn a i z, = 2 cos- cos---------+I 2| [ l 2 2j 7n) . (— +isen ;4 ) l z3 = 72^cos^a -ij .. o a F (na). fria)] d) z. = 2sen— cos------- + isen----------2[ <2 2) (.2 2J] zA = 2sen^cos^-5í.-|J + isen^ cos-^- + isen-^pj ; b) 4(cos0 + isen0); c) 4872| cos— + isen— |; d) 30| cos-^ + isen— ]. I 12 12j V 2 2) 8. a) |z| = 12, argz = 0. Argz = 2krt, argz = 0, arg(-z) = rt; 71 2 n 2 2 3 _ . i jn . 3n) 0 z6 = 41 cos—+ isen—I. 5. a) z, = 12^ x 4n 4nc) z-> = cos — -isen—;3 3 3 e) z5 = 7Í3^cos^ „ __ 3n 2 2816 | Respostas, Dicas e Soluções dos Problemas Propostos c) |z| = 2' , argz e {0, n}. 10. Se z = rfcost +isent) e n = -m , onde m é um inteiro positivo, então zn = z + isen + isen 6.1.4 As raízes enésimas da unidade (pág. 52) 1. a) zk = ‘J2 , k e {0,1}; 2 2 2 2 2 d) zk = 2 . k e {0,1}; 2 2 1 zm n(5n - a) 2 n(n-a) 2 n(5n-a) 2 se a e [n,2n]; se a e [0,n); 1 1 cosO-HsenO rm (cosmt+ isenmt) rm cosmt + isenmt ^ + 2kn - + 2kn b) zk = cos——— + isen-2—-—, k e {0,1}; 2" sen"— cos 2 11. a) 2nsenn— cos b) |z| = 14>/2, argz = ly, Argz = ly + 2kn, argz = ^y, arg(-z) = i. . , n 1 — n rtb) z h-----= 2cos— . zn 6 4n ™ 4n— + 2k>t — + 2kn cos —----------+ isen -3---------- —+ 2kn - + 2kn c) z =cos—-— + isen-4—-—, k e {0,1}; = — [cos(0 - m)t + isen(0 - m)t] = r-m(cos(-mt) + isen(-mt)) = r" (cos nt + isen nt)' n(n-a) 2 9.a) |z| = 213+-í-,argz = ^; b) |z| = 2-, argz = n ; I 5nn| cos----I 3 I + 2krt + 2kn 4 4cos —---------+ —--------- 2 Números Complexo de A a Z282 e) z0 = 4 - 3i, z, =-4+ 3i. , ke{0,1,2};+ isen c) zk = 72 , k e {0.1.2}; 3 3 3 e) z0 = 3 + i, z, = (3 + i)e, z2 = (3 + í)e2 ,onde 1,e,e2 são as raizes cúbicas da unidade. 3. a) zk = 72 ,ke{0.1,2,3}; b) zk = 72 , k e {0,1,2,3}; 4 4 d) zk = 72 , k e {0,12,3); 44 e) z0 = 2 + i, z, = —2 — i, z2 = -1 + 2Í, z3 = 1 — 2i. 3 4 3 4 3 4 3 2 n + 2kn 3 3 n + 2kn 3 —+ 2kn ■2-------- ,ke {0,12,3}; —+ 2kn - + 2kn 4 4cos—-------- + isen—--------- 5. a) Considere e( = E^.q. = ck, onde e = cos— + isen—. Então e, Ek = e] - + 2kn, 2c) zk =cos—— -----+ isen — + 2kn — + 2kn 2 2cos —----------+ isen —---------- 5n 5n— + 2kn — + 2kn 4 4cos —---------- + ise n —---------- 5n 5n Ol— + 2kn — + 2k/t d) zk = cos 3 q----- + isen-^—------- , k e {0,1,2); 4. zk= cos — + isenk e {0,1,...,n-l),n e {5,6,7,8,12}. í + 2kn 6 cos-^-------- + isen 3rt 3n----- r2kn — + 2kn 2. a) zk =cos-2 ----- + isen-2—------ , k <= {0,1,2}; :i*k. Seja r - + 2kn 6_____ 3 4 b) zk = 3^cos 5n 3__ ' 3 2836 | Respostas, Dicas e Soluções dos Problemas Propostos n • |. ke {0,1,2,3};b) zk = 2 cos + isen , k e {0,1,2};c) zk = 4 , ke {0,1,2}; 33 zk = 1 + cos , k = 0,1,3,4. 6.1.5 Algumas Transformações Geométricas no Plano Complexo (pág 160) 8. É fácil ver que qualquer solução é diferente de zero. Multiplicando por z, a equação é equivalente à z5-5z4+10z3-10z2+5z-1 =-1, z * 0. Obtemos a equação binomial (z -1)5 = -1, z * 0 . As soluções são E1 (2k + 1) 5 n —+ isen —+ 2kn2 cos—--------- + isen (2k + 1)n 5 o resto módulo n de j + k . Temos que j + k = p n + r, r e {0,1 n -1} e ^n+r=(en) ■sr=Er=BreEj • Ek = E 4 = e' EJ J + 2k7T 2 + isen —-------- ^ + 2kn d) zk = 3 cos — 3n Ol ----- f 2kn 2_____ 3 :n-JeUn.b) Podemos escrever e, ’ = — £j - v 2kn 2kn'j ,6. a) Z|, = 5| cos-^- + isen-y-I, k e {0,1,2}; rt + 2kn n + 2kn ---------- + isen----------- 4 4 7. a) A equação é equivalente à (z4 - i){z3 - 2i) = 0 . b) Podemos escrever a equação como (z3 + l)(z3 +1-1) = 0 . c) A equação é equivalente â z6 = -1 + i. d) Podemos escrever a equação equivalente como (zs -2)(z5 +i) = 0 . 3n 2___ 3 284 Números Complexo de A a Z z + 1 + 2i.5. A função fé o resultado de uma rotação z -> -iz com a translação z 6.2 SOLUÇÕES DOS PROBLEMAS PARA OLIMPÍADAS 6.2.1 Problemas envolvendo Módulos e Conjugados (págs 175-176) Problema 21. 1, i.e., |1 + az|2< |z + a|2. Isto é, Problema 22. 1. Suponha que f,g são isometrias. Então, para todos os complexos a,b, temos que |f (g(a))-f (g(b))| = |g(a)-g(b)| = |a — b|, portanto f °g também é uma isometria. 2. Suponha que f é uma isometria e seja C um ponto da reta AB. Seja f (C) = M. Então MA = f (C)f (A) = AC e, analogamente, MB = BC . Portanto, |MA -MB| = AB . Assim A, M, B são colineares. Agora, de MA = AC e MB = BC , concluímos que M = C . Consequentemente, f (M) = M, de onde segue a conclusão. 4. A função f é o resultado da rotação z -> iz, da translação z —> z + 4 - i e da reflexão em relação ao eixo real. Claramente, f é uma isometria. Obtemos z = ^—^ =-f(-y), assim |z| = |f (-y)| < 1, como desejado. 1 + azl z + a | Primeiramente, provemos que a função f está bem definida, i.e., |f (z)| < 1 para todo z com |z| < 1. Certamente, temos que |f(z)| < 1 se e somente se A última relação é equivalente á (1 + az)(l + az] < (z + a)(z + a). 1 + |a|2 |z|2 < |a|2 + |z|2 ou, de modo equivalente, (|a|2-l)(|z|2-lj < 0 . A última desigualdade é óbvia, já que |z| < 1 e |a| > 1. Para provar que f é bijetiva, basta observar que para qualquer y e A existe um *1 i 7 único zeA tal que f (z) = —— = y. 3. Isso segue imediatamente do fato de que qualquer isometria f é da forma f (z) = az + b ou f (z) = az + b, com |a| = 1. 2856 | Respostas, Dicas e Soluções dos Problemas Propostos z* cos(i 1)cp = cosnípcoscp - sennq>sen<p e 3 , como desejado. Problema 23. se zs 1. Então teR se e somente se t = t. A ,i.e.,-(z-1)(z + l) = (z + 1)[z-l). Problema 24. -l| = 2|senn<p| Basta provar que senncpeQ. Provamos por indução em n que senncp e cosncp são ambos números racionais. A afirmaçãoé óbvia para n = 1. Suponha que senncp.cosncp e Q. Então sen(n + 1)<p = senn<pcos<p + cosn<(>cosip e 3 e z-1 i(1 + z) r> 1 + ti , , De z =------ obtemos t = 1-ti |z2n Seja z = cos cp + isencp, com cos <p, sentp e Q . Então :2n -1 = cos2n<p + isen2nip-1 = 1-2sen2mp + 2isenn<pcosn<p-1 = -2senn<p(sen ntp-icosntp) e última relação é equivalente à z-1 _ z-1 i(1+z) -i(i + z)' Segue que 2zz = 2, i.e., |z| = 1, assim a imagem da função f é o conjunto {z e K | |z| = 1 e z * -1}, a circunferência unitária sem o ponto de coordenada z = -1. Para provar que a função f é injetora, seja f(a) = f(b). Então 1 + ai = l*1?'.. |sso é 1 — ai 1-bi equivalente à 1 + ab + (a -b)i = 1 + ab + (b -a)i, i.e., a = b, como requerido. A imagem da função f é o conjunto de complexos z e C tais que existe t e R com z = f(t) = l±lí. 1 ' 1-ti 286 Números Complexo de A a Z Além disso. |z, + z2 +...+z„| < |z,| + |z2|... + |zn|. (2) Multiplicando por |zk| a desigualdade (1) e por |z, +z2 +... + zn| a desigualdade (2), fazendo o somatório com k variando, obtemos: Solução alternativa. Sejam A, B, C os lugares geométricos dos complexos z,. z2, z, +z2 .respectivamente. No paralelogramoOACB temos que OA = OB = OC assim AOB = 120°. Então Problema 25.________________________________________________ Primeiramente provemos a desigualdade |Zk|^|Zl|+|z2|+... + |zk_,|+|zk+1| + ... + |zn| + |z1+Z2+... + Zn| — = cos120°+isen120° , Z2 Seja — = t e C . Então |z2 + z,t| = |z,| = |z,t| ou |1 +1| = |t| = 1. Z1 Segue que tt = 1 e 1 = |1 +1|2 = (1 + t)(l +1) = 1 +1 +1 + 1, assim t2 +1 +1 = 0 . Portanto, t é uma raiz cúbica da unidade não real. |z1|2+|z2|2+- + |z„|2+|Zl+Z2+- + Zn|2 <|z1+Z2+...-rZn|£|zk|+£|zk|Sk . k=1 k=1 Adicionando em ambos os lados da desigualdade a expressão |z,|2 +|z2|2 +... + |zn|2 +|z, +z2 +... + zn|2 para todo k e {1,2 n}. De fato, |zk| = |(z,+z2 + ... + zk_1 +zk +zk,, + ... + zn) “(Zi+Z2 +... + zk_1 + zk4.1 +... + zn)| <|z1+z2+... + zn| + |z1| + ... + |zk.1| + |zk,l| + ... + |zn|, como afirmado. Denote Sk = |z1| + ... + |zk_1| + |zk+1| + ... + |zn| para todo k. Então |zk|sSk +|z, + z2+... + zn|, para todo k. (1) — = cos120°+isen120° ou Zl portanto íl = cos —±isen—■ z, 3 3 2876 | Respostas, Dicas e Soluções oos Problemas Propostos como desejado. Problema 26. Z2 + Z2n-1 Figura 6.1. t|)] = 4r2cos2= 2r2 [l + cos(t2n. Solução alternativa. Considere zk = r(costk + isentk), k = 1,2 2n e observe que para qualquer j = 1,2 n, temos Sejam M,,M2 M2n pontos de coordenadas z,,z2 z2n e sejam A,.A2 An os pontos médios dos segmentos M,M2n,M2M2n_1,...,MnMnt1. Os pontos M, ,i = 1,2n situam-se na semicircunferência superior centrada na origem e de raio 1. Além disso, os comprimentos das cordas M1M2n,M2M2n_1 MnMn,, estão em ordem decrescente, assim OA,,OA2 OAn está crescendo. Portanto hr* de onde segue a conclusão. Zn+Zn-1 2 |zj+z2n-jti|2 =|r[(costi + cost2n_jk1) + i(sentj + sent2n.j„)]| = r2 jjcos tj + cos t2n_j+1 )2 + (sentj + sent2n_jt1 )2 ] = r2[2 + 2(costjCOSt2n_i+1+sentjSent2n_H)] *2n-j.1 ~ (i 2 obtemos 2(|z,|2 + |z2|2 +... + |zn|2 +|z,+z2 +... + zn|2) S(|Zl| + -+|Zn| + |Zl+Z2+... + Zn|)2, Números Complexo de A a Z288 e as desigualdades Problema 27. Claramente, devemos fazer a substituição 7x = u, 7y = v • O sistema se torna 4722 u-iv U + ÍV + , onde os sinais + e - correspondem.de soluções x = 472) 472) 77 J' são equivalentes à t2n -1, > t2n_, -12 2... 2 tn+1 - tn. Como 0 < t, S t2 <... < t2n < n, as últimas desigualdades são obviamente satisfeitas. Problema 28._______________________________________________________ A implicação direta é óbvia. Analisando o sentido inverso, seja |z,| =|z2 +z3|, |z2| = |z, +z3|, |z3| = |z, +z2|. 2 Iz1 + z2n| Ê |z2 + z2n.,| <... s |z„ + zn4.,| 2 .472 u2+v2 ’ 73+l .73 721. Portanto |z, + z2n_j+1 = 2rcos O quociente (u-iv)/(u2 + v2) é igual a z/|z|2 = z/(zz) = 1/z , portanto a equação acima se torna 2 .472 73+l ^±72 Isso mostra que o sistema inicial possui as soluções ' 1 2 )2 ( .7ã TãiJ ’ y l onde os sinais + e - correspondem. 77 J Assim, z satisfaz a equação quadrática z2 -í-JL + i^^-lz + l = 0 ’ V0 u2+v2J 77uíl+ , 1 - ! = -= I u2 + v2 ) 73 Porém, u2+v2 é o quadrado do módulo do número complexo z = u + iv. Isso sugere que devemos somar a segunda equação multiplicada por i com a primeira. Assim, obtemos 2896 | Respostas, Dicas e Soluções dos Problemas Propostos Segue que N2 +|z2|2 +Iz3|2 = lzz +zs|2 +Iz3 +zi|2 +Izi +z2|2- Escrevemos a última relação como + z2 + z3)(z, + Zj + z3) = 0 e obtemos |z, + z2 + z3|2 = 0, i.e., z, + z2 + z3 =0, como desejado. Problema 29. e (z3z4 - •zn + z1z4 - zn+ - + zlz2 -zn_2) = 0; Problema 30. portanto cos(t,-t2) = Então Isso é equivalente a Z,Z, + Z2Z2 + Z3Z3 = Z2Z2 + Z2Zj + Z2Z3 + ZjZj + z3z, + z,z3 + z,z, + z,z, + z,z2 + z2z, + z2Zj, i.e., z,z, + z2z2 + z3z3 + z,z2 + z2z, + z,z3 + z,z3 + z2z3 + z3z2 = 0. Temos que 1 = |z + a| = ^(r, cos t, + r2 cos t2 )2 + (qsent, + r2sent2) = 4r? + r2 + 2rlrZ COS (t, -12 ) . a4 ZlZ2 -Zn assim z,z2 + z2z3 + ■ • • + zn_1zn = 0 , como desejado. 1-r2-r2 2r,r2 a = r2 (cost2 + isent2).Seja z = qjcost, +isent,) e |z2 +a2| = |r,2 (cos2t, +isen2t,) + r2 (cos2t2 +isen2t2)| 2 Seja a = |z,| = |z2| = ... = |zn|. Então zk = —, k = 1,n zk --------------------------------------- n-1---------- n-1 a“ z,z2 + z2z3 + ... + Zn_,Zn - £ - X —— ■ k=1 k-rZkZk»! 290 Números Complexo de A A Z = 72r,4 +2r24 + 1-2r,2 -2r2 .= Jr4 + r2 + 2r2r2 ■ 2l 2r4 + 2r2 +1 - 2r,2 - 2r2 2 , i.e., Caso 1. Se a = b , a equação (1) se torna (a + ia)n = (a -ia)n . Que é equivalente a Caso 2. Se a=-b,a equação(1) pode ser reescrita como (a-ia)n =-(a+ ia)n . Isso é, 1 + i 1-i Tomando o módulo de ambos os lados da equação azn=b(z)" (1) que apresenta soluções somente para n = 4k, k e Z . Nesse caso, as soluções são z = a (1 + i), a # 0 . Problema 31. É fácil ver que z = 0 é uma raiz da equação. Considere z = a + ib * 0,a,b e R . Observe que se a = 0 , então b = 0 ese b = 0, então a = 0 . Portanto, vamos assumir que a.b * 0. h-2H 72 4r4 + 4r24 - 4r,2 - 4r2 + 2 > 1 - 4r2 + 4r24 . (1~2ri)2 2 A desigualdade |z2 +a2| > l-rf-rg 2r,r2 ,n = 1, i.e., in =1, Obtemos (2r2 -l)2 > 0 , e assim está provado. obtemos |a| = |b| ou a = ±b. M=-1, i.e., (-i)" = 1 + iJ v ' = ^(r2cos2t, +r2 cos2t2)2 +(r2sen2t, +r2sen2t2) = Jr,4 + r2 + 2r2r22 cos 2 (t, -12) = + r24 + 2r2r22 (2 cos2 (t, -12) -1) -1 2916 | Respostas, Dicas e Soluções dos Problemas Propostos Problema 33. como queríamos provar. Obtemos 8a2 - 8a + 2 s (8a2 - 8a + 2^cos(t, -t2), i.e., 1 â cos(t, -12), que, obviamente, é verdade. Isso é, 2^'a2 + b2 + 2ab cos(t, -12) à ^2 + cos(t, -t2), i.e., 4a2 + 4(1-a)2 +83(1-3)005(1, -12) > 2 + 2003(1, -12). que apresenta soluções somente para n = 4k + 2, k e Z. Portanto, z = a (1 — i), a * 0. Seja r = |z,| = |z2| = • ■ • = |zn| > 0 . Então A + A +... + A = = -1(zÍ+4+... + zk) = o, Problema 32. Seja z, = cos t, + isent, e z2 = cos t2 + isent2. A desigualdade |az,+bz2|SlA|Al é equivalente a ^(acost, +bcost2)2 +(asent, +bsent2)2 i 1 J(cos t, + cos t2 )2 + (sent, + sent2 )2 . + H r2“' A igualdade ocorre se e somente se t, = t2, i.e., z, = z2 ou a = b = 1 Para concluir, a) se n é ímpar, então z = 0; b) se n = 4k, k e Z , então z = {a(1 + i) | a e R} ,i.e., uma reta passando pela origem; c) se n = 4k + 2,keZ, então z = {a(1-i) | a e R}, i.e., uma reta passando pela origem. 292 Números Complexo de A A Z 6.2.2 Equações algébricas e polinômios (pág 181) Problema 11. bb |c| c a a a Isso é, —(x, + x2).|x,x2| = -(x, + x2).x,x2, i.e., (x, + x2)r2 = |x,|2x2 +x,|x2|2 . Segue que (x, + x2)r2 = (x, + x2)r2, o que é claramente verdade. ab|c| |a|bc ãa|a| ãa|a| Problema 12. Observe que z2 = z2 =1 e z| = z4 = -1. Se n = 6k + r, k e Zer e {0,1,2,3,4,5}, então z?+zS =z',+zí. e zj+zj =zr3+z,4. A igualdade z" + z2 = zj +zj é equivalente a z\ + z2 = z3 + z4 e ocorre somente para r e {0,2,4}. De fato. i) se r = 0 , então z° + z2= 2 = z3 + zj; ii) se r = 2 . então z2 + z2 = (z, + z2)2 - 2z,z2 = (-1)2 — 2 ■ 1 = —1 e z3 + z2 = (z3 + z4) — 2z3z4 = 12—2-1 = —1, iii) se r = 4 , então z\ +z2 = z, +z2 = 1 e z| +z4 = -(z3 +z4) = -(-1) = 1. Seja r = |z,| = |z2|. A relação ãb|c| = |a|bc é equivalente a Os outros casos são: iv) r = 1, então z4 + z2 = -1 # z3 + z4 = 1; v) r = 3 , então z? + z2 = 1 +1 = 2 * z3 + z4 = -1 -1 = -2 ; vi) r = 5 . então z® +z| = z2 +z| = -1 * z3 +z4 = ~(z3 +z4) = 1. Portanto, os números desejados são pares. Essa relação pode ser escrita como a 2936 | Respostas, Dicas e Soluções dos Problemas Propostos Problema 13. Seja f (x) = x6 + ax5 + bx4 + cx3 + bx2 + ax +1 6 como queríamos provar. Problema 14. A equação az , e assim está provado. Portanto, temos que P(z0) + P(-z0) = 2(|z|i + i) = 2i(l + |a|). e s = n (x - xk) = n (xk - X), para todo x e C. k=1 k=1 +isen(H) Temos que n(x£ +l)= f[ (xk + i)- f[ (xk -i) = f (-i) f (i) k=1' ' k-1 k=1 = (i6 + ai5 + bi4 + ci3 + bi2 + ai +1) • (i6 - ai5 + bi4 - ci3 + bi2 - ai +1) = (2ai-ci)(-2ai + ci) = (2a -c)2, |a|ié equivalente a z2 = cos^-d + isen^-j-1^. Definindo zn = cos| — | <4 2) Problema 15._______________________________________________________ Seja z um número complexo raiz do polinômio f. Da relação dada, segue que 2z3 também é raiz de f. Observe que se |z| > 1, então |2z3+z| = |z||2z2+l|>|z|(2|z|2-l)>|z|. Para um número complexo z com |z| = 1, observe que P(z) + P(-z) = az2 +bz + i + az2 -bz + i = 2(az2 +i). Basta escolher z0 tal que azj; = |a|i. Seja a = |a|(cost + isent), te[0,2n). Tomando o módulo de ambos os lados segue que |P(z0)| + |P(-z0)| a 2(1 + |a|). Istoé, |P(z0)| a 1 + |a| ou |P(-z0)| > 1 + |a|. Perceba que |z0| = |-z0| = 1, como necessário. 294 Números Complexo de A á Z ,2k ,2k e 1 = 1. f(2x2) = x: Consequentemente, se f tem uma raiz z, com então f tem uma raiz z2 = 2zf + z, com |z21 > |z,|. Podemos continuar com esse procedimento e obter infinitas raizes de f,zvz2,... com ••■ > |z2| > |z,|, um absurdo. Portanto, todas as raizes satisfazem |z| < 1. Iremos mostrar que f não é divisível por x. Suponha, por absurdo, o contrário e escolha o maior k > 1 com a propriedade que xkdivide f. Segue que f (x) = xk (a + xg(x)), com a * 0 , assim onde g,g,,g2 são polinômios e a, *0 é um número real. Arelação f(x)f(2x2) = f(2x3 + x) é equivalente a xk (a + xg(x))x2k (a, +xg, (x)) = xk (a + xg2 (x)) que é impossível para a * 0 e k>0. Seja m o grau do polinômio f. Os polinômios f (2x2) e f(2x3 + x) possuem graus 2m e 3m , respectivamente. Se f(x) = brnxm + ... + b0,então f(2x2) = 2mbmx2m + ... e f(2x3 + x) = 2mbmx3m + ... Da relação dada, encontramos que bm • 2m ■ bm = 2m ■ bm , assim bm = 1. Novamente, usando a relação dada, segue que f2 (0) = f (0), i.e., b„ = b0 , assim b 1. O produto das raizes do polinômio f é ±1. Levando em consideração que para qualquer raiz z se f temos que |z| < 1, segue que as raizes de f têm módulo 1. Considere z uma raiz de f. Então |z| = 1 e 1 = |2z3 + z| = |z||2z2 +1| = |2z2 +1| > 12z21 -1 = 2|z21 - A igualdade é possível se e somente se os números complexos 2z2 e -1 possuem o mesmo argumento; isto é, z = ±i. (al+2k*1x2g(2x2)) = x2k(al + xg,(x)) f(2x’ + x) = xk(2x2 + 1)k (a + (2x2 + 1 )xg(x)) = xk(a + xg2(x)), Como f tem coeficiente reais e suas raizes são ±i, segue que f é da forma (x2 +1) para algum inteiro positivo n. Usando a identidade (x2 + l)(4x4 +1) = (2x3 + x)2 +1 obtemos que os polinômios desejados são f(x) = (x2+lj", onde n é um inteiro positivo arbitrário. 2956 | Respostas. Dicas e Soluções oos Problemas Propostos 6.2.3 De identidades algébricas a propriedades geométricas (pág 190) Problema 12. e usando as Problema 13. Sz,z2z3z4 ; Zziz2z3 ; Sejam A.B.C.D pontos de coordenadas a.b.c.d , respectivamente. Se a + b = 0 , então c + d = 0. Assim a + b = c + d , i.e., ABCD é um paralelogramo inscrito em uma circunferência de raio R = |a| e assim está terminado. Se a + b * 0 , então os pontos MeNde coordenadas a + b e c + d , respectivamente, são simétricos em relação a origem O do plano complexo. Já que AB é a diagonal do losango OAMB, segue que AB é a bissetriz perpendicular ao segmento OM. Do mesmo modo, CD é a bissetriz perpendicular ao segmento ON. Portanto, A. B. C, D são os pontos de interseção da circunferência de raio R com as bissetrizes perpendiculares aos segmentos OM e ON, assim A, B, C, D são vértices de um retângulo. d) • y Já que a d = b c, segue que 1 2 1 2com raizes zk, k = 1,5.Então a =-£z, e b =£z,z2 = — (íz,) --£z, =0. Denotando por ro módulo usual e calculando os conjugados, também obtemos r2 Z1Z2Z3Z4ZS r4 Z,Z2ZjZ4Z5 Considere o polinômio P(X) = X5 + aX4 + bX3 + cX2 + dX + e Solução alternativa. Primeiramente, perceba que de a + b + c + d = 0 segue que a + d = -(b + c), i.e., |a + d| = |b + c|. Assim, |a+d|2 = |b+ c|2 propriedades de produto real encontramos que (a + d) (a + d) = (b + c) (b -c). Isto é, |a|2+|d|2+2a-d = |b|2+|c|2+ 2b c . Levando em consideração que |a| = |b| = |c| = |d|, obtemos que a • d = b • c. Por outro lado, AD2 = |d-a|2 = (d - a) (d-a) = |d|2 +|a|2 -2a d = 2(R2 - a • Analogamente, temos que BC2=2(R2-b-c AD = BC. portanto ABCD é um retângulo. ------ r4 de onde se obtém d = 0 e 0 = £z,z2 = Z------ = Z1Z2 - r2 0 = Ez, =E—= Z1 Números Complexo de A a Z296 que, ao serem somadas, nos dá a conclusão desejada. ■^ = 1,(1) = 1.(2) CA Figura 6.2. portanto c = 0 . Segue que P(X) = X5 +e, logo z,,z2 z5 são as raizes quintuplas de e e a conclusão está provada. senBAM senCBM senACM senMAC senMBA senMCB senBAN senCBN senACN senNAC senNBA senNCB Problema 15.__________________________________________________________ Sejam a. b. c, m e n as cooordenadas de A, B, C. M e N, respectivamente. Já que as retas AM, BM e CM são concorrentes, bem como as retas AN, BN e CN, segue do teorema de Ceva que Por hipótese, BAM = NAC e MBA = CBN . Assim, BAN = MAC e NBA = CBM . Combinando (1) com (2), essas igualdades implicam senACMsenACN = senMCB senNCB . Problema 14._________________________________________________________ a) Considere um plano complexo de origem em M. Denote por a, b, c as coordenada de A, B, C, respectivamente. Como a(b-c) = b(a-c) + c(b-a) temos que |a||b - a| = |b(a - c) + c(b -a)| < |b||a - c| + |c||b - a|. Portanto, AM ■ BC < BM AC + CM ■ AB ou 2R ■ AM ■ senA < 2R BM ■ senB + 2R ■ CM ■ senC o que nos dá AM ■ senA < BM ■ senB + CM ■ senC . De a) temos as relações AA, ■ sena < AB, ■ sen[3 + AC, ■ seny , BB, ■ sen|i < BA, ■ sena + BC, • seny, CC, • seny < CA, • sena + CB, ■ senp , 6 | Respostas, Dicas e Soluções dos Problemas Propostos 297 Assim, cada uma dessas razões é igual ao seu valor em módulo e. portanto. = 1. 6.2.4 Resolvendo problemas geométricos (págs 211-213) Segue que a2 -2b2 -2c2 -ab-ac + 5b c = 0, i.e., (a c-a n-a’ m-a b-a A última relação é equivalente a c2 - 2a c + a2 +b2 -2a b + a2 = 5c2 -10b c + 5b2 , i.e . 2a2 - 4b2 - 4c2 - 2a ■ b - 2a ■ c +10b c = 0 . Problema 26,__________________________________________________________ Sejam a. b, c as coordenadas dos pontos A, 8. C, respectivamente. Usando o produto real de números complexos, temos que AC2 + AB2 = 5BC2 se e somente se |c - a|2 + |b - a|2 ■- 5|c - b|2. i.e.. (c-a)(c-a) + (b-a)(b-a) = 5(c-b)(c-b). Problema 27.__________________________________________________________ Denotando por letra minúscula a coordenada de um ponto denotado pela respectiva letra maiúscula, obtemos AM-AN AB AC BM-BN BA-BC CM-CN CA-CB Portanto, cos(NCM + 2ACM] -cosNCM = cosfNCM + 2NCB)-cosNCM . e, consequentemente, ACM = NCB . Já que BAM = NAC.MBA = CBN e ÃCN = MCB , as seguintes razões complexas são todas reais positivos: ) = °- , a-kb c =-------- 1-k 2b) (a + b-2c) = 0, assim -b |-( a -c Aúltima relação mostra que as medianas que partem de B e de C são perpendiculares, como desejado. _(m-a)(n-a) (m-b)(n-b) (m-c)(n-c) (b-a)(c-a) (a-b)(c-b) (b-c)(a-c) , , c- ka b =--------. 1-k , b-kc a =-------- , 1-k m-b c-b ------- :-------- e a-b n-b m-c,a-c b-c n-c a-b 2 Números Complexo de A a Z298 Então. o que prova que os triângulos ABC e A"B"C" são semelhantes. Assim am„ < Rs . Assim, umci < Rs , como queríamos provar. b + d 2 Problema 28._________________________________________________________ Considere o plano complexo com origem no circuncirculo do triângulo ABC e sejam z,,z2,z3 as coordenadas dos pontos A, B, C. Problema 29.__________________________________________________________ Considere o plano complexo com origem no circuncentro O e sejam a, b, c, d as coordenadas dos pontos A, B, C, D. Os pontos médios E e F das diagonais AC e BD têm coordenadas a+— e Usando o produto escalar de números complexos temos que £ AB2 + BC2 + CD2 + DA2 = 8R2 se e somente se (b-a) (b-a) + (c-b)-(c-b) + (d-c) (d-c) + (a-d) (a-d) = 8R2, i.e., 2a ■ b + 2b • c + 2c • d + 2d ■ a = 0 . R m K 2K A desigualdade — > é equivalente a 2rm„ < Rh<t, i.e., 2—m„ < R— . Usando números complexos, temos 2am„ = 2|z2 - z3||z, -= |(z2 - z3 )(2z, -z2 - z3 )| = |z2(z1-Z2) + Z,(z2-Z3) + Z3(z3 ~ *1 )| S |z2||zi -z2| + |zi||z2 -z3| + |Z3||Z3 -Z,| = R(a +p + r) = 2Rs . „ c'-kb' e a =--------- 1-k 1-k a'-kc' (l + k2)b-k(a + c) 1-k " (1-k)2 (l + k2)c -k(b + a) c"-a" (i + k2)(c-a)-k(a-c) c_a b"-a" (i + k2j(b-a)-k(a-b) b~a (l + k2)a-k(b + c) bi_ (1-k)2 2996 | Respostas. Dicas e Soluções dos Problemas Propostos A última relação é equivalente a b(a + c) + d(a + c) = O, i.e., (b-rd)(a+c) = O. Isto é, AC 1 BD ou uma das diagonais AC e BD é diâmetro da circunferência C. (m + n + p + q) + (a + b onde r. = cosa + isena . Já que e + f = g + h , então EGFH é um paralelogramo, como queríamos provar. Sejam E. F, G. H os pontos médios das diagonais BD, AC. MP. NQ, respectivamente; então e como e + e2 = -1 segue que m + n + p + q = a + b + c + d . Portanto, os quadriláteros ABCD e MNPQ possuem o mesmo baricentro. Já que os triângulos ABM, BCN, COP e DAQ são equiláteros, temos que m + be + as2=0. n + CE + bE2=0, p + dc + cc2 = 0, q-at^di:2 = 0 . Problema 32,__________________________________________ Considere os pontos E, F, G, H tais que OE1AB, OE = CD, OF X BC, OF=AD, OG JL CD, OG = AB, OH 1 AD, OH = BC. onde O é o circuncentro de ABCD. Problema 30,__________________________________________________________ Denote por letra minúscula a coordenada de um ponto denotado pela respectiva letra maiscula e seja s = cos 120° +isen120° . c + (b + d-a -c)c 2 Fazendo o somatório dessas igualdades, obtemos + c + d)(e + E2) = 0 , b + d e =-------, 2 e h = — b + d a + c Encontramos —- -------— = 0, i.e., OE ± OF ou E = O ou F = O. Problema 31.__________________________________________________________ Denote por letra minúscula a coordenada de um ponto denotado pela respectiva letra maiúscula. Usando a fórmula de rotação, obtemos m = b + (a-b)E, n = c + (b-c)e, p = d + (c-d)E, q = a + (d-a)e. , a+c b + d + (a + c-b-d)cf =------- q ----------------------------- — 2 2 300 Números Complexo de A A Z Temos a> = g = o. 0 . Como e + g = f+ h , obtemos AD + BCcos(A+B) + iBCsen(A+ B) = CD(cosD -isenD) + AB(cos A + isenA) e a conclusão segue. Problema 33.__________________________________________________________ Considere o plano complexo com origem no circuncentro O do triângulo. Sejam a,b,c,w,g,Z|as coordenadas dos pontos A,B,C,O9,G,I, respectivamente. Sem perda de generalidade, vamos supor que o circunraio do triângulo ABC é igual a 1. assim, |a| = |b| = |c| = 1. Provamos que EFGH é um paralelogramo. Já que OE = CD,OF = AD e EOF = 180°-ABC = ADC , segue que os triângulos EOF e ADC sâo congruentes, assim EF = GH . Analogamente, FG = EH e a afirmação está provada. Considere o plano complexo com origem em O tal que F situa-se no eixo real positivo. Denote por letra minúscula a coordenada de um ponto denotado pela respectiva letra maiúscula. Temos |e| = CD, |f| = AD, |g| = AB, |h| = BC . Além disso, FÕG = 180°-C =Â, GÕH=B, HÕE=C, assim f = |f| = AD, g =|g|(cosA+ isenA) = AD(cosA+isenA), h = |h|[cos(A + B) + isen(A + B)] = BC[cos(A + B) + isen(A + B)] , e =|e|[cos(A + B + C) + isen(A + B + C)] = CD(cosD-isenD). a + b + c (a-b)|a-c| + (a-c)|a-b| 6 |a-b|+|b-c| + |a-c| Isto é equivalente a (a + b + c)[(a -b)|a - c| + (a -c)|a - b|] = 0, i.e., Re {(a + b + c)[ja -b)|a-c| + (a-c)|a-b|^ = ( Encontramos Re ||a - c| (aa + ba + ca - ab - bb - cb) +|a -b|(aa + ba + ca - ac -bc -ccjj = 0 . (1) a + b-c a + b + c a|b-c| + b|a-c| + c|a-b| 2 ’ 9 3 ' Z| |a — b| + |b —c| + |a —c| Usando as propriedades de produto escalar de números complexos, temos OgG 1 Al se e somente se (<u — g) ■ (a — z,) = 0, i.e.. 3016 | Respostas. Dicas e Soluções dos Problemas Propostos Segue que | 0, i.e.. 0. Isso é equivalente à AC • BC2 - AC3 + AB ■ BC2 - AB3 = 0. representa uma rotação é igual a A última relação pode ser escrita como BC2(AC + AB) = (AC + AB)(AC2 - AC AB - AB2 ). assim, AC ■ AB = AC2 + AB2 - BC2. m1 + m2 ’ 2 ^(o, +z(m) -o,)) + ^(o2 + z(m2 -o2)) = o + z(m' - o), i.e., o lugar geométrico de Méa circunferência centrada em O de raio OM'. (b) Devemos usar ângulos no sentido anti-horário de módulo n . Observe que QMX2 = QPM2 = QPÕj = OO/5 2 . Problema 34.__________________________________________________________ (a) Denote por letra minúscula o número complexo associado ao ponto representado pela correspondente letra maiúscula. Sejam M', M e O denotando os pontos médios dos segmentos [m',M2J. [M,M2] e [O,O2] ■ respectivamente. Seja também a - c |[(bb - bc - cb + cc)J - (aa - ca - ca + cc) +|a-b|[(bb-bc-cb- |a-c|(|b-c|2-|a-c| Observe que aa = bb = cc = 1 e Re(ba-ab) = Re(ca -ac) = 0, assim, a relação (1) é equivalente a Re(|a -c|(ca -cb) + |a- b|(ba-bc)| = 0. i.e., |a -c|(ca + ca -cb -cb) + |a -b|(ãb + ab- bc -bc) = 0 . + z(m;-o,)) + ^(o2 i.e., Â=", 3 z = ——— = ———, assim a multiplicação por z m) - o, m2 - o2 em relação a origem através de algum ângulo. Então, m + cc) - (aa - ab - ab + bb) J = :|2) + |a-b|2(|b-c|2-|a-b|2) = Obtemos cos A = -5 como desejado. 302 Números Complexo de A a Z Semelhantemente, QM2M, = QO2O, , implicando que os triângulos QM,M2 e QO,O2 Problema 35. ou (lembrando que o, = 0 e t = 2p ) °3 ~ °2 3,-0, q-o, ou, equivalentemente, —----- - = q-o2 Sem perda de generalidade, suponha que o triângulo A,A2A3 é orientado no sentido antihorário (i.e., o ângulo A,A2A3 é orientado no sentido horário). Seja P a reflexão de O, em relação à T. Usaremos os números complexos com origem O,, onde cada ponto denotado por letra maiúscula é representado pelo número complexo com a correspondente letra minúscula. Seja Çk = ak/p para k = 1,2 , tal que zi->Çk(z-z0) é uma transformação através do ângulo PO,Ak com raio O,A3/O,P em relação ao ponto correspondente a z0. Como O, e A, situam-se em lados opostos da reta A2A3 , os ângulos A2A3O, e A2A3A, possuem orientações opostas, i.e., o primeiro é orientado no sentido anti-horário. Portanto, os ângulos PA3O, e A2O3A, são ambos orientados no sentido anti-horário. Como PA3O, = 2A2A3O2 = A2O3A,, segue que os triângulos isósceles PA3O, e A2O3A., são semelhantes e possuem a mesma orientação. Assim, o3 = a, + Ç3 (a2 - a,). De modo análogo, o2 = a, + Ç2 (a3 - a,). Consequentemente, °3 - °2 = (C2 “ Çj )a, + Çaa2 “ Çza3 ~ ?2 (a2 - a3 ) + Ç3 (Ç2P) -Í2 (Ç3P ) = Í2 (a2 ~ a3 ) ’ _ a2 ~ a3 _. 1 a2 ~ a3 2 P - O, 2 t - O, são semelhantes com mesma orientação. Assim, -5—— = -——, q c?2 q rrig (q-mi)-(q-°i) . o,-m, _o,-m, (q - m2) - (q - o2) o2 — m2 o2 — m2 Como as retas O,m', e O2M2 se intersectam, o, - n-i, * o2 - m2 e podemos resolver essa equação para encontrar um único valor de q 3036 | Respostas. Dicas e Soluções dos Problemas Propostos Portanto, temos a igualdade (ü,c2 + <ú2e + <o3 = 0 . (2) Essa igualdade pode ser escrita de acordo com a forma equivalente abaixo co3 = u>i(1 + e) + (_s)<u2 ■ (3) No nosso caso, para n = 662 , obtemos z1996 = z0 +662(1-e)(<o,e2+(o2eProblema 36.__________________________________________________________ Suponha que a origem O do sistema de coordenadas no plano complexo seja o centro da circunferência circunscrita. Então, os vértices A,. A2. A3 são representados pelos números complexos a>,,a>2,<i)3 tais que |<o,| = |<o2| = |o>3| = R . co3) = z0. O ponto P2 é representado por z2 = z0e2 + (1 - s)(DiE + (1 - e)ci3 , e P3 por z3 = z0£3 + (1 - e)w1s2 + (1-s)oj2e + (1- e)w3 = z0+(1-e)(o>fE2 + co2e + U)3 ) . Por meio de uma simples indução em n, mostra-se que após n ciclos de três rotações obtemos que P3n é representado por Z3n = z0 + n (1 - e)(w1e2 + “zE + “3 ) • Seja e = cos^ + isen^-. Então e2+e + 1 = 0 e s3 = 1. Suponha que Po é representado pelo número complexo z0. O ponto P, é representado pelo número complexo z, = zoe + (1-e)w,. (1) Levando em consideração que 1 + e = cos-2. + isen^, a igualdade (3) pode ser interpretada, usando o lema da página 218, como segue: o ponto A3 é obtido sob que é n/2 . Além disso, O2O3/O,A, = — A3A2/O,T ou O,A,/O2O3 = 2O,T/A2A3 . Isso completa a prova. 2 rotação do ponto A, com centro em A2 através do ângulo de . Isso prova que A,A2A3 é um triângulo equilátero. Portanto, o ângulo entre [O,A,] e [O2O3] é igual ao ângulo entre [O,T] e [A3A2], 304 Números Complexo de A a Z Problema 37. y pA coB (zn-Zm) = 0.z - Figura 6.3. O resultado requirido é equivalente ao que segue: a reta bissetriz lMN do segmento MN passa por um ponto fixo P(x0,y0). Seja R o ponto médio do segmento MN. ZM + ZN 2 Usando z = x + iy , obtemos x(c - b)(1 - cost)-y(c -b)sent = -l(|zN|2 - |zm|2 ) e zN = iaw + (1-u)c . 1 Então zR = —(zM + zN). O ponto Z do plano é um ponto de lMN se e somente se as retas RZ e MN são ortogonais. Usando o produto real de números complexos, obtemos Após um simples cálculo, obtemos |zM|2 = 2b2 + a2 - 2b2 cos t - 2absent e |zN|2 = 2c2+a2-2c2 cost - 2acsent. Isso é equivalente a z ■ (zN - zM) = ^(|zN|2 -|zM|2) Seja B(b,0),C(c,0) os centros das circunferências dadas e seja A(0,a),X(0,-a) os seus pontos de interseção. Os números complexos associados a esses pontos são zB =b,zc =c,zA = ia e zx =-ia .respectivamente. Depois de rotacionar A através de um ângulo t em relação a B, obtemos um ponto M e depois de rotacionar A em relação a C, obtemos o ponto N. Seus números complexos correspondentes são dados pelas fórmulas: zM = (ia - b)w + b = iam + (1 - io)b 3056 | Respostas. Dicas e Soluções dos Problemas Propostos Zp Problema 38. . Já que K é representado por K(-i(j3 -l)j. 2-731 C(-t -í) 0(1 - >) Figura 6.4. R e o ponto médio de BL é S o ponto médio de AN éo <) Sejam A(1 + i).B (-1 + i),C(-1 - i).D (1 - i) os vértices do quadrado. Usando a simetria da configuração dos pontos, em relação aos eixos e ao centro O do quadrado, iremos fazer os cálculos dos pontos situados no primeiro quadrante. Então, L e Portanto, a condição de ortogonalidade se torna x(1-cost)-ysent = (b + c) - (b + c)cost - asent. Isso pode ser escrito na forma (x-b-c)(1-cost) = (y-a)sent. Essa equação mostra que o ponto P(x0,y0), onde x0 =b-c,y0 =a é um ponto fixo da família de retas lMN . O ponto P pertence a reta que passa por A paralela a BC e é o ponto simétrico de X em relação ao ponto médio do segmento BC. Isso segue da igualdade b + c zx= — M são representados pelos complexos l(73 -1),m((73 -l)i). O ponto médio do -2 + 73 i ---------------------- í- _ 2 2 2-73 . _i_ 2 +2 segmento LM é pÍ^—+i^2 1 Números Complexo de A a Z306 2 RP2 =|ZP-ZR|2 = + i ■0 C 2 v Figura 6.5. Problema 39.__________________________________________________________ Sejam 1,e,e2as coordenadas dos pontos A, B, C, M, respectivamente, onde s = cos 120° +isen120° . 7-473 2-273 2 (12-7^)(7 + 473 ) 2(7-4V3)(7 + 4V3) Isso prova que SRP = — . Do mesmo modo, cosRPQ = —e RPQ = 6 2 6 273 -3 .73-2 = -------------+ l------------ 2 2 Por reflexão em OA, também temos que PQ2 = RP2 = 7 - 473 . Para os ângulos, temos Basta provar que SR=RP=PQ e SRP = RPQ = -^. Para qualquer ponto X 6 denotemos por Zxo número complexo correspondente. Temos que RS2 = |ZS-ZR|2 =(-2 + 73)Z =7-473’ 75-1 .75-1 2-75 I ----------+1 2------ 2 2 2 2 (273-3)2+(273-2)2 28-1673 _7 4 4 3-273 cos SRP =---- 2---- 3076 | Respostas, Dicas e Soluções dos Problemas Propostos Usando a fórmula de rotação, obtemos e assim b| s |a — b||a +b|, ou |m2 - a2| + |m2 - b2| > |a2 - b2|. Isso segue imediatamente da desigualdade triangular. (b-cjd-(b-c)a + (bc-bc) = 0 (1)ou Como b - c = - fazendo a interseção dessas expressões em (1). obtemos que d-b c - b a-b) c-b/ Essa relação mostra que Méo ponto médio do segmento [AV], de onde segue a conclusão. Considere o ponto V tal que MEVD é um paralelogramo. Se d, e. v são as coordenadas dos pontos D, E, V, respectivamente, então v = e + d-m. |m + a| + |m - b||m + d = m + (r. -m)c R2(b-c) bc R2(-a-r-b + c) R2(h-2a) bc bc R2(b2-c2) bc e = m + (r.2 -m)>:2. = m + ez + e - mie2 + c) = m -1 + m = 2m -1. v = m + e2 - mc + m + e4 -mc2 -m e bc - bc = Problema 41.__________________________________________________________ Sejam a, b, c, h e o as coordenadas de A, B, C, H e O, respectivamente. Consequentemente, aã = bb = cc = R2 e h = a + b + c . Já que D é simétrico a A em relação à reta BC, as coordenadas d e a satisfazem k - 2bc g a . -bc + ca + ab d =------------------- a Problema 40,__________________________________________________________ Considere o plano complexo de origem no centro do paralelogramo ABCD. Sejam a, b, c, d, m as coordenadas dos pontos A, B, C. D, M, respectivamente. Segue que c = -a e d = -b . Basta provar que |m — a| ■ |i 308 Números Complexo oe A a Z onde k = bc-rc + ab. Analogamente, temos que . ê , f = Como A = a2b2c2 2 é igual a b-c a-c a-b c-b AC)/BC3 . Por outro lado, fazendo algumas Problema 42.__________________________________________________________ Sejam a, b, c, d e e as coordenadas de A, B, C, D e E, respectivamente. Assim, d - (2b + c)/3 e e = 2d - a . Já que ACB = 2ABC , a razão k-2ab c R2 (a -b)(h- 2c) abc R2 (a-c)(h-2b) abc e-c . b-c ,2 R2 (h-2b) ca 7c-b? (e-bj f 4(b-a)-(c-a) l 3 é real e positiva. Ela é igual a (AB2 ■ contas, mostra-se que a razão e - R2(h-2c) ab (b-a) + 2(c-a) ~ 3 = _4_ (b-a)2(c-a) 4 AB2 AC 27 + (b_c)2 27 BC3 que é um número real. Assim, os argumentos de (e - c)/(b - c) e (c - b)2/(e -b)2, ECB e. 2EBC , respectivamente, diferem um do outro por um múltiplo inteiro de 180° Facilmente encontramos também que ÉCB = 2ÉBC ou ÉCB = 2ÉBC -180°, dependendo se a razão é positiva ou negativa. Para provar que a segunda ocorre, 1 ---------7*(b-c)3 (b-a)(k-2ab) ab (c-a)(k -2ca) ca _ R2(c-a)(a-b) -(ck-2abc) (h-2c) (bk-2abc) -(h-2b) -R2(b-c)(c-a)(a-b)(hk-4abc) e h = R2k/abc , segue que D, E e F são colineares se e somente se A = 0 . Isso é equivalente a hk - 4abc = 0 , i.e., hh = 4R2 . Da última igualdade, obtemos OH = 2R . le-d ê-dl|f-d f-d| d d 1 e ê 1 f f 1 k-2ca e =---------- b 6 | Respostas, Dicas e Soluções dos Problemas Propostos 309 Figura 6.6. de onde segue a conclusão. 6.2.5 Resolvendo problemas trigonométricos (pág 220) temos de mostrar que AB2 • AC/BC3 é maior que 4/27 . Escolha um ponto F sobre o lado AC tal que CF = CB . _4_ 27 ' Problema 11,_________________ (i) Considere o número complexo AB2 AC BC3 1 z =------- (cos6 + isen9). cos0' ' Como ACBF é isósceles e ACB = 2ABC , temos que CFB = ABC. Portanto, AABF e AACB são semelhantes e AB : AF = AC : AB . Já que AF = AC + BC , AB2 = AC (AC + BC). Seja AC = u2 e AC+ BC = v2 . Então AB = uve BC = v2 -u2 . De AB + AC > BC , obtemos que u/v > 1/2 . Portanto U4V2 (u/v)4 ; (1/2)4 (v2-u2)3 (l-u2/v2)3 >(1-1/4)3 n—1 1_ Da identidade £ zk = -=- (1) k=0 1-Z 310 Números Complexo oe A a Z obtemos (cosnO + isennO)cosO- Segue que 0 = i cotgO- Finalmente, basta tomar 0 = 30° no somatório acima. sennO senOcos"' n Segue que £cosk0coskG = k=1 n , Zzk = k.1 n Assim,£ cosk 0(cosk0+isenk0) k=l Problema 12, Seja senn0cos' senO sennecos1 senO 0(cosn0 + isenn0) senO senn0 senOcos"’ e, assim, basta substituir 0 = 30° . (ii) Procedemos de modo análogo, porém, agora considerando o número complexo z = cos0(cos0 + isen0). Usando a identidade (1) obtemos - + i 0 cos" O-cosn0 senOcos11-10 ,n+1g 2n . 2n co = cos— + isen— n n cosk0 k»OCOSk 0 z-znM 1 —z 1 cosn10 -isen0 cos0*1 OcosnO senO cos 0 (cos 0 + isenO) - cosn*10(cos (n +1)0 +isen (n +1)0) sen20-icos0sen0 . cos 0(cos 0 + isenO) - cosn+10 (cos (n +1) 0 + isen (n +1) 0) sen0(cos0 + isen0) cos"*1 >"’10 / , „ ------ + i cotg0- n_, 1 1------- —(cosnO+ isenn0) É----r—(cosk0 + isenkO) = —ǰslO-------------------------- k’ocos 0 1—— (cos0 + isen0) cos0v ' 6 | Respostas, Dicas e Soluções dos Problemas Propostos 311 para algum inteiro n. Considere a soma Para todo k = 1 n -1, temos ,2n ,2n . (1) Por outro lado, usando a expansão binomial, temos que ,ik As relações (1) e (2) dão a identidade desejada. Problema 13. (cosa -isena)2p^j. 2 1-a>|n 1 —ü>* (n~1)* n®+cos: n-1, , U Assim Sn =4" + Z(l + <ok) e (l + a>k )2" = 2‘ = 4" 1 + cos2n 4 k . 2k?t . 2krt _ kn ( kn kn1 + co = 1 + cos----- + isen------= 2cos— cos—-t-isen— n n n V n n . Sn=4"++(1 + a>)2n+(l + <o2)2n ;2n—(COs2kn + isen2kn) = 4" cos2" — .I2ncos: Para p = 0, tome a0 = 1. Se p > 1, seja z = cosa + isena e observe que z2p = cos2pa + isen2pa , z-2p = cos2pa -isen2pa e :'2p 1r, . .2p -----= — ((cos a + isena) + z2pcos 2pa =---- = 2n + n(nn)- (2) n-1 / t, sn= E 1 + <0k) k=0' ' i*n i*n 312 Números Complexo oe A a Z cos2pa 6.2.6 Mais sobre Raizes Enésimas da unidade (págs. 228 - 229) Problema 11. Observação Uma solução alternativa pode ser obtida usando o fato que é um inteiro positivo para qualquer inteiro positivo a > 1 e n > k . te[0,2n). Usando a expansão binomial dos termos acima, obtemos cos2p-2 asen2ct + ...+ (-!)' d c Problema 12.____________ Reescreva a equação como bx + atzY ax + bõ) = ( TH ao<ai«--> ap +í2pl(-1)p l2pJV ’ .+P<o.\2pJJ (ak-l)(a’ in"1-l)...(an-k+1-l) -1)-(a-1) Seja p = 1,2 m e seja zeUp. Então zp = 1. Perceba que n-m + 1,n-m + 2 n são m inteiros consecutivos e, como p<m, existe um inteiro k e {n -m + 1,n -m + 2 n) tal que p divide k. Sejak = k'p. Segue que zk = (zp)k =1, portanto z e Uk <= Un_m+1 u Un_m+2 u...uU„, como afirmado. Como |c| = |d|, temos que |-| = 1 e considere — = cost + isent, p2Pjc°s2p-2 asen2ct +... + (-1)P QPj Assim, cos2paé um polinõmio de grau p em sen2a , portanto, existem e !R tais que cos2pa = a0 + a,sen2a + ... + apsen2pa paratodoaeR, ,2pa- sen2pa. d c 3136 | Respostas, Dicas e Soluções dos Problemas Propostos Segue que = uk, (1) k = 0, n -1.onde uk = cos + isen , k = 0,n-1A relação (1) implica que xk = = xk, k = 0, n -1xk = como queríamos provar. Multiplicando ambos os lados por x e derivando novamente, chegamos em = g(x). Problema 13,_____________________ Derivando a identidade familiar abaixo t + 2kx n t + 2kn n bxk + ao. axk + bã bauk -aa b-auk bauk - aa b-auk aa - bauk auk -b + (n + 1)2x"-x-1 Í3 em relação a x, obtemos £ kxk“’ k=1 xn*1 -1 x-1 -(n + 1)xn +1 7^? £k2xk’1 k=1 -l)xnt1 Tomando x = z e usando |z| = 1 (que nos foi dado), obtemos n2xn+2-(2n2 +2n onde g(x) =-------------------------- Zzk = k=0 nxn+1 Para provar que as raizes xk,k=0,n-1 são números reais, basta mostrar que xk = xk para todo k = 0,n -1. Denote |a| = |b| = r. Então r2 1 r2 — a----------a b uk a r^_rM b a uk k=1 o Números Complexo de A A Z314 9(z) = ■ (2) Problema 14. e M e. consequentemente, Problema 15. , onde Portanto xkx,•xkx2■• • xkxn = x,x2 ■ • • xn, assim xk = 1, isto é, xk é uma raiz enésima da unidade. O número xk foi escolhido arbitrariamente, assim M éo conjunto das raizes enésimas da unidade, como afirmado. U„,) + Z S(Unk 1 ’ k<l<s v K (n + 1)(2n + 1) 6 Por outro lado, levando em consideração que zn = 1, z * 1, obtemos n(nz2-2(n + 1)z + n + 2) n(nz-(n + 2)) (z-1)2 De (1) e (2) nós, portanto, concluímos que |nz-(n + 2)| £ Un| nllns ) + ... = um inteiro . Se x e y são elementos arbitrários de M, então x,y' x•—- = xy e M . y Sejam x,,x2,...,xn elementos do conjunto M , tome aleatoriamente um elemento xk e M, k = 1,n . Já que xk * 0 para todo k = 1,n , os números xkx,,xkx2 xkxn são distintos e pertencem ao conjunto M, assim {xkx„xkx2 xkxn} = {x„x2 xn}. X 1 i Definindo x = y e M temos 1 = — <= M. Para x = 1 e y e M obtemos - = y e M y y S(A)=XS(U^)-XS(U^ m números são as d-ésimas raizes da unidade. Isso mostra que A \ {0} = U U„k Up = {z e C | zp = 1}. Já que S(Up) = 0 para p a 2, S(U,) = 1 e Up nU, = U(pq) obtemos a) Iremos denotar por S(X)a soma dos elementos de um conjunto finito X. Suponha que 0 #ze A . Como A é finito, existe inteiros positivos m < n tais que zm = zn, de onde zn-m = 1. Seja d o menor inteiro positivo k tal que zk = 1. Então, 1,z,z2 zd’1 são diferentes e a d-ésima potência de cada é igual a 1; portanto esses 3156 | Respostas, Dicas e Soluções oos Problemas Propostos ) 6.2.7 Problemas envolvendo polígonos (pág. 237) Problema 12. k = 0,1 1989 Assim, se existe A tal que S (A) = k , então existem BeCtaisque S(B) = k-1 e S (C) = k + 2. A conclusão agora segue facilmente. ^P2P4P6UPlP4P5 S(AoUpi) = S(A) + S(Upi)-S(AnUp,) = k-S(U,) = k-1. 1939 Como Y AkAk+1 = 0,o problema pode ser redefinido como segue: encontre uma k=o permutação (n0,n, n,989) dos números 1 ,2 1990 tal que 'paPsPe ) 1989 £ nkco = 0 . k=0 Suponha que exista um 1990-ágono e sejam A0A,...A1989 seus vértices. Os lados AkAk+1, k = 0,1 1989 definem os vetores AkAki1 que podem ser representados no plano complexo pleos números zk =nkwk, onde to = cos +isen . Aig90=A0 e n0.n1l...,n1989 representam uma 1990 1990 permutação dos números 12,22,...,19902 . Além disso. S(AoUpiP2P3 P2P4P6 ^PiP4P5 nUt m b) Suponha que para algum inteiro k existe A = U Lk tal que S(A) = k .Sejam k=1 K P1.P2--P6 primos distintos que não são divisores de de qualquer nk. Então = s(A)+s(uP1P2P3)+s(uP1P4P5)+s(uP2P4P6)+s(uI -S(AnUp1P2P3) + SIA n UP1P2P3 r> UP1P4P5 ) +...-S(AnUpiP2P3 r-iUp^ps nUW4Pe mUP3P5P6J = k + 40-4S(U1)-£ S(UPk) + 10S(U1)-5S(U1) + S(U1) =k-4+10-5+1=k+2 316 Números Complexo de A a Z que dividem a circunferência unitária do plano coordenado em 995 arcos iguais. Sejam Bo = 1. B,.... B994 os vértices de um 995-ágono regular inscrito em uma circunferência unitária. Pretendemos relacionar as medidas 3,7,11.... 3979 aos vetores unitários OB0,OB,.... OB994 de tal forma que a soma dos vetores obtidos é zero. Dividimos 995 medidas em 199 grupos de tamanho 5: (3,7,11,15,19),(23,27,31,35,39) (3963,3967,3971,3975,3979)- Figura 6.7. 2k + 3,2k + 7,2k +11,2k +15,2k +19 (k = 0.... 198). Observe que 1990 = 2- 5-199 .A estratégia é somar vetores após um agrupamento conveniente de 2, 5 e 199 vetores de tal modo que essas somas parciais podem dar o resultado adequado. Para começar, considere os pares de números (l2,22),(32,42 ).... (19882,19892) e associe essas medidas aos pares de vetores opostos, respectivamente: (cok, ), k = 0 994 Somando os vetores obtidos, obtemos 995 vetores de medidas 22-12=3; 42-32 =7; 62 -52 = 11;...; 19892 -19882 = 3979 Seja C = cos—— + isen——, <o = cos——- + isen-—— as raizes primitivas da unidade 5 5 199 199 de ordem 5 e 199, respectivamente. Seja P, o pentágono de vértices 1,Ç,Ç2,Ç3,Ç4 . Assim, rotacionamos P, em relação a origem com as coordenadas através 2kndos ângulos 9k =—-, k = 1.... 198, para obter novos pentágonos P2.....P198, 199 respectivamente. Os vértices de Pk+, são o>k ,cükÇ,cokÇ2,cokÇ3,™kÇ4, k = 0.... 198 . Nós atribuímos aos vetores unitários definidos pelos vértices Pk das respectivas medidas: 6 | Respostas, Dicas e Soluções dos Problemas Propostos 317 198 = 0 , segue que a soma é igual — em relação ao centro O do círculo circunscrito ao octógono. Dessa forma, Figura 6.8. Portanto, temos de calcular asoma: T r(2k + 3)wk + (2k + 7)cokÇ + (2k +11)cokÇ2 + (2k + 15)cokÇ3 + (2k+ 19)wkÇ4] k=0L J 198 198 Z 2kwk(l + Ç + Ç2 + Ç3+Ç4) + (3 + 7Ç + 11Ç2 + 15Ç3 + 19Ç4) Z uk 2n n . . ... .. .. ~8 ~ 4 um caminho é uma sequência dessas rotações, submetidas a algumas condições. Problema 13.___________________________________________________________ É conveniente tomar um octógono regular inscrito em uma circunferência e denotar seus vértices como seguem: A = Ao, A,, A2, A3, A4 — E, A_3, A_2, A_1 Imaginamos um passo no caminho como sendo uma rotação de um ângulo de Se a rotação é no sentido anti-horário, nós somamos o ângulo de —; 4 se a rotação é no sentido horário nós adicionamos o ângulo de —y. O ponto 4 de partida é Ao, que é representado pelo número complexo z0 =cosO + isenO. Qualquer vértice Ak do octógono é representado por zk = cos-^^ + isen—. É 8 8 2knconveniente trabalhar somente com os ângulos - 4 < k < 4 . Porém, esses k's são inteiros de módulo 8, de tal forma que z4 = z_4 e A4 = A^ . Como 1 + Ç + Ç2 +Ç3 + Ç4 = 0 e 1 + cü + w2 + ... + 0 a zero. Números Complexo de A a Z318 a) uk = ±1 para qualquer k = 1,2.... n ; mais precisamente u, = +1 se o arco (Pk_,Pk) Yn = xn_2+2yn_2. (1) Devemos associar a um caminho de comprimento n, dito (P0P, • • Pn), uma sequência ordenada (u,,u2,...,un) de inteiros que satisfazem as seguintes condições: Essa igualdade vem das seguintes construções. A sequência (u,.....un_2) para qual sn_2 = ±2 dá origem a uma única sequência de comprimento n com sn = 0 estendendo-a também para (u,.... un_2,1,1) ou (ur u2,.... u„_,, - 1, - 1). Além disso, a sequência (u1,...,un_2) com sn_2 = 0 dá origem a sequência (u1,...,un_2,1,-1) ou é e uk =-1 se o arco (Pk_-|Pk) é ~; b) u,+u2+... + uk e {-3,-2,-1,0,1,2,3} para todo k = 1,2 n —1; c) u, +u2 + ,.. + un = ±4 . Por exemplo, a sequência associada ao caminho (Ag.A.,,A0,A1,A2,A3,A4) é (-1,1,1,1,1,1). A partir de agora, iremos considerar somente sequências que satisfazem a), b) e c). Obviamente as condições a), b) e c) definem uma função bijetiva entre o conjunto de caminhos e o conjunto de sequências. Para qualquer sequência u„u2,...un e qualquer k,1 < k < n , iremos denominar a soma sk = u, +u2 +... + uk uma soma parcial da sequência. É fácil ver que para qualquer k, sk é um número par se e somente se k é par. Portanto, a2n_, = 0 . Assim, temos de provar a fórmula para números pares. Para n pequeno, temos a2 = 0,a4 =2 ; por exemplo, somente as sequências (1,1,1,1) e (-1,-1,-1,-1) de comprimento 4 satisfazem as condições a)-c). A seguir, iremos provar uma relação recorrente entre os números a„, com n par.O primeiro passo é observar que se sn = ±4 , então sn_2 = ±2 . Além disso, se (u,.u2....un_2) é uma sequência que satisfaz a), b) e sn_2 = ±2 , existem somente dois caminhos para estendê-la a uma sequência que satisfaz c) também: ou a sequência (u,.u2.... un_2,+1, +1) ou a sequência (u,.u2.....Un_2,-1,-1). Portanto, se denotarmos por xn o número de sequências que satisfazem a), b) e sn = ±2 , então n é par e an = xn_2. Seja yn o número de sequências que satisfazem a), b) e sn = 0. Assim, n é par e temos a igualdade 3196 | Respostas, Dicas e Soluções dos Problemas Propostos n>4, (3) C2n Seja zM = cost + isent, t e [0,2n) as coordenadas do ponto M. Partindo da hipótese 2 ' Problema 14._______________________________________________ ___________ Considere o plano complexo de origem no centro do polígono. Sem perda de generalidade, vamos assumir que as coordenadas de A.B.C são 1,e,e2 , 1 ( / <— \n-1= -U 2 + 72T2I? > é facilmente demonstrável. (C2n )n21 ■ (2 - 72 )n 1) obedece as mesmas condições. Isso an.2 =4a„-2a, com valores iniciais a2 = 0,a4 = 2 . A sequência (an), n>2, n par, é unicamente definida por a2=0,a4=2 e a relação (3). Portanto, para responder a questão, basta provar que a sequência , . An .encontramos que t > — . Assim, n MA = |zM -1| = ^(cost-1)2 +sen2t = 72-2cost = 2sen—; (u, un_2,-1,1). Finalmente, toda sequência de comprimento n com sn = 0 termina em uma das seguintes "terminações": (-1,-1),(1,1),(1,-1),(-1,1). A seguinte igualdade também é verificada: x = 2x 2 + 2y 2. (2) Isso corresponde a propriedade de que qualquer sequência de comprimento n, para a qual sn = ±2 , pode ser obtida ou a partir de uma sequência semelhante de comprimento n-2adicionando a terminação (1, - 1) ou a terminação (-1.1), ou a partir de uma sequência de comprimento n - 2 para a qual sn_2 = 0 adicionando a terminação (1,1) ou a terminação (-1,-1). Agora, o problema é obter an = xn_2 , partindo das relações (1) e (2). Subtraindo (1) de (2)obtemosxn_2 = xn -yn,paratodon > 4par.Portanto,y„_2 = xn_2 - xn_4. Substituindo a última igualdade em (2) obtemos a relação recorrente: xn = 4xn_2 -2xn_4, para todo n > 4 , n par. Levando em consideração que xn = ant2 , obtemos a relação recorrente linear respectivamente, onde e = cos—+ isen—. n n „ (1 71 2sen------- (2 n MB = |zM -e| = ^2-2cos(t-^ = 320 Números Complexo de A à Z MC AB =|e- Temos MB2 - AB2 = 4sen2 = MAMC, 2 n Problema 15. zn = -1, i.e.. para os pontos médios dos arcos AjAj. Rotacione o polígono A,A2 • • • An de tal forma que as coordenadas dos seus vértices são as raizes complexas da unidade de ordem n, c1,e2,...,en . Denomine por z a coordenada do ponto P localizado sobre o circuncirculo do polígono e perceba que |z| = 1. _ (t 2n'|2sen---------12 n ) 2n = -2-2sen — como queríamos pfevar. n ne = cos—+ isen— . n n para k = -1. Então £ PAk+1 • PA„. k=0 ‘"+k+1-k?ol“’Ek| la + ek| 2n n sen— Problema 16.__________________________________________________________ Sem perda de generalidade, suponha que os pontos Ak tenham coordenadas ek"1 para k = 1.... 2n , onde A igualdade zn -1 = n(2-f:j) I2" ~1| = Fl]2 — Ej| = PIPAj . j=i j=i 1=1 Como |z" -1| < |z|n +1 = 2, segue que o máximo valor de fl PA2 é 2 e é obtido para j = 1.....n , onde A„41 = A,. = |zM -s2| = ^2- 2cos^t - —= 1| = ,/2-2cos— = 2sen —. v n n Seja a a coordenada do ponto P, |cc| = 1. Temos PAk+1 = |cr — ek | e PA„+k+1 = |a-en^| = |a + sk|, í t n'l . 2 ® 2n------- -4sen — = 2 cos— (2 nJ n ít-—1 ,.L = 2sen—-2senf—- — 2 2 6 | Respostas, Dicas e Soluções dos Problemas Propostos 321 = 2n-a2 ã2VJ = 2n. 6.2.8 Números Complexos e Combinatória (pág. 245) Problema 11. A relação acima é equivalente a 2cos^ I (cost + isent)k . senkt. t . t> cos—+ isen— . 2 2) já que t e [0,tt] . De (2), segue que nt cos—, 2 S —(z + 1)szr (1) osklrsnklslrP ’ { ’ k+s+r=n s+2r=n Osk.ssn k+s=n k-oVkJ j (coskt + isenkt) = 2cosir2k(' Considere o número complexo z = cost + isent e a soma tn = EÍ? Observe que k=°lkj = X[(a-ek)(a-êk)][(a + ek)(ã + êk)]=Z(2-aêk-ãEk)(2 + aêk+ãek) I (2 - a2ê2k - ã2 e2k) = 2n - a2 Z ê2k - ã 2X e2k k=0x ' k=0 ê2k-1_2 e2n-1 „ ê2-1 e2-1 como queríamos provar. k=0\^K)\ KJ No produto (1 + X)n (1 + zX)n = (l + (z +1) X + zX2) , encontrar o coeficiente de Xn equivale a calcular nt . nt-^ cos— + isen— , . 2 2 ' assim Mf n Y2kV t T‘2k Sn = Z L. k 2COS“I k=o\2kk r2k ntsen—.) 2 s("hk = k-O^kJ A forma trigonométrica do complexo 1 + z é dada por 1 + cost + isent = 2cos2 — + 2isen—cos— = 2cos—11 2 2 2 2V t 2 Números Complexo de A A Z322 Observação 2n n se n é impar, n se n é par. Problema 12. 1) No Problema 4 considere p = 4 para obter 2) Considere p = 5 no Problema 4. Encontramos que e 1_ 4 2*1 2 + 22 2nn cos----- 5 nn cos— 4 2n 4 assim A2 + B2 + C2 - A„B„ - BnC„ - C„A„ = (A„ + Bne + Cne2)(An + Bne2 + C„e) = (1-c)n(l-c2)n = (l-e-e2+l)" =3". / y 1 + 2 cos—l 4J f Y 1 + 2 cos—l 5j ínY2k>(2kAkJ Y2k'|2n‘2k = Jv k J 2) Obviamente. An+Bn+Cn=O. Substituindo Cn=-(An+Bn) na identidade anterior obtemos A2 + AnBn + C2 = 3""1. Se z = -1, então E (-1)k k=0 Se z = 1, então £ I k=o : cos— k 5 ) 2 I 2n-k ,, . , . _ n 75+1Usando as famosas relações cos— =-------- desejada segue. J Problema 13.___________________________ 1) Seja c uma raiz cúbica da unidade diferente de 1. Temos